You are on page 1of 82

Answer Sheets

1 A
O B
O C
O D
O E
O 6 A
O B
O C
O D
O E
O 11 A
O B
O C
O D
O E
O 16 A
O B
O C
O D
O E
O
SECTION 2 A
O B
O C
O D
O E
O 7 A
O B
O C
O D
O E
O 12 A
O B
O C
O D
O E
O 17 A
O B
O C
O D
O E
O

1 3
4
5
A
O
A
O
A
O
B
O

B
O
C
O

B
O

C
O
D
O

C
O
D
O
E
O

D
O
E
O
E
O
8
9
10
A
O
A
O
A
O
B
O

B
O
C
O

B
O

C
O
D
O

C
O
D
O
E
O

D
O
E
O
E
O
13
14
15
A
O
A
O
A
O
B
O

B
O
C
O

B
O

C
O
D
O

C
O
D
O
E
O

D
O
E
O
E
O
18
19
20
A
O
A
O
A
O
B
O

B
O
B
O
C
O

C
O

C
O
D
O

D
O
D
O
E
O

E
O
E
O

1 A
O B
O C
O D
O E
O 7 A
O B
O C
O D
O E
O 12 A
O B
O C
O D
O E
O 17 A
O B
O C
O D
O E
O

SECTION 2 A
O B
O C
O D
O E
O 8 A
O B
O C
O D
O E
O 13 A
O B
O C
O D
O E
O 18 A
O B
O C
O D
O E
O

3 A
O B
O C
O D
O E
O 9 A
O B
O C
O D
O E
O 14 A
O B
O C
O D
O E
O 19 A
O B
O C
O D
O E
O

2 4
5
6
A
O
A
O
A
O
B
O

B
O
C
O
B
O

C
O
D
O

D
O
E
O

C
O
E
O
D
O E
O
10
11
A
O
A
O
B
O C
O
B
O
D
O E
O

C
O D
O E
O
15
16
A
O
A
O
B
O C
O
B
O
D
O E
O

C
O D
O E
O
20
21
A
O
A
O
B
O
B
O
C
O

C
O
D
O
D
O
E
O
E
O

1 A
O B
O C
O D
O E
O 9 A
O B
O C
O D
O E
O 17 A
O B
O C
O D
O E
O 24 A
O B
O C
O D
O E
O

2 A
O B
O C
O D
O E
O 10 A
O B
O C
O D
O E
O 18 A
O B
O C
O D
O E
O 25 A
O B
O C
O D
O E
O

SECTION 3 A
O B
O C
O D
O E
O 11 A
O B
O C
O D
O E
O 19 A
O B
O C
O D
O E
O 26 A
O B
O C
O D
O E
O

4 A
O B
O C
O D
O E
O 12 A
O B
O C
O D
O E
O 20 A
O B
O C
O D
O E
O 27 A
O B
O C
O D
O E
O

3 5
6
7
A
O
A
O
A
O
B
O

B
O
B
O

C
O

C
O
D
O

D
O
C
O
E
O

E
O
D
O E
O 13
14
15
A
O
A
O
A
O
B
O

B
O
B
O

C
O

C
O
D
O

D
O
C
O
E
O

E
O
D
O E
O 21
22
23
A
O
A
O
A
O
B
O

B
O
B
O

C
O

C
O
D
O

D
O
C
O
E
O

E
O
D
O E
O 28
29
30
A
O
A
O
A
O
B
O
B
O

B
O
C
O

C
O

C
O
D
O
D
O

D
O
E
O
E
O

E
O

8 A
O B
O C
O D
O E
O 16 A
O B
O C
O D
O E
O

1 A
O B
O C
O D
O E
O 8 A
O B
O C
O D
O E
O 15 A
O B
O C
O D
O E
O 22 A
O B
O C
O D
O E
O

2 A
O B
O C
O D
O E
O 9 A
O B
O C
O D
O E
O 16 A
O B
O C
O D
O E
O 23 A
O B
O C
O D
O E
O
SECTION 3 A
O B
O C
O D
O E
O 10 A
O B
O C
O D
O E
O 17 A
O B
O C
O D
O E
O 24 A
O B
O C
O D
O E
O

4 4
5
6
A
O
A
O
A
O
B
O
B
O
C
O

C
O
B
O
D
O
D
O

C
O
E
O
E
O
D
O E
O
11
12
13
A
O
A
O
A
O
B
O
B
O
C
O

C
O
B
O
D
O
D
O

C
O
E
O
E
O
D
O E
O
18
19
20
A
O
A
O
A
O
B
O
B
O
C
O

C
O
B
O
D
O
D
O

C
O
E
O
E
O
D
O E
O
25
26
27
A
O
A
O
A
O
B
O
B
O
B
O
C
O

C
O

C
O
D
O
D
O
D
O
E
O
E
O
E
O

7 A
O B
O C
O D
O E
O 14 A
O B
O C
O D
O E
O 21 A
O B
O C
O D
O E
O

1 A
O B
O C
O D
O E
O 7 A
O B
O C
O D
O E
O 12 A
O B
O C
O D
O E
O 17 A
O B
O C
O D
O E
O

SECTION 2 A
O B
O C
O D
O E
O 8 A
O B
O C
O D
O E
O 13 A
O B
O C
O D
O E
O 18 A
O B
O C
O D
O E
O

3 A
O B
O C
O D
O E
O 9 A
O B
O C
O D
O E
O 14 A
O B
O C
O D
O E
O 19 A
O B
O C
O D
O E
O

5 4
5
6
A
O
A
O
A
O
B
O

B
O
C
O
B
O

C
O
D
O

D
O
E
O

C
O

E
O
D
O E
O
10
11
A
O
A
O
B
O C
O
B
O
D
O E
O

C
O D
O E
O
15
16
A
O
A
O
B
O C
O
B
O
D
O E
O

C
O D
O E
O
20
21
A
O
A
O
B
O
B
O
C
O

C
O
D
O
D
O
E
O
E
O

Copyright © 2005 Thomson Peterson’s, a part of The Thomson Corporaton 1


SAT is a registered trademark of the College Entrance Examination Board, which
was not involved in the production of and does not endorse this product.
Answer Sheets

SECTION 1 A
O B
O C
O D
O E
O 5 A
O B
O C
O D
O E
O 9 A
O B
O C
O D
O E
O 13 A
O B
O C
O D
O E
O

2 A
O B
O C
O D
O E
O 6 A
O B
O C
O D
O E
O 10 A
O B
O C
O D
O E
O 14 A
O B
O C
O D
O E
O

6 3
4
A
O
A
O
B
O C
O

B
O
D
O

C
O
E
O

D
O E
O
7
8
A
O
A
O
B
O C
O

B
O
D
O

C
O
E
O

D
O E
O
11
12
A
O
A
O
B
O C
O

B
O
D
O

C
O
E
O

D
O E
O
15
16
A
O
A
O
B
O

B
O
C
O

C
O
D
O

D
O
E
O

E
O

SECTION
For Questions 1–13:

1
7 Only answers entered in the ovals in each grid area will be scored.
You will not receive credit for anything written in the boxes above the ovals.
2 3 4 5

6 7 8 9 10

11 12 13

2 Copyright © 2005 Thomson Peterson’s, a part of The Thomson Corporaton


SAT is a registered trademark of the College Entrance Examination Board, which
was not involved in the production of and does not endorse this product.
Practice Test

1
Section 1
20 Questions j Time—25 Minutes
12345678901234567890123456789012123456789012345678901234567890121234567
12234567890123456789012345678901212345678901234567890123456789012123456 7
1234567890123456789012345678901212345678901234567890123456789012123456 7
1234567890123456789012345678901212345678901234567890123456789012123456 7
1234567890123456789012345678901212345678901234567890123456789012123456 7
1234567890123456789012345678901212345678901234567890123456789012123456
Directions: Read each of the passages carefully, then answer the questions that come after them. 7
34567890123456789012345678901212345678901234567890123456789012123456
1234567890123456789012345678901212345678901234567890123456789012123456 7
1 The answer to each question may be stated overtly or only implied. You will not have to use 7
12234567890123456789012345678901212345678901234567890123456789012123456 7
1234567890123456789012345678901212345678901234567890123456789012123456
outside knowledge to answer the questions—all the material you will need will be in the passage 7
1234567890123456789012345678901212345678901234567890123456789012123456 7
1234567890123456789012345678901212345678901234567890123456789012123456 77
34567890123456789012345678901212345678901234567890123456789012123456
itself. In some cases, you will be asked to read two related passages and answer questions about
1234567890123456789012345678901212345678901234567890123456789012123456
their relationship to one another. Mark the letter of your choice on your answer sheet.
1234567890123456789012345678901212345678901234567890123456789012123456 7
1234567890123456789012345678901212345678901234567890123456789012123456 77
1234567890123456789012345678901212345678901234567890123456789012123456 7
1234567890123456789012345678901212345678901234567890123456789012123456 7
1234567890123456789012345678901212345678901234567890123456789012123456 7
1234567890123456789012345678901212345678901234567890123456789012123456
Musical notes, like all sounds, are a result of 1. In this passage, musical notes are used
1234567890123456789012345678901212345678901234567890123456789012123456 7
1 the sound waves created by movement, like the primarily to 7
1234567890123456789012345678901212345678901234567890123456789012123456 77
2 34567890123456789012345678901212345678901234567890123456789012123456
1234567890123456789012345678901212345678901234567890123456789012123456
1 rush of air through a trumpet. Musical notes (A) illustrate the difference between 7
1234567890123456789012345678901212345678901234567890123456789012123456
are very regular sound waves. The qualities of 7
12234567890123456789012345678901212345678901234567890123456789012123456
34567890123456789012345678901212345678901234567890123456789012123456
human-produced and nonhuman- 7
7
1234567890123456789012345678901212345678901234567890123456789012123456
these waves—how much they displace mol-
1234567890123456789012345678901212345678901234567890123456789012123456 produced sound. 7
1 ecules, and how often they do so—give the note 7
12234567890123456789012345678901212345678901234567890123456789012123456 (B) demonstrate the difference between 7
1234567890123456789012345678901212345678901234567890123456789012123456
its particular sound. How much a sound wave 7
34567890123456789012345678901212345678901234567890123456789012123456
1234567890123456789012345678901212345678901234567890123456789012123456 musical sound and all other sound. 7
1234567890123456789012345678901212345678901234567890123456789012123456
displaces molecules affects the volume of the 7
1 note. How frequently a sound wave reaches (C) provide an example of sound 7
1234567890123456789012345678901212345678901234567890123456789012123456 7
1234567890123456789012345678901212345678901234567890123456789012123456 properties common to all sound. 7
2 34567890123456789012345678901212345678901234567890123456789012123456
1234567890123456789012345678901212345678901234567890123456789012123456
your ear determines whether the note is high- 7
1234567890123456789012345678901212345678901234567890123456789012123456 (D) convey the difference between 7
1234567890123456789012345678901212345678901234567890123456789012123456
or low-pitched. When scientists describe how 7
1234567890123456789012345678901212345678901234567890123456789012123456 musical pitch and frequency pitch. 7
1 high or low a sound is, they use a numerical 7
2 34567890123456789012345678901212345678901234567890123456789012123456
1234567890123456789012345678901212345678901234567890123456789012123456 (E) explain the connection between 7
1234567890123456789012345678901212345678901234567890123456789012123456
measurement of its frequency, such as “440 7
1234567890123456789012345678901212345678901234567890123456789012123456 number and letter names for sounds. 7
1234567890123456789012345678901212345678901234567890123456789012123456 77
vibrations per second,” rather than the letters
1 musicians use.
12234567890123456789012345678901212345678901234567890123456789012123456 7
1234567890123456789012345678901212345678901234567890123456789012123456 7
1234567890123456789012345678901212345678901234567890123456789012123456 7
1234567890123456789012345678901212345678901234567890123456789012123456 7
1 34567890123456789012345678901212345678901234567890123456789012123456 7
1234567890123456789012345678901212345678901234567890123456789012123456 77
2 34567890123456789012345678901212345678901234567890123456789012123456
1
12234567890123456789012345678901212345678901234567890123456789012123456 7
1 34567890123456789012345678901212345678901234567890123456789012123456 7
12234567890123456789012345678901212345678901234567890123456789012123456 7
1 34567890123456789012345678901212345678901234567890123456789012123456 7
1234567890123456789012345678901212345678901234567890123456789012123456 7
1234567890123456789012345678901212345678901234567890123456789012123456 7
1234567890123456789012345678901212345678901234567890123456789012123456 7
12345678901234567890123456789012123456789012345678901234567890121234567
Copyright © 2005 Thomson Peterson’s, a part of The Thomson Corporaton 3
SAT is a registered trademark of the College Entrance Examination Board, which
was not involved in the production of and does not endorse this product.
12345678901234567890123456789012123456789012345678901234567890121234567
1234567890123456789012345678901212345678901234567890123456789012123456 7
12 34567890123456789012345678901212345678901234567890123456789012123456 7
1234567890123456789012345678901212345678901234567890123456789012123456
2. All of the following are true statements 3. The passage cites Walker’s interaction 7
12 34567890123456789012345678901212345678901234567890123456789012123456 7
12 34567890123456789012345678901212345678901234567890123456789012123456
about pitch, according to the passage, with Langston Hughes as 7
1234567890123456789012345678901212345678901234567890123456789012123456 7
12
2
34567890123456789012345678901212345678901234567890123456789012123456
EXCEPT:
34567890123456789012345678901212345678901234567890123456789012123456
7
7
1 (A) instrumental in her early work being
12 34567890123456789012345678901212345678901234567890123456789012123456 7
12 34567890123456789012345678901212345678901234567890123456789012123456 7
(A) Nonmusical sounds cannot be published.
12 34567890123456789012345678901212345678901234567890123456789012123456
(B) influential in her decision to study at 7
12
2
34567890123456789012345678901212345678901234567890123456789012123456
referred to in terms of pitch.
34567890123456789012345678901212345678901234567890123456789012123456
7
7
1
12 34567890123456789012345678901212345678901234567890123456789012123456
(B) Pitch is solely determined by the Northwestern University. 7
12 34567890123456789012345678901212345678901234567890123456789012123456 7
12 34567890123456789012345678901212345678901234567890123456789012123456 7
frequency of the sound wave. (C) not as important at the time it
12 34567890123456789012345678901212345678901234567890123456789012123456 7
1234567890123456789012345678901212345678901234567890123456789012123456
(C) Pitch is closely related to the happened as it is now, due to 7
12 34567890123456789012345678901212345678901234567890123456789012123456 7
12 34567890123456789012345678901212345678901234567890123456789012123456 7
vibration of molecules. Hughes’ fame.
12 34567890123456789012345678901212345678901234567890123456789012123456 7
12
2
34567890123456789012345678901212345678901234567890123456789012123456
(D) Pitch cannot be accurately described (D) a great encouragement for Walker’s
34567890123456789012345678901212345678901234567890123456789012123456
7
7
1 with letter names. confidence as a poet.
12 34567890123456789012345678901212345678901234567890123456789012123456 7
12 34567890123456789012345678901212345678901234567890123456789012123456
(E) Humans’ perception of pitch is not (E) important to her choice to study at 7
12 34567890123456789012345678901212345678901234567890123456789012123456 7
12 34567890123456789012345678901212345678901234567890123456789012123456
affected by the intensity of the New Orleans University. 7
12 34567890123456789012345678901212345678901234567890123456789012123456 7
12 34567890123456789012345678901212345678901234567890123456789012123456
sound wave. 7
12 34567890123456789012345678901212345678901234567890123456789012123456 7
12 34567890123456789012345678901212345678901234567890123456789012123456
4. The passage suggests that Walker’s 7
12 34567890123456789012345678901212345678901234567890123456789012123456
Line Margaret Walker, who would become 7
2 34567890123456789012345678901212345678901234567890123456789012123456
1234567890123456789012345678901212345678901234567890123456789012123456
decision to become a poet 7
1 one of the most important twentieth- 7
12 34567890123456789012345678901212345678901234567890123456789012123456 7
12 34567890123456789012345678901212345678901234567890123456789012123456 7
century African-American poets, was (A) occurred before she entered college.
12 34567890123456789012345678901212345678901234567890123456789012123456
(B) was primarily a result of her interac- 7
12 34567890123456789012345678901212345678901234567890123456789012123456
born in Birmingham, Alabama, in 1915. 7
12 34567890123456789012345678901212345678901234567890123456789012123456 7
12 34567890123456789012345678901212345678901234567890123456789012123456 7
(5) Her parents, a minister and a music tion with Hughes.
12 34567890123456789012345678901212345678901234567890123456789012123456
(C) was not surprising, given her 7
12 34567890123456789012345678901212345678901234567890123456789012123456
teacher, encouraged her to read poetry 7
2 34567890123456789012345678901212345678901234567890123456789012123456
1234567890123456789012345678901212345678901234567890123456789012123456
upbringing. 7
1234567890123456789012345678901212345678901234567890123456789012123456
and philosophy even as a child. Walker 7
1234567890123456789012345678901212345678901234567890123456789012123456
(D) occurred after her transfer to 7
1234567890123456789012345678901212345678901234567890123456789012123456 77
completed her high school education at
1 Northwestern University.
12 34567890123456789012345678901212345678901234567890123456789012123456
Gilbert Academy in New Orleans and 7
12 34567890123456789012345678901212345678901234567890123456789012123456
went on to attend New Orleans Univer- (E) was sudden and immediately 7
1234567890123456789012345678901212345678901234567890123456789012123456
(10)
7
1234567890123456789012345678901212345678901234567890123456789012123456
sity for two years. It was then that the successful. 7
12
2
34567890123456789012345678901212345678901234567890123456789012123456 7
34567890123456789012345678901212345678901234567890123456789012123456 7
1234567890123456789012345678901212345678901234567890123456789012123456
important Harlem Renaissance poet
1234567890123456789012345678901212345678901234567890123456789012123456 7
1 Langston Hughes recognized her talent 7
12 34567890123456789012345678901212345678901234567890123456789012123456 7
Questions 5–10 are based on the following
12 34567890123456789012345678901212345678901234567890123456789012123456
and persuaded her to continue her 7
12 34567890123456789012345678901212345678901234567890123456789012123456
passage. 7
12 34567890123456789012345678901212345678901234567890123456789012123456
(15) education in the North. She transferred 7
1234567890123456789012345678901212345678901234567890123456789012123456
Line F. Scott Fitzgerald was a prominent 7
1234567890123456789012345678901212345678901234567890123456789012123456
to Northwestern University in Illinois, 7
1234567890123456789012345678901212345678901234567890123456789012123456
American writer of the twentieth century. 7
2 34567890123456789012345678901212345678901234567890123456789012123456
1234567890123456789012345678901212345678901234567890123456789012123456
where she received a degree in English in
This passage comes from one of his short 7
7
1234567890123456789012345678901212345678901234567890123456789012123456 7
1234567890123456789012345678901212345678901234567890123456789012123456
1935. Her poem, “For My People,”
stories and tells the story of a young John 7
1234567890123456789012345678901212345678901234567890123456789012123456 7
1 which would remain one of her most
12 34567890123456789012345678901212345678901234567890123456789012123456
(5) Unger leaving home for boarding school. 7
12 34567890123456789012345678901212345678901234567890123456789012123456
(20) important works, was also her first 7
12 34567890123456789012345678901212345678901234567890123456789012123456
John T. Unger came from a family 7
12 34567890123456789012345678901212345678901234567890123456789012123456
publication, appearing in Poetry maga- 7
1234567890123456789012345678901212345678901234567890123456789012123456
that had been well known in Hades—a 7
2 34567890123456789012345678901212345678901234567890123456789012123456
small town on the Mississippi River—for 7
1234567890123456789012345678901212345678901234567890123456789012123456
zine in 1937.
1234567890123456789012345678901212345678901234567890123456789012123456 7
1234567890123456789012345678901212345678901234567890123456789012123456 7
1234567890123456789012345678901212345678901234567890123456789012123456 77
several generations. John’s father had
1
12 34567890123456789012345678901212345678901234567890123456789012123456
(10) held the amateur golf championship 7
1234567890123456789012345678901212345678901234567890123456789012123456 7
12 34567890123456789012345678901212345678901234567890123456789012123456 7
through many a heated contest; Mrs.
1234567890123456789012345678901212345678901234567890123456789012123456 7
12
2
34567890123456789012345678901212345678901234567890123456789012123456
Unger was known “from hot-box to
34567890123456789012345678901212345678901234567890123456789012123456
7
7
1
1234567890123456789012345678901212345678901234567890123456789012123456
hot-bed,” as the local phrase went, for 7
1234567890123456789012345678901212345678901234567890123456789012123456 77
1234567890123456789012345678901212345678901234567890123456789012123456
12345678901234567890123456789012123456789012345678901234567890121234567
4 Copyright © 2005 Thomson Peterson’s, a part of The Thomson Corporaton
SAT is a registered trademark of the College Entrance Examination Board, which
was not involved in the production of and does not endorse this product.
12345678901234567890123456789012123456789012345678901234567890121234567
1234567890123456789012345678901212345678901234567890123456789012123456 7
12234567890123456789012345678901212345678901234567890123456789012123456 7
1 34567890123456789012345678901212345678901234567890123456789012123456
her political addresses; and young John So the old man and the young shook 7
12234567890123456789012345678901212345678901234567890123456789012123456 7
1234567890123456789012345678901212345678901234567890123456789012123456
(15) T. Unger, who had just turned sixteen, hands, and John walked away with tears 7
1 34567890123456789012345678901212345678901234567890123456789012123456 7
12234567890123456789012345678901212345678901234567890123456789012123456
had danced all the latest dances from streaming from his eyes. Ten minutes
34567890123456789012345678901212345678901234567890123456789012123456
7
7
1
12234567890123456789012345678901212345678901234567890123456789012123456
New York before he put on long trou- later he had passed outside the city limits 7
1234567890123456789012345678901212345678901234567890123456789012123456 7
1234567890123456789012345678901212345678901234567890123456789012123456
sers. And now, for a certain time, he was (60) and he stopped to glance back for the last 7
1234567890123456789012345678901212345678901234567890123456789012123456 7
1 34567890123456789012345678901212345678901234567890123456789012123456
to be away from home. time. Over the gates the old-fashioned 7
12234567890123456789012345678901212345678901234567890123456789012123456 7
1234567890123456789012345678901212345678901234567890123456789012123456
(20) That respect for a New England Victorian motto seemed strangely 7
1234567890123456789012345678901212345678901234567890123456789012123456 7
1234567890123456789012345678901212345678901234567890123456789012123456
education which is the bane of all attractive
34567890123456789012345678901212345678901234567890123456789012123456
to him. His father had tried 7
1 time and time again to have it changed to 7
12234567890123456789012345678901212345678901234567890123456789012123456
provincial places, which drains them 7
1234567890123456789012345678901212345678901234567890123456789012123456 7
1234567890123456789012345678901212345678901234567890123456789012123456
yearly of their most promising young (65) something with a little more push and 7
1234567890123456789012345678901212345678901234567890123456789012123456
34567890123456789012345678901212345678901234567890123456789012123456
7
7
1 men, had seized upon his parents. verve about it, such as “Hades—Your
12234567890123456789012345678901212345678901234567890123456789012123456 7
1234567890123456789012345678901212345678901234567890123456789012123456
(25) Nothing would suit them but that he Opportunity,” or else a plain “Welcome” 7
1234567890123456789012345678901212345678901234567890123456789012123456
should go to St. Midas’s School near 7
1234567890123456789012345678901212345678901234567890123456789012123456
sign set over a hearty handshake pricked 7
34567890123456789012345678901212345678901234567890123456789012123456
out in electric lights. The old motto was a 7
1234567890123456789012345678901212345678901234567890123456789012123456
Boston—Hades was too small to hold
1234567890123456789012345678901212345678901234567890123456789012123456 7
1234567890123456789012345678901212345678901234567890123456789012123456
their darling and gifted son. Now in (70) little depressing, Mr. Unger had 7
1234567890123456789012345678901212345678901234567890123456789012123456 77
1234567890123456789012345678901212345678901234567890123456789012123456
Hades—as you know if you ever have thought—but now ...
1234567890123456789012345678901212345678901234567890123456789012123456 7
1 (30) been there—the names of the more So John took his look and then set 7
12234567890123456789012345678901212345678901234567890123456789012123456 7
1234567890123456789012345678901212345678901234567890123456789012123456
fashionable preparatory schools and his face resolutely toward his destination. 7
1234567890123456789012345678901212345678901234567890123456789012123456 7
1234567890123456789012345678901212345678901234567890123456789012123456
colleges mean very little. The inhabitants And, as he turned away, the lights of 7
1234567890123456789012345678901212345678901234567890123456789012123456 7
1234567890123456789012345678901212345678901234567890123456789012123456
have been so long out of the world that, (75) Hades against the sky seemed full of a 7
1234567890123456789012345678901212345678901234567890123456789012123456 7
1234567890123456789012345678901212345678901234567890123456789012123456
though they make a show of keeping warm and passionate beauty. 7
34567890123456789012345678901212345678901234567890123456789012123456
1234567890123456789012345678901212345678901234567890123456789012123456 7
1234567890123456789012345678901212345678901234567890123456789012123456
(35) up-to-date in dress and manners and 7
1234567890123456789012345678901212345678901234567890123456789012123456 7
1234567890123456789012345678901212345678901234567890123456789012123456
literature, they depend to a great extent The tone of line 28 can best be described as 7
1 5. 7
1234567890123456789012345678901212345678901234567890123456789012123456 77
2 34567890123456789012345678901212345678901234567890123456789012123456
on hearsay, and a function that in Hades
1234567890123456789012345678901212345678901234567890123456789012123456
(A) compassionate.
7
1 would be considered elaborate would
1234567890123456789012345678901212345678901234567890123456789012123456
(B) sincere. 7
12234567890123456789012345678901212345678901234567890123456789012123456
doubtless be hailed by a Chicago
34567890123456789012345678901212345678901234567890123456789012123456
7
7
1234567890123456789012345678901212345678901234567890123456789012123456
(C) sardonic.
1234567890123456789012345678901212345678901234567890123456789012123456
(40) beef-princess as “perhaps a little tacky.” 7
1 (D) dismayed. 7
12234567890123456789012345678901212345678901234567890123456789012123456
John T. Unger was on the eve of 7
1234567890123456789012345678901212345678901234567890123456789012123456
(E) understated. 7
1234567890123456789012345678901212345678901234567890123456789012123456 77
34567890123456789012345678901212345678901234567890123456789012123456
departure. Mrs. Unger, with maternal
1234567890123456789012345678901212345678901234567890123456789012123456
fatuity, packed his trunks full of linen
1 6. The “Chicago beef-princess” (lines 39–40) 7
1234567890123456789012345678901212345678901234567890123456789012123456 7
suits and electric fans, and Mr. Unger
1234567890123456789012345678901212345678901234567890123456789012123456 7
2 34567890123456789012345678901212345678901234567890123456789012123456
1234567890123456789012345678901212345678901234567890123456789012123456
(45) presented his son with an asbestos
can best be described as representing the 7
1234567890123456789012345678901212345678901234567890123456789012123456
Chicago upper class by way of which 7
1234567890123456789012345678901212345678901234567890123456789012123456
pocket-book stuffed with money. 7
1234567890123456789012345678901212345678901234567890123456789012123456
literary device? 7
1 “Remember, you are always welcome 7
2 34567890123456789012345678901212345678901234567890123456789012123456
1234567890123456789012345678901212345678901234567890123456789012123456 7
1234567890123456789012345678901212345678901234567890123456789012123456
here,” he said. “You can be sure, boy, (A) Anachronism 7
1234567890123456789012345678901212345678901234567890123456789012123456 7
1234567890123456789012345678901212345678901234567890123456789012123456
that we’ll keep the home fires burning.” (B) Simile 7
1 (50) 7
1234567890123456789012345678901212345678901234567890123456789012123456 77
2 34567890123456789012345678901212345678901234567890123456789012123456
“I know,” answered John huskily. (C) Apostrophe
1234567890123456789012345678901212345678901234567890123456789012123456 7
1234567890123456789012345678901212345678901234567890123456789012123456
“Don’t forget who you are and where (D) Metaphor
7
1234567890123456789012345678901212345678901234567890123456789012123456 7
1 you come from,” continued his father (E) Neologism
2 34567890123456789012345678901212345678901234567890123456789012123456
1234567890123456789012345678901212345678901234567890123456789012123456 7
1 proudly, “and you can do nothing to 7
12234567890123456789012345678901212345678901234567890123456789012123456 7
1 34567890123456789012345678901212345678901234567890123456789012123456 7
harm you. You are an Unger—from
12234567890123456789012345678901212345678901234567890123456789012123456 7
1 34567890123456789012345678901212345678901234567890123456789012123456
(55) Hades.” 7
1234567890123456789012345678901212345678901234567890123456789012123456 7
1234567890123456789012345678901212345678901234567890123456789012123456 7
1234567890123456789012345678901212345678901234567890123456789012123456 7
12345678901234567890123456789012123456789012345678901234567890121234567
Copyright © 2005 Thomson Peterson’s, a part of The Thomson Corporaton 5
SAT is a registered trademark of the College Entrance Examination Board, which
was not involved in the production of and does not endorse this product.
12345678901234567890123456789012123456789012345678901234567890121234567
1234567890123456789012345678901212345678901234567890123456789012123456 7
12 34567890123456789012345678901212345678901234567890123456789012123456 7
1234567890123456789012345678901212345678901234567890123456789012123456
7. The phrase “maternal fatuity” (line 10. The names Hades, St. Midas, and Unger 7
12 34567890123456789012345678901212345678901234567890123456789012123456 7
12 34567890123456789012345678901212345678901234567890123456789012123456
42–43), suggests that suggest that the passage can be considered 7
1234567890123456789012345678901212345678901234567890123456789012123456 7
12
2
34567890123456789012345678901212345678901234567890123456789012123456
a(n)
34567890123456789012345678901212345678901234567890123456789012123456
7
7
1 (A) John will not need linen suits and
12 34567890123456789012345678901212345678901234567890123456789012123456 7
12 34567890123456789012345678901212345678901234567890123456789012123456 7
electric fans at St. Midas’s. (A) epic poem.
12 34567890123456789012345678901212345678901234567890123456789012123456 7
12
2
34567890123456789012345678901212345678901234567890123456789012123456
(B) John’s mother packed frantically and (B) euphemism.
34567890123456789012345678901212345678901234567890123456789012123456
7
7
1
12 34567890123456789012345678901212345678901234567890123456789012123456
ineffectively. (C) aphorism. 7
12 34567890123456789012345678901212345678901234567890123456789012123456 7
12 34567890123456789012345678901212345678901234567890123456789012123456 7
(C) John’s mother was excessively (D) satire.
12 34567890123456789012345678901212345678901234567890123456789012123456 7
1234567890123456789012345678901212345678901234567890123456789012123456
doting. (E) allegory. 7
12 34567890123456789012345678901212345678901234567890123456789012123456 7
12 34567890123456789012345678901212345678901234567890123456789012123456 7
(D) John resented his mother packing for
12 34567890123456789012345678901212345678901234567890123456789012123456 7
12
2
34567890123456789012345678901212345678901234567890123456789012123456
him. Questions 11–20 are based on the following
34567890123456789012345678901212345678901234567890123456789012123456
7
7
1 (E) John never enjoyed linen suits or
12 34567890123456789012345678901212345678901234567890123456789012123456
passage. 7
12 34567890123456789012345678901212345678901234567890123456789012123456
electric fans. 7
12 34567890123456789012345678901212345678901234567890123456789012123456 7
12 34567890123456789012345678901212345678901234567890123456789012123456 7
This passage discusses the work of Abe Kobo, a
2 34567890123456789012345678901212345678901234567890123456789012123456
1234567890123456789012345678901212345678901234567890123456789012123456 7
1234567890123456789012345678901212345678901234567890123456789012123456
8. From the conversation between John and Japanese novelist of the twentieth century. 7
1234567890123456789012345678901212345678901234567890123456789012123456 7
1234567890123456789012345678901212345678901234567890123456789012123456
his father in paragraphs 3–6, it can be Line Abe Kobo is one of the great writers of 7
1234567890123456789012345678901212345678901234567890123456789012123456 7
1234567890123456789012345678901212345678901234567890123456789012123456
inferred that John feels postwar Japan. His literature is richer, 7
1 7
12 34567890123456789012345678901212345678901234567890123456789012123456
less predictable, and wider-ranging than 7
12 34567890123456789012345678901212345678901234567890123456789012123456 7
(A) rejected and angry.
12 34567890123456789012345678901212345678901234567890123456789012123456
that of his famed contemporaries, 7
12 34567890123456789012345678901212345678901234567890123456789012123456
(B) melancholic but composed. 7
12 34567890123456789012345678901212345678901234567890123456789012123456
(5) Mishima Yukio and Nobel laureate Oe 7
12 34567890123456789012345678901212345678901234567890123456789012123456
(C) impassive and indifferent. 7
12 34567890123456789012345678901212345678901234567890123456789012123456
(D) resigned but filled with dread. Kenzaburo. It is infused with the passion 7
12 34567890123456789012345678901212345678901234567890123456789012123456 7
12 34567890123456789012345678901212345678901234567890123456789012123456
(E) relieved but apprehensive. and strangeness of his experiences in 7
12 34567890123456789012345678901212345678901234567890123456789012123456 7
12 34567890123456789012345678901212345678901234567890123456789012123456
Manchuria, which was a Japanese colony 7
12 34567890123456789012345678901212345678901234567890123456789012123456 7
1234567890123456789012345678901212345678901234567890123456789012123456
9. John’s meditation on the town’s sign in on mainland China before World War II. 7
2 34567890123456789012345678901212345678901234567890123456789012123456
Abe spent his childhood and much of his 7
1234567890123456789012345678901212345678901234567890123456789012123456
1234567890123456789012345678901212345678901234567890123456789012123456
paragraph 6 serves in the passage prima- (10) 7
1 7
1234567890123456789012345678901212345678901234567890123456789012123456 7
rily to suggest a contrast between youth in Manchuria, and, as a result,
12 34567890123456789012345678901212345678901234567890123456789012123456 7
12 34567890123456789012345678901212345678901234567890123456789012123456
the orbit of his work would be far less 7
12 34567890123456789012345678901212345678901234567890123456789012123456
(A) John’s love of Victorian things and 7
1234567890123456789012345678901212345678901234567890123456789012123456
controlled by the oppressive gravitational 7
12 34567890123456789012345678901212345678901234567890123456789012123456
his father’s love of modern things. 7
12 34567890123456789012345678901212345678901234567890123456789012123456
pull of the themes of furusato (home- 7
2 34567890123456789012345678901212345678901234567890123456789012123456
1234567890123456789012345678901212345678901234567890123456789012123456
(B) his father’s commercialism and 7
1234567890123456789012345678901212345678901234567890123456789012123456
John’s sentimentality.
(15) town) and the emperor than his contem- 7
1 7
1234567890123456789012345678901212345678901234567890123456789012123456
(C) John’s previous role as a part of the
poraries’. 7
1234567890123456789012345678901212345678901234567890123456789012123456 7
2 34567890123456789012345678901212345678901234567890123456789012123456
1234567890123456789012345678901212345678901234567890123456789012123456
town and his new role as nostalgic
Abe, like most of the sons of Japa- 7
1234567890123456789012345678901212345678901234567890123456789012123456
nese families living in Manchuria, did 7
1234567890123456789012345678901212345678901234567890123456789012123456
outsider. 7
return to Japan for schooling. He entered 7
1234567890123456789012345678901212345678901234567890123456789012123456
1 (D) his father’s naivety and John’s 7
12 34567890123456789012345678901212345678901234567890123456789012123456
(20) medical school in Tokyo in 1944—just in 7
12 34567890123456789012345678901212345678901234567890123456789012123456
pragmatism. 7
12 34567890123456789012345678901212345678901234567890123456789012123456
time to forge himself a medical certificate 7
12 34567890123456789012345678901212345678901234567890123456789012123456
(E) the old-fashioned atmosphere in the 7
1234567890123456789012345678901212345678901234567890123456789012123456
claiming ill health; this allowed him to 7
2 34567890123456789012345678901212345678901234567890123456789012123456
1234567890123456789012345678901212345678901234567890123456789012123456
town before John’s father influenced 7
1234567890123456789012345678901212345678901234567890123456789012123456
avoid fighting in the war that Japan was 7
1234567890123456789012345678901212345678901234567890123456789012123456
it and its current modernity. 7
1234567890123456789012345678901212345678901234567890123456789012123456
already losing and return to Manchuria. 7
1 7
1234567890123456789012345678901212345678901234567890123456789012123456 77
2 34567890123456789012345678901212345678901234567890123456789012123456
(25) When Japan lost the war, however, it also
1
12 34567890123456789012345678901212345678901234567890123456789012123456
lost its Manchurian colony. The Japanese 7
1234567890123456789012345678901212345678901234567890123456789012123456 7
12
2
34567890123456789012345678901212345678901234567890123456789012123456
living there were attacked by the Soviet
34567890123456789012345678901212345678901234567890123456789012123456
7
7
1
1234567890123456789012345678901212345678901234567890123456789012123456
Army and various guerrilla bands. They 7
1234567890123456789012345678901212345678901234567890123456789012123456 77
1234567890123456789012345678901212345678901234567890123456789012123456
12345678901234567890123456789012123456789012345678901234567890121234567
6 Copyright © 2005 Thomson Peterson’s, a part of The Thomson Corporaton
SAT is a registered trademark of the College Entrance Examination Board, which
was not involved in the production of and does not endorse this product.
12345678901234567890123456789012123456789012345678901234567890121234567
1234567890123456789012345678901212345678901234567890123456789012123456 7
12234567890123456789012345678901212345678901234567890123456789012123456 7
1 34567890123456789012345678901212345678901234567890123456789012123456
suddenly found themselves refugees, accurate to say that the novel simply 7
12234567890123456789012345678901212345678901234567890123456789012123456 7
1234567890123456789012345678901212345678901234567890123456789012123456
(30) desperate for food. Many unfit men were marked a turning point in his career, 7
1 34567890123456789012345678901212345678901234567890123456789012123456 7
12234567890123456789012345678901212345678901234567890123456789012123456
abandoned in the Manchurian desert. At
34567890123456789012345678901212345678901234567890123456789012123456
when Abe turned away from the experi- 7
7
1
12234567890123456789012345678901212345678901234567890123456789012123456
this apocalyptic time, Abe lost his father (75) mental and heavily political work of his 7
1234567890123456789012345678901212345678901234567890123456789012123456 7
1234567890123456789012345678901212345678901234567890123456789012123456
to cholera. earlier career. Fortunately, he did not 7
1234567890123456789012345678901212345678901234567890123456789012123456
34567890123456789012345678901212345678901234567890123456789012123456
7
7
1 He returned to mainland Japan once then turn to furusato and the emperor
12234567890123456789012345678901212345678901234567890123456789012123456 7
1234567890123456789012345678901212345678901234567890123456789012123456
(35) more, where the young were turning to after all, but rather began a somewhat 7
1234567890123456789012345678901212345678901234567890123456789012123456 7
1234567890123456789012345678901212345678901234567890123456789012123456
Marxism as a rejection of the militarism
34567890123456789012345678901212345678901234567890123456789012123456
more realistic exploration of his continu- 7
7
1
12234567890123456789012345678901212345678901234567890123456789012123456
of the war. After a brief, unsuccessful (80) ing obsession with homelessness and 7
1234567890123456789012345678901212345678901234567890123456789012123456 7
1234567890123456789012345678901212345678901234567890123456789012123456
stint at medical school, he became part of alienation. Not completely a stranger to 7
1234567890123456789012345678901212345678901234567890123456789012123456
34567890123456789012345678901212345678901234567890123456789012123456
7
7
1 a Marxist group of avant-garde artists. his earlier commitment to Marxism, Abe
12234567890123456789012345678901212345678901234567890123456789012123456 7
1234567890123456789012345678901212345678901234567890123456789012123456
(40) His work at this time was passionate and turned his attention, beginning in the 7
1234567890123456789012345678901212345678901234567890123456789012123456
outspoken on political matters, adopting 7
1234567890123456789012345678901212345678901234567890123456789012123456 sixties, to the effects on the individual of 7
34567890123456789012345678901212345678901234567890123456789012123456
1234567890123456789012345678901212345678901234567890123456789012123456
black humor as its mode of critique. Japan’s rapidly urbanizing, growth- 7
(85)
1234567890123456789012345678901212345678901234567890123456789012123456 7
1234567890123456789012345678901212345678901234567890123456789012123456
During this time, Abe worked in the driven, increasingly corporate society. 7
1234567890123456789012345678901212345678901234567890123456789012123456 77
1234567890123456789012345678901212345678901234567890123456789012123456
genres of theater, music, and photogra-
1234567890123456789012345678901212345678901234567890123456789012123456 7
1 (45) phy. Eventually, he mimeographed fifty 7
12234567890123456789012345678901212345678901234567890123456789012123456
11. The word “infused” in line 6 most closely 7
1234567890123456789012345678901212345678901234567890123456789012123456
copies of his first “published” literary 7
1234567890123456789012345678901212345678901234567890123456789012123456 means 7
1234567890123456789012345678901212345678901234567890123456789012123456 77
34567890123456789012345678901212345678901234567890123456789012123456
work, entitled Anonymous Poems, in
1234567890123456789012345678901212345678901234567890123456789012123456
1234567890123456789012345678901212345678901234567890123456789012123456
1947. It was a politically charged set of (A) illuminated. 7
1234567890123456789012345678901212345678901234567890123456789012123456 (B) saturated. 7
1234567890123456789012345678901212345678901234567890123456789012123456 77
poems dedicated to the memory of his
1234567890123456789012345678901212345678901234567890123456789012123456 (C) influenced.
1234567890123456789012345678901212345678901234567890123456789012123456
(50) father and friends who had died in 7
1234567890123456789012345678901212345678901234567890123456789012123456 (D) bewildered. 7
1234567890123456789012345678901212345678901234567890123456789012123456 77
Manchuria. Shortly thereafter, he
1 (E) nuanced.
12234567890123456789012345678901212345678901234567890123456789012123456
published his first novel, For a Signpost 7
1234567890123456789012345678901212345678901234567890123456789012123456 7
1 34567890123456789012345678901212345678901234567890123456789012123456
at the End of a Road, which imagined 7
1234567890123456789012345678901212345678901234567890123456789012123456
12. The author refers to “the orbit” of Abe’s 7
12234567890123456789012345678901212345678901234567890123456789012123456
another life for his best friend who had
34567890123456789012345678901212345678901234567890123456789012123456
7
7
1234567890123456789012345678901212345678901234567890123456789012123456 work (lines 12–13) to emphasize that
1234567890123456789012345678901212345678901234567890123456789012123456
(55) died in the Manchurian desert. Abe was 7
1 7
12234567890123456789012345678901212345678901234567890123456789012123456
also active in the Communist Party, (A) his work covers a wide range of 7
1234567890123456789012345678901212345678901234567890123456789012123456 7
1234567890123456789012345678901212345678901234567890123456789012123456 77
34567890123456789012345678901212345678901234567890123456789012123456
organizing literary groups for working- themes.
1234567890123456789012345678901212345678901234567890123456789012123456
men. (B) the emperor is often compared to a 7
1
1234567890123456789012345678901212345678901234567890123456789012123456
Unfortunately, most of this radical sun. 7
1234567890123456789012345678901212345678901234567890123456789012123456 7
2 34567890123456789012345678901212345678901234567890123456789012123456
1234567890123456789012345678901212345678901234567890123456789012123456
(60) early work is unknown outside Japan C. Abe’s travels were the primary 7
1234567890123456789012345678901212345678901234567890123456789012123456 7
1234567890123456789012345678901212345678901234567890123456789012123456
and underappreciated even in Japan. In themes in his work. 7
1234567890123456789012345678901212345678901234567890123456789012123456 77
1 early 1962, Abe was dismissed from the D. Abe’s work is so different from his
12234567890123456789012345678901212345678901234567890123456789012123456 7
1234567890123456789012345678901212345678901234567890123456789012123456
Japanese Liberalist Party. Four months contemporaries’ that it is like 7
1234567890123456789012345678901212345678901234567890123456789012123456 7
1234567890123456789012345678901212345678901234567890123456789012123456
later, he published the work that would another solar system. 7
1 34567890123456789012345678901212345678901234567890123456789012123456 7
1234567890123456789012345678901212345678901234567890123456789012123456 77
2 34567890123456789012345678901212345678901234567890123456789012123456
(65) blind us to his earlier oeuvre, Woman in (E) conventional themes can limit an
1234567890123456789012345678901212345678901234567890123456789012123456 7
1234567890123456789012345678901212345678901234567890123456789012123456
the Dunes. It was director Teshigahara author’s individuality.
7
1234567890123456789012345678901212345678901234567890123456789012123456 7
1 Hiroshi’s film adaptation of Woman in
2 34567890123456789012345678901212345678901234567890123456789012123456
1234567890123456789012345678901212345678901234567890123456789012123456 7
1 the Dunes that brought Abe’s work to 7
12234567890123456789012345678901212345678901234567890123456789012123456 7
1 34567890123456789012345678901212345678901234567890123456789012123456 7
the international stage. The movie’s fame
12234567890123456789012345678901212345678901234567890123456789012123456 7
1 34567890123456789012345678901212345678901234567890123456789012123456
(70) has wrongly led readers to view the novel 7
1234567890123456789012345678901212345678901234567890123456789012123456 7
1234567890123456789012345678901212345678901234567890123456789012123456 77
as Abe’s masterpiece. It would be more
1234567890123456789012345678901212345678901234567890123456789012123456
12345678901234567890123456789012123456789012345678901234567890121234567
Copyright © 2005 Thomson Peterson’s, a part of The Thomson Corporaton 7
SAT is a registered trademark of the College Entrance Examination Board, which
was not involved in the production of and does not endorse this product.
12345678901234567890123456789012123456789012345678901234567890121234567
1234567890123456789012345678901212345678901234567890123456789012123456 7
12 34567890123456789012345678901212345678901234567890123456789012123456 7
1234567890123456789012345678901212345678901234567890123456789012123456
13. From the sentence beginning “He entered 16. Which of the following does the passage 7
12 34567890123456789012345678901212345678901234567890123456789012123456 7
12 34567890123456789012345678901212345678901234567890123456789012123456
medical school. . . “ in lines 19–24, it can present as a fact? 7
1234567890123456789012345678901212345678901234567890123456789012123456 7
12
2
34567890123456789012345678901212345678901234567890123456789012123456
be inferred that
34567890123456789012345678901212345678901234567890123456789012123456
7
7
1 (A) Abe was a better playwright than
12 34567890123456789012345678901212345678901234567890123456789012123456 7
12 34567890123456789012345678901212345678901234567890123456789012123456 7
(A) Abe entered medical school because novelist.
12 34567890123456789012345678901212345678901234567890123456789012123456 7
12
2
34567890123456789012345678901212345678901234567890123456789012123456
he was sick. (B) Abe’s early work was of greater
34567890123456789012345678901212345678901234567890123456789012123456
7
7
1
12 34567890123456789012345678901212345678901234567890123456789012123456
(B) sick people were sent to Manchuria quality than his later work. 7
12 34567890123456789012345678901212345678901234567890123456789012123456 7
12 34567890123456789012345678901212345678901234567890123456789012123456 7
during World War II. (C) The group of avant-garde artists of
12 34567890123456789012345678901212345678901234567890123456789012123456 7
1234567890123456789012345678901212345678901234567890123456789012123456
(C) Abe wanted to help the ill and which Abe was a part were influ- 7
12 34567890123456789012345678901212345678901234567890123456789012123456 7
12 34567890123456789012345678901212345678901234567890123456789012123456 7
injured in World War II, rather than enced by Marxism.
12 34567890123456789012345678901212345678901234567890123456789012123456 7
12
2
34567890123456789012345678901212345678901234567890123456789012123456
fight. (D) The themes of furusato and the
34567890123456789012345678901212345678901234567890123456789012123456
7
7
1 (D) illness would excuse one from emperor have precluded Japanese
12 34567890123456789012345678901212345678901234567890123456789012123456 7
12 34567890123456789012345678901212345678901234567890123456789012123456
military duty in World War II Japan. literature from playing a major role 7
12 34567890123456789012345678901212345678901234567890123456789012123456 7
12 34567890123456789012345678901212345678901234567890123456789012123456
(E) Abe never intended to practice in world literature. 7
12 34567890123456789012345678901212345678901234567890123456789012123456 7
12 34567890123456789012345678901212345678901234567890123456789012123456
medicine. (E) Abe’s work is richer than his 7
12 34567890123456789012345678901212345678901234567890123456789012123456 7
12 34567890123456789012345678901212345678901234567890123456789012123456
contemporaries’ because he included 7
12 34567890123456789012345678901212345678901234567890123456789012123456 7
2 34567890123456789012345678901212345678901234567890123456789012123456
1234567890123456789012345678901212345678901234567890123456789012123456
14. The author uses the word “apocalyptic” autobiographical elements. 7
1 7
12 34567890123456789012345678901212345678901234567890123456789012123456 7
to emphasize that
12 34567890123456789012345678901212345678901234567890123456789012123456 7
12 34567890123456789012345678901212345678901234567890123456789012123456
17. The phrase “blind us” in lines 65–66 7
2 34567890123456789012345678901212345678901234567890123456789012123456
1234567890123456789012345678901212345678901234567890123456789012123456
(A) Manchuria suffered intensely as a 7
1234567890123456789012345678901212345678901234567890123456789012123456
refers to the 7
1234567890123456789012345678901212345678901234567890123456789012123456 77
result of the use of nuclear weapons
1234567890123456789012345678901212345678901234567890123456789012123456
in World War II. (A) absence of film adaptations for Abe’s 7
1234567890123456789012345678901212345678901234567890123456789012123456
1234567890123456789012345678901212345678901234567890123456789012123456
(B) Abe was deeply affected by the loss other novels. 7
1234567890123456789012345678901212345678901234567890123456789012123456 7
1234567890123456789012345678901212345678901234567890123456789012123456
of his father. (B) excessive critical attention to Abe’s 7
1234567890123456789012345678901212345678901234567890123456789012123456 77
1 (C) there was massive famine in Man- novel, Woman in the Dunes.
12 34567890123456789012345678901212345678901234567890123456789012123456 7
12 34567890123456789012345678901212345678901234567890123456789012123456
churia at the end of World War II. (C) difficulty in reconciling Woman in 7
1234567890123456789012345678901212345678901234567890123456789012123456 7
1234567890123456789012345678901212345678901234567890123456789012123456
(D) postwar Manchuria experienced the Dunes and other later works 7
12 34567890123456789012345678901212345678901234567890123456789012123456 7
1234567890123456789012345678901212345678901234567890123456789012123456 77
2 34567890123456789012345678901212345678901234567890123456789012123456
exhilarating change. with the form and content of his
1234567890123456789012345678901212345678901234567890123456789012123456 7
1 (E) conditions in Manchuria after World earlier works.
12 34567890123456789012345678901212345678901234567890123456789012123456 7
12 34567890123456789012345678901212345678901234567890123456789012123456 7
War II were generally horrific. (D) challenge of interpreting Abe’s more
2 34567890123456789012345678901212345678901234567890123456789012123456
1234567890123456789012345678901212345678901234567890123456789012123456 7
1234567890123456789012345678901212345678901234567890123456789012123456
experimental works. 7
1 15. The word “avant-garde” (line 39) could 7
1234567890123456789012345678901212345678901234567890123456789012123456 7
(E) overwhelming power of Abe’s novel,
1234567890123456789012345678901212345678901234567890123456789012123456 7
12 34567890123456789012345678901212345678901234567890123456789012123456
best be replaced by Woman in the Dunes. 7
12 34567890123456789012345678901212345678901234567890123456789012123456 7
12 34567890123456789012345678901212345678901234567890123456789012123456
(A) experimental. 7
12 34567890123456789012345678901212345678901234567890123456789012123456 7
1234567890123456789012345678901212345678901234567890123456789012123456
(B) dramatic. 7
2 34567890123456789012345678901212345678901234567890123456789012123456
1234567890123456789012345678901212345678901234567890123456789012123456 7
1234567890123456789012345678901212345678901234567890123456789012123456
(C) novel. 7
1234567890123456789012345678901212345678901234567890123456789012123456 7
1234567890123456789012345678901212345678901234567890123456789012123456
(D) profound. 7
1 7
1234567890123456789012345678901212345678901234567890123456789012123456 77
2 34567890123456789012345678901212345678901234567890123456789012123456
(E) realistic.
1234567890123456789012345678901212345678901234567890123456789012123456 7
1234567890123456789012345678901212345678901234567890123456789012123456 7
1234567890123456789012345678901212345678901234567890123456789012123456 7
1
12 34567890123456789012345678901212345678901234567890123456789012123456 7
1234567890123456789012345678901212345678901234567890123456789012123456 7
12 34567890123456789012345678901212345678901234567890123456789012123456 7
1234567890123456789012345678901212345678901234567890123456789012123456 7
12 34567890123456789012345678901212345678901234567890123456789012123456 7
1234567890123456789012345678901212345678901234567890123456789012123456 7
1234567890123456789012345678901212345678901234567890123456789012123456 7
1234567890123456789012345678901212345678901234567890123456789012123456 7
1234567890123456789012345678901212345678901234567890123456789012123456 7
12345678901234567890123456789012123456789012345678901234567890121234567
8 Copyright © 2005 Thomson Peterson’s, a part of The Thomson Corporaton
SAT is a registered trademark of the College Entrance Examination Board, which
was not involved in the production of and does not endorse this product.
12345678901234567890123456789012123456789012345678901234567890121234567
1234567890123456789012345678901212345678901234567890123456789012123456 7
12234567890123456789012345678901212345678901234567890123456789012123456 7
1 34567890123456789012345678901212345678901234567890123456789012123456
18. The author’s main purpose in the passage 20. The author’s attitude toward Marxism 7
12234567890123456789012345678901212345678901234567890123456789012123456 7
1234567890123456789012345678901212345678901234567890123456789012123456
is to can best be described as 7
1 34567890123456789012345678901212345678901234567890123456789012123456 7
12234567890123456789012345678901212345678901234567890123456789012123456 7
1 34567890123456789012345678901212345678901234567890123456789012123456
(A) defend Abe’s later works against the (A) contemptuous derision. 7
12234567890123456789012345678901212345678901234567890123456789012123456 7
1234567890123456789012345678901212345678901234567890123456789012123456
prevalent criticism of it. (B) reverent espousal. 7
1234567890123456789012345678901212345678901234567890123456789012123456 7
1234567890123456789012345678901212345678901234567890123456789012123456
(B) advocate for Abe’s work over that of (C)
34567890123456789012345678901212345678901234567890123456789012123456
skeptical tolerance. 7
7
1
12234567890123456789012345678901212345678901234567890123456789012123456
his contemporaries. (D) respectful interest. 7
1234567890123456789012345678901212345678901234567890123456789012123456 7
1234567890123456789012345678901212345678901234567890123456789012123456
(C) explain the differences between Abe’s (E) restrained impatience. 7
1234567890123456789012345678901212345678901234567890123456789012123456
34567890123456789012345678901212345678901234567890123456789012123456
7
7
1 earlier and later works.
12234567890123456789012345678901212345678901234567890123456789012123456 7
1234567890123456789012345678901212345678901234567890123456789012123456
(D) argue that Abe is an even greater 7
1234567890123456789012345678901212345678901234567890123456789012123456 7
1234567890123456789012345678901212345678901234567890123456789012123456
writer and artist than generally
34567890123456789012345678901212345678901234567890123456789012123456
7
7
1 perceived.
12234567890123456789012345678901212345678901234567890123456789012123456 7
1234567890123456789012345678901212345678901234567890123456789012123456
(E) demonstrate that Abe’s work became 7
1234567890123456789012345678901212345678901234567890123456789012123456 7
1234567890123456789012345678901212345678901234567890123456789012123456
less interesting after he left Manchu- 7
1234567890123456789012345678901212345678901234567890123456789012123456 77
34567890123456789012345678901212345678901234567890123456789012123456
1234567890123456789012345678901212345678901234567890123456789012123456
ria.
1234567890123456789012345678901212345678901234567890123456789012123456 77
1234567890123456789012345678901212345678901234567890123456789012123456 7
1234567890123456789012345678901212345678901234567890123456789012123456
1234567890123456789012345678901212345678901234567890123456789012123456
19. The author of the passage is most likely a 7
1 7
12234567890123456789012345678901212345678901234567890123456789012123456 7
1234567890123456789012345678901212345678901234567890123456789012123456
(A) film critic. 7
1234567890123456789012345678901212345678901234567890123456789012123456 7
1234567890123456789012345678901212345678901234567890123456789012123456 77
34567890123456789012345678901212345678901234567890123456789012123456
(B) literary critic.
1234567890123456789012345678901212345678901234567890123456789012123456
1234567890123456789012345678901212345678901234567890123456789012123456
(C) avant-garde artist. 7
1234567890123456789012345678901212345678901234567890123456789012123456
(D) translator. 7
1234567890123456789012345678901212345678901234567890123456789012123456 77
1234567890123456789012345678901212345678901234567890123456789012123456
(E) novelist.
1234567890123456789012345678901212345678901234567890123456789012123456 7
1234567890123456789012345678901212345678901234567890123456789012123456 77
1234567890123456789012345678901212345678901234567890123456789012123456 7
1
12234567890123456789012345678901212345678901234567890123456789012123456 7
1234567890123456789012345678901212345678901234567890123456789012123456 7
1 34567890123456789012345678901212345678901234567890123456789012123456 7
1234567890123456789012345678901212345678901234567890123456789012123456 7
12234567890123456789012345678901212345678901234567890123456789012123456 7
1234567890123456789012345678901212345678901234567890123456789012123456 7
1234567890123456789012345678901212345678901234567890123456789012123456 7
1 34567890123456789012345678901212345678901234567890123456789012123456 7
12234567890123456789012345678901212345678901234567890123456789012123456 7
1234567890123456789012345678901212345678901234567890123456789012123456 7
1234567890123456789012345678901212345678901234567890123456789012123456 77
34567890123456789012345678901212345678901234567890123456789012123456
1234567890123456789012345678901212345678901234567890123456789012123456 7
1
1234567890123456789012345678901212345678901234567890123456789012123456 7
1234567890123456789012345678901212345678901234567890123456789012123456 7
12234567890123456789012345678901212345678901234567890123456789012123456 7
1234567890123456789012345678901212345678901234567890123456789012123456 7
1234567890123456789012345678901212345678901234567890123456789012123456 7
1234567890123456789012345678901212345678901234567890123456789012123456 7
1 34567890123456789012345678901212345678901234567890123456789012123456 7
1234567890123456789012345678901212345678901234567890123456789012123456 77
2 34567890123456789012345678901212345678901234567890123456789012123456
1234567890123456789012345678901212345678901234567890123456789012123456 7
1234567890123456789012345678901212345678901234567890123456789012123456 7
1234567890123456789012345678901212345678901234567890123456789012123456 7
1
12234567890123456789012345678901212345678901234567890123456789012123456 7
1234567890123456789012345678901212345678901234567890123456789012123456 7
1234567890123456789012345678901212345678901234567890123456789012123456 7
1234567890123456789012345678901212345678901234567890123456789012123456 7
1 34567890123456789012345678901212345678901234567890123456789012123456 7
1234567890123456789012345678901212345678901234567890123456789012123456 77
2 34567890123456789012345678901212345678901234567890123456789012123456
1
12234567890123456789012345678901212345678901234567890123456789012123456 7
1 34567890123456789012345678901212345678901234567890123456789012123456 7
12234567890123456789012345678901212345678901234567890123456789012123456 7
STOP
1 34567890123456789012345678901212345678901234567890123456789012123456
Do not proceed to the next section until time is up.
1234567890123456789012345678901212345678901234567890123456789012123456 7
7
1234567890123456789012345678901212345678901234567890123456789012123456 7
1234567890123456789012345678901212345678901234567890123456789012123456 7
12345678901234567890123456789012123456789012345678901234567890121234567
Copyright © 2005 Thomson Peterson’s, a part of The Thomson Corporaton 9
SAT is a registered trademark of the College Entrance Examination Board, which
was not involved in the production of and does not endorse this product.
Section 2
21 Questions j Time—25 Minutes

12345678901234567890123456789012123456789012345678901234567890121234567
12 34567890123456789012345678901212345678901234567890123456789012123456 7
12 34567890123456789012345678901212345678901234567890123456789012123456 7
1 34567890123456789012345678901212345678901234567890123456789012123456 7
2
12 34567890123456789012345678901212345678901234567890123456789012123456 7
12 34567890123456789012345678901212345678901234567890123456789012123456
Directions: Solve the following problems using any available space on the page for scratchwork. 7
12 34567890123456789012345678901212345678901234567890123456789012123456
Mark the letter of your choice on the answer sheet that best corresponds to the correct answer. 7
12 34567890123456789012345678901212345678901234567890123456789012123456 7
1234567890123456789012345678901212345678901234567890123456789012123456 77
2 34567890123456789012345678901212345678901234567890123456789012123456
1234567890123456789012345678901212345678901234567890123456789012123456
Notes:
1234567890123456789012345678901212345678901234567890123456789012123456 7
1234567890123456789012345678901212345678901234567890123456789012123456
1. You may use a calculator. All of the numbers used are real numbers. 7
1234567890123456789012345678901212345678901234567890123456789012123456 77
1234567890123456789012345678901212345678901234567890123456789012123456 7
1 2. You may use the figures that accompany the problems to help you find the solution. Unless
12 34567890123456789012345678901212345678901234567890123456789012123456 7
12 34567890123456789012345678901212345678901234567890123456789012123456 7
the instructions say that a figure is not drawn to scale, assume that it has been drawn
12 34567890123456789012345678901212345678901234567890123456789012123456 7
12 34567890123456789012345678901212345678901234567890123456789012123456
accurately. Each figure lies in a plane unless the instructions say otherwise. 7
12 34567890123456789012345678901212345678901234567890123456789012123456 7
12 34567890123456789012345678901212345678901234567890123456789012123456 7
12 34567890123456789012345678901212345678901234567890123456789012123456 7
12 34567890123456789012345678901212345678901234567890123456789012123456 7
1234567890123456789012345678901212345678901234567890123456789012123456
2 7
Reference Information

2s
1234567890123456789012345678901212345678901234567890123456789012123456
r w 2x
45
7
1234567890123456789012345678901212345678901234567890123456789012123456
h h
r
c x s 60
7
1234567890123456789012345678901212345678901234567890123456789012123456 h b 7
1 34567890123456789012345678901212345678901234567890123456789012123456 7
30
45
w
2 34567890123456789012345678901212345678901234567890123456789012123456
1234567890123456789012345678901212345678901234567890123456789012123456 7
1234567890123456789012345678901212345678901234567890123456789012123456
A 5 pr2 b a 3x s 7
1 1 7
1234567890123456789012345678901212345678901234567890123456789012123456 7
2 2 2 2
C 5 2pr A 5 ,w A 5 bh V 5 ,wh V 5 pr h c 5 a 1 b Special Right Triangles
12
2
34567890123456789012345678901212345678901234567890123456789012123456 7
2
34567890123456789012345678901212345678901234567890123456789012123456 7
1234567890123456789012345678901212345678901234567890123456789012123456
The number of degrees of arc in a circle is 360.
1234567890123456789012345678901212345678901234567890123456789012123456 7
1 The measure in degrees of a straight angle is 180. 7
12 34567890123456789012345678901212345678901234567890123456789012123456 7
12 34567890123456789012345678901212345678901234567890123456789012123456 7
The sum of the measures in degrees of the angles of a triangle is 180.
1234567890123456789012345678901212345678901234567890123456789012123456 77
2 34567890123456789012345678901212345678901234567890123456789012123456
1234567890123456789012345678901212345678901234567890123456789012123456 7
1
1234567890123456789012345678901212345678901234567890123456789012123456 7
1234567890123456789012345678901212345678901234567890123456789012123456
1. What percentage of 75 is 12? 2. If a circle is inscribed in a square of area 7
2 34567890123456789012345678901212345678901234567890123456789012123456
1234567890123456789012345678901212345678901234567890123456789012123456 7
1234567890123456789012345678901212345678901234567890123456789012123456
36, what is the area of the circle? 7
1234567890123456789012345678901212345678901234567890123456789012123456 77
(A) 8%
1234567890123456789012345678901212345678901234567890123456789012123456
(B) 12% (A) 36p 7
1
2 34567890123456789012345678901212345678901234567890123456789012123456
1234567890123456789012345678901212345678901234567890123456789012123456
(C) 16% (B) 24p 7
1234567890123456789012345678901212345678901234567890123456789012123456 7
1234567890123456789012345678901212345678901234567890123456789012123456
(D) 18% (C) 12p 7
1234567890123456789012345678901212345678901234567890123456789012123456 77
1 (E) 20% (D) 9p
12 34567890123456789012345678901212345678901234567890123456789012123456 7
12 34567890123456789012345678901212345678901234567890123456789012123456
(E) 6p 7
12 34567890123456789012345678901212345678901234567890123456789012123456 7
12 34567890123456789012345678901212345678901234567890123456789012123456 7
1234567890123456789012345678901212345678901234567890123456789012123456 7
1234567890123456789012345678901212345678901234567890123456789012123456
2 7
1 34567890123456789012345678901212345678901234567890123456789012123456 7
12 34567890123456789012345678901212345678901234567890123456789012123456 7
1234567890123456789012345678901212345678901234567890123456789012123456 7
12 34567890123456789012345678901212345678901234567890123456789012123456 7
1234567890123456789012345678901212345678901234567890123456789012123456 7
1234567890123456789012345678901212345678901234567890123456789012123456 7
1234567890123456789012345678901212345678901234567890123456789012123456 7
1234567890123456789012345678901212345678901234567890123456789012123456 7
12345678901234567890123456789012123456789012345678901234567890121234567
10 Copyright © 2005 Thomson Peterson’s, a part of The Thomson Corporaton
SAT is a registered trademark of the College Entrance Examination Board, which
was not involved in the production of and does not endorse this product.
12345678901234567890123456789012123456789012345678901234567890121234567
1234567890123456789012345678901212345678901234567890123456789012123456 7
12234567890123456789012345678901212345678901234567890123456789012123456 7
1 34567890123456789012345678901212345678901234567890123456789012123456
3. Which of the following could have a slope 4. The first four terms of a series are 1, 4, 9, 7
12234567890123456789012345678901212345678901234567890123456789012123456 7
1234567890123456789012345678901212345678901234567890123456789012123456
of one? and 16. What is the eight term of this 7
1 34567890123456789012345678901212345678901234567890123456789012123456 7
12234567890123456789012345678901212345678901234567890123456789012123456
series?
34567890123456789012345678901212345678901234567890123456789012123456
7
7
1 (A)
12234567890123456789012345678901212345678901234567890123456789012123456 7
1234567890123456789012345678901212345678901234567890123456789012123456
(A) 49 7
1234567890123456789012345678901212345678901234567890123456789012123456 7
1234567890123456789012345678901212345678901234567890123456789012123456
(B) 56
34567890123456789012345678901212345678901234567890123456789012123456
7
7
1
12234567890123456789012345678901212345678901234567890123456789012123456
(C) 64 7
1234567890123456789012345678901212345678901234567890123456789012123456 7
1234567890123456789012345678901212345678901234567890123456789012123456
(D) 72 7
1234567890123456789012345678901212345678901234567890123456789012123456
34567890123456789012345678901212345678901234567890123456789012123456
7
7
1 (E) 81
12234567890123456789012345678901212345678901234567890123456789012123456 7
1234567890123456789012345678901212345678901234567890123456789012123456 7
1234567890123456789012345678901212345678901234567890123456789012123456
5. 7
1234567890123456789012345678901212345678901234567890123456789012123456
34567890123456789012345678901212345678901234567890123456789012123456
7
7
1
12234567890123456789012345678901212345678901234567890123456789012123456
(B) 7
1234567890123456789012345678901212345678901234567890123456789012123456 7
1234567890123456789012345678901212345678901234567890123456789012123456 7
1234567890123456789012345678901212345678901234567890123456789012123456 7
1234567890123456789012345678901212345678901234567890123456789012123456 77
34567890123456789012345678901212345678901234567890123456789012123456
1234567890123456789012345678901212345678901234567890123456789012123456 7
1234567890123456789012345678901212345678901234567890123456789012123456 7
1234567890123456789012345678901212345678901234567890123456789012123456
1234567890123456789012345678901212345678901234567890123456789012123456
What is the area of the above parallelo- 7
1234567890123456789012345678901212345678901234567890123456789012123456 7
1 gram? 7
12234567890123456789012345678901212345678901234567890123456789012123456 7
1234567890123456789012345678901212345678901234567890123456789012123456 7
1234567890123456789012345678901212345678901234567890123456789012123456
(A) 16 7
34567890123456789012345678901212345678901234567890123456789012123456
1234567890123456789012345678901212345678901234567890123456789012123456 7
1234567890123456789012345678901212345678901234567890123456789012123456
(B) 18 7
1234567890123456789012345678901212345678901234567890123456789012123456
(C) (C) 22 7
1234567890123456789012345678901212345678901234567890123456789012123456 77
1234567890123456789012345678901212345678901234567890123456789012123456
(D) 24
1234567890123456789012345678901212345678901234567890123456789012123456 7
1234567890123456789012345678901212345678901234567890123456789012123456
(E) 32 7
1234567890123456789012345678901212345678901234567890123456789012123456 77
1234567890123456789012345678901212345678901234567890123456789012123456 7
1
12234567890123456789012345678901212345678901234567890123456789012123456
6. Which of the following fractions has the 7
1234567890123456789012345678901212345678901234567890123456789012123456 7
1 34567890123456789012345678901212345678901234567890123456789012123456
greatest reciprocal? 7
1234567890123456789012345678901212345678901234567890123456789012123456 7
12234567890123456789012345678901212345678901234567890123456789012123456 7
1234567890123456789012345678901212345678901234567890123456789012123456
2 7
1234567890123456789012345678901212345678901234567890123456789012123456
(A) 7
1 34567890123456789012345678901212345678901234567890123456789012123456
(D) 9 7
12234567890123456789012345678901212345678901234567890123456789012123456 7
1234567890123456789012345678901212345678901234567890123456789012123456
4 7
34567890123456789012345678901212345678901234567890123456789012123456
1234567890123456789012345678901212345678901234567890123456789012123456
(B) 7
1234567890123456789012345678901212345678901234567890123456789012123456
5 7
1 7
1234567890123456789012345678901212345678901234567890123456789012123456
7 7
1234567890123456789012345678901212345678901234567890123456789012123456 7
2 34567890123456789012345678901212345678901234567890123456789012123456
1234567890123456789012345678901212345678901234567890123456789012123456
(C) 7
1234567890123456789012345678901212345678901234567890123456789012123456
3 7
1234567890123456789012345678901212345678901234567890123456789012123456 7
1234567890123456789012345678901212345678901234567890123456789012123456
2 7
1 (D) 7
1234567890123456789012345678901212345678901234567890123456789012123456 77
2 34567890123456789012345678901212345678901234567890123456789012123456
3
1234567890123456789012345678901212345678901234567890123456789012123456 7
1234567890123456789012345678901212345678901234567890123456789012123456
3
7
1234567890123456789012345678901212345678901234567890123456789012123456
(E) (E)
7
1 13
1234567890123456789012345678901212345678901234567890123456789012123456 77
2 34567890123456789012345678901212345678901234567890123456789012123456
1234567890123456789012345678901212345678901234567890123456789012123456 7
1234567890123456789012345678901212345678901234567890123456789012123456 7
1234567890123456789012345678901212345678901234567890123456789012123456 7
1
12234567890123456789012345678901212345678901234567890123456789012123456 7
1 34567890123456789012345678901212345678901234567890123456789012123456 7
12234567890123456789012345678901212345678901234567890123456789012123456 7
1 34567890123456789012345678901212345678901234567890123456789012123456 7
12234567890123456789012345678901212345678901234567890123456789012123456 7
1 34567890123456789012345678901212345678901234567890123456789012123456 7
1234567890123456789012345678901212345678901234567890123456789012123456 7
1234567890123456789012345678901212345678901234567890123456789012123456 7
1234567890123456789012345678901212345678901234567890123456789012123456 7
12345678901234567890123456789012123456789012345678901234567890121234567
Copyright © 2005 Thomson Peterson’s, a part of The Thomson Corporaton 11
SAT is a registered trademark of the College Entrance Examination Board, which
was not involved in the production of and does not endorse this product.
12345678901234567890123456789012123456789012345678901234567890121234567
1234567890123456789012345678901212345678901234567890123456789012123456 7
12 34567890123456789012345678901212345678901234567890123456789012123456 7
1234567890123456789012345678901212345678901234567890123456789012123456
7. 9. 7
12 34567890123456789012345678901212345678901234567890123456789012123456 7
12 34567890123456789012345678901212345678901234567890123456789012123456 7
1234567890123456789012345678901212345678901234567890123456789012123456 7
12 34567890123456789012345678901212345678901234567890123456789012123456 7
1234567890123456789012345678901212345678901234567890123456789012123456 7
12 34567890123456789012345678901212345678901234567890123456789012123456 7
12 34567890123456789012345678901212345678901234567890123456789012123456 7
12 34567890123456789012345678901212345678901234567890123456789012123456 7
12 34567890123456789012345678901212345678901234567890123456789012123456 7
1 34567890123456789012345678901212345678901234567890123456789012123456 7
2
12 34567890123456789012345678901212345678901234567890123456789012123456
According to the chart above, Company B 7
12 34567890123456789012345678901212345678901234567890123456789012123456 7
12 34567890123456789012345678901212345678901234567890123456789012123456 7
12
2
34567890123456789012345678901212345678901234567890123456789012123456 7
produced approximately how
34567890123456789012345678901212345678901234567890123456789012123456
many more
7
1 widgets than Company A?
12 34567890123456789012345678901212345678901234567890123456789012123456 7
12 34567890123456789012345678901212345678901234567890123456789012123456 7
12 34567890123456789012345678901212345678901234567890123456789012123456
(A) 75,000 7
12
2
34567890123456789012345678901212345678901234567890123456789012123456
34567890123456789012345678901212345678901234567890123456789012123456
7
7
1 (B) 150,000
12 34567890123456789012345678901212345678901234567890123456789012123456 7
12 34567890123456789012345678901212345678901234567890123456789012123456
(C) 225,000 7
12 34567890123456789012345678901212345678901234567890123456789012123456
If a 5 60 and c 5 50 then x = 7
12 34567890123456789012345678901212345678901234567890123456789012123456 7
(D) 300,000
2 34567890123456789012345678901212345678901234567890123456789012123456
1234567890123456789012345678901212345678901234567890123456789012123456 7
1234567890123456789012345678901212345678901234567890123456789012123456
(A) 40 (E) 375,000 7
1234567890123456789012345678901212345678901234567890123456789012123456 7
1234567890123456789012345678901212345678901234567890123456789012123456
(B) 65 7
1234567890123456789012345678901212345678901234567890123456789012123456
x 7
1234567890123456789012345678901212345678901234567890123456789012123456 77
(C) 75 10. If x is odd, and x is an integer, then the
1
12 34567890123456789012345678901212345678901234567890123456789012123456
(D) 85 value of x must be 7
12 34567890123456789012345678901212345678901234567890123456789012123456 7
12 34567890123456789012345678901212345678901234567890123456789012123456
(E) 110 (A) odd. 7
1234567890123456789012345678901212345678901234567890123456789012123456 77
2 34567890123456789012345678901212345678901234567890123456789012123456
1234567890123456789012345678901212345678901234567890123456789012123456
(B) even.
1234567890123456789012345678901212345678901234567890123456789012123456
8. @~22 1 22!21#22 5 7
1234567890123456789012345678901212345678901234567890123456789012123456
(C) less than one. 7
1234567890123456789012345678901212345678901234567890123456789012123456 7
1234567890123456789012345678901212345678901234567890123456789012123456
1 (D) an irrational number. 7
1234567890123456789012345678901212345678901234567890123456789012123456
(A) 7
1234567890123456789012345678901212345678901234567890123456789012123456
64 (E) None of the above. 7
1234567890123456789012345678901212345678901234567890123456789012123456 77
1 1
12 34567890123456789012345678901212345678901234567890123456789012123456
(B) 7
12 34567890123456789012345678901212345678901234567890123456789012123456
16 7
1234567890123456789012345678901212345678901234567890123456789012123456 7
1234567890123456789012345678901212345678901234567890123456789012123456 7
12
2
34567890123456789012345678901212345678901234567890123456789012123456
1
34567890123456789012345678901212345678901234567890123456789012123456
7
7
1234567890123456789012345678901212345678901234567890123456789012123456
(C)
1234567890123456789012345678901212345678901234567890123456789012123456
8 7
1 7
12 34567890123456789012345678901212345678901234567890123456789012123456 7
(D) 16
12 34567890123456789012345678901212345678901234567890123456789012123456 7
12 34567890123456789012345678901212345678901234567890123456789012123456
(E) 64 7
12 34567890123456789012345678901212345678901234567890123456789012123456 7
1234567890123456789012345678901212345678901234567890123456789012123456 7
1234567890123456789012345678901212345678901234567890123456789012123456 7
1234567890123456789012345678901212345678901234567890123456789012123456 7
12 34567890123456789012345678901212345678901234567890123456789012123456 7
12 34567890123456789012345678901212345678901234567890123456789012123456 7
12 34567890123456789012345678901212345678901234567890123456789012123456 7
12 34567890123456789012345678901212345678901234567890123456789012123456 7
1234567890123456789012345678901212345678901234567890123456789012123456 7
1234567890123456789012345678901212345678901234567890123456789012123456
2 7
1234567890123456789012345678901212345678901234567890123456789012123456 7
1234567890123456789012345678901212345678901234567890123456789012123456 7
1234567890123456789012345678901212345678901234567890123456789012123456 7
1 34567890123456789012345678901212345678901234567890123456789012123456 7
1234567890123456789012345678901212345678901234567890123456789012123456 77
2 34567890123456789012345678901212345678901234567890123456789012123456
1234567890123456789012345678901212345678901234567890123456789012123456 7
1234567890123456789012345678901212345678901234567890123456789012123456 7
1234567890123456789012345678901212345678901234567890123456789012123456 7
1
12 34567890123456789012345678901212345678901234567890123456789012123456 7
1234567890123456789012345678901212345678901234567890123456789012123456 7
12 34567890123456789012345678901212345678901234567890123456789012123456 7
1234567890123456789012345678901212345678901234567890123456789012123456 7
12 34567890123456789012345678901212345678901234567890123456789012123456 7
1234567890123456789012345678901212345678901234567890123456789012123456 7
1234567890123456789012345678901212345678901234567890123456789012123456 7
1234567890123456789012345678901212345678901234567890123456789012123456 7
1234567890123456789012345678901212345678901234567890123456789012123456 7
12345678901234567890123456789012123456789012345678901234567890121234567
12 Copyright © 2005 Thomson Peterson’s, a part of The Thomson Corporaton
SAT is a registered trademark of the College Entrance Examination Board, which
was not involved in the production of and does not endorse this product.
12345678901234567890123456789012123456789012345678901234567890121234567
1234567890123456789012345678901212345678901234567890123456789012123456 7
12234567890123456789012345678901212345678901234567890123456789012123456 7
1 34567890123456789012345678901212345678901234567890123456789012123456
The following description applies to questions 14. If the determinant of this matrix is 26, 7
12234567890123456789012345678901212345678901234567890123456789012123456 7
1234567890123456789012345678901212345678901234567890123456789012123456
11–13. what is the value of n? 7
1 34567890123456789012345678901212345678901234567890123456789012123456 7
12234567890123456789012345678901212345678901234567890123456789012123456 7
The factorial of a number is the product of all
the integers from one to the number. For
n 4
1 34567890123456789012345678901212345678901234567890123456789012123456
12234567890123456789012345678901212345678901234567890123456789012123456
1234567890123456789012345678901212345678901234567890123456789012123456
5 27
S D 7
7
7
1234567890123456789012345678901212345678901234567890123456789012123456
example, 5 factorial is 5 3 4 3 3 3 2 3 1. The 7
1234567890123456789012345678901212345678901234567890123456789012123456
34567890123456789012345678901212345678901234567890123456789012123456
(A) 7
7
1 notation for a factorial is the number followed 22
12234567890123456789012345678901212345678901234567890123456789012123456 7
1234567890123456789012345678901212345678901234567890123456789012123456
by an exclamation point. 3 7
1234567890123456789012345678901212345678901234567890123456789012123456
(B) 25 7
12345678901234567890123456789012123456789012345678901234567890121234565
34567890123456789012345678901212345678901234567890123456789012123456
7
7
1 Thus 5! 5 5 3 4 3 3 3 2 3 1.
12234567890123456789012345678901212345678901234567890123456789012123456 7
1234567890123456789012345678901212345678901234567890123456789012123456
(C) 27 7
1234567890123456789012345678901212345678901234567890123456789012123456
6! 7
1234567890123456789012345678901212345678901234567890123456789012123456
(D) 28
34567890123456789012345678901212345678901234567890123456789012123456
7
7
1 11. 5
12234567890123456789012345678901212345678901234567890123456789012123456
3! 7
12345678901234567890123456789012123456789012345678901234567890121234563 7
1234567890123456789012345678901212345678901234567890123456789012123456
(A) 2 (E) 28 7
12345678901234567890123456789012123456789012345678901234567890121234564 7
1234567890123456789012345678901212345678901234567890123456789012123456 77
34567890123456789012345678901212345678901234567890123456789012123456
(B) 16
1234567890123456789012345678901212345678901234567890123456789012123456
15. If p 1 q 5 2q 1 6, which of the following 7
1234567890123456789012345678901212345678901234567890123456789012123456
(C) 30
1234567890123456789012345678901212345678901234567890123456789012123456 7
1234567890123456789012345678901212345678901234567890123456789012123456
(D) 88 statements must be true? 7
1234567890123456789012345678901212345678901234567890123456789012123456 7
1 (E) 120 7
12234567890123456789012345678901212345678901234567890123456789012123456
I. p is even 7
1234567890123456789012345678901212345678901234567890123456789012123456 7
1234567890123456789012345678901212345678901234567890123456789012123456
2
12. If f(x) 5 (x!) then f(3) 5 II. q is even 7
34567890123456789012345678901212345678901234567890123456789012123456
1234567890123456789012345678901212345678901234567890123456789012123456 7
1234567890123456789012345678901212345678901234567890123456789012123456
III. pq is even 7
1234567890123456789012345678901212345678901234567890123456789012123456
(A) 16 7
1234567890123456789012345678901212345678901234567890123456789012123456 7
1234567890123456789012345678901212345678901234567890123456789012123456 77
(B) 36 (A) I only
1234567890123456789012345678901212345678901234567890123456789012123456
1234567890123456789012345678901212345678901234567890123456789012123456
(C) 172 (B) II only 7
1234567890123456789012345678901212345678901234567890123456789012123456 7
1234567890123456789012345678901212345678901234567890123456789012123456 77
(D) 1080 (C) II and III only
1
12234567890123456789012345678901212345678901234567890123456789012123456
(E) 6282 (D) I, II, and III 7
1234567890123456789012345678901212345678901234567890123456789012123456 7
1 34567890123456789012345678901212345678901234567890123456789012123456 7
(E) None
1234567890123456789012345678901212345678901234567890123456789012123456
13. If y 1 2 5 x , and y and x are integers, 7
12234567890123456789012345678901212345678901234567890123456789012123456
34567890123456789012345678901212345678901234567890123456789012123456
7
7
1234567890123456789012345678901212345678901234567890123456789012123456
y! 16.
1234567890123456789012345678901212345678901234567890123456789012123456
then 5 7
1 x! 7
12234567890123456789012345678901212345678901234567890123456789012123456 7
1234567890123456789012345678901212345678901234567890123456789012123456 7
34567890123456789012345678901212345678901234567890123456789012123456
1234567890123456789012345678901212345678901234567890123456789012123456
1 7
1234567890123456789012345678901212345678901234567890123456789012123456
(A) 7
1 ~ y 1 2!~ y 1 1 ! 7
1234567890123456789012345678901212345678901234567890123456789012123456 7
1234567890123456789012345678901212345678901234567890123456789012123456
(B) y 7
1234567890123456789012345678901212345678901234567890123456789012123456 77
2 34567890123456789012345678901212345678901234567890123456789012123456
1234567890123456789012345678901212345678901234567890123456789012123456
For the above graph, for which values of x 7
1234567890123456789012345678901212345678901234567890123456789012123456
1
1234567890123456789012345678901212345678901234567890123456789012123456
(C) 2 7
1 y is y . 0? 7
1234567890123456789012345678901212345678901234567890123456789012123456 77
2 34567890123456789012345678901212345678901234567890123456789012123456
1234567890123456789012345678901212345678901234567890123456789012123456
y
7
1234567890123456789012345678901212345678901234567890123456789012123456
(D) (A) 23 , 21 and 1 , x , 3
7
1234567890123456789012345678901212345678901234567890123456789012123456
x
7
1 (B) x , 23 and 21 , x , 1 and x . 3
2 34567890123456789012345678901212345678901234567890123456789012123456
1234567890123456789012345678901212345678901234567890123456789012123456 7
1234567890123456789012345678901212345678901234567890123456789012123456
(E) y(y 2 1) (C) x , 23 and 1 , x , 3 and x . 3 7
1234567890123456789012345678901212345678901234567890123456789012123456 7
1234567890123456789012345678901212345678901234567890123456789012123456 77
(D) x . 3 and x . 21
1
12234567890123456789012345678901212345678901234567890123456789012123456
(E) 23 . x . 3 7
1 34567890123456789012345678901212345678901234567890123456789012123456 7
12234567890123456789012345678901212345678901234567890123456789012123456 7
1 34567890123456789012345678901212345678901234567890123456789012123456 7
12234567890123456789012345678901212345678901234567890123456789012123456 7
1 34567890123456789012345678901212345678901234567890123456789012123456 7
1234567890123456789012345678901212345678901234567890123456789012123456 7
1234567890123456789012345678901212345678901234567890123456789012123456 7
1234567890123456789012345678901212345678901234567890123456789012123456 7
12345678901234567890123456789012123456789012345678901234567890121234567
Copyright © 2005 Thomson Peterson’s, a part of The Thomson Corporaton 13
SAT is a registered trademark of the College Entrance Examination Board, which
was not involved in the production of and does not endorse this product.
12345678901234567890123456789012123456789012345678901234567890121234567
1234567890123456789012345678901212345678901234567890123456789012123456 7
12 34567890123456789012345678901212345678901234567890123456789012123456 7
1234567890123456789012345678901212345678901234567890123456789012123456
17. 19. Raising n2p 7
12 34567890123456789012345678901212345678901234567890123456789012123456 n by which of the following will 7
12 34567890123456789012345678901212345678901234567890123456789012123456 7
1234567890123456789012345678901212345678901234567890123456789012123456 7
give the result n?
12 34567890123456789012345678901212345678901234567890123456789012123456 7
1234567890123456789012345678901212345678901234567890123456789012123456 7
12 345678901234567890123456789012123456789012345678901234567890121234562p 7
12 34567890123456789012345678901212345678901234567890123456789012123456
The numbers from the number set pn 7
12 34567890123456789012345678901212345678901234567890123456789012123456
(A) 7
12
2
34567890123456789012345678901212345678901234567890123456789012123456
{9, 11, 12, 15, 16} must be put in the
34567890123456789012345678901212345678901234567890123456789012123456
7
7
1 above boxes according to these condi-
n
12 34567890123456789012345678901212345678901234567890123456789012123456
(B) np 7
12 34567890123456789012345678901212345678901234567890123456789012123456
tions: 7
12 34567890123456789012345678901212345678901234567890123456789012123456
p 7
12
2
34567890123456789012345678901212345678901234567890123456789012123456
34567890123456789012345678901212345678901234567890123456789012123456
7
7
1 Boxes A, C, and D contain numbers (C)
12 34567890123456789012345678901212345678901234567890123456789012123456
n 7
12 34567890123456789012345678901212345678901234567890123456789012123456
divisible by three. 7
12 34567890123456789012345678901212345678901234567890123456789012123456
2n 7
12
2
34567890123456789012345678901212345678901234567890123456789012123456
(D)
34567890123456789012345678901212345678901234567890123456789012123456
7
7
1 Box B contains a prime number. p
12 34567890123456789012345678901212345678901234567890123456789012123456 7
12 34567890123456789012345678901212345678901234567890123456789012123456
n1p 7
12 34567890123456789012345678901212345678901234567890123456789012123456
Which number must be in Box E? (E) 7
12 34567890123456789012345678901212345678901234567890123456789012123456 p 7
2 34567890123456789012345678901212345678901234567890123456789012123456
1234567890123456789012345678901212345678901234567890123456789012123456
(A) 9 7
1234567890123456789012345678901212345678901234567890123456789012123456 7
1234567890123456789012345678901212345678901234567890123456789012123456
(B) 11 If n is a positive integer and n, n 2, and 7
1234567890123456789012345678901212345678901234567890123456789012123456 77
20. 2
1234567890123456789012345678901212345678901234567890123456789012123456
(C) 12 n 1 2 are each prime numbers, then the
1234567890123456789012345678901212345678901234567890123456789012123456 7
1 (D) 15 set of those three numbers is called a 7
12 34567890123456789012345678901212345678901234567890123456789012123456 7
12 34567890123456789012345678901212345678901234567890123456789012123456
(E) 16 prime triplet. How many different prime 7
12 34567890123456789012345678901212345678901234567890123456789012123456 7
2 34567890123456789012345678901212345678901234567890123456789012123456
1234567890123456789012345678901212345678901234567890123456789012123456
triplets are there where none of the set is 7
1234567890123456789012345678901212345678901234567890123456789012123456 7
1234567890123456789012345678901212345678901234567890123456789012123456
18. greater than fifty? 7
1234567890123456789012345678901212345678901234567890123456789012123456 77
1234567890123456789012345678901212345678901234567890123456789012123456
1234567890123456789012345678901212345678901234567890123456789012123456
(A) 1 7
1234567890123456789012345678901212345678901234567890123456789012123456 7
1234567890123456789012345678901212345678901234567890123456789012123456
(B) 2 7
1234567890123456789012345678901212345678901234567890123456789012123456 7
1 (C) 3 7
2 34567890123456789012345678901212345678901234567890123456789012123456
1234567890123456789012345678901212345678901234567890123456789012123456 7
1234567890123456789012345678901212345678901234567890123456789012123456
(D) 4 7
1 7
1234567890123456789012345678901212345678901234567890123456789012123456 7
If,AC 5 8, AB 5 4, and D and H are (E) None.
12 34567890123456789012345678901212345678901234567890123456789012123456 7
12 34567890123456789012345678901212345678901234567890123456789012123456
midpoints on CE and AG respectively, 7
12 34567890123456789012345678901212345678901234567890123456789012123456
21. If a circle has four tangents, each of which 7
1234567890123456789012345678901212345678901234567890123456789012123456 7
what percentage of the rectangle is
12 34567890123456789012345678901212345678901234567890123456789012123456
is perpendicular to two of the other 7
12 34567890123456789012345678901212345678901234567890123456789012123456
shaded? 7
1234567890123456789012345678901212345678901234567890123456789012123456 77
2 34567890123456789012345678901212345678901234567890123456789012123456
tangents, then
1234567890123456789012345678901212345678901234567890123456789012123456 7
1 (A) 12.5%
1234567890123456789012345678901212345678901234567890123456789012123456
(B) 16.33% (A) at most one pair of tangent lines is 7
1234567890123456789012345678901212345678901234567890123456789012123456 7
2 34567890123456789012345678901212345678901234567890123456789012123456
1234567890123456789012345678901212345678901234567890123456789012123456
(C) 20% parallel. 7
1234567890123456789012345678901212345678901234567890123456789012123456 7
1234567890123456789012345678901212345678901234567890123456789012123456
(D) 25% (B) at most two pairs of tangent lines are 7
1234567890123456789012345678901212345678901234567890123456789012123456 7
1 (E) 30% parallel. 7
2 34567890123456789012345678901212345678901234567890123456789012123456
1234567890123456789012345678901212345678901234567890123456789012123456 7
1234567890123456789012345678901212345678901234567890123456789012123456
(C) all four tangent lines are parallel. 7
1234567890123456789012345678901212345678901234567890123456789012123456 7
1234567890123456789012345678901212345678901234567890123456789012123456 77
(D) a square is inscribed in the circle.
1
12 34567890123456789012345678901212345678901234567890123456789012123456
(E) one particular diameter could be 7
12 34567890123456789012345678901212345678901234567890123456789012123456 7
12 34567890123456789012345678901212345678901234567890123456789012123456 7
perpendicular to all four tangent
12 34567890123456789012345678901212345678901234567890123456789012123456 7
1234567890123456789012345678901212345678901234567890123456789012123456
lines. 7
1234567890123456789012345678901212345678901234567890123456789012123456 77
2 34567890123456789012345678901212345678901234567890123456789012123456
1
12 34567890123456789012345678901212345678901234567890123456789012123456 7
1234567890123456789012345678901212345678901234567890123456789012123456 7
12 34567890123456789012345678901212345678901234567890123456789012123456 7
STOP
1234567890123456789012345678901212345678901234567890123456789012123456
Do not proceed to the next section until time is up.
1234567890123456789012345678901212345678901234567890123456789012123456 7
7
1234567890123456789012345678901212345678901234567890123456789012123456 7
1234567890123456789012345678901212345678901234567890123456789012123456 7
12345678901234567890123456789012123456789012345678901234567890121234567
14 Copyright © 2005 Thomson Peterson’s, a part of The Thomson Corporaton
SAT is a registered trademark of the College Entrance Examination Board, which
was not involved in the production of and does not endorse this product.
Section 3
30 Questions j Time—25 Minutes

12345678901234567890123456789012123456789012345678901234567890121234567
12234567890123456789012345678901212345678901234567890123456789012123456 7
1234567890123456789012345678901212345678901234567890123456789012123456
34567890123456789012345678901212345678901234567890123456789012123456
7
7
1 Identifying Sentence Errors
12234567890123456789012345678901212345678901234567890123456789012123456 7
1234567890123456789012345678901212345678901234567890123456789012123456 7
1234567890123456789012345678901212345678901234567890123456789012123456 7
1234567890123456789012345678901212345678901234567890123456789012123456
Directions: Mark the letter of your choice on the answer sheet that best corresponds to the 7
1234567890123456789012345678901212345678901234567890123456789012123456 77
34567890123456789012345678901212345678901234567890123456789012123456
1234567890123456789012345678901212345678901234567890123456789012123456
correct answer.
1234567890123456789012345678901212345678901234567890123456789012123456 77
1234567890123456789012345678901212345678901234567890123456789012123456
1234567890123456789012345678901212345678901234567890123456789012123456
Notes: 7
1234567890123456789012345678901212345678901234567890123456789012123456 77
1
12234567890123456789012345678901212345678901234567890123456789012123456
1. The following questions test your knowledge of the rules of English grammar, as well as 7
1234567890123456789012345678901212345678901234567890123456789012123456 7
1234567890123456789012345678901212345678901234567890123456789012123456
word usage, word choice, and idioms. 7
1234567890123456789012345678901212345678901234567890123456789012123456 77
34567890123456789012345678901212345678901234567890123456789012123456
1234567890123456789012345678901212345678901234567890123456789012123456
2. Some sentences are correct, but others contain a single error. No sentence contains more
1234567890123456789012345678901212345678901234567890123456789012123456 7
1234567890123456789012345678901212345678901234567890123456789012123456
than one error. 7
1234567890123456789012345678901212345678901234567890123456789012123456 77
1234567890123456789012345678901212345678901234567890123456789012123456
1234567890123456789012345678901212345678901234567890123456789012123456
3. Any errors that occur will be found in the underlined portion of the sentence. Choose the 7
1234567890123456789012345678901212345678901234567890123456789012123456 7
1234567890123456789012345678901212345678901234567890123456789012123456
letter underneath the error to indicate the part of the sentence that must be changed. 7
1 7
2 34567890123456789012345678901212345678901234567890123456789012123456
1234567890123456789012345678901212345678901234567890123456789012123456 7
1234567890123456789012345678901212345678901234567890123456789012123456
4. If there is no error, pick answer choice (E). 7
1 7
1234567890123456789012345678901212345678901234567890123456789012123456 7
12234567890123456789012345678901212345678901234567890123456789012123456
5. There will be no change in any parts of the sentence that are not underlined. 7
1234567890123456789012345678901212345678901234567890123456789012123456 77
34567890123456789012345678901212345678901234567890123456789012123456
1234567890123456789012345678901212345678901234567890123456789012123456 7
1
12234567890123456789012345678901212345678901234567890123456789012123456 7
1234567890123456789012345678901212345678901234567890123456789012123456 7
1234567890123456789012345678901212345678901234567890123456789012123456 77
34567890123456789012345678901212345678901234567890123456789012123456
1. Despite the enormous voter drive, there 3. The FDA did not conclude that the
1234567890123456789012345678901212345678901234567890123456789012123456 7
1 A B A
1234567890123456789012345678901212345678901234567890123456789012123456
are still many city-dwellers who are not negative side affects of the drug offset the 7
1234567890123456789012345678901212345678901234567890123456789012123456 7
12234567890123456789012345678901212345678901234567890123456789012123456
C B C 7
1234567890123456789012345678901212345678901234567890123456789012123456 7
1234567890123456789012345678901212345678901234567890123456789012123456
registered to vote. No error drug’s positive benefits . No error 7
1234567890123456789012345678901212345678901234567890123456789012123456 7
1 34567890123456789012345678901212345678901234567890123456789012123456 7
D E D E
1234567890123456789012345678901212345678901234567890123456789012123456 77
2 34567890123456789012345678901212345678901234567890123456789012123456
1234567890123456789012345678901212345678901234567890123456789012123456 7
1234567890123456789012345678901212345678901234567890123456789012123456
2. Debating the energy bill was the first 4. Over the last decade, the information
7
1234567890123456789012345678901212345678901234567890123456789012123456 7
1 A A
2 34567890123456789012345678901212345678901234567890123456789012123456
1234567890123456789012345678901212345678901234567890123456789012123456 7
1234567890123456789012345678901212345678901234567890123456789012123456
order of business for the Senate; to set the industry had grown into a multi-billion- 7
1234567890123456789012345678901212345678901234567890123456789012123456 7
1234567890123456789012345678901212345678901234567890123456789012123456 77
B C B
1 calendar for the upcoming session dollar industry that employs tens of
12234567890123456789012345678901212345678901234567890123456789012123456 7
1 34567890123456789012345678901212345678901234567890123456789012123456 C D 7
12234567890123456789012345678901212345678901234567890123456789012123456 7
1 34567890123456789012345678901212345678901234567890123456789012123456 7
was to follow . No error (E) thousands of workers. No error
12234567890123456789012345678901212345678901234567890123456789012123456 7
1 34567890123456789012345678901212345678901234567890123456789012123456
D E E 7
1234567890123456789012345678901212345678901234567890123456789012123456 7
1234567890123456789012345678901212345678901234567890123456789012123456 7
1234567890123456789012345678901212345678901234567890123456789012123456 7
12345678901234567890123456789012123456789012345678901234567890121234567
12345678901234567890123456789012123456789012345678901234567890121234567
1234567890123456789012345678901212345678901234567890123456789012123456 7
12 34567890123456789012345678901212345678901234567890123456789012123456 7
1234567890123456789012345678901212345678901234567890123456789012123456
5. Reading widely in her field, making herself 8. Of Armigo’s two films, most critics agree 7
12 34567890123456789012345678901212345678901234567890123456789012123456 7
12 34567890123456789012345678901212345678901234567890123456789012123456
A B A B 7
1234567890123456789012345678901212345678901234567890123456789012123456 7
12
2
34567890123456789012345678901212345678901234567890123456789012123456 7
available to students, and that the second
34567890123456789012345678901212345678901234567890123456789012123456
is best . No error
7
1
12 34567890123456789012345678901212345678901234567890123456789012123456 C D E 7
12 34567890123456789012345678901212345678901234567890123456789012123456
her sophisticated research paid off for 7
12 34567890123456789012345678901212345678901234567890123456789012123456 7
12
2
34567890123456789012345678901212345678901234567890123456789012123456
C D 9. Most people
34567890123456789012345678901212345678901234567890123456789012123456
in the neighborhood 7
7
1
12 34567890123456789012345678901212345678901234567890123456789012123456
Professor Jackson: she was awarded 7
12 34567890123456789012345678901212345678901234567890123456789012123456 7
12 34567890123456789012345678901212345678901234567890123456789012123456 7
agree that it is reasonable for the represen-
12 34567890123456789012345678901212345678901234567890123456789012123456 7
1234567890123456789012345678901212345678901234567890123456789012123456
tenure last year. No error A B 7
12 34567890123456789012345678901212345678901234567890123456789012123456
tative to not acquiesce to the demands of 7
12 34567890123456789012345678901212345678901234567890123456789012123456 7
E
12 34567890123456789012345678901212345678901234567890123456789012123456 C D 7
12
2
34567890123456789012345678901212345678901234567890123456789012123456
34567890123456789012345678901212345678901234567890123456789012123456
7
7
1 6. One cannot perform multiple tasks the transit authority. No error
12 34567890123456789012345678901212345678901234567890123456789012123456 7
12 34567890123456789012345678901212345678901234567890123456789012123456 E 7
12 34567890123456789012345678901212345678901234567890123456789012123456
simultaneously if one is easily distracted 7
12 34567890123456789012345678901212345678901234567890123456789012123456 7
2 34567890123456789012345678901212345678901234567890123456789012123456
1234567890123456789012345678901212345678901234567890123456789012123456
A B C 10. Although in many ways Canada is a 7
1234567890123456789012345678901212345678901234567890123456789012123456 7
1234567890123456789012345678901212345678901234567890123456789012123456
by one’s surroundings. No error A 7
1234567890123456789012345678901212345678901234567890123456789012123456 7
1234567890123456789012345678901212345678901234567890123456789012123456 77
D E staunch ally of the United States,
1234567890123456789012345678901212345678901234567890123456789012123456 7
1 B
12 34567890123456789012345678901212345678901234567890123456789012123456
7. In many ways emblematic of the sweeping they have made their differences 7
12 34567890123456789012345678901212345678901234567890123456789012123456 7
12 34567890123456789012345678901212345678901234567890123456789012123456
A C 7
12 34567890123456789012345678901212345678901234567890123456789012123456 7
12 34567890123456789012345678901212345678901234567890123456789012123456
changes the state’s agricultural industry known on a number of important issues. 7
12 34567890123456789012345678901212345678901234567890123456789012123456 7
12 34567890123456789012345678901212345678901234567890123456789012123456 7
B D
12 34567890123456789012345678901212345678901234567890123456789012123456 7
12 34567890123456789012345678901212345678901234567890123456789012123456
has undergone, strawberry farming No error 7
12 34567890123456789012345678901212345678901234567890123456789012123456 7
12 34567890123456789012345678901212345678901234567890123456789012123456
C E 7
12 34567890123456789012345678901212345678901234567890123456789012123456
had exploded in Central California. 7
1234567890123456789012345678901212345678901234567890123456789012123456 7
1234567890123456789012345678901212345678901234567890123456789012123456 77
2 34567890123456789012345678901212345678901234567890123456789012123456
D
1234567890123456789012345678901212345678901234567890123456789012123456 7
1 No error
1234567890123456789012345678901212345678901234567890123456789012123456 7
12 34567890123456789012345678901212345678901234567890123456789012123456 7
E
1234567890123456789012345678901212345678901234567890123456789012123456 77
2 34567890123456789012345678901212345678901234567890123456789012123456
1234567890123456789012345678901212345678901234567890123456789012123456 7
1
12 34567890123456789012345678901212345678901234567890123456789012123456 7
12 34567890123456789012345678901212345678901234567890123456789012123456 7
1234567890123456789012345678901212345678901234567890123456789012123456
2 7
1234567890123456789012345678901212345678901234567890123456789012123456 7
1 34567890123456789012345678901212345678901234567890123456789012123456 7
1234567890123456789012345678901212345678901234567890123456789012123456 7
1234567890123456789012345678901212345678901234567890123456789012123456 7
12 34567890123456789012345678901212345678901234567890123456789012123456 7
12 34567890123456789012345678901212345678901234567890123456789012123456 7
12 34567890123456789012345678901212345678901234567890123456789012123456 7
12 34567890123456789012345678901212345678901234567890123456789012123456 7
1234567890123456789012345678901212345678901234567890123456789012123456 7
1234567890123456789012345678901212345678901234567890123456789012123456
2 7
1234567890123456789012345678901212345678901234567890123456789012123456 7
1234567890123456789012345678901212345678901234567890123456789012123456 7
1234567890123456789012345678901212345678901234567890123456789012123456 7
1 34567890123456789012345678901212345678901234567890123456789012123456 7
1234567890123456789012345678901212345678901234567890123456789012123456 77
2 34567890123456789012345678901212345678901234567890123456789012123456
1234567890123456789012345678901212345678901234567890123456789012123456 7
1234567890123456789012345678901212345678901234567890123456789012123456 7
1234567890123456789012345678901212345678901234567890123456789012123456 7
1
12 34567890123456789012345678901212345678901234567890123456789012123456 7
1234567890123456789012345678901212345678901234567890123456789012123456 7
12 34567890123456789012345678901212345678901234567890123456789012123456 7
1234567890123456789012345678901212345678901234567890123456789012123456 7
12 34567890123456789012345678901212345678901234567890123456789012123456 7
1234567890123456789012345678901212345678901234567890123456789012123456 7
1234567890123456789012345678901212345678901234567890123456789012123456 7
1234567890123456789012345678901212345678901234567890123456789012123456 7
1234567890123456789012345678901212345678901234567890123456789012123456 7
12345678901234567890123456789012123456789012345678901234567890121234567
16 Copyright © 2005 Thomson Peterson’s, a part of The Thomson Corporaton
SAT is a registered trademark of the College Entrance Examination Board, which
was not involved in the production of and does not endorse this product.
12345678901234567890123456789012123456789012345678901234567890121234567
1234567890123456789012345678901212345678901234567890123456789012123456 7
12234567890123456789012345678901212345678901234567890123456789012123456 7
1 34567890123456789012345678901212345678901234567890123456789012123456
Improving Sentences 7
12234567890123456789012345678901212345678901234567890123456789012123456 7
1234567890123456789012345678901212345678901234567890123456789012123456 7
1 34567890123456789012345678901212345678901234567890123456789012123456 7
12234567890123456789012345678901212345678901234567890123456789012123456
Directions: 7
1 34567890123456789012345678901212345678901234567890123456789012123456 7
12234567890123456789012345678901212345678901234567890123456789012123456 7
1234567890123456789012345678901212345678901234567890123456789012123456
1. The following questions test your knowledge of English grammar, word usage, word 7
1234567890123456789012345678901212345678901234567890123456789012123456 7
1234567890123456789012345678901212345678901234567890123456789012123456
choice, sentence construction, and punctuation.
34567890123456789012345678901212345678901234567890123456789012123456
7
7
1
12234567890123456789012345678901212345678901234567890123456789012123456
2. Every sentence contains a portion that is underlined. 7
1234567890123456789012345678901212345678901234567890123456789012123456 7
1234567890123456789012345678901212345678901234567890123456789012123456 7
1234567890123456789012345678901212345678901234567890123456789012123456
3. Any errors that occur will be found in the underlined
34567890123456789012345678901212345678901234567890123456789012123456
portion of the sentence. If you 7
7
1
12234567890123456789012345678901212345678901234567890123456789012123456
believe there is an error, choose the answer choice that corrects the original mistake. 7
1234567890123456789012345678901212345678901234567890123456789012123456 7
1234567890123456789012345678901212345678901234567890123456789012123456
Answer choices (B), (C), (D), and (E) contain alternative phrasings of the underlined 7
1234567890123456789012345678901212345678901234567890123456789012123456
34567890123456789012345678901212345678901234567890123456789012123456
7
7
1 portion. If the sentence contains an error, one of these alternate phrasings will correct it.
12234567890123456789012345678901212345678901234567890123456789012123456 7
1234567890123456789012345678901212345678901234567890123456789012123456
4. Choice (A) repeats the original underlined portion. If you believe the underlined portion 7
1234567890123456789012345678901212345678901234567890123456789012123456 7
1234567890123456789012345678901212345678901234567890123456789012123456
does not contain any errors, select answer choice (A). 7
1234567890123456789012345678901212345678901234567890123456789012123456 77
34567890123456789012345678901212345678901234567890123456789012123456
1234567890123456789012345678901212345678901234567890123456789012123456
5. There will be no change in any parts of the sentence that are not underlined.
1234567890123456789012345678901212345678901234567890123456789012123456 7
1234567890123456789012345678901212345678901234567890123456789012123456 77
1234567890123456789012345678901212345678901234567890123456789012123456 7
1234567890123456789012345678901212345678901234567890123456789012123456 7
1
12234567890123456789012345678901212345678901234567890123456789012123456
11. Her first novel having been published, the 13. After working on his serve for several 7
1234567890123456789012345678901212345678901234567890123456789012123456 7
1234567890123456789012345678901212345678901234567890123456789012123456
author began to take notes for her second. days, rumors circulated that the challenger 7
1234567890123456789012345678901212345678901234567890123456789012123456 77
34567890123456789012345678901212345678901234567890123456789012123456
1234567890123456789012345678901212345678901234567890123456789012123456 would win the rematch.
1234567890123456789012345678901212345678901234567890123456789012123456
(A) Her first novel having been published 7
1234567890123456789012345678901212345678901234567890123456789012123456 7
1234567890123456789012345678901212345678901234567890123456789012123456
(B) Having been her most recent novel (A) After working on his serve for 7
1234567890123456789012345678901212345678901234567890123456789012123456 7
1234567890123456789012345678901212345678901234567890123456789012123456
published several days, rumors circulated that 7
1234567890123456789012345678901212345678901234567890123456789012123456 7
1234567890123456789012345678901212345678901234567890123456789012123456
(C) Her first novel, having been pub- the challenger would win the 7
1 7
1234567890123456789012345678901212345678901234567890123456789012123456 77
2 34567890123456789012345678901212345678901234567890123456789012123456
lished rematch.
1234567890123456789012345678901212345678901234567890123456789012123456 7
1 (D) When having had her first novel (B) After working on his serve for
1234567890123456789012345678901212345678901234567890123456789012123456 7
12234567890123456789012345678901212345678901234567890123456789012123456
published
34567890123456789012345678901212345678901234567890123456789012123456
several days, the challenger circu- 7
7
1234567890123456789012345678901212345678901234567890123456789012123456
1234567890123456789012345678901212345678901234567890123456789012123456
(E) Having published her first novel lated rumors that he would win the 7
1 7
12234567890123456789012345678901212345678901234567890123456789012123456 rematch. 7
1234567890123456789012345678901212345678901234567890123456789012123456
12. Van Gogh’s early work has often been 7
1234567890123456789012345678901212345678901234567890123456789012123456 77
34567890123456789012345678901212345678901234567890123456789012123456
(C) Rumors circulated that the chal-
1234567890123456789012345678901212345678901234567890123456789012123456
described as being in sharp contrast with lenger, after working on his serve for 7
1
1234567890123456789012345678901212345678901234567890123456789012123456
his later work, despite there is a funda- several days, would win the rematch. 7
1234567890123456789012345678901212345678901234567890123456789012123456 7
2 34567890123456789012345678901212345678901234567890123456789012123456
1234567890123456789012345678901212345678901234567890123456789012123456
mental continuity between the two. (D) After having worked on his serve for 7
1234567890123456789012345678901212345678901234567890123456789012123456 7
1234567890123456789012345678901212345678901234567890123456789012123456 several days, the rematch was 7
1234567890123456789012345678901212345678901234567890123456789012123456 77
(A) with his later work, despite
1 rumored to be won by the challenger.
12234567890123456789012345678901212345678901234567890123456789012123456
(B) with his later work; despite the fact 7
1234567890123456789012345678901212345678901234567890123456789012123456 (E) After working on his serve for 7
1234567890123456789012345678901212345678901234567890123456789012123456
that 7
1234567890123456789012345678901212345678901234567890123456789012123456 several days, rumors circulated, the 7
1 34567890123456789012345678901212345678901234567890123456789012123456
(C) with his later work, rather, 7
2 34567890123456789012345678901212345678901234567890123456789012123456
1234567890123456789012345678901212345678901234567890123456789012123456 challenger would win the rematch. 7
1234567890123456789012345678901212345678901234567890123456789012123456
(D) with his later work, but 7
1234567890123456789012345678901212345678901234567890123456789012123456 7
1234567890123456789012345678901212345678901234567890123456789012123456 77
(E) with his later work, notwithstanding
1
12234567890123456789012345678901212345678901234567890123456789012123456 7
1 34567890123456789012345678901212345678901234567890123456789012123456 7
12234567890123456789012345678901212345678901234567890123456789012123456 7
1 34567890123456789012345678901212345678901234567890123456789012123456 7
12234567890123456789012345678901212345678901234567890123456789012123456 7
1 34567890123456789012345678901212345678901234567890123456789012123456 7
1234567890123456789012345678901212345678901234567890123456789012123456 7
1234567890123456789012345678901212345678901234567890123456789012123456 7
1234567890123456789012345678901212345678901234567890123456789012123456 7
12345678901234567890123456789012123456789012345678901234567890121234567
Copyright © 2005 Thomson Peterson’s, a part of The Thomson Corporaton 17
SAT is a registered trademark of the College Entrance Examination Board, which
was not involved in the production of and does not endorse this product.
12345678901234567890123456789012123456789012345678901234567890121234567
1234567890123456789012345678901212345678901234567890123456789012123456 7
12 34567890123456789012345678901212345678901234567890123456789012123456 7
1234567890123456789012345678901212345678901234567890123456789012123456
14. The artist thought that it was important 16. Coffee shops, which were formerly found 7
12 34567890123456789012345678901212345678901234567890123456789012123456 7
12 34567890123456789012345678901212345678901234567890123456789012123456
both to portray the subject truthfully, no only in urban settings and near college 7
1234567890123456789012345678901212345678901234567890123456789012123456 7
12
2
34567890123456789012345678901212345678901234567890123456789012123456 7
matter the difficulty, and revealing campuses, have
34567890123456789012345678901212345678901234567890123456789012123456
been expanding in the last
7
1
12 34567890123456789012345678901212345678901234567890123456789012123456
something new about the subject. few years outside these circumspect 7
12 34567890123456789012345678901212345678901234567890123456789012123456 7
12 34567890123456789012345678901212345678901234567890123456789012123456
domains. 7
12
2
34567890123456789012345678901212345678901234567890123456789012123456 7
(A) and revealing something new about
34567890123456789012345678901212345678901234567890123456789012123456 7
1
12 34567890123456789012345678901212345678901234567890123456789012123456
the subject. (A) which were formerly found only in 7
12 34567890123456789012345678901212345678901234567890123456789012123456 7
12 34567890123456789012345678901212345678901234567890123456789012123456 7
(B) and so he revealed something new urban settings and near college
12 34567890123456789012345678901212345678901234567890123456789012123456 7
1234567890123456789012345678901212345678901234567890123456789012123456
about the subject. campuses 7
12 34567890123456789012345678901212345678901234567890123456789012123456 7
12 34567890123456789012345678901212345678901234567890123456789012123456 7
(C) and to reveal something new about (B) being formerly found only in urban
12 34567890123456789012345678901212345678901234567890123456789012123456 7
12
2
the subject. settings and near college campuses
34567890123456789012345678901212345678901234567890123456789012123456
34567890123456789012345678901212345678901234567890123456789012123456
7
1 (D) having thereby revealed something (C) which have been found formerly only 7
12 34567890123456789012345678901212345678901234567890123456789012123456 7
12 34567890123456789012345678901212345678901234567890123456789012123456
new about the subject. in urban settings and near college 7
12 34567890123456789012345678901212345678901234567890123456789012123456 7
12 34567890123456789012345678901212345678901234567890123456789012123456
(E) and revealing something about the campuses 7
12 34567890123456789012345678901212345678901234567890123456789012123456 7
12 34567890123456789012345678901212345678901234567890123456789012123456
subject that is new. (D) which were formerly found only in 7
12 34567890123456789012345678901212345678901234567890123456789012123456 7
12 34567890123456789012345678901212345678901234567890123456789012123456
urban settings or near college 7
12 34567890123456789012345678901212345678901234567890123456789012123456 7
1234567890123456789012345678901212345678901234567890123456789012123456 77
2 34567890123456789012345678901212345678901234567890123456789012123456
15. Max Planck was not only one of the campuses
1
12 34567890123456789012345678901212345678901234567890123456789012123456
founders of quantum mechanics, but an (E) that were formerly found only in 7
12 34567890123456789012345678901212345678901234567890123456789012123456 7
12 34567890123456789012345678901212345678901234567890123456789012123456 7
accomplished pianist. urban settings and near college
2 34567890123456789012345678901212345678901234567890123456789012123456
1234567890123456789012345678901212345678901234567890123456789012123456 7
1234567890123456789012345678901212345678901234567890123456789012123456
campuses 7
1234567890123456789012345678901212345678901234567890123456789012123456 77
(A) mechanics, but an accomplished
1234567890123456789012345678901212345678901234567890123456789012123456
pianist.
1234567890123456789012345678901212345678901234567890123456789012123456 7
1234567890123456789012345678901212345678901234567890123456789012123456
(B) mechanics; but he was also an
17. Until the Chin dynasty changed this 7
1234567890123456789012345678901212345678901234567890123456789012123456 7
practice, most
1234567890123456789012345678901212345678901234567890123456789012123456
accomplished pianist.
Chinese intellectuals did 7
1234567890123456789012345678901212345678901234567890123456789012123456
not travel to the imperial court but 7
1 (C) mechanics; and he was also an 7
2 34567890123456789012345678901212345678901234567890123456789012123456
remained in their native provincial centers. 7
1234567890123456789012345678901212345678901234567890123456789012123456
1234567890123456789012345678901212345678901234567890123456789012123456
accomplished pianist. 7
1 7
1234567890123456789012345678901212345678901234567890123456789012123456
(D) mechanics, and an accomplished (A) but remained in their native provin- 7
12 34567890123456789012345678901212345678901234567890123456789012123456 7
1234567890123456789012345678901212345678901234567890123456789012123456 77
2 34567890123456789012345678901212345678901234567890123456789012123456
pianist. cial centers.
1234567890123456789012345678901212345678901234567890123456789012123456 7
1 (E) mechanics, but also an accomplished (B) and remained in their native provin-
12 34567890123456789012345678901212345678901234567890123456789012123456 7
12 34567890123456789012345678901212345678901234567890123456789012123456 7
piano. cial centers.
2 34567890123456789012345678901212345678901234567890123456789012123456
1234567890123456789012345678901212345678901234567890123456789012123456 7
1234567890123456789012345678901212345678901234567890123456789012123456
(C) but rather they remained in the 7
1 7
1234567890123456789012345678901212345678901234567890123456789012123456 7
native provinces.
1234567890123456789012345678901212345678901234567890123456789012123456 7
12 34567890123456789012345678901212345678901234567890123456789012123456
(D) yet they remained in their native 7
12 34567890123456789012345678901212345678901234567890123456789012123456 7
12 34567890123456789012345678901212345678901234567890123456789012123456 7
provincial centers.
12 34567890123456789012345678901212345678901234567890123456789012123456 7
1234567890123456789012345678901212345678901234567890123456789012123456
(E) but remained in the provinces to 7
2 34567890123456789012345678901212345678901234567890123456789012123456
1234567890123456789012345678901212345678901234567890123456789012123456 7
1234567890123456789012345678901212345678901234567890123456789012123456
which they were native. 7
1234567890123456789012345678901212345678901234567890123456789012123456 77
1234567890123456789012345678901212345678901234567890123456789012123456 7
1
12 34567890123456789012345678901212345678901234567890123456789012123456 7
12 34567890123456789012345678901212345678901234567890123456789012123456 7
12 34567890123456789012345678901212345678901234567890123456789012123456 7
12 34567890123456789012345678901212345678901234567890123456789012123456 7
1234567890123456789012345678901212345678901234567890123456789012123456 7
1234567890123456789012345678901212345678901234567890123456789012123456
2 7
1 34567890123456789012345678901212345678901234567890123456789012123456 7
12 34567890123456789012345678901212345678901234567890123456789012123456 7
1234567890123456789012345678901212345678901234567890123456789012123456 7
12 34567890123456789012345678901212345678901234567890123456789012123456 7
1234567890123456789012345678901212345678901234567890123456789012123456 7
1234567890123456789012345678901212345678901234567890123456789012123456 7
1234567890123456789012345678901212345678901234567890123456789012123456 7
1234567890123456789012345678901212345678901234567890123456789012123456 7
12345678901234567890123456789012123456789012345678901234567890121234567
18 Copyright © 2005 Thomson Peterson’s, a part of The Thomson Corporaton
SAT is a registered trademark of the College Entrance Examination Board, which
was not involved in the production of and does not endorse this product.
12345678901234567890123456789012123456789012345678901234567890121234567
1234567890123456789012345678901212345678901234567890123456789012123456 7
12234567890123456789012345678901212345678901234567890123456789012123456 7
1 34567890123456789012345678901212345678901234567890123456789012123456
18. The artwork of the late Renaissance was 20. Philology, the study of words, no longer 7
12234567890123456789012345678901212345678901234567890123456789012123456 7
1234567890123456789012345678901212345678901234567890123456789012123456
characterized by a deep sympathy for the exists in academia as a distinct discipline 7
1 34567890123456789012345678901212345678901234567890123456789012123456 7
12234567890123456789012345678901212345678901234567890123456789012123456
human subject, often portraying human because
34567890123456789012345678901212345678901234567890123456789012123456
it has been subsumed under the 7
7
1
12234567890123456789012345678901212345678901234567890123456789012123456
frailties and failings. study of linguistics. 7
1234567890123456789012345678901212345678901234567890123456789012123456 7
1234567890123456789012345678901212345678901234567890123456789012123456 7
1234567890123456789012345678901212345678901234567890123456789012123456
(A) often portraying human frailties and (A) it
34567890123456789012345678901212345678901234567890123456789012123456
has been subsumed under the study 7
7
1
12234567890123456789012345678901212345678901234567890123456789012123456
failings. of linguistics. 7
1234567890123456789012345678901212345678901234567890123456789012123456 7
1234567890123456789012345678901212345678901234567890123456789012123456
(B) and it often portrayed human (B) it was subsumed in the past under 7
1234567890123456789012345678901212345678901234567890123456789012123456
34567890123456789012345678901212345678901234567890123456789012123456
7
7
1 frailties and failings. the study of linguistics.
12234567890123456789012345678901212345678901234567890123456789012123456 7
1234567890123456789012345678901212345678901234567890123456789012123456
(C) human frailties and human failings (C) it has been subsumed with the study 7
1234567890123456789012345678901212345678901234567890123456789012123456 7
being often portrayed.
1234567890123456789012345678901212345678901234567890123456789012123456of
34567890123456789012345678901212345678901234567890123456789012123456
linguistics. 7
7
1 (D) although it often portrayed human (D) linguistics previously having sub-
12234567890123456789012345678901212345678901234567890123456789012123456 7
1234567890123456789012345678901212345678901234567890123456789012123456
frailties and failings. sumed it. 7
1234567890123456789012345678901212345678901234567890123456789012123456 7
1234567890123456789012345678901212345678901234567890123456789012123456
(E) though portraying human frailties (E) it had been subsumed under the 7
1234567890123456789012345678901212345678901234567890123456789012123456 77
34567890123456789012345678901212345678901234567890123456789012123456
1234567890123456789012345678901212345678901234567890123456789012123456
and failings. study of linguistics.
1234567890123456789012345678901212345678901234567890123456789012123456 77
1234567890123456789012345678901212345678901234567890123456789012123456 7
1234567890123456789012345678901212345678901234567890123456789012123456
1234567890123456789012345678901212345678901234567890123456789012123456
19. Pancho Villa’s raid on Columbus, New 7
1 7
12234567890123456789012345678901212345678901234567890123456789012123456
Mexico, which was part of the tumult of 7
1234567890123456789012345678901212345678901234567890123456789012123456 7
1234567890123456789012345678901212345678901234567890123456789012123456
the Mexican revolution, therefore 7
34567890123456789012345678901212345678901234567890123456789012123456
1234567890123456789012345678901212345678901234567890123456789012123456 7
1234567890123456789012345678901212345678901234567890123456789012123456
prompted a retaliatory expedition led by 7
1234567890123456789012345678901212345678901234567890123456789012123456 7
1234567890123456789012345678901212345678901234567890123456789012123456 77
General Pershing.
1234567890123456789012345678901212345678901234567890123456789012123456 7
1234567890123456789012345678901212345678901234567890123456789012123456
(A) revolution, therefore prompted a
1234567890123456789012345678901212345678901234567890123456789012123456 7
1234567890123456789012345678901212345678901234567890123456789012123456
retaliatory expedition led by General 7
1234567890123456789012345678901212345678901234567890123456789012123456 77
1 Pershing.
12234567890123456789012345678901212345678901234567890123456789012123456 7
1234567890123456789012345678901212345678901234567890123456789012123456
(B) revolution, thereby prompting a 7
1 34567890123456789012345678901212345678901234567890123456789012123456 7
1234567890123456789012345678901212345678901234567890123456789012123456
retaliatory expedition led by General 7
12234567890123456789012345678901212345678901234567890123456789012123456 7
1234567890123456789012345678901212345678901234567890123456789012123456 77
34567890123456789012345678901212345678901234567890123456789012123456
Pershing.
1234567890123456789012345678901212345678901234567890123456789012123456 7
1 (C) revolution, had prompted General
12234567890123456789012345678901212345678901234567890123456789012123456 7
1234567890123456789012345678901212345678901234567890123456789012123456
Pershing to lead a retaliatory 7
34567890123456789012345678901212345678901234567890123456789012123456
1234567890123456789012345678901212345678901234567890123456789012123456 7
1234567890123456789012345678901212345678901234567890123456789012123456
expedition. 7
1 7
1234567890123456789012345678901212345678901234567890123456789012123456 7
(D) revolution; a retaliatory expedition
1234567890123456789012345678901212345678901234567890123456789012123456 7
12234567890123456789012345678901212345678901234567890123456789012123456
led by General Pershing thereby 7
1234567890123456789012345678901212345678901234567890123456789012123456 7
1234567890123456789012345678901212345678901234567890123456789012123456
prompted. 7
1234567890123456789012345678901212345678901234567890123456789012123456 7
1 34567890123456789012345678901212345678901234567890123456789012123456 7
(E) revolution, prompted a retaliatory
2 34567890123456789012345678901212345678901234567890123456789012123456
1234567890123456789012345678901212345678901234567890123456789012123456 7
1234567890123456789012345678901212345678901234567890123456789012123456
expedition led by General Pershing. 7
1234567890123456789012345678901212345678901234567890123456789012123456 77
1234567890123456789012345678901212345678901234567890123456789012123456 7
1
12234567890123456789012345678901212345678901234567890123456789012123456 7
1234567890123456789012345678901212345678901234567890123456789012123456 7
1234567890123456789012345678901212345678901234567890123456789012123456 7
1234567890123456789012345678901212345678901234567890123456789012123456 7
1 34567890123456789012345678901212345678901234567890123456789012123456 7
1234567890123456789012345678901212345678901234567890123456789012123456 77
2 34567890123456789012345678901212345678901234567890123456789012123456
1
12234567890123456789012345678901212345678901234567890123456789012123456 7
1 34567890123456789012345678901212345678901234567890123456789012123456 7
12234567890123456789012345678901212345678901234567890123456789012123456 7
1 34567890123456789012345678901212345678901234567890123456789012123456 7
1234567890123456789012345678901212345678901234567890123456789012123456 7
1234567890123456789012345678901212345678901234567890123456789012123456 7
1234567890123456789012345678901212345678901234567890123456789012123456 7
12345678901234567890123456789012123456789012345678901234567890121234567
Copyright © 2005 Thomson Peterson’s, a part of The Thomson Corporaton 19
SAT is a registered trademark of the College Entrance Examination Board, which
was not involved in the production of and does not endorse this product.
12345678901234567890123456789012123456789012345678901234567890121234567
1234567890123456789012345678901212345678901234567890123456789012123456 7
12 34567890123456789012345678901212345678901234567890123456789012123456 7
1234567890123456789012345678901212345678901234567890123456789012123456
Improving Paragraphs 7
12 34567890123456789012345678901212345678901234567890123456789012123456 7
12 34567890123456789012345678901212345678901234567890123456789012123456 7
1234567890123456789012345678901212345678901234567890123456789012123456 7
12 34567890123456789012345678901212345678901234567890123456789012123456
Directions: 7
1234567890123456789012345678901212345678901234567890123456789012123456 7
12 34567890123456789012345678901212345678901234567890123456789012123456 7
12 34567890123456789012345678901212345678901234567890123456789012123456
1. The following questions test your knowledge of paragraph and sentence construction. 7
12 34567890123456789012345678901212345678901234567890123456789012123456 7
12 34567890123456789012345678901212345678901234567890123456789012123456 7
1234567890123456789012345678901212345678901234567890123456789012123456
2. The following passage is a rough draft of an essay. This rough draft contains various 7
12 34567890123456789012345678901212345678901234567890123456789012123456 7
12 34567890123456789012345678901212345678901234567890123456789012123456 7
errors.
12 34567890123456789012345678901212345678901234567890123456789012123456 7
12 34567890123456789012345678901212345678901234567890123456789012123456 7
1 34567890123456789012345678901212345678901234567890123456789012123456
2 3. Read the rough draft and then answer the questions that follow. Some questions will focus 7
12 34567890123456789012345678901212345678901234567890123456789012123456 7
12 34567890123456789012345678901212345678901234567890123456789012123456 7
on specific sentences and ask if there are any problems with that sentence’s word choice,
12 34567890123456789012345678901212345678901234567890123456789012123456
word usage, or overall structure. Other questions will ask about the paragraph itself. 7
12
2
34567890123456789012345678901212345678901234567890123456789012123456
34567890123456789012345678901212345678901234567890123456789012123456
7
7
1 These questions will focus on paragraph organization and development.
12 34567890123456789012345678901212345678901234567890123456789012123456 7
12 34567890123456789012345678901212345678901234567890123456789012123456 7
12 34567890123456789012345678901212345678901234567890123456789012123456
4. Select the answer that best reflects the rules of English grammar and proper essay and 7
12 34567890123456789012345678901212345678901234567890123456789012123456 7
1234567890123456789012345678901212345678901234567890123456789012123456 77
2 34567890123456789012345678901212345678901234567890123456789012123456
paragraph writing.
1234567890123456789012345678901212345678901234567890123456789012123456 7
1234567890123456789012345678901212345678901234567890123456789012123456 7
1234567890123456789012345678901212345678901234567890123456789012123456 7
1234567890123456789012345678901212345678901234567890123456789012123456 7
1234567890123456789012345678901212345678901234567890123456789012123456 7
1 Questions 21–25 are based on the following 21. Which of the following offers the best
12 34567890123456789012345678901212345678901234567890123456789012123456 7
12 34567890123456789012345678901212345678901234567890123456789012123456
passage. combination of sentences 1 and 2 (repro- 7
12 34567890123456789012345678901212345678901234567890123456789012123456 7
1234567890123456789012345678901212345678901234567890123456789012123456 77
2 34567890123456789012345678901212345678901234567890123456789012123456
duced below)?
1234567890123456789012345678901212345678901234567890123456789012123456
An incredible hot-air balloon exhibition hap- 7
1234567890123456789012345678901212345678901234567890123456789012123456
(1) An incredible hot-air balloon exhibition
1234567890123456789012345678901212345678901234567890123456789012123456 7
1234567890123456789012345678901212345678901234567890123456789012123456
happened on September 5, 1862. (2) It was 7
1234567890123456789012345678901212345678901234567890123456789012123456
pened on September 5, 1862. It was given 7
1234567890123456789012345678901212345678901234567890123456789012123456
given by Glaisher and Coxwell, two English- 7
1234567890123456789012345678901212345678901234567890123456789012123456
men. (3) There was no compressed oxygen for
by Glaisher and Coxwell, two Englishmen. 7
1234567890123456789012345678901212345678901234567890123456789012123456 77
1 them to breathe in those days. (4) They got so (A) An incredible hot-air balloon
12 34567890123456789012345678901212345678901234567890123456789012123456 7
12 34567890123456789012345678901212345678901234567890123456789012123456
high that they couldn’t use their limbs. (5) exhibition was given September 5, 7
1234567890123456789012345678901212345678901234567890123456789012123456 7
1234567890123456789012345678901212345678901234567890123456789012123456
Coxwell had to open the descending valve with 1862 by Glaisher and Coxwell, two 7
12
2
34567890123456789012345678901212345678901234567890123456789012123456
34567890123456789012345678901212345678901234567890123456789012123456
7
7
1234567890123456789012345678901212345678901234567890123456789012123456
his teeth. (6) Before Glaisher passed out, he Englishmen.
1234567890123456789012345678901212345678901234567890123456789012123456 7
1 recorded an elevation of twenty-nine thousand (B) An incredibly hot-air balloon 7
12 34567890123456789012345678901212345678901234567890123456789012123456 7
12 34567890123456789012345678901212345678901234567890123456789012123456
feet. (7) Many believe they got eight thousand exhibition happened on September 5, 7
2 34567890123456789012345678901212345678901234567890123456789012123456
1234567890123456789012345678901212345678901234567890123456789012123456 7
1234567890123456789012345678901212345678901234567890123456789012123456
feet higher before they began to descend, 1862, given by Glaisher and Cox- 7
1 making their ascent the highest in the nine- 7
1234567890123456789012345678901212345678901234567890123456789012123456 well, two Englishmen. 7
1234567890123456789012345678901212345678901234567890123456789012123456 7
1234567890123456789012345678901212345678901234567890123456789012123456 77
2 34567890123456789012345678901212345678901234567890123456789012123456
teenth century. (C) Given by Glaisher and Coxwell, two
1234567890123456789012345678901212345678901234567890123456789012123456 7
1234567890123456789012345678901212345678901234567890123456789012123456
(8) Now the largest balloon to go up in the Englishmen, an incredible hot-air
7
1234567890123456789012345678901212345678901234567890123456789012123456 7
1 nineteenth century was “The Giant.” (9) The balloon exhibition happened on
2 34567890123456789012345678901212345678901234567890123456789012123456
1234567890123456789012345678901212345678901234567890123456789012123456 7
1234567890123456789012345678901212345678901234567890123456789012123456
balloon held 215,000 cubic feet of air and was September 5, 1862. 7
1234567890123456789012345678901212345678901234567890123456789012123456 7
1234567890123456789012345678901212345678901234567890123456789012123456 77
74 feet wide. (10) It could carry four and a half (D) Glaisher and Coxwell, two English-
1 tons of cargo. (11) Its flight began in Paris, in
12 34567890123456789012345678901212345678901234567890123456789012123456 men, gave an incredible hot-air 7
12 34567890123456789012345678901212345678901234567890123456789012123456 7
12 34567890123456789012345678901212345678901234567890123456789012123456 7
1853, with fifteen passengers. (12) All of whom balloon exhibition, happening on
12 34567890123456789012345678901212345678901234567890123456789012123456 7
1234567890123456789012345678901212345678901234567890123456789012123456
returned safely. (13) The successful trip received September 5, 1862. 7
2 34567890123456789012345678901212345678901234567890123456789012123456
(E) Two Englishmen, Glaisher and 7
1234567890123456789012345678901212345678901234567890123456789012123456
1 a great deal of national and international press 7
12 34567890123456789012345678901212345678901234567890123456789012123456
because many thought the hot-air balloon would Coxwell, gave an incredible hot-air bal- 7
1234567890123456789012345678901212345678901234567890123456789012123456 7
12 34567890123456789012345678901212345678901234567890123456789012123456
become a form of common transportation. loon exhibition on September 5, 1862. 7
1234567890123456789012345678901212345678901234567890123456789012123456 7
1234567890123456789012345678901212345678901234567890123456789012123456 7
1234567890123456789012345678901212345678901234567890123456789012123456 7
1234567890123456789012345678901212345678901234567890123456789012123456 7
12345678901234567890123456789012123456789012345678901234567890121234567
20 Copyright © 2005 Thomson Peterson’s, a part of The Thomson Corporaton
SAT is a registered trademark of the College Entrance Examination Board, which
was not involved in the production of and does not endorse this product.
12345678901234567890123456789012123456789012345678901234567890121234567
1234567890123456789012345678901212345678901234567890123456789012123456 7
12234567890123456789012345678901212345678901234567890123456789012123456 7
1 34567890123456789012345678901212345678901234567890123456789012123456
22. Which of the following sentences in the 24. Which of the following is the best way to 7
12234567890123456789012345678901212345678901234567890123456789012123456 7
1234567890123456789012345678901212345678901234567890123456789012123456
first paragraph appears to be out of order? combine sentences 9 and 10 (reproduced 7
1 34567890123456789012345678901212345678901234567890123456789012123456 7
12234567890123456789012345678901212345678901234567890123456789012123456
below)?
34567890123456789012345678901212345678901234567890123456789012123456
7
7
1 (A) There was no compressed oxygen for
12234567890123456789012345678901212345678901234567890123456789012123456 7
1234567890123456789012345678901212345678901234567890123456789012123456
them to breathe in those days. The balloon held 215,000 cubic feet of air 7
1234567890123456789012345678901212345678901234567890123456789012123456 7
1234567890123456789012345678901212345678901234567890123456789012123456
(B) They got so high that they couldn’t and
34567890123456789012345678901212345678901234567890123456789012123456
was 74 feet wide. It could handle four 7
7
1
12234567890123456789012345678901212345678901234567890123456789012123456
use their limbs. and a half tons of cargo. 7
1234567890123456789012345678901212345678901234567890123456789012123456 7
1234567890123456789012345678901212345678901234567890123456789012123456
(C) Coxwell had to open the descending 7
1234567890123456789012345678901212345678901234567890123456789012123456
(A)
34567890123456789012345678901212345678901234567890123456789012123456
The balloon held 215,000 cubic feet 7
7
1 valve with his teeth.
of air and was 74 feet wide, which
12234567890123456789012345678901212345678901234567890123456789012123456 7
1234567890123456789012345678901212345678901234567890123456789012123456
(D) Before Glaisher passed out, he
could handle four and a half tons of 7
1234567890123456789012345678901212345678901234567890123456789012123456 7
1234567890123456789012345678901212345678901234567890123456789012123456
recorded an elevation of 29 thousand
34567890123456789012345678901212345678901234567890123456789012123456 cargo. 7
7
1 feet.
12234567890123456789012345678901212345678901234567890123456789012123456
(B) The balloon held 215,000 cubic feet 7
1234567890123456789012345678901212345678901234567890123456789012123456
(E) Many believe they got 8 thousand 7
1234567890123456789012345678901212345678901234567890123456789012123456 of air and was 74 feet wide, handling 7
1234567890123456789012345678901212345678901234567890123456789012123456
feet higher before they began to 7
1234567890123456789012345678901212345678901234567890123456789012123456 four and a half tons of cargo. 7
1234567890123456789012345678901212345678901234567890123456789012123456
descend. 7
1234567890123456789012345678901212345678901234567890123456789012123456
(C) The balloon held 215,000 cubic feet 7
1234567890123456789012345678901212345678901234567890123456789012123456 7
1234567890123456789012345678901212345678901234567890123456789012123456 of air and was 74 feet wide; it could 7
34567890123456789012345678901212345678901234567890123456789012123456
1234567890123456789012345678901212345678901234567890123456789012123456
23. Which of the following is the best revision
handle four and a half tons of cargo. 7
7
1
12234567890123456789012345678901212345678901234567890123456789012123456
for sentence 8 (reproduced below)?
(D) The balloon held 215,000 cubic feet 7
1234567890123456789012345678901212345678901234567890123456789012123456 7
1234567890123456789012345678901212345678901234567890123456789012123456 7
1234567890123456789012345678901212345678901234567890123456789012123456 77
34567890123456789012345678901212345678901234567890123456789012123456
Now the largest balloon to go up in the of air and was 74 feet wide, and it
1234567890123456789012345678901212345678901234567890123456789012123456 could handle four and a half tons of 7
1234567890123456789012345678901212345678901234567890123456789012123456
nineteenth century was “The Giant.”
1234567890123456789012345678901212345678901234567890123456789012123456 7
1234567890123456789012345678901212345678901234567890123456789012123456
(A) Move “in the nineteenth century” to cargo. 7
(E) The balloon held 215,000 cubic feet 7
1234567890123456789012345678901212345678901234567890123456789012123456
1234567890123456789012345678901212345678901234567890123456789012123456
the beginning of the sentence and 7
1234567890123456789012345678901212345678901234567890123456789012123456 7
1234567890123456789012345678901212345678901234567890123456789012123456
delete “Now” of air and was 74 feet wide, but it 7
1 7
1234567890123456789012345678901212345678901234567890123456789012123456 77
2 34567890123456789012345678901212345678901234567890123456789012123456
(B) Add a comma after “Now.” could carry four and a half tons of
1234567890123456789012345678901212345678901234567890123456789012123456 cargo. 7
1 (C) Begin the sentence with “Moreover,”
1234567890123456789012345678901212345678901234567890123456789012123456 7
12234567890123456789012345678901212345678901234567890123456789012123456
(D) Delete “now.”
34567890123456789012345678901212345678901234567890123456789012123456
7
7
1234567890123456789012345678901212345678901234567890123456789012123456
25. Which of the following is the best way to
1234567890123456789012345678901212345678901234567890123456789012123456
(E) Replace “to go up” with “exhibition.” 7
1 revise sentences 11 and 12 (reproduced 7
12234567890123456789012345678901212345678901234567890123456789012123456 7
1234567890123456789012345678901212345678901234567890123456789012123456
below)? 7
1234567890123456789012345678901212345678901234567890123456789012123456 77
34567890123456789012345678901212345678901234567890123456789012123456
1234567890123456789012345678901212345678901234567890123456789012123456 7
1 Its flight began in Paris, in 1853, with
1234567890123456789012345678901212345678901234567890123456789012123456 7
1234567890123456789012345678901212345678901234567890123456789012123456 7
fifteen passengers. All of whom returned
2 34567890123456789012345678901212345678901234567890123456789012123456
1234567890123456789012345678901212345678901234567890123456789012123456 7
1234567890123456789012345678901212345678901234567890123456789012123456
safely. 7
1234567890123456789012345678901212345678901234567890123456789012123456 77
1234567890123456789012345678901212345678901234567890123456789012123456
(A) Replace “whom” with “who.”
7
1
2 34567890123456789012345678901212345678901234567890123456789012123456
1234567890123456789012345678901212345678901234567890123456789012123456
(B) Make the second sentence read 7
1234567890123456789012345678901212345678901234567890123456789012123456 7
1234567890123456789012345678901212345678901234567890123456789012123456 “Who all returned safely.” 7
1234567890123456789012345678901212345678901234567890123456789012123456 77
1 (C) Delete “of”
12234567890123456789012345678901212345678901234567890123456789012123456 7
1234567890123456789012345678901212345678901234567890123456789012123456
(D) Replace the period at the end of 7
1234567890123456789012345678901212345678901234567890123456789012123456 7
1234567890123456789012345678901212345678901234567890123456789012123456 sentence 11 with a comma. 7
1 34567890123456789012345678901212345678901234567890123456789012123456 7
1234567890123456789012345678901212345678901234567890123456789012123456 77
2 34567890123456789012345678901212345678901234567890123456789012123456
(E) Delete the period at the end of
1
12234567890123456789012345678901212345678901234567890123456789012123456 sentence 11 and change “returned” 7
1 34567890123456789012345678901212345678901234567890123456789012123456 7
12234567890123456789012345678901212345678901234567890123456789012123456
34567890123456789012345678901212345678901234567890123456789012123456
to “returning” 7
7
1
1234567890123456789012345678901212345678901234567890123456789012123456 7
1234567890123456789012345678901212345678901234567890123456789012123456 7
1234567890123456789012345678901212345678901234567890123456789012123456 7
12345678901234567890123456789012123456789012345678901234567890121234567
Copyright © 2005 Thomson Peterson’s, a part of The Thomson Corporaton 21
SAT is a registered trademark of the College Entrance Examination Board, which
was not involved in the production of and does not endorse this product.
12345678901234567890123456789012123456789012345678901234567890121234567
1234567890123456789012345678901212345678901234567890123456789012123456 7
12 34567890123456789012345678901212345678901234567890123456789012123456 7
1234567890123456789012345678901212345678901234567890123456789012123456
Questions 26–30 are based on the following 26. Which of the following is the revision of 7
12 34567890123456789012345678901212345678901234567890123456789012123456 7
12 34567890123456789012345678901212345678901234567890123456789012123456
passage. sentence 4 (reproduced below)? 7
1234567890123456789012345678901212345678901234567890123456789012123456 7
12 34567890123456789012345678901212345678901234567890123456789012123456 One of the officials from the literacy pro- 7
1234567890123456789012345678901212345678901234567890123456789012123456 7
12 34567890123456789012345678901212345678901234567890123456789012123456
(1) On my nineteenth birthday, I began my trip 7
12 34567890123456789012345678901212345678901234567890123456789012123456 gram I was working was there to meet me. 7
12 34567890123456789012345678901212345678901234567890123456789012123456 7
to Mali, West Africa. (2) Some 24 hours later I
12 34567890123456789012345678901212345678901234567890123456789012123456 7
1234567890123456789012345678901212345678901234567890123456789012123456
arrived in Bamako, the capital of Mali. (3) The (A) As it is now. 7
12 34567890123456789012345678901212345678901234567890123456789012123456 7
12 34567890123456789012345678901212345678901234567890123456789012123456 7
sun had set and the night was starless. (4) One (B) One of the literacy program I was
12 34567890123456789012345678901212345678901234567890123456789012123456 working’s officials was there to meet 7
12
2
34567890123456789012345678901212345678901234567890123456789012123456
of the officials from the literacy program I was
34567890123456789012345678901212345678901234567890123456789012123456
7
7
1 working was there to meet me. (5) After the me.
12 34567890123456789012345678901212345678901234567890123456789012123456 7
12 34567890123456789012345678901212345678901234567890123456789012123456
melee in the baggage claim, we proceeded to his (C) There, was one of the officials from 7
12 34567890123456789012345678901212345678901234567890123456789012123456 7
12 34567890123456789012345678901212345678901234567890123456789012123456
car. (6) Actually, it was a truck. (7) I was soon the literacy program I was working 7
1234567890123456789012345678901212345678901234567890123456789012123456 7
12 34567890123456789012345678901212345678901234567890123456789012123456
to learn that most people in Mali that had to meet me. 7
12 34567890123456789012345678901212345678901234567890123456789012123456 7
12 34567890123456789012345678901212345678901234567890123456789012123456
automobiles actually had trucks or SUVs. (8) (D) One of the officials from the literacy 7
12 34567890123456789012345678901212345678901234567890123456789012123456 7
2 34567890123456789012345678901212345678901234567890123456789012123456
1234567890123456789012345678901212345678901234567890123456789012123456
Apparently, there not just a convenience but a program where I worked had been 7
1234567890123456789012345678901212345678901234567890123456789012123456 7
1234567890123456789012345678901212345678901234567890123456789012123456
necessity when you live on the edge of the there to meet me. 7
1234567890123456789012345678901212345678901234567890123456789012123456 7
1234567890123456789012345678901212345678901234567890123456789012123456 77
Sahara. (9) I threw my bags into the bed of the (E) One of the officials from the literacy
1234567890123456789012345678901212345678901234567890123456789012123456 7
1 truck, and hopped in to the back of the cab. program where I would be working
12 34567890123456789012345678901212345678901234567890123456789012123456 7
12 34567890123456789012345678901212345678901234567890123456789012123456 7
(10) Riding to my welcome dinner, I stared out was there to meet me.
12 34567890123456789012345678901212345678901234567890123456789012123456 7
12 34567890123456789012345678901212345678901234567890123456789012123456
the windows of the truck and took in the city. 7
12 34567890123456789012345678901212345678901234567890123456789012123456 7
12 34567890123456789012345678901212345678901234567890123456789012123456 7
(11) It was truly a foreign land to me, and I 27. Which of the following is the best way to
12 34567890123456789012345678901212345678901234567890123456789012123456 revise sentence 7 (reproduced below)? 7
12 34567890123456789012345678901212345678901234567890123456789012123456
knew that I was an alien there. (12) “What am 7
2 34567890123456789012345678901212345678901234567890123456789012123456
1234567890123456789012345678901212345678901234567890123456789012123456 I was soon to learn that most people in 7
1234567890123456789012345678901212345678901234567890123456789012123456
I doing here?” I thought. 7
1234567890123456789012345678901212345678901234567890123456789012123456 Mali that had automobiles actually had 7
1234567890123456789012345678901212345678901234567890123456789012123456 77
(13) It is hard to believe but seven months
1 later I returned to the same airport along the trucks or SUVs.
12 34567890123456789012345678901212345678901234567890123456789012123456 7
12 34567890123456789012345678901212345678901234567890123456789012123456 7
1234567890123456789012345678901212345678901234567890123456789012123456
same road that I had traveled on that first night (A) Change “I was soon to learn” to “I 7
1234567890123456789012345678901212345678901234567890123456789012123456
in Bamako, and my perspective on the things 7
12
2
34567890123456789012345678901212345678901234567890123456789012123456 7
34567890123456789012345678901212345678901234567890123456789012123456
was soon learning”
7
1234567890123456789012345678901212345678901234567890123456789012123456
that I saw had completely changed. (14) The
1234567890123456789012345678901212345678901234567890123456789012123456 (B) Change “that had automobiles” to 7
1 landscape that had once seemed so desolate and 7
12 34567890123456789012345678901212345678901234567890123456789012123456 7 “who had automobiles”
12 34567890123456789012345678901212345678901234567890123456789012123456
lifeless now was the homeland of people that I 7
12 34567890123456789012345678901212345678901234567890123456789012123456 (C) Replace “or” with “and” 7
12 34567890123456789012345678901212345678901234567890123456789012123456
had come to love. (15) When I looked back at 7
1234567890123456789012345678901212345678901234567890123456789012123456 (D) Add commas after “Mali” and 7
1234567890123456789012345678901212345678901234567890123456789012123456
the capital, Bamako, fast receding on the 7
1234567890123456789012345678901212345678901234567890123456789012123456 “automobiles” 7
2 34567890123456789012345678901212345678901234567890123456789012123456
1234567890123456789012345678901212345678901234567890123456789012123456
horizon, I did not see a city foreboding and (E) Add an apostrophe to make “SUVs” 7
1234567890123456789012345678901212345678901234567890123456789012123456 7
1234567890123456789012345678901212345678901234567890123456789012123456
wild in its foreignness. (16) I saw the city which 7
1234567890123456789012345678901212345678901234567890123456789012123456 read “SUV’s“ 7
1 held so many dear friends. (17) I saw tea- 7
2 34567890123456789012345678901212345678901234567890123456789012123456
1234567890123456789012345678901212345678901234567890123456789012123456 7
1234567890123456789012345678901212345678901234567890123456789012123456
drinking sessions going late into the night. 7
1234567890123456789012345678901212345678901234567890123456789012123456 7
1234567890123456789012345678901212345678901234567890123456789012123456 77
(18) I saw the hospitality and open-heartedness
1 of the people of Mali. (19) The second time,
12 34567890123456789012345678901212345678901234567890123456789012123456 7
12 34567890123456789012345678901212345678901234567890123456789012123456 7
12 34567890123456789012345678901212345678901234567890123456789012123456
everything looked completely different, and I 7
12 34567890123456789012345678901212345678901234567890123456789012123456 7
1234567890123456789012345678901212345678901234567890123456789012123456 7
knew that it was I who had changed and not it.
1234567890123456789012345678901212345678901234567890123456789012123456 77
2 34567890123456789012345678901212345678901234567890123456789012123456
1
12 34567890123456789012345678901212345678901234567890123456789012123456 7
1234567890123456789012345678901212345678901234567890123456789012123456 7
12 34567890123456789012345678901212345678901234567890123456789012123456 7
1234567890123456789012345678901212345678901234567890123456789012123456 7
1234567890123456789012345678901212345678901234567890123456789012123456 7
1234567890123456789012345678901212345678901234567890123456789012123456 7
1234567890123456789012345678901212345678901234567890123456789012123456 7
12345678901234567890123456789012123456789012345678901234567890121234567
22 Copyright © 2005 Thomson Peterson’s, a part of The Thomson Corporaton
SAT is a registered trademark of the College Entrance Examination Board, which
was not involved in the production of and does not endorse this product.
12345678901234567890123456789012123456789012345678901234567890121234567
1234567890123456789012345678901212345678901234567890123456789012123456 7
12234567890123456789012345678901212345678901234567890123456789012123456 7
1 34567890123456789012345678901212345678901234567890123456789012123456
28. Sentence 13 (reproduced below) would 29. If you were to combine sentences 16–18 7
12234567890123456789012345678901212345678901234567890123456789012123456 7
1234567890123456789012345678901212345678901234567890123456789012123456
best be revised to which of the following (reproduced below) into one sentence, 7
1 34567890123456789012345678901212345678901234567890123456789012123456 7
12234567890123456789012345678901212345678901234567890123456789012123456
choices?
34567890123456789012345678901212345678901234567890123456789012123456
which of the following would be the best 7
7
1
12234567890123456789012345678901212345678901234567890123456789012123456 choice? 7
1234567890123456789012345678901212345678901234567890123456789012123456
It is hard to believe but seven months later 7
1234567890123456789012345678901212345678901234567890123456789012123456 7
1234567890123456789012345678901212345678901234567890123456789012123456
I returned to the same airport along the
34567890123456789012345678901212345678901234567890123456789012123456
I saw the city which held so many dear 7
1
12234567890123456789012345678901212345678901234567890123456789012123456
same road that I had traveled on that first friends. I saw tea-drinking sessions going 7 7
1234567890123456789012345678901212345678901234567890123456789012123456 7
1234567890123456789012345678901212345678901234567890123456789012123456
night in Bamako, and my perspective on late into the night. I saw the hospitality 7
1234567890123456789012345678901212345678901234567890123456789012123456
34567890123456789012345678901212345678901234567890123456789012123456
7
7
1 the things that I saw had completely and open-heartedness of the people of
12234567890123456789012345678901212345678901234567890123456789012123456 7
1234567890123456789012345678901212345678901234567890123456789012123456
changed. Mali. 7
1234567890123456789012345678901212345678901234567890123456789012123456 7
1234567890123456789012345678901212345678901234567890123456789012123456
(A) As it is now.
34567890123456789012345678901212345678901234567890123456789012123456(A) I saw the city which held so many 7
7
1
12234567890123456789012345678901212345678901234567890123456789012123456
(B) It is hard to believe, but seven dear friends; I saw tea-drinking 7
1234567890123456789012345678901212345678901234567890123456789012123456 7
1234567890123456789012345678901212345678901234567890123456789012123456
months later I returned to the same sessions going late into the night; I 7
1234567890123456789012345678901212345678901234567890123456789012123456 7
1234567890123456789012345678901212345678901234567890123456789012123456 77
34567890123456789012345678901212345678901234567890123456789012123456
airport along the same road that I saw the hospitality and open-
1234567890123456789012345678901212345678901234567890123456789012123456
1234567890123456789012345678901212345678901234567890123456789012123456
had traveled on that first night in heartedness of the people of Mali. 7
1234567890123456789012345678901212345678901234567890123456789012123456 7
1234567890123456789012345678901212345678901234567890123456789012123456 77
Bamako: my perspective on the (B) I saw the city which held so many
1234567890123456789012345678901212345678901234567890123456789012123456 dear friends, drinking tea into late in 7
1 things I saw had completely changed.
12234567890123456789012345678901212345678901234567890123456789012123456 7
1234567890123456789012345678901212345678901234567890123456789012123456
(C) It is hard to believe but seven months the night, and the hospitality and 7
1234567890123456789012345678901212345678901234567890123456789012123456 7
1234567890123456789012345678901212345678901234567890123456789012123456 77
34567890123456789012345678901212345678901234567890123456789012123456
later I returned to the same airport open-heartedness of the people of
1234567890123456789012345678901212345678901234567890123456789012123456
1234567890123456789012345678901212345678901234567890123456789012123456
along the same road that I had Mali. 7
1234567890123456789012345678901212345678901234567890123456789012123456 7
1234567890123456789012345678901212345678901234567890123456789012123456 77
traveled on that first night in (C) I saw the city which held so many
1234567890123456789012345678901212345678901234567890123456789012123456
1234567890123456789012345678901212345678901234567890123456789012123456
Bamako, and my perspective dear friends, I saw tea-drinking 7
1234567890123456789012345678901212345678901234567890123456789012123456 7
1234567890123456789012345678901212345678901234567890123456789012123456 77
completely changed on the things I sessions going late into the night, I
1
12234567890123456789012345678901212345678901234567890123456789012123456
saw. saw the hospitality and open- 7
1234567890123456789012345678901212345678901234567890123456789012123456 7
(D) It is hard to believe, but seven
1 34567890123456789012345678901212345678901234567890123456789012123456 7 heartedness of the people of Mali.
1234567890123456789012345678901212345678901234567890123456789012123456
months later, when I returned to the (D) I saw the city which held so many 7
12234567890123456789012345678901212345678901234567890123456789012123456
34567890123456789012345678901212345678901234567890123456789012123456
7
7
1234567890123456789012345678901212345678901234567890123456789012123456
same airport along the same road dear friends, tea-drinking sessions
1234567890123456789012345678901212345678901234567890123456789012123456 7
1 that I had traveled on that first night going late into the night, the hospi- 7
12234567890123456789012345678901212345678901234567890123456789012123456 7
1234567890123456789012345678901212345678901234567890123456789012123456
in Bamako, my perspective on the tality and open-heartedness of the 7
34567890123456789012345678901212345678901234567890123456789012123456
1234567890123456789012345678901212345678901234567890123456789012123456 7
1234567890123456789012345678901212345678901234567890123456789012123456
things I saw had completely changed. people of Mali. 7
1 7
1234567890123456789012345678901212345678901234567890123456789012123456
(E) It is hard to believe, but seven (E) I saw the city which held so many 7
1234567890123456789012345678901212345678901234567890123456789012123456 7
1234567890123456789012345678901212345678901234567890123456789012123456 77
2 34567890123456789012345678901212345678901234567890123456789012123456
months later I returned to the same dear friends: tea-drinking sessions
1234567890123456789012345678901212345678901234567890123456789012123456 7
1234567890123456789012345678901212345678901234567890123456789012123456
airport along the same road that I going late into the night, the hospi-
7
1234567890123456789012345678901212345678901234567890123456789012123456 7
1 had traveled on that first night in tality and open-heartedness of the
2 34567890123456789012345678901212345678901234567890123456789012123456
1234567890123456789012345678901212345678901234567890123456789012123456 7
1234567890123456789012345678901212345678901234567890123456789012123456
Bamako, and my perspective on the people of Mali. 7
1234567890123456789012345678901212345678901234567890123456789012123456 7
1234567890123456789012345678901212345678901234567890123456789012123456 77
things that I saw having completely
1
12234567890123456789012345678901212345678901234567890123456789012123456
changed. 7
1234567890123456789012345678901212345678901234567890123456789012123456 7
1234567890123456789012345678901212345678901234567890123456789012123456 7
1234567890123456789012345678901212345678901234567890123456789012123456 7
1 34567890123456789012345678901212345678901234567890123456789012123456 7
1234567890123456789012345678901212345678901234567890123456789012123456 77
2 34567890123456789012345678901212345678901234567890123456789012123456
1
12234567890123456789012345678901212345678901234567890123456789012123456 7
1 34567890123456789012345678901212345678901234567890123456789012123456 7
12234567890123456789012345678901212345678901234567890123456789012123456 7
1 34567890123456789012345678901212345678901234567890123456789012123456 7
1234567890123456789012345678901212345678901234567890123456789012123456 7
1234567890123456789012345678901212345678901234567890123456789012123456 7
1234567890123456789012345678901212345678901234567890123456789012123456 7
12345678901234567890123456789012123456789012345678901234567890121234567
Copyright © 2005 Thomson Peterson’s, a part of The Thomson Corporaton 23
SAT is a registered trademark of the College Entrance Examination Board, which
was not involved in the production of and does not endorse this product.
12345678901234567890123456789012123456789012345678901234567890121234567
1234567890123456789012345678901212345678901234567890123456789012123456 7
12 34567890123456789012345678901212345678901234567890123456789012123456 7
1234567890123456789012345678901212345678901234567890123456789012123456
30. Which of the following must be done to 7
12 34567890123456789012345678901212345678901234567890123456789012123456 7
12 34567890123456789012345678901212345678901234567890123456789012123456
sentence 8 (reproduced below) to make it 7
1234567890123456789012345678901212345678901234567890123456789012123456 7
12
2
34567890123456789012345678901212345678901234567890123456789012123456 7
conform to the rules of written English?
34567890123456789012345678901212345678901234567890123456789012123456 7
1
12 34567890123456789012345678901212345678901234567890123456789012123456 7
12 34567890123456789012345678901212345678901234567890123456789012123456 7
Apparently, there not just a convenience
12 34567890123456789012345678901212345678901234567890123456789012123456 7
12
2
34567890123456789012345678901212345678901234567890123456789012123456
but a necessity when you live on the edge
34567890123456789012345678901212345678901234567890123456789012123456
7
7
1
12 34567890123456789012345678901212345678901234567890123456789012123456
of the Sahara. 7
12 34567890123456789012345678901212345678901234567890123456789012123456 7
12 34567890123456789012345678901212345678901234567890123456789012123456 7
12
2
34567890123456789012345678901212345678901234567890123456789012123456
(A) Eliminate the comma after “Appar-
34567890123456789012345678901212345678901234567890123456789012123456
7
7
1 ently”
12 34567890123456789012345678901212345678901234567890123456789012123456 7
12 34567890123456789012345678901212345678901234567890123456789012123456
(B) Change “there” to “they are” 7
12 34567890123456789012345678901212345678901234567890123456789012123456 7
12 34567890123456789012345678901212345678901234567890123456789012123456
(C) Add commas after “convenience” 7
1234567890123456789012345678901212345678901234567890123456789012123456 7
12 34567890123456789012345678901212345678901234567890123456789012123456
and “necessity” 7
12 34567890123456789012345678901212345678901234567890123456789012123456 7
12 34567890123456789012345678901212345678901234567890123456789012123456
(D) Change “you live” to “one lives” 7
12 34567890123456789012345678901212345678901234567890123456789012123456 7
1234567890123456789012345678901212345678901234567890123456789012123456 77
2 34567890123456789012345678901212345678901234567890123456789012123456
(E) Add “Desert” after “Sahara“
1234567890123456789012345678901212345678901234567890123456789012123456 7
1234567890123456789012345678901212345678901234567890123456789012123456 7
1234567890123456789012345678901212345678901234567890123456789012123456 7
1234567890123456789012345678901212345678901234567890123456789012123456 7
1234567890123456789012345678901212345678901234567890123456789012123456 7
1
12 34567890123456789012345678901212345678901234567890123456789012123456 7
12 34567890123456789012345678901212345678901234567890123456789012123456 7
12 34567890123456789012345678901212345678901234567890123456789012123456 7
1234567890123456789012345678901212345678901234567890123456789012123456
2 7
1234567890123456789012345678901212345678901234567890123456789012123456 7
1234567890123456789012345678901212345678901234567890123456789012123456 7
1234567890123456789012345678901212345678901234567890123456789012123456 7
1234567890123456789012345678901212345678901234567890123456789012123456 7
1234567890123456789012345678901212345678901234567890123456789012123456 77
34567890123456789012345678901212345678901234567890123456789012123456
1234567890123456789012345678901212345678901234567890123456789012123456 7
1234567890123456789012345678901212345678901234567890123456789012123456 7
1234567890123456789012345678901212345678901234567890123456789012123456 7
1
12 34567890123456789012345678901212345678901234567890123456789012123456 7
12 34567890123456789012345678901212345678901234567890123456789012123456 7
1234567890123456789012345678901212345678901234567890123456789012123456 7
1234567890123456789012345678901212345678901234567890123456789012123456 7
12 34567890123456789012345678901212345678901234567890123456789012123456 7
12 34567890123456789012345678901212345678901234567890123456789012123456 7
12 34567890123456789012345678901212345678901234567890123456789012123456 7
1234567890123456789012345678901212345678901234567890123456789012123456 7
12 34567890123456789012345678901212345678901234567890123456789012123456 7
12 34567890123456789012345678901212345678901234567890123456789012123456 7
1234567890123456789012345678901212345678901234567890123456789012123456
2 7
1234567890123456789012345678901212345678901234567890123456789012123456 7
1 34567890123456789012345678901212345678901234567890123456789012123456 7
1234567890123456789012345678901212345678901234567890123456789012123456 7
1234567890123456789012345678901212345678901234567890123456789012123456 7
12 34567890123456789012345678901212345678901234567890123456789012123456 7
12 34567890123456789012345678901212345678901234567890123456789012123456 7
12 34567890123456789012345678901212345678901234567890123456789012123456 7
12 34567890123456789012345678901212345678901234567890123456789012123456 7
1234567890123456789012345678901212345678901234567890123456789012123456 7
1234567890123456789012345678901212345678901234567890123456789012123456
2 7
1234567890123456789012345678901212345678901234567890123456789012123456 7
1234567890123456789012345678901212345678901234567890123456789012123456 7
1234567890123456789012345678901212345678901234567890123456789012123456 7
1 34567890123456789012345678901212345678901234567890123456789012123456 7
1234567890123456789012345678901212345678901234567890123456789012123456 77
2 34567890123456789012345678901212345678901234567890123456789012123456
1234567890123456789012345678901212345678901234567890123456789012123456 7
1234567890123456789012345678901212345678901234567890123456789012123456 7
1234567890123456789012345678901212345678901234567890123456789012123456 7
1
12 34567890123456789012345678901212345678901234567890123456789012123456 7
1234567890123456789012345678901212345678901234567890123456789012123456 7
12 34567890123456789012345678901212345678901234567890123456789012123456 7
1234567890123456789012345678901212345678901234567890123456789012123456 7
12 34567890123456789012345678901212345678901234567890123456789012123456 7
STOP
1234567890123456789012345678901212345678901234567890123456789012123456
Do not proceed to the next section until time is up.
1234567890123456789012345678901212345678901234567890123456789012123456 7
7
1234567890123456789012345678901212345678901234567890123456789012123456 7
1234567890123456789012345678901212345678901234567890123456789012123456 7
12345678901234567890123456789012123456789012345678901234567890121234567
24 Copyright © 2005 Thomson Peterson’s, a part of The Thomson Corporaton
SAT is a registered trademark of the College Entrance Examination Board, which
was not involved in the production of and does not endorse this product.
Section 4
27 Questions j Time—25 Minutes

12345678901234567890123456789012123456789012345678901234567890121234567
12234567890123456789012345678901212345678901234567890123456789012123456 7
1234567890123456789012345678901212345678901234567890123456789012123456
34567890123456789012345678901212345678901234567890123456789012123456
7
7
1
12234567890123456789012345678901212345678901234567890123456789012123456 7
1234567890123456789012345678901212345678901234567890123456789012123456
Directions: Each sentence below has either one or two blanks in it and is followed by five 7
1234567890123456789012345678901212345678901234567890123456789012123456
choices, labeled (A) through (E). These choices represent words or phrases that have been left 7
1234567890123456789012345678901212345678901234567890123456789012123456 7
34567890123456789012345678901212345678901234567890123456789012123456
1234567890123456789012345678901212345678901234567890123456789012123456
out. Choose the word or phrase that, if inserted into the sentence, would best fit the meaning of 7
1234567890123456789012345678901212345678901234567890123456789012123456 7
1234567890123456789012345678901212345678901234567890123456789012123456
the sentence as a whole. 7
1234567890123456789012345678901212345678901234567890123456789012123456 77
1234567890123456789012345678901212345678901234567890123456789012123456
1234567890123456789012345678901212345678901234567890123456789012123456
Example: 7
1 7
12234567890123456789012345678901212345678901234567890123456789012123456 7
1234567890123456789012345678901212345678901234567890123456789012123456
Canine massage is a veterinary technique 7
1234567890123456789012345678901212345678901234567890123456789012123456 7
1234567890123456789012345678901212345678901234567890123456789012123456 77
34567890123456789012345678901212345678901234567890123456789012123456
for calming dogs that are extremely __________.
1234567890123456789012345678901212345678901234567890123456789012123456 7
1234567890123456789012345678901212345678901234567890123456789012123456
1234567890123456789012345678901212345678901234567890123456789012123456
(A) inept 7
1234567890123456789012345678901212345678901234567890123456789012123456 7
1234567890123456789012345678901212345678901234567890123456789012123456 77
(B) disciplined
1234567890123456789012345678901212345678901234567890123456789012123456
(C) controlled
1234567890123456789012345678901212345678901234567890123456789012123456 7
1234567890123456789012345678901212345678901234567890123456789012123456
(D) stressed 7
1 7
2 34567890123456789012345678901212345678901234567890123456789012123456
1234567890123456789012345678901212345678901234567890123456789012123456
(E) restrained 7
1234567890123456789012345678901212345678901234567890123456789012123456 7
1 7
1234567890123456789012345678901212345678901234567890123456789012123456
A
O Þ
Þ B
O Þ C
O Þ O ÞE 7
12234567890123456789012345678901212345678901234567890123456789012123456 7
1234567890123456789012345678901212345678901234567890123456789012123456 77
34567890123456789012345678901212345678901234567890123456789012123456
1234567890123456789012345678901212345678901234567890123456789012123456 7
1
12234567890123456789012345678901212345678901234567890123456789012123456 7
1234567890123456789012345678901212345678901234567890123456789012123456
1. The professor’s oldest colleague was 2. The new team member’s __________ was 7
34567890123456789012345678901212345678901234567890123456789012123456
1234567890123456789012345678901212345678901234567890123456789012123456
an encouragement to the rest of the team, 7
7
1234567890123456789012345678901212345678901234567890123456789012123456
selected to give the __________ at the
7
1
1234567890123456789012345678901212345678901234567890123456789012123456
funeral. who had become __________ by the string 7
1234567890123456789012345678901212345678901234567890123456789012123456 7
12234567890123456789012345678901212345678901234567890123456789012123456
of defeats. 7
1234567890123456789012345678901212345678901234567890123456789012123456
(A) eulogy 7
1234567890123456789012345678901212345678901234567890123456789012123456 7
1234567890123456789012345678901212345678901234567890123456789012123456
(B) elegy (A) enthusiasm. .elated 7
1 34567890123456789012345678901212345678901234567890123456789012123456 7
1234567890123456789012345678901212345678901234567890123456789012123456 77
2 34567890123456789012345678901212345678901234567890123456789012123456
(C) epigraph (B) vigor. .inundated
1234567890123456789012345678901212345678901234567890123456789012123456 7
1234567890123456789012345678901212345678901234567890123456789012123456
(D) eponymy (C) ebullience. .dispirited
7
1234567890123456789012345678901212345678901234567890123456789012123456 7
1 (E) epitaph (D) dourness. .undone
2 34567890123456789012345678901212345678901234567890123456789012123456
1234567890123456789012345678901212345678901234567890123456789012123456
(E) excessiveness. .downcast 7
1234567890123456789012345678901212345678901234567890123456789012123456 7
1234567890123456789012345678901212345678901234567890123456789012123456 77
1234567890123456789012345678901212345678901234567890123456789012123456 7
1
12234567890123456789012345678901212345678901234567890123456789012123456 7
1 34567890123456789012345678901212345678901234567890123456789012123456 7
12234567890123456789012345678901212345678901234567890123456789012123456 7
1 34567890123456789012345678901212345678901234567890123456789012123456 7
12234567890123456789012345678901212345678901234567890123456789012123456 7
1 34567890123456789012345678901212345678901234567890123456789012123456 7
1234567890123456789012345678901212345678901234567890123456789012123456 7
1234567890123456789012345678901212345678901234567890123456789012123456 7
1234567890123456789012345678901212345678901234567890123456789012123456 7
12345678901234567890123456789012123456789012345678901234567890121234567
12345678901234567890123456789012123456789012345678901234567890121234567
1234567890123456789012345678901212345678901234567890123456789012123456 7
12 34567890123456789012345678901212345678901234567890123456789012123456 7
1234567890123456789012345678901212345678901234567890123456789012123456
3. By the end of the campaign both candi- 7. The outcome of the race seemed 7
12 34567890123456789012345678901212345678901234567890123456789012123456 7
12 34567890123456789012345678901212345678901234567890123456789012123456
dates had resorted to __________ the __________ before the leader’s misstep on 7
1234567890123456789012345678901212345678901234567890123456789012123456 7
12
2
34567890123456789012345678901212345678901234567890123456789012123456
other. the final leg gave her competitors a(n)
34567890123456789012345678901212345678901234567890123456789012123456
7
7
1
12 34567890123456789012345678901212345678901234567890123456789012123456
__________ of winning the title. 7
12 34567890123456789012345678901212345678901234567890123456789012123456 7
(A) commending
12 34567890123456789012345678901212345678901234567890123456789012123456 7
12
2
34567890123456789012345678901212345678901234567890123456789012123456
(B) denigrating (A) dubious. .prospect
34567890123456789012345678901212345678901234567890123456789012123456
7
7
1
12 34567890123456789012345678901212345678901234567890123456789012123456
(C) mollifying (B) inevitable. .hope 7
12 34567890123456789012345678901212345678901234567890123456789012123456 7
12 34567890123456789012345678901212345678901234567890123456789012123456 7
(D) conceding (C) indubitable. .air
12 34567890123456789012345678901212345678901234567890123456789012123456 7
1234567890123456789012345678901212345678901234567890123456789012123456
(E) swindling (D) assured. .expectation 7
12 34567890123456789012345678901212345678901234567890123456789012123456 7
12 34567890123456789012345678901212345678901234567890123456789012123456 7
(E) partial. .endeavor
12 34567890123456789012345678901212345678901234567890123456789012123456
4. The cat __________ crept across the lawn, 7
12
2
34567890123456789012345678901212345678901234567890123456789012123456
34567890123456789012345678901212345678901234567890123456789012123456
7
7
1 gracefully __________ the dog. 8. Though the new pharmaceutical regime
12 34567890123456789012345678901212345678901234567890123456789012123456 7
12 34567890123456789012345678901212345678901234567890123456789012123456
was intended to be beneficial, its actual 7
12 34567890123456789012345678901212345678901234567890123456789012123456
(A) felicitously. .enticing 7
12 34567890123456789012345678901212345678901234567890123456789012123456
effect was __________, a result the 7
2 34567890123456789012345678901212345678901234567890123456789012123456
1234567890123456789012345678901212345678901234567890123456789012123456
(B) swiftly. .defeating 7
1234567890123456789012345678901212345678901234567890123456789012123456
medical community __________. 7
1234567890123456789012345678901212345678901234567890123456789012123456 77
(C) acrobatically. .apprehending
1234567890123456789012345678901212345678901234567890123456789012123456
1234567890123456789012345678901212345678901234567890123456789012123456
(D) maladroitly. .undermining (A) harmful. .heralded 7
1234567890123456789012345678901212345678901234567890123456789012123456 7
1 (E) deftly. .eluding (B) abundant. .castigated 7
12 34567890123456789012345678901212345678901234567890123456789012123456 7
12 34567890123456789012345678901212345678901234567890123456789012123456 7
(C) fortuitous. .ignored
12 34567890123456789012345678901212345678901234567890123456789012123456
5. The storyteller’s __________ anecdotes 7
12 34567890123456789012345678901212345678901234567890123456789012123456
(D) detrimental. .lamented 7
12 34567890123456789012345678901212345678901234567890123456789012123456
earned her the __________ attention of 7
12 34567890123456789012345678901212345678901234567890123456789012123456
(E) negative. .projected 7
12 34567890123456789012345678901212345678901234567890123456789012123456
the crowd. 7
12 34567890123456789012345678901212345678901234567890123456789012123456 7
2 34567890123456789012345678901212345678901234567890123456789012123456
1234567890123456789012345678901212345678901234567890123456789012123456
9. The life of the lightening bug is 7
1234567890123456789012345678901212345678901234567890123456789012123456
(A) compelling. .rapt
__________ to human eyes: They live only 7
7
1234567890123456789012345678901212345678901234567890123456789012123456
1234567890123456789012345678901212345678901234567890123456789012123456
(B) pointed. .spellbound 7
1 twenty-four hours. 7
1234567890123456789012345678901212345678901234567890123456789012123456 77
2 34567890123456789012345678901212345678901234567890123456789012123456
(C) moribund. .lucid
1234567890123456789012345678901212345678901234567890123456789012123456 7
1 (D) poignant. .abrasive (A) ludicrous
1234567890123456789012345678901212345678901234567890123456789012123456 7
12
2
34567890123456789012345678901212345678901234567890123456789012123456 7
(E) meandering. .distracted (B) ephemeral
34567890123456789012345678901212345678901234567890123456789012123456 7
1234567890123456789012345678901212345678901234567890123456789012123456
1234567890123456789012345678901212345678901234567890123456789012123456
(C) epic 7
1 6. The bill became bogged down in a(n) 7
12 34567890123456789012345678901212345678901234567890123456789012123456 7
(D) ecstatic
12 34567890123456789012345678901212345678901234567890123456789012123456
__________ of contentious issues in a 7
12 34567890123456789012345678901212345678901234567890123456789012123456
(E) incandescent 7
12 34567890123456789012345678901212345678901234567890123456789012123456
Senate subcommittee. 7
1234567890123456789012345678901212345678901234567890123456789012123456 7
1234567890123456789012345678901212345678901234567890123456789012123456
10. The kangaroo species __________ in the 7
1234567890123456789012345678901212345678901234567890123456789012123456
(A) marsh 7
2 34567890123456789012345678901212345678901234567890123456789012123456
1234567890123456789012345678901212345678901234567890123456789012123456
new environment where there was an 7
1234567890123456789012345678901212345678901234567890123456789012123456
(B) sequence 7
1234567890123456789012345678901212345678901234567890123456789012123456
abundant supply of food and a(n) 7
1234567890123456789012345678901212345678901234567890123456789012123456
(C) iota 7
1 __________ of predators. 7
1234567890123456789012345678901212345678901234567890123456789012123456 77
2 34567890123456789012345678901212345678901234567890123456789012123456
(D) conundrum
1234567890123456789012345678901212345678901234567890123456789012123456 7
1234567890123456789012345678901212345678901234567890123456789012123456
(E) quagmire (A) stagnated. .excess
7
1234567890123456789012345678901212345678901234567890123456789012123456 7
1 (B) bolstered. .paucity
2 34567890123456789012345678901212345678901234567890123456789012123456
1234567890123456789012345678901212345678901234567890123456789012123456 7
1234567890123456789012345678901212345678901234567890123456789012123456
(C) exploded. .abundance 7
1234567890123456789012345678901212345678901234567890123456789012123456 7
1234567890123456789012345678901212345678901234567890123456789012123456 77
(D) flagged. .absence
1 (E) flourished. .dearth
12 34567890123456789012345678901212345678901234567890123456789012123456 7
1234567890123456789012345678901212345678901234567890123456789012123456 7
12 34567890123456789012345678901212345678901234567890123456789012123456 7
1234567890123456789012345678901212345678901234567890123456789012123456 7
12 34567890123456789012345678901212345678901234567890123456789012123456 7
1234567890123456789012345678901212345678901234567890123456789012123456 7
1234567890123456789012345678901212345678901234567890123456789012123456 7
1234567890123456789012345678901212345678901234567890123456789012123456 7
1234567890123456789012345678901212345678901234567890123456789012123456 7
12345678901234567890123456789012123456789012345678901234567890121234567
26 Copyright © 2005 Thomson Peterson’s, a part of The Thomson Corporaton
SAT is a registered trademark of the College Entrance Examination Board, which
was not involved in the production of and does not endorse this product.
12345678901234567890123456789012123456789012345678901234567890121234567
1234567890123456789012345678901212345678901234567890123456789012123456 7
12234567890123456789012345678901212345678901234567890123456789012123456 7
1 34567890123456789012345678901212345678901234567890123456789012123456
11. With her speech, the politician attempted 15. The senior official __________ at the 7
12234567890123456789012345678901212345678901234567890123456789012123456 7
1234567890123456789012345678901212345678901234567890123456789012123456
to __________ the fears of the __________ insinuation that his country’s international 7
1 34567890123456789012345678901212345678901234567890123456789012123456 7
12234567890123456789012345678901212345678901234567890123456789012123456
citizens. trade policies were directly __________ the 7
34567890123456789012345678901212345678901234567890123456789012123456 7
1
12234567890123456789012345678901212345678901234567890123456789012123456
region’s economic woes. 7
1234567890123456789012345678901212345678901234567890123456789012123456
(A) intensify. .disingenuous 7
1234567890123456789012345678901212345678901234567890123456789012123456 7
1234567890123456789012345678901212345678901234567890123456789012123456
(B) ignore. .alarmed (A) balked.
34567890123456789012345678901212345678901234567890123456789012123456
.responsible for 7
7
1
12234567890123456789012345678901212345678901234567890123456789012123456
(C) assuage. .concerned (B) wrinkled. .at fault for 7
1234567890123456789012345678901212345678901234567890123456789012123456 7
1234567890123456789012345678901212345678901234567890123456789012123456
(D) quell. .disaffected (C) staggered. .inhibiting 7
1234567890123456789012345678901212345678901234567890123456789012123456
34567890123456789012345678901212345678901234567890123456789012123456
7
7
1 (E) exploit. .serene (D) blundered. .implicated in
12234567890123456789012345678901212345678901234567890123456789012123456 7
1234567890123456789012345678901212345678901234567890123456789012123456
(E) riled. .accountable to 7
1234567890123456789012345678901212345678901234567890123456789012123456
12. The fencing champion was __________ 7
1234567890123456789012345678901212345678901234567890123456789012123456
34567890123456789012345678901212345678901234567890123456789012123456
7
7
1 with her rapier, but in most other sports
12234567890123456789012345678901212345678901234567890123456789012123456
Questions 16–27 are based on the following 7
1234567890123456789012345678901212345678901234567890123456789012123456
she was rather __________. 7
1234567890123456789012345678901212345678901234567890123456789012123456
passage. 7
1234567890123456789012345678901212345678901234567890123456789012123456 7
34567890123456789012345678901212345678901234567890123456789012123456
(A) adroit. .awkward
The following passage was written by John 7
1234567890123456789012345678901212345678901234567890123456789012123456
1234567890123456789012345678901212345678901234567890123456789012123456 7
1234567890123456789012345678901212345678901234567890123456789012123456
(B) adept. .lithe 7
1234567890123456789012345678901212345678901234567890123456789012123456
Janovec, an ecologist who has worked in the Los 7
1234567890123456789012345678901212345678901234567890123456789012123456
(C) tenacious. .passable 7
1234567890123456789012345678901212345678901234567890123456789012123456
Amigos watershed in Peru. 7
1 (D) incompetent. .clumsy 7
12234567890123456789012345678901212345678901234567890123456789012123456
Line The Amazonian wilderness harbors the 7
1234567890123456789012345678901212345678901234567890123456789012123456
(E) deft. .skillful 7
1234567890123456789012345678901212345678901234567890123456789012123456
greatest number of species on this planet 7
1234567890123456789012345678901212345678901234567890123456789012123456 77
34567890123456789012345678901212345678901234567890123456789012123456
1234567890123456789012345678901212345678901234567890123456789012123456
13. Jane Goodall was at first a(n) __________ and is an irreplaceable resource for
1234567890123456789012345678901212345678901234567890123456789012123456 7
1234567890123456789012345678901212345678901234567890123456789012123456
in her field, but since then she has received present and future generations. Amazo- 7
1234567890123456789012345678901212345678901234567890123456789012123456 7
1234567890123456789012345678901212345678901234567890123456789012123456 77
many accolades for her work. (5) nia is crucial for maintaining global
1234567890123456789012345678901212345678901234567890123456789012123456
1234567890123456789012345678901212345678901234567890123456789012123456
climate and genetic resources, and its 7
1234567890123456789012345678901212345678901234567890123456789012123456
(A) acolyte 7
1 forest and rivers provide vital sources of 7
2 34567890123456789012345678901212345678901234567890123456789012123456
1234567890123456789012345678901212345678901234567890123456789012123456
(B) maverick 7
1234567890123456789012345678901212345678901234567890123456789012123456
food, building materials, pharmaceuti- 7
1 (C) luminary 7
1234567890123456789012345678901212345678901234567890123456789012123456
cals, and water needed by wildlife and 7
12234567890123456789012345678901212345678901234567890123456789012123456
(D) charlatan
34567890123456789012345678901212345678901234567890123456789012123456
7
7
1234567890123456789012345678901212345678901234567890123456789012123456
(10) humanity.
1234567890123456789012345678901212345678901234567890123456789012123456
(E) miser 7
1 The Los Amigos watershed in the 7
12234567890123456789012345678901212345678901234567890123456789012123456 7
1234567890123456789012345678901212345678901234567890123456789012123456
14. Alston was impressed by the philosopher’s state of Madre de Dios, southeastern 7
34567890123456789012345678901212345678901234567890123456789012123456
1234567890123456789012345678901212345678901234567890123456789012123456 7
1234567890123456789012345678901212345678901234567890123456789012123456
lecture, but Mario thought the lecture was Peru, is representative of the pristine 7
1 7
1234567890123456789012345678901212345678901234567890123456789012123456
better characterized as __________ than as lowland moist forest once found 7
1234567890123456789012345678901212345678901234567890123456789012123456 7
1234567890123456789012345678901212345678901234567890123456789012123456 77
2 34567890123456789012345678901212345678901234567890123456789012123456
erudite. (15) throughout most of upper Amazonian
1234567890123456789012345678901212345678901234567890123456789012123456
South America. Threats to tropical 7
1234567890123456789012345678901212345678901234567890123456789012123456 7
1234567890123456789012345678901212345678901234567890123456789012123456
(A) translucent forests occur in the form of fishing,
7
1
2 34567890123456789012345678901212345678901234567890123456789012123456
hunting, gold mining, timber extraction, 7
1234567890123456789012345678901212345678901234567890123456789012123456
(B) recondite
1234567890123456789012345678901212345678901234567890123456789012123456 7
1234567890123456789012345678901212345678901234567890123456789012123456
(C) impeccable impending road construction, and 7
1234567890123456789012345678901212345678901234567890123456789012123456 7
1 (D) specious (20) slash-and-burn agriculture. The Los 7
2 34567890123456789012345678901212345678901234567890123456789012123456
1234567890123456789012345678901212345678901234567890123456789012123456 7
1234567890123456789012345678901212345678901234567890123456789012123456
(E) fictitious Amigos watershed, consisting of 1.6 7
1234567890123456789012345678901212345678901234567890123456789012123456
million hectares (3.95 million acres), still 7
7
1234567890123456789012345678901212345678901234567890123456789012123456 7
1
12234567890123456789012345678901212345678901234567890123456789012123456
offers the increasingly scarce opportunity 7
1 34567890123456789012345678901212345678901234567890123456789012123456 7
12234567890123456789012345678901212345678901234567890123456789012123456
to study rainforest as it was before the 7
1 34567890123456789012345678901212345678901234567890123456789012123456 7
12234567890123456789012345678901212345678901234567890123456789012123456
(25) disruptive
34567890123456789012345678901212345678901234567890123456789012123456
encroachment of modern 7
7
1
1234567890123456789012345678901212345678901234567890123456789012123456
human civilization. Because of its 7
1234567890123456789012345678901212345678901234567890123456789012123456 77
1234567890123456789012345678901212345678901234567890123456789012123456
12345678901234567890123456789012123456789012345678901234567890121234567
Copyright © 2005 Thomson Peterson’s, a part of The Thomson Corporaton 27
SAT is a registered trademark of the College Entrance Examination Board, which
was not involved in the production of and does not endorse this product.
12345678901234567890123456789012123456789012345678901234567890121234567
1234567890123456789012345678901212345678901234567890123456789012123456 7
12 34567890123456789012345678901212345678901234567890123456789012123456 7
1234567890123456789012345678901212345678901234567890123456789012123456
relatively pristine condition and the Amigos area and the Madre de Dios 7
12 34567890123456789012345678901212345678901234567890123456789012123456 7
12 34567890123456789012345678901212345678901234567890123456789012123456
immediate need to justify it as a conser- (70) watershed in general. 7
1234567890123456789012345678901212345678901234567890123456789012123456 7
12
2
34567890123456789012345678901212345678901234567890123456789012123456 7
vation zone, this area deserves intensive,
34567890123456789012345678901212345678901234567890123456789012123456
With support from the Amazon
7
1 (30) long-term projects aimed at botanical
12 34567890123456789012345678901212345678901234567890123456789012123456
Conservation Association, and in 7
12 34567890123456789012345678901212345678901234567890123456789012123456 7
12 34567890123456789012345678901212345678901234567890123456789012123456 7
training, ecotourism, biological inven- collaboration with U.S. and Peruvian
12 34567890123456789012345678901212345678901234567890123456789012123456
colleagues, the Botany of the Los Amigos 7
1234567890123456789012345678901212345678901234567890123456789012123456
tory, and information synthesis. 7
12 34567890123456789012345678901212345678901234567890123456789012123456 7
12 34567890123456789012345678901212345678901234567890123456789012123456 7
On July 24, 2001, the government of (75) project has been initiated. At Los
12 34567890123456789012345678901212345678901234567890123456789012123456 7
12
2
34567890123456789012345678901212345678901234567890123456789012123456
Peru and the Amazon Conservation Amigos, we are attempting to develop a
34567890123456789012345678901212345678901234567890123456789012123456
7
7
1 (35) Association signed a contractual agree-
12 34567890123456789012345678901212345678901234567890123456789012123456
system of preservation, sustainability, 7
12 34567890123456789012345678901212345678901234567890123456789012123456 7
12 34567890123456789012345678901212345678901234567890123456789012123456 7
ment creating the first long-term perma- and scientific research; a marriage
12 34567890123456789012345678901212345678901234567890123456789012123456
between various disciplines, from human 7
1234567890123456789012345678901212345678901234567890123456789012123456
nently renewable conservation conces- 7
12 34567890123456789012345678901212345678901234567890123456789012123456 7
12 34567890123456789012345678901212345678901234567890123456789012123456 7
sion. To our knowledge this is the first (80) ecology to economic botany, product
12 34567890123456789012345678901212345678901234567890123456789012123456
such agreement to be implemented in the 7
12 34567890123456789012345678901212345678901234567890123456789012123456
marketing to forest management. The 7
2 34567890123456789012345678901212345678901234567890123456789012123456
1234567890123456789012345678901212345678901234567890123456789012123456
world. The conservation concession complexity of the ecosystem will best be 7
(40)
1234567890123456789012345678901212345678901234567890123456789012123456 7
1234567890123456789012345678901212345678901234567890123456789012123456
protects 340,000 acres of old-growth understood through a multidisciplinary 7
1234567890123456789012345678901212345678901234567890123456789012123456 77
1234567890123456789012345678901212345678901234567890123456789012123456
Amazonian forest in the Los Amigos approach, and improved understanding
1234567890123456789012345678901212345678901234567890123456789012123456 7
1 watershed, which is located in southeast- (85) of the complexity will lead to better 7
12 34567890123456789012345678901212345678901234567890123456789012123456 7
12 34567890123456789012345678901212345678901234567890123456789012123456
ern Peru. This watershed protects the management. The future of these forests 7
12 34567890123456789012345678901212345678901234567890123456789012123456 7
1234567890123456789012345678901212345678901234567890123456789012123456 77
2 34567890123456789012345678901212345678901234567890123456789012123456
(45) eastern flank of Manu National Park and will depend on sustainable management
1234567890123456789012345678901212345678901234567890123456789012123456
1234567890123456789012345678901212345678901234567890123456789012123456
is part of the lowland forest corridor that and development of alternative practices 7
1234567890123456789012345678901212345678901234567890123456789012123456 7
1234567890123456789012345678901212345678901234567890123456789012123456 77
links it to Bahuaja-Sonene National Park. and products that do not require
1234567890123456789012345678901212345678901234567890123456789012123456
1234567890123456789012345678901212345678901234567890123456789012123456
The Los Amigos conservation concession (90) irreversible destruction. 7
1234567890123456789012345678901212345678901234567890123456789012123456 7
1234567890123456789012345678901212345678901234567890123456789012123456 77
will serve as a mechanism for the The botanical project will provide a
1 (50) development of a regional center of
12 34567890123456789012345678901212345678901234567890123456789012123456
foundation of information that is 7
12 34567890123456789012345678901212345678901234567890123456789012123456 7
1234567890123456789012345678901212345678901234567890123456789012123456 7
excellence in natural forest management essential to other programs at Los
1234567890123456789012345678901212345678901234567890123456789012123456 7
12
2
34567890123456789012345678901212345678901234567890123456789012123456
and biodiversity science. Amigos. By combining botanical studies
34567890123456789012345678901212345678901234567890123456789012123456
7
7
1234567890123456789012345678901212345678901234567890123456789012123456
1234567890123456789012345678901212345678901234567890123456789012123456
Several major projects are being (95) with fisheries and mammology, we will 7
1 7
12 34567890123456789012345678901212345678901234567890123456789012123456 7
implemented at the Los Amigos Conser- better understand plant/animal interac-
12 34567890123456789012345678901212345678901234567890123456789012123456 7
12 34567890123456789012345678901212345678901234567890123456789012123456
(55) vation Area. Louise Emmons is initiating tions. By providing names, the botanical 7
12 34567890123456789012345678901212345678901234567890123456789012123456
studies of mammal diversity and ecology program will facilitate accurate commu- 7
1234567890123456789012345678901212345678901234567890123456789012123456 7
1234567890123456789012345678901212345678901234567890123456789012123456
in the Los Amigos area. Other projects nication about plants and the animals 7
1234567890123456789012345678901212345678901234567890123456789012123456 7
2 34567890123456789012345678901212345678901234567890123456789012123456
1234567890123456789012345678901212345678901234567890123456789012123456
involve studies of the diversity of (100) that use them. Included in this scenario 7
1234567890123456789012345678901212345678901234567890123456789012123456 7
1234567890123456789012345678901212345678901234567890123456789012123456
arthropods, amphibians, reptiles, and are humans, as we will dedicate time to 7
1234567890123456789012345678901212345678901234567890123456789012123456 77
1 (60) birds. Robin Foster has conducted people-plant interactions in order to
12 34567890123456789012345678901212345678901234567890123456789012123456 7
12 34567890123456789012345678901212345678901234567890123456789012123456
botanical studies at Los Amigos, resulting learn what plants are used by people in 7
12 34567890123456789012345678901212345678901234567890123456789012123456 7
12 34567890123456789012345678901212345678901234567890123456789012123456
in the labeling of hundreds of plant the Los Amigos area, and what plants 7
1234567890123456789012345678901212345678901234567890123456789012123456 7
1234567890123456789012345678901212345678901234567890123456789012123456 77
2 34567890123456789012345678901212345678901234567890123456789012123456
species along two kilometers of trail in (105) could potentially be used by people.
1234567890123456789012345678901212345678901234567890123456789012123456 7
1234567890123456789012345678901212345678901234567890123456789012123456
upland and lowland forest. Michael To be informed, we must develop
7
1234567890123456789012345678901212345678901234567890123456789012123456 7
1 (65) Goulding is leading a fisheries and knowledge. To develop knowledge, we
2 34567890123456789012345678901212345678901234567890123456789012123456
1234567890123456789012345678901212345678901234567890123456789012123456 7
1 aquatic ecology program, which aims to must collect, organize, and disseminate 7
12 34567890123456789012345678901212345678901234567890123456789012123456 7
1234567890123456789012345678901212345678901234567890123456789012123456 7
document the diversity of fish, their information. In this sense, botanical
12 34567890123456789012345678901212345678901234567890123456789012123456 7
1234567890123456789012345678901212345678901234567890123456789012123456
ecologies, and their habitats in the Los (110) information has conservation value. 7
1234567890123456789012345678901212345678901234567890123456789012123456 7
1234567890123456789012345678901212345678901234567890123456789012123456 77
Before we can use plant-based products
1234567890123456789012345678901212345678901234567890123456789012123456
12345678901234567890123456789012123456789012345678901234567890121234567
28 Copyright © 2005 Thomson Peterson’s, a part of The Thomson Corporaton
SAT is a registered trademark of the College Entrance Examination Board, which
was not involved in the production of and does not endorse this product.
12345678901234567890123456789012123456789012345678901234567890121234567
1234567890123456789012345678901212345678901234567890123456789012123456 7
12234567890123456789012345678901212345678901234567890123456789012123456 7
1 34567890123456789012345678901212345678901234567890123456789012123456
from the forest, we must know what 18. In line 40, “concession” could be re- 7
12234567890123456789012345678901212345678901234567890123456789012123456 7
1234567890123456789012345678901212345678901234567890123456789012123456
species are useful and we must know placed, without changing the meaning, 7
1 34567890123456789012345678901212345678901234567890123456789012123456 7
12234567890123456789012345678901212345678901234567890123456789012123456
their names. We must be able to identify with
34567890123456789012345678901212345678901234567890123456789012123456
7
7
1 (115) them, to know where they occur in the
12234567890123456789012345678901212345678901234567890123456789012123456 7
1234567890123456789012345678901212345678901234567890123456789012123456
(A) grant. 7
1234567890123456789012345678901212345678901234567890123456789012123456
forest, how many of them exist, how they 7
1234567890123456789012345678901212345678901234567890123456789012123456
(B) acknowledgement.
34567890123456789012345678901212345678901234567890123456789012123456
7
7
1 are pollinated and when they produce
12234567890123456789012345678901212345678901234567890123456789012123456
(C) food supply. 7
1234567890123456789012345678901212345678901234567890123456789012123456
fruit (or other useful products). Aside 7
1234567890123456789012345678901212345678901234567890123456789012123456
(D) apology. 7
1234567890123456789012345678901212345678901234567890123456789012123456
from understanding the species as they
34567890123456789012345678901212345678901234567890123456789012123456
7
7
1 (120) occur locally at Los Amigos, we must (E) compromise.
12234567890123456789012345678901212345678901234567890123456789012123456 7
1234567890123456789012345678901212345678901234567890123456789012123456 7
1234567890123456789012345678901212345678901234567890123456789012123456
have information about their overall 19. The author implies in paragraph 3 that the 7
1234567890123456789012345678901212345678901234567890123456789012123456 7
1 34567890123456789012345678901212345678901234567890123456789012123456
distribution in tropical America in order agreement between Peru and the Amazon 7
12234567890123456789012345678901212345678901234567890123456789012123456 7
1234567890123456789012345678901212345678901234567890123456789012123456
to better understand and manage the Conservation Association is historic 7
1234567890123456789012345678901212345678901234567890123456789012123456
distribution, variation, and viability of 7
1234567890123456789012345678901212345678901234567890123456789012123456
primarily because it 7
34567890123456789012345678901212345678901234567890123456789012123456
1234567890123456789012345678901212345678901234567890123456789012123456
(125) their genetic diversity. This involves a 7
1234567890123456789012345678901212345678901234567890123456789012123456 7
1234567890123456789012345678901212345678901234567890123456789012123456
more complete understanding of the (A) was the first time a South American 7
1234567890123456789012345678901212345678901234567890123456789012123456 7
1234567890123456789012345678901212345678901234567890123456789012123456
species through studies in the field and government had made an agreement 7
1234567890123456789012345678901212345678901234567890123456789012123456 7
1 herbarium. of any kind with the Amazon 7
12234567890123456789012345678901212345678901234567890123456789012123456 7
1234567890123456789012345678901212345678901234567890123456789012123456
Conservation Association. 7
1234567890123456789012345678901212345678901234567890123456789012123456 7
34567890123456789012345678901212345678901234567890123456789012123456
1234567890123456789012345678901212345678901234567890123456789012123456
(B) was the first long-term agreement 7
1234567890123456789012345678901212345678901234567890123456789012123456
16. In line 6, “genetic resources” refers to 7
1234567890123456789012345678901212345678901234567890123456789012123456 77
regarding land in the Amazon
1234567890123456789012345678901212345678901234567890123456789012123456
(A) plant seeds. Rainforest.
1234567890123456789012345678901212345678901234567890123456789012123456 7
1234567890123456789012345678901212345678901234567890123456789012123456
(B) different races of people. (C) represented the first time a South 7
1234567890123456789012345678901212345678901234567890123456789012123456 7
American government had agreed to 7
1234567890123456789012345678901212345678901234567890123456789012123456
(C) natural resources, such as oil.
1234567890123456789012345678901212345678901234567890123456789012123456 77
1 (D) diverse species of plants and animals. renew a conservation agreement.
12234567890123456789012345678901212345678901234567890123456789012123456 7
1234567890123456789012345678901212345678901234567890123456789012123456
(E) cells that can be used in genetic cures (D) is essentially a permanent conserva- 7
1 34567890123456789012345678901212345678901234567890123456789012123456 7
1234567890123456789012345678901212345678901234567890123456789012123456
for diseases. tion agreement. 7
12234567890123456789012345678901212345678901234567890123456789012123456 7
1234567890123456789012345678901212345678901234567890123456789012123456 77
34567890123456789012345678901212345678901234567890123456789012123456
(E) represents the first time such an
1234567890123456789012345678901212345678901234567890123456789012123456 7
1 17. In paragraph 2, the author emphasizes agreement had been made in the
12234567890123456789012345678901212345678901234567890123456789012123456 7
1234567890123456789012345678901212345678901234567890123456789012123456
that the current environmental condition form of a renewable contract. 7
34567890123456789012345678901212345678901234567890123456789012123456
1234567890123456789012345678901212345678901234567890123456789012123456 7
1234567890123456789012345678901212345678901234567890123456789012123456
of Amazonian South America is 7
1 7
1234567890123456789012345678901212345678901234567890123456789012123456
(A) mostly unscathed. 7
1234567890123456789012345678901212345678901234567890123456789012123456 7
2 34567890123456789012345678901212345678901234567890123456789012123456
1234567890123456789012345678901212345678901234567890123456789012123456
(B) largely unknown. 7
1234567890123456789012345678901212345678901234567890123456789012123456 7
1234567890123456789012345678901212345678901234567890123456789012123456
(C) restorable through his project. 7
1234567890123456789012345678901212345678901234567890123456789012123456 77
1 (D) irredeemable everywhere but in the
12234567890123456789012345678901212345678901234567890123456789012123456 7
1234567890123456789012345678901212345678901234567890123456789012123456
Los Amigos watershed. 7
1234567890123456789012345678901212345678901234567890123456789012123456 7
1234567890123456789012345678901212345678901234567890123456789012123456
(E) varying from destroyed to virtually 7
1 34567890123456789012345678901212345678901234567890123456789012123456 7
1234567890123456789012345678901212345678901234567890123456789012123456 77
2 34567890123456789012345678901212345678901234567890123456789012123456
pristine.
1234567890123456789012345678901212345678901234567890123456789012123456 7
1234567890123456789012345678901212345678901234567890123456789012123456 7
1234567890123456789012345678901212345678901234567890123456789012123456 7
1
12234567890123456789012345678901212345678901234567890123456789012123456 7
1 34567890123456789012345678901212345678901234567890123456789012123456 7
12234567890123456789012345678901212345678901234567890123456789012123456 7
1 34567890123456789012345678901212345678901234567890123456789012123456 7
12234567890123456789012345678901212345678901234567890123456789012123456 7
1 34567890123456789012345678901212345678901234567890123456789012123456 7
1234567890123456789012345678901212345678901234567890123456789012123456 7
1234567890123456789012345678901212345678901234567890123456789012123456 7
1234567890123456789012345678901212345678901234567890123456789012123456 7
12345678901234567890123456789012123456789012345678901234567890121234567
Copyright © 2005 Thomson Peterson’s, a part of The Thomson Corporaton 29
SAT is a registered trademark of the College Entrance Examination Board, which
was not involved in the production of and does not endorse this product.
12345678901234567890123456789012123456789012345678901234567890121234567
1234567890123456789012345678901212345678901234567890123456789012123456 7
12 34567890123456789012345678901212345678901234567890123456789012123456 7
1234567890123456789012345678901212345678901234567890123456789012123456
20. The author’s main purpose in the passage 22. The work of Louise Emmons, Robin 7
12 34567890123456789012345678901212345678901234567890123456789012123456 7
12 34567890123456789012345678901212345678901234567890123456789012123456
is to Foster, and Michael Goulding (in the 7
1234567890123456789012345678901212345678901234567890123456789012123456 7
12
2
34567890123456789012345678901212345678901234567890123456789012123456
fourth paragraph)
34567890123456789012345678901212345678901234567890123456789012123456
are employed in the 7
7
1 (A) demonstrate that conservation efforts
12 34567890123456789012345678901212345678901234567890123456789012123456
passage as 7
12 34567890123456789012345678901212345678901234567890123456789012123456 7
have been historically successful and
12 34567890123456789012345678901212345678901234567890123456789012123456 7
12
2
34567890123456789012345678901212345678901234567890123456789012123456
so should be continued. (A) colleagues of the author’s in his
34567890123456789012345678901212345678901234567890123456789012123456
7
7
1
12 34567890123456789012345678901212345678901234567890123456789012123456
(B) garner support for opposition to botanical project. 7
12 34567890123456789012345678901212345678901234567890123456789012123456 7
12 34567890123456789012345678901212345678901234567890123456789012123456 7
destructive activities in the Los (B) examples of the kinds of activities
12 34567890123456789012345678901212345678901234567890123456789012123456 7
1234567890123456789012345678901212345678901234567890123456789012123456
Amigos watershed. the author and his colleagues are 7
12 34567890123456789012345678901212345678901234567890123456789012123456 7
12 34567890123456789012345678901212345678901234567890123456789012123456 7
(C) position the Los Amigos watershed trying to halt.
12 34567890123456789012345678901212345678901234567890123456789012123456
(C) examples of the influence of interna- 7
12
2
34567890123456789012345678901212345678901234567890123456789012123456
agreement as a success towards the
34567890123456789012345678901212345678901234567890123456789012123456
7
7
1 achievement of the vital goal of con- tional scientists in Peru.
12 34567890123456789012345678901212345678901234567890123456789012123456 7
12 34567890123456789012345678901212345678901234567890123456789012123456
serving the Amazonian rainforests. (D) scientists who represent new trends 7
12 34567890123456789012345678901212345678901234567890123456789012123456 7
12 34567890123456789012345678901212345678901234567890123456789012123456
(D) uphold the Peruvian government’s of study in Amazonian botany. 7
12 34567890123456789012345678901212345678901234567890123456789012123456 7
12 34567890123456789012345678901212345678901234567890123456789012123456
progressive policies on management (E) scientists involved in projects related 7
12 34567890123456789012345678901212345678901234567890123456789012123456 7
12 34567890123456789012345678901212345678901234567890123456789012123456
of the Los Amigos watershed as an and amenable to the author’s. 7
12 34567890123456789012345678901212345678901234567890123456789012123456 7
1234567890123456789012345678901212345678901234567890123456789012123456 77
2 34567890123456789012345678901212345678901234567890123456789012123456
example of government policy
1
12 34567890123456789012345678901212345678901234567890123456789012123456
working toward conservation. 23. The author’s botanical project involves all 7
12 34567890123456789012345678901212345678901234567890123456789012123456 7
12 34567890123456789012345678901212345678901234567890123456789012123456
(E) argue that the study of pristine of the following EXCEPT 7
2 34567890123456789012345678901212345678901234567890123456789012123456
1234567890123456789012345678901212345678901234567890123456789012123456 7
1234567890123456789012345678901212345678901234567890123456789012123456
rainforests is essential for document- 7
1234567890123456789012345678901212345678901234567890123456789012123456
(A) studying plants in a laboratory. 7
1234567890123456789012345678901212345678901234567890123456789012123456
ing and studying the myriad new (B) studying how plants are used by 7
1234567890123456789012345678901212345678901234567890123456789012123456 7
1234567890123456789012345678901212345678901234567890123456789012123456
species that the forests contain. humans and animals. 7
1234567890123456789012345678901212345678901234567890123456789012123456 7
1234567890123456789012345678901212345678901234567890123456789012123456
(C) facilitating pharmaceutical use of 7
1234567890123456789012345678901212345678901234567890123456789012123456
21. The author’s tone in the passage can best 7
1 plants. 7
2 34567890123456789012345678901212345678901234567890123456789012123456
1234567890123456789012345678901212345678901234567890123456789012123456
be described as 7
1234567890123456789012345678901212345678901234567890123456789012123456
(D) providing information on how to 7
1 7
1234567890123456789012345678901212345678901234567890123456789012123456
(A) advocacy for his project over other keep plant species flourishing. 7
12 34567890123456789012345678901212345678901234567890123456789012123456 7
1234567890123456789012345678901212345678901234567890123456789012123456 77
2 34567890123456789012345678901212345678901234567890123456789012123456
competing projects. (E) labeling plants in the Los Amigos
1234567890123456789012345678901212345678901234567890123456789012123456 7
1 (B) general praise for conservation area.
12 34567890123456789012345678901212345678901234567890123456789012123456 7
12 34567890123456789012345678901212345678901234567890123456789012123456 7
projects in Amazonian South
2 34567890123456789012345678901212345678901234567890123456789012123456
1234567890123456789012345678901212345678901234567890123456789012123456 7
1234567890123456789012345678901212345678901234567890123456789012123456
America. 7
1 7
1234567890123456789012345678901212345678901234567890123456789012123456 7
(C) condemnation for the government of
1234567890123456789012345678901212345678901234567890123456789012123456 7
12 34567890123456789012345678901212345678901234567890123456789012123456
Peru for allowing destruction of the 7
12 34567890123456789012345678901212345678901234567890123456789012123456 7
12 34567890123456789012345678901212345678901234567890123456789012123456 7
rainforest.
12 34567890123456789012345678901212345678901234567890123456789012123456 7
1234567890123456789012345678901212345678901234567890123456789012123456
(D) passionate support for his and 7
2 34567890123456789012345678901212345678901234567890123456789012123456
1234567890123456789012345678901212345678901234567890123456789012123456 7
1234567890123456789012345678901212345678901234567890123456789012123456
related projects. 7
1234567890123456789012345678901212345678901234567890123456789012123456 7
1234567890123456789012345678901212345678901234567890123456789012123456 77
(E) zealous advocacy for his point of view.
1
12 34567890123456789012345678901212345678901234567890123456789012123456 7
12 34567890123456789012345678901212345678901234567890123456789012123456 7
12 34567890123456789012345678901212345678901234567890123456789012123456 7
12 34567890123456789012345678901212345678901234567890123456789012123456 7
1234567890123456789012345678901212345678901234567890123456789012123456 7
1234567890123456789012345678901212345678901234567890123456789012123456
2 7
1 34567890123456789012345678901212345678901234567890123456789012123456 7
12 34567890123456789012345678901212345678901234567890123456789012123456 7
1234567890123456789012345678901212345678901234567890123456789012123456 7
12 34567890123456789012345678901212345678901234567890123456789012123456 7
1234567890123456789012345678901212345678901234567890123456789012123456 7
1234567890123456789012345678901212345678901234567890123456789012123456 7
1234567890123456789012345678901212345678901234567890123456789012123456 7
1234567890123456789012345678901212345678901234567890123456789012123456 7
12345678901234567890123456789012123456789012345678901234567890121234567
30 Copyright © 2005 Thomson Peterson’s, a part of The Thomson Corporaton
SAT is a registered trademark of the College Entrance Examination Board, which
was not involved in the production of and does not endorse this product.
12345678901234567890123456789012123456789012345678901234567890121234567
1234567890123456789012345678901212345678901234567890123456789012123456 7
12234567890123456789012345678901212345678901234567890123456789012123456 7
1 34567890123456789012345678901212345678901234567890123456789012123456
24. When the author says that the botanical 26. Which of the following issues does the 7
12234567890123456789012345678901212345678901234567890123456789012123456 7
1234567890123456789012345678901212345678901234567890123456789012123456
project will “provide names,” (line passage NOT address? 7
1 34567890123456789012345678901212345678901234567890123456789012123456 7
12234567890123456789012345678901212345678901234567890123456789012123456
97–100) he means that the project will
34567890123456789012345678901212345678901234567890123456789012123456
7
7
1 (A) Positive contributions of scientific
12234567890123456789012345678901212345678901234567890123456789012123456 7
1234567890123456789012345678901212345678901234567890123456789012123456
(A) help recognize new species. research for conservation efforts 7
1234567890123456789012345678901212345678901234567890123456789012123456 7
1234567890123456789012345678901212345678901234567890123456789012123456
(B) aid in the standardization of names (B) Pollution
34567890123456789012345678901212345678901234567890123456789012123456
of water sources in 7
7
1
12234567890123456789012345678901212345678901234567890123456789012123456
for new species. Amazonian Peru 7
1234567890123456789012345678901212345678901234567890123456789012123456 7
1234567890123456789012345678901212345678901234567890123456789012123456
(C) participate in naming the region’s (C) Economic importance of conserving 7
1234567890123456789012345678901212345678901234567890123456789012123456
34567890123456789012345678901212345678901234567890123456789012123456
7
7
1 different zones. the Amazon rainforests
12234567890123456789012345678901212345678901234567890123456789012123456 7
1234567890123456789012345678901212345678901234567890123456789012123456
(D) publish information for corporations (D) Specific efforts of the Peruvian 7
1234567890123456789012345678901212345678901234567890123456789012123456 7
1234567890123456789012345678901212345678901234567890123456789012123456
and researchers regarding the most government
34567890123456789012345678901212345678901234567890123456789012123456
to maintain the integrity 7
7
1 appropriate names for specific of Peruvian rainforests
12234567890123456789012345678901212345678901234567890123456789012123456 7
1234567890123456789012345678901212345678901234567890123456789012123456
plants. (E) Examples of previous scientific 7
1234567890123456789012345678901212345678901234567890123456789012123456 7
1234567890123456789012345678901212345678901234567890123456789012123456
(E) clarify the confusion surrounding the research in Los Amigos 7
1234567890123456789012345678901212345678901234567890123456789012123456 77
34567890123456789012345678901212345678901234567890123456789012123456
1234567890123456789012345678901212345678901234567890123456789012123456
names of different organizations
27. The author mentions areas outside the Los 7
1234567890123456789012345678901212345678901234567890123456789012123456
1234567890123456789012345678901212345678901234567890123456789012123456
working in Amazonia. 7
1234567890123456789012345678901212345678901234567890123456789012123456 7
1234567890123456789012345678901212345678901234567890123456789012123456 77
Amigos watershed primarily in order to
1 25. When the author says that, “botanical
12234567890123456789012345678901212345678901234567890123456789012123456 7
1234567890123456789012345678901212345678901234567890123456789012123456
(A) imply that his future research will 7
1234567890123456789012345678901212345678901234567890123456789012123456
information has conservation value,” 7
34567890123456789012345678901212345678901234567890123456789012123456
1234567890123456789012345678901212345678901234567890123456789012123456 focus on these areas. 7
1234567890123456789012345678901212345678901234567890123456789012123456
(lines 109–110) he means that 7
1234567890123456789012345678901212345678901234567890123456789012123456 77
(B) draw a comparison between work in
1234567890123456789012345678901212345678901234567890123456789012123456
(A) a robust understanding of conserva- those areas and work in the Los
1234567890123456789012345678901212345678901234567890123456789012123456 7
1234567890123456789012345678901212345678901234567890123456789012123456
tionism is aided by botanical Amigos area. 7
1234567890123456789012345678901212345678901234567890123456789012123456 7
(C) underscore the interrelatedness of the 7
1234567890123456789012345678901212345678901234567890123456789012123456
information.
1234567890123456789012345678901212345678901234567890123456789012123456 77
1 (B) conservationists should strive to ecosystems.
12234567890123456789012345678901212345678901234567890123456789012123456 7
1234567890123456789012345678901212345678901234567890123456789012123456
preserve botanical information. (D) emphasize that Los Amigos is the 7
1 34567890123456789012345678901212345678901234567890123456789012123456 7
1234567890123456789012345678901212345678901234567890123456789012123456
(C) speciation is important for most pristine locale. 7
12234567890123456789012345678901212345678901234567890123456789012123456 7
1234567890123456789012345678901212345678901234567890123456789012123456 77
34567890123456789012345678901212345678901234567890123456789012123456
conservation. (E) praise the Peruvian government for its
1234567890123456789012345678901212345678901234567890123456789012123456 7
1 (D) political discussions about other conservationist undertakings.
12234567890123456789012345678901212345678901234567890123456789012123456 7
1234567890123456789012345678901212345678901234567890123456789012123456
conservation should use botanical 7
34567890123456789012345678901212345678901234567890123456789012123456
1234567890123456789012345678901212345678901234567890123456789012123456 7
1234567890123456789012345678901212345678901234567890123456789012123456
nomenclature. 7
1 7
1234567890123456789012345678901212345678901234567890123456789012123456 7
(E) new drugs will be developed in the re-
1234567890123456789012345678901212345678901234567890123456789012123456 7
12234567890123456789012345678901212345678901234567890123456789012123456
gions protected by conservationism. 7
1234567890123456789012345678901212345678901234567890123456789012123456 7
1234567890123456789012345678901212345678901234567890123456789012123456 7
1234567890123456789012345678901212345678901234567890123456789012123456 7
1 34567890123456789012345678901212345678901234567890123456789012123456 7
1234567890123456789012345678901212345678901234567890123456789012123456 77
2 34567890123456789012345678901212345678901234567890123456789012123456
1234567890123456789012345678901212345678901234567890123456789012123456 7
1234567890123456789012345678901212345678901234567890123456789012123456 7
1234567890123456789012345678901212345678901234567890123456789012123456 7
1
12234567890123456789012345678901212345678901234567890123456789012123456 7
1234567890123456789012345678901212345678901234567890123456789012123456 7
1234567890123456789012345678901212345678901234567890123456789012123456 7
1234567890123456789012345678901212345678901234567890123456789012123456 7
1 34567890123456789012345678901212345678901234567890123456789012123456 7
1234567890123456789012345678901212345678901234567890123456789012123456 77
2 34567890123456789012345678901212345678901234567890123456789012123456
1
12234567890123456789012345678901212345678901234567890123456789012123456 7
1 34567890123456789012345678901212345678901234567890123456789012123456 7
12234567890123456789012345678901212345678901234567890123456789012123456 7
STOP
1 34567890123456789012345678901212345678901234567890123456789012123456
Do not proceed to the next section until time is up.
1234567890123456789012345678901212345678901234567890123456789012123456 7
7
1234567890123456789012345678901212345678901234567890123456789012123456 7
1234567890123456789012345678901212345678901234567890123456789012123456 7
12345678901234567890123456789012123456789012345678901234567890121234567
Copyright © 2005 Thomson Peterson’s, a part of The Thomson Corporaton 31
SAT is a registered trademark of the College Entrance Examination Board, which
was not involved in the production of and does not endorse this product.
Section 5
21 Questions j Time—25 Minutes

12345678901234567890123456789012123456789012345678901234567890121234567
12 34567890123456789012345678901212345678901234567890123456789012123456 7
12 34567890123456789012345678901212345678901234567890123456789012123456 7
1 34567890123456789012345678901212345678901234567890123456789012123456 7
2
12 34567890123456789012345678901212345678901234567890123456789012123456 7
12 34567890123456789012345678901212345678901234567890123456789012123456
Directions: Solve the following problems using any available space on the page for scratchwork. 7
12 34567890123456789012345678901212345678901234567890123456789012123456
Mark the letter of your choice on the answer sheet that best corresponds to the correct answer. 7
12 34567890123456789012345678901212345678901234567890123456789012123456 7
1234567890123456789012345678901212345678901234567890123456789012123456 77
2 34567890123456789012345678901212345678901234567890123456789012123456
1234567890123456789012345678901212345678901234567890123456789012123456
Notes:
1234567890123456789012345678901212345678901234567890123456789012123456 7
1234567890123456789012345678901212345678901234567890123456789012123456
1. You may use a calculator. All of the numbers used are real numbers. 7
1234567890123456789012345678901212345678901234567890123456789012123456 77
1234567890123456789012345678901212345678901234567890123456789012123456 7
1 2. You may use the figures that accompany the problems to help you find the solution. Unless
12 34567890123456789012345678901212345678901234567890123456789012123456 7
12 34567890123456789012345678901212345678901234567890123456789012123456 7
the instructions say that a figure is not drawn to scale, assume that it has been drawn
12 34567890123456789012345678901212345678901234567890123456789012123456 7
12 34567890123456789012345678901212345678901234567890123456789012123456
accurately. Each figure lies in a plane unless the instructions say otherwise. 7
12 34567890123456789012345678901212345678901234567890123456789012123456 7
12 34567890123456789012345678901212345678901234567890123456789012123456 7
12 34567890123456789012345678901212345678901234567890123456789012123456 7
12 34567890123456789012345678901212345678901234567890123456789012123456 7
1234567890123456789012345678901212345678901234567890123456789012123456
2 7
Reference Information

2s
1234567890123456789012345678901212345678901234567890123456789012123456
r w 2x
45
7
1234567890123456789012345678901212345678901234567890123456789012123456
h h
r
c x s
60
7
1234567890123456789012345678901212345678901234567890123456789012123456 h b 7
1 34567890123456789012345678901212345678901234567890123456789012123456 7
30
45
w
2 34567890123456789012345678901212345678901234567890123456789012123456
1234567890123456789012345678901212345678901234567890123456789012123456 7
1234567890123456789012345678901212345678901234567890123456789012123456
A 5 pr2 b a 3x s 7
1 1 7
1234567890123456789012345678901212345678901234567890123456789012123456 7
2 2 2 2
C 5 2pr A 5 ,w A 5 bh V 5 ,wh V 5 pr h c 5 a 1 b Special Right Triangles
12
2
34567890123456789012345678901212345678901234567890123456789012123456 7
2
34567890123456789012345678901212345678901234567890123456789012123456 7
1234567890123456789012345678901212345678901234567890123456789012123456
The number of degrees of arc in a circle is 360.
1234567890123456789012345678901212345678901234567890123456789012123456 7
1 The measure in degrees of a straight angle is 180. 7
12 34567890123456789012345678901212345678901234567890123456789012123456 7
12 34567890123456789012345678901212345678901234567890123456789012123456 7
The sum of the measures in degrees of the angles of a triangle is 180.
1234567890123456789012345678901212345678901234567890123456789012123456 77
2 34567890123456789012345678901212345678901234567890123456789012123456
1234567890123456789012345678901212345678901234567890123456789012123456 7
1
1234567890123456789012345678901212345678901234567890123456789012123456 7
1234567890123456789012345678901212345678901234567890123456789012123456
1. Consider two sets of numbers: Set A 2. If four sweaters cost p dollars, and the 7
2 34567890123456789012345678901212345678901234567890123456789012123456
1234567890123456789012345678901212345678901234567890123456789012123456 7
1234567890123456789012345678901212345678901234567890123456789012123456
includes all the positive integers and Set B sweaters go on a half-off sale, how much 7
1234567890123456789012345678901212345678901234567890123456789012123456 7
1234567890123456789012345678901212345678901234567890123456789012123456 77
includes all the negative integers. Which would 12 sweaters cost in dollars?
1
12 34567890123456789012345678901212345678901234567890123456789012123456
set has more members?
p 7
12 34567890123456789012345678901212345678901234567890123456789012123456 7
12 34567890123456789012345678901212345678901234567890123456789012123456
(A) A
(A)
2 7
12 34567890123456789012345678901212345678901234567890123456789012123456 7
1234567890123456789012345678901212345678901234567890123456789012123456
(B) B 7
12 34567890123456789012345678901212345678901234567890123456789012123456 3p 7
12 34567890123456789012345678901212345678901234567890123456789012123456
(C) They contain an equal number. (B) 7
12 34567890123456789012345678901212345678901234567890123456789012123456 2 7
12 34567890123456789012345678901212345678901234567890123456789012123456
(D) Neither. 7
1234567890123456789012345678901212345678901234567890123456789012123456 (C) 2p 7
1234567890123456789012345678901212345678901234567890123456789012123456 77
2 34567890123456789012345678901212345678901234567890123456789012123456
(E) It cannot be determined.
1
12 34567890123456789012345678901212345678901234567890123456789012123456(D) 4p 7
1234567890123456789012345678901212345678901234567890123456789012123456 7
12 34567890123456789012345678901212345678901234567890123456789012123456 7
1234567890123456789012345678901212345678901234567890123456789012123456 (E) 6p 7
1234567890123456789012345678901212345678901234567890123456789012123456 7
1234567890123456789012345678901212345678901234567890123456789012123456 7
1234567890123456789012345678901212345678901234567890123456789012123456 7
12345678901234567890123456789012123456789012345678901234567890121234567
32 Copyright © 2005 Thomson Peterson’s, a part of The Thomson Corporaton
SAT is a registered trademark of the College Entrance Examination Board, which
was not involved in the production of and does not endorse this product.
12345678901234567890123456789012123456789012345678901234567890121234567
1234567890123456789012345678901212345678901234567890123456789012123456 7
12234567890123456789012345678901212345678901234567890123456789012123456 7
1 34567890123456789012345678901212345678901234567890123456789012123456
2 5. If y 5 mx 1 b where m is a negative 7
12234567890123456789012345678901212345678901234567890123456789012123456 7
1234567890123456789012345678901212345678901234567890123456789012123456
3 constant and b is a positive constant, 7
1 34567890123456789012345678901212345678901234567890123456789012123456 7
12234567890123456789012345678901212345678901234567890123456789012123456
3. 5 which of the following
34567890123456789012345678901212345678901234567890123456789012123456
could be a possible 7
7
1 4
12234567890123456789012345678901212345678901234567890123456789012123456
graph of y 5 mx 1 b? 7
1234567890123456789012345678901212345678901234567890123456789012123456
5 7
1234567890123456789012345678901212345678901234567890123456789012123456 7
1234567890123456789012345678901212345678901234567890123456789012123456
1 (A)
34567890123456789012345678901212345678901234567890123456789012123456
7
7
1 (A)
12234567890123456789012345678901212345678901234567890123456789012123456
3 7
1234567890123456789012345678901212345678901234567890123456789012123456 7
1234567890123456789012345678901212345678901234567890123456789012123456
8 7
1234567890123456789012345678901212345678901234567890123456789012123456
34567890123456789012345678901212345678901234567890123456789012123456
7
7
1 (B)
15
12234567890123456789012345678901212345678901234567890123456789012123456 7
1234567890123456789012345678901212345678901234567890123456789012123456 7
1234567890123456789012345678901212345678901234567890123456789012123456
5 7
1234567890123456789012345678901212345678901234567890123456789012123456
(C)
34567890123456789012345678901212345678901234567890123456789012123456
7
7
1 6
12234567890123456789012345678901212345678901234567890123456789012123456 7
1234567890123456789012345678901212345678901234567890123456789012123456
15 7
1234567890123456789012345678901212345678901234567890123456789012123456
(D) 7
1234567890123456789012345678901212345678901234567890123456789012123456
8 7
1234567890123456789012345678901212345678901234567890123456789012123456 77
34567890123456789012345678901212345678901234567890123456789012123456
(B)
1234567890123456789012345678901212345678901234567890123456789012123456
1234567890123456789012345678901212345678901234567890123456789012123456
6 7
1234567890123456789012345678901212345678901234567890123456789012123456
(E) 7
1234567890123456789012345678901212345678901234567890123456789012123456 77
5
1234567890123456789012345678901212345678901234567890123456789012123456 7
1
12234567890123456789012345678901212345678901234567890123456789012123456
10 7
1234567890123456789012345678901212345678901234567890123456789012123456
4. If 3y 5 x and y 5 , what is the value of 7
1234567890123456789012345678901212345678901234567890123456789012123456
z 7
34567890123456789012345678901212345678901234567890123456789012123456
1234567890123456789012345678901212345678901234567890123456789012123456
z when x 5 3? 7
1234567890123456789012345678901212345678901234567890123456789012123456 7
1234567890123456789012345678901212345678901234567890123456789012123456 7
1234567890123456789012345678901212345678901234567890123456789012123456
(A) 1 7
1234567890123456789012345678901212345678901234567890123456789012123456 7
1234567890123456789012345678901212345678901234567890123456789012123456
(B) 3 7
1234567890123456789012345678901212345678901234567890123456789012123456
(C) 7
1234567890123456789012345678901212345678901234567890123456789012123456
(C) 5 7
1234567890123456789012345678901212345678901234567890123456789012123456 7
1 (D) 7 7
2 34567890123456789012345678901212345678901234567890123456789012123456
1234567890123456789012345678901212345678901234567890123456789012123456 7
1234567890123456789012345678901212345678901234567890123456789012123456
(E) 10 7
1 7
1234567890123456789012345678901212345678901234567890123456789012123456 7
12234567890123456789012345678901212345678901234567890123456789012123456 7
1234567890123456789012345678901212345678901234567890123456789012123456 7
1234567890123456789012345678901212345678901234567890123456789012123456 7
1 34567890123456789012345678901212345678901234567890123456789012123456 7
12234567890123456789012345678901212345678901234567890123456789012123456 7
1234567890123456789012345678901212345678901234567890123456789012123456 7
1234567890123456789012345678901212345678901234567890123456789012123456 77
34567890123456789012345678901212345678901234567890123456789012123456
1234567890123456789012345678901212345678901234567890123456789012123456
(D)
7
1
1234567890123456789012345678901212345678901234567890123456789012123456 7
1234567890123456789012345678901212345678901234567890123456789012123456 7
12234567890123456789012345678901212345678901234567890123456789012123456 7
1234567890123456789012345678901212345678901234567890123456789012123456 7
1234567890123456789012345678901212345678901234567890123456789012123456 7
1234567890123456789012345678901212345678901234567890123456789012123456 7
1 34567890123456789012345678901212345678901234567890123456789012123456 7
1234567890123456789012345678901212345678901234567890123456789012123456 77
2 34567890123456789012345678901212345678901234567890123456789012123456
1234567890123456789012345678901212345678901234567890123456789012123456 7
1234567890123456789012345678901212345678901234567890123456789012123456 7
1234567890123456789012345678901212345678901234567890123456789012123456 7
1
12234567890123456789012345678901212345678901234567890123456789012123456 7
1234567890123456789012345678901212345678901234567890123456789012123456 7
1234567890123456789012345678901212345678901234567890123456789012123456 7
1234567890123456789012345678901212345678901234567890123456789012123456 7
1 34567890123456789012345678901212345678901234567890123456789012123456 7
1234567890123456789012345678901212345678901234567890123456789012123456 77
2 34567890123456789012345678901212345678901234567890123456789012123456
1
12234567890123456789012345678901212345678901234567890123456789012123456 7
1 34567890123456789012345678901212345678901234567890123456789012123456 7
12234567890123456789012345678901212345678901234567890123456789012123456 7
1 34567890123456789012345678901212345678901234567890123456789012123456 7
1234567890123456789012345678901212345678901234567890123456789012123456 7
1234567890123456789012345678901212345678901234567890123456789012123456 7
1234567890123456789012345678901212345678901234567890123456789012123456 7
12345678901234567890123456789012123456789012345678901234567890121234567
Copyright © 2005 Thomson Peterson’s, a part of The Thomson Corporaton 33
SAT is a registered trademark of the College Entrance Examination Board, which
was not involved in the production of and does not endorse this product.
12345678901234567890123456789012123456789012345678901234567890121234567
1234567890123456789012345678901212345678901234567890123456789012123456 7
12 34567890123456789012345678901212345678901234567890123456789012123456 7
1234567890123456789012345678901212345678901234567890123456789012123456
6. Small Medium Large 9. If f(x) 5 x2 is graphed on a standard 7
12 34567890123456789012345678901212345678901234567890123456789012123456 7
12 34567890123456789012345678901212345678901234567890123456789012123456
(S) (M) (L) xy-axis and then the graph is rotated 90° 7
1234567890123456789012345678901212345678901234567890123456789012123456 7
12
2
34567890123456789012345678901212345678901234567890123456789012123456
clockwise, which of
34567890123456789012345678901212345678901234567890123456789012123456
the following graphs 7
7
1 Hats (H) $8 $12 $12
12 34567890123456789012345678901212345678901234567890123456789012123456
would result? 7
12 34567890123456789012345678901212345678901234567890123456789012123456 7
Shirts (SH) $12 $12 $14
12 34567890123456789012345678901212345678901234567890123456789012123456 7
12
2
34567890123456789012345678901212345678901234567890123456789012123456
(A)
34567890123456789012345678901212345678901234567890123456789012123456
7
7
1 Hat & Shirt Prices at Moe’s
12 34567890123456789012345678901212345678901234567890123456789012123456 7
12 34567890123456789012345678901212345678901234567890123456789012123456
If Moe’s has a 25% off sale on medium- 7
12 34567890123456789012345678901212345678901234567890123456789012123456 7
12
2
34567890123456789012345678901212345678901234567890123456789012123456
sized items, how much would it cost, in
34567890123456789012345678901212345678901234567890123456789012123456
7
7
1
12 34567890123456789012345678901212345678901234567890123456789012123456
dollars, to order 2 H-M, 2 H-L, and 7
12 34567890123456789012345678901212345678901234567890123456789012123456 7
12 34567890123456789012345678901212345678901234567890123456789012123456
1SH-M? 7
12
2
34567890123456789012345678901212345678901234567890123456789012123456
34567890123456789012345678901212345678901234567890123456789012123456
7
7
1 (B)
12 34567890123456789012345678901212345678901234567890123456789012123456
(A) 51 7
12 34567890123456789012345678901212345678901234567890123456789012123456 7
12 34567890123456789012345678901212345678901234567890123456789012123456 7
(B) 53
12 34567890123456789012345678901212345678901234567890123456789012123456 7
12 34567890123456789012345678901212345678901234567890123456789012123456
(C) 55 7
12 34567890123456789012345678901212345678901234567890123456789012123456 7
12 34567890123456789012345678901212345678901234567890123456789012123456 7
(D) 56
12 34567890123456789012345678901212345678901234567890123456789012123456 7
12 34567890123456789012345678901212345678901234567890123456789012123456
(E) 58 7
1234567890123456789012345678901212345678901234567890123456789012123456 77
2 34567890123456789012345678901212345678901234567890123456789012123456
1
12 34567890123456789012345678901212345678901234567890123456789012123456
7. If 5=x 1 15 5 30, then x 5 7
12 34567890123456789012345678901212345678901234567890123456789012123456
(C) 7
12 34567890123456789012345678901212345678901234567890123456789012123456 7
2 34567890123456789012345678901212345678901234567890123456789012123456
1234567890123456789012345678901212345678901234567890123456789012123456
(A) 9 7
1234567890123456789012345678901212345678901234567890123456789012123456 7
1234567890123456789012345678901212345678901234567890123456789012123456
(B) 10 7
1234567890123456789012345678901212345678901234567890123456789012123456 7
1234567890123456789012345678901212345678901234567890123456789012123456
(C) 12 7
1234567890123456789012345678901212345678901234567890123456789012123456 7
1234567890123456789012345678901212345678901234567890123456789012123456
(D) 14 7
1234567890123456789012345678901212345678901234567890123456789012123456 7
1234567890123456789012345678901212345678901234567890123456789012123456
(E) 15 7
1 7
2 34567890123456789012345678901212345678901234567890123456789012123456
1234567890123456789012345678901212345678901234567890123456789012123456 7
1234567890123456789012345678901212345678901234567890123456789012123456
2 7
1 8. 2 x 2 x 2 12 7
1234567890123456789012345678901212345678901234567890123456789012123456
5 7
12
2
34567890123456789012345678901212345678901234567890123456789012123456 7
2x 1 2x 2 12 (D)
34567890123456789012345678901212345678901234567890123456789012123456 7
1234567890123456789012345678901212345678901234567890123456789012123456
1234567890123456789012345678901212345678901234567890123456789012123456
x22 7
1 (A) 7
12 34567890123456789012345678901212345678901234567890123456789012123456
x24 7
12 34567890123456789012345678901212345678901234567890123456789012123456 7
2 34567890123456789012345678901212345678901234567890123456789012123456
1234567890123456789012345678901212345678901234567890123456789012123456
3~x 2 2! 7
1234567890123456789012345678901212345678901234567890123456789012123456 7
1 (B) 7
1234567890123456789012345678901212345678901234567890123456789012123456 7
x 2 4
1234567890123456789012345678901212345678901234567890123456789012123456 7
12 34567890123456789012345678901212345678901234567890123456789012123456
4~x 2 3! 7
12 34567890123456789012345678901212345678901234567890123456789012123456
(C) 7
12 34567890123456789012345678901212345678901234567890123456789012123456
x22 7
12 34567890123456789012345678901212345678901234567890123456789012123456 7
1234567890123456789012345678901212345678901234567890123456789012123456
x24 7
12 34567890123456789012345678901212345678901234567890123456789012123456
(D) (E) 7
12 34567890123456789012345678901212345678901234567890123456789012123456
2~x 2 2! 7
12 34567890123456789012345678901212345678901234567890123456789012123456 7
12 34567890123456789012345678901212345678901234567890123456789012123456
4~x 2 3! 7
1234567890123456789012345678901212345678901234567890123456789012123456 7
2 34567890123456789012345678901212345678901234567890123456789012123456
1234567890123456789012345678901212345678901234567890123456789012123456
(E) 7
1234567890123456789012345678901212345678901234567890123456789012123456
3~x 2 2! 7
1234567890123456789012345678901212345678901234567890123456789012123456 77
1234567890123456789012345678901212345678901234567890123456789012123456 7
1
12 34567890123456789012345678901212345678901234567890123456789012123456 7
1234567890123456789012345678901212345678901234567890123456789012123456 7
12 34567890123456789012345678901212345678901234567890123456789012123456 7
1234567890123456789012345678901212345678901234567890123456789012123456 7
12 34567890123456789012345678901212345678901234567890123456789012123456 7
1234567890123456789012345678901212345678901234567890123456789012123456 7
1234567890123456789012345678901212345678901234567890123456789012123456 7
1234567890123456789012345678901212345678901234567890123456789012123456 7
1234567890123456789012345678901212345678901234567890123456789012123456 7
12345678901234567890123456789012123456789012345678901234567890121234567
34 Copyright © 2005 Thomson Peterson’s, a part of The Thomson Corporaton
SAT is a registered trademark of the College Entrance Examination Board, which
was not involved in the production of and does not endorse this product.
12345678901234567890123456789012123456789012345678901234567890121234567
1234567890123456789012345678901212345678901234567890123456789012123456 7
12234567890123456789012345678901212345678901234567890123456789012123456 7
1 34567890123456789012345678901212345678901234567890123456789012123456
10. For which of the following ordered pairs 13. 7
12234567890123456789012345678901212345678901234567890123456789012123456 7
1234567890123456789012345678901212345678901234567890123456789012123456
(x,y) is x 2 y . 2 and x 1 y . 4? 7
1 34567890123456789012345678901212345678901234567890123456789012123456 7
12234567890123456789012345678901212345678901234567890123456789012123456 7
1 34567890123456789012345678901212345678901234567890123456789012123456
(A) (1,3) 7
12234567890123456789012345678901212345678901234567890123456789012123456 7
1234567890123456789012345678901212345678901234567890123456789012123456
(B) (2,3) 7
1234567890123456789012345678901212345678901234567890123456789012123456 7
1234567890123456789012345678901212345678901234567890123456789012123456
(C) (4,0)
34567890123456789012345678901212345678901234567890123456789012123456
7
7
1
12234567890123456789012345678901212345678901234567890123456789012123456
(D) (3,2) If the two triangles above are congruent 7
1234567890123456789012345678901212345678901234567890123456789012123456 7
1234567890123456789012345678901212345678901234567890123456789012123456
(E) (4,1) then b 5 7
1234567890123456789012345678901212345678901234567890123456789012123456
34567890123456789012345678901212345678901234567890123456789012123456
7
7
1 (A) 30
12234567890123456789012345678901212345678901234567890123456789012123456
x 1 7
1234567890123456789012345678901212345678901234567890123456789012123456
11. If x and y are positive integers and 5 , (B) 40 7
1234567890123456789012345678901212345678901234567890123456789012123456
y 2 7
1234567890123456789012345678901212345678901234567890123456789012123456
(C) 45
34567890123456789012345678901212345678901234567890123456789012123456
7
7
1 then x 1 y 5
12234567890123456789012345678901212345678901234567890123456789012123456
(D) 60 7
1234567890123456789012345678901212345678901234567890123456789012123456 7
1234567890123456789012345678901212345678901234567890123456789012123456
(A) 3x (E) It cannot be determined. 7
1234567890123456789012345678901212345678901234567890123456789012123456 7
1234567890123456789012345678901212345678901234567890123456789012123456 77
34567890123456789012345678901212345678901234567890123456789012123456
(B) 5x
1234567890123456789012345678901212345678901234567890123456789012123456
(C) y
1234567890123456789012345678901212345678901234567890123456789012123456
14. 7
1234567890123456789012345678901212345678901234567890123456789012123456
(D) 2y 7
1234567890123456789012345678901212345678901234567890123456789012123456 77
1234567890123456789012345678901212345678901234567890123456789012123456
(E) 3y
7
1
12234567890123456789012345678901212345678901234567890123456789012123456 7
1234567890123456789012345678901212345678901234567890123456789012123456 7
1234567890123456789012345678901212345678901234567890123456789012123456
12. Quentin buys three hot dogs with a 7
34567890123456789012345678901212345678901234567890123456789012123456
1234567890123456789012345678901212345678901234567890123456789012123456 7
1234567890123456789012345678901212345678901234567890123456789012123456
ten-dollar bill and receives seven dollars 7
1234567890123456789012345678901212345678901234567890123456789012123456 7
1234567890123456789012345678901212345678901234567890123456789012123456 77
and thirty-four cents in change. If the sales
1234567890123456789012345678901212345678901234567890123456789012123456
1234567890123456789012345678901212345678901234567890123456789012123456
tax is seven cents per dollar (rounding to 7
1234567890123456789012345678901212345678901234567890123456789012123456 7
1234567890123456789012345678901212345678901234567890123456789012123456 77
the nearest penny), which of the following
1234567890123456789012345678901212345678901234567890123456789012123456 7
1 choices, in cents, is closest to the actual What is the area of the above figure?
12234567890123456789012345678901212345678901234567890123456789012123456 7
1234567890123456789012345678901212345678901234567890123456789012123456
price of a hot dog? 7
1 34567890123456789012345678901212345678901234567890123456789012123456
(A) 2=2 7
1234567890123456789012345678901212345678901234567890123456789012123456
(A) 78 7
12234567890123456789012345678901212345678901234567890123456789012123456
34567890123456789012345678901212345678901234567890123456789012123456
7
7
1234567890123456789012345678901212345678901234567890123456789012123456
(B) 82 (B) 4
1234567890123456789012345678901212345678901234567890123456789012123456 7
1 (C) 86 (C) 8 7
12234567890123456789012345678901212345678901234567890123456789012123456 7
1234567890123456789012345678901212345678901234567890123456789012123456
(D) 89 (D) 9 7
34567890123456789012345678901212345678901234567890123456789012123456
1234567890123456789012345678901212345678901234567890123456789012123456 7
1234567890123456789012345678901212345678901234567890123456789012123456
(E) 92 7
1 (E) 12 7
1234567890123456789012345678901212345678901234567890123456789012123456 7
1234567890123456789012345678901212345678901234567890123456789012123456 7
12234567890123456789012345678901212345678901234567890123456789012123456 7
1234567890123456789012345678901212345678901234567890123456789012123456 7
1234567890123456789012345678901212345678901234567890123456789012123456 7
1234567890123456789012345678901212345678901234567890123456789012123456 7
1 34567890123456789012345678901212345678901234567890123456789012123456 7
1234567890123456789012345678901212345678901234567890123456789012123456 77
2 34567890123456789012345678901212345678901234567890123456789012123456
1234567890123456789012345678901212345678901234567890123456789012123456 7
1234567890123456789012345678901212345678901234567890123456789012123456 7
1234567890123456789012345678901212345678901234567890123456789012123456 7
1
12234567890123456789012345678901212345678901234567890123456789012123456 7
1234567890123456789012345678901212345678901234567890123456789012123456 7
1234567890123456789012345678901212345678901234567890123456789012123456 7
1234567890123456789012345678901212345678901234567890123456789012123456 7
1 34567890123456789012345678901212345678901234567890123456789012123456 7
1234567890123456789012345678901212345678901234567890123456789012123456 77
2 34567890123456789012345678901212345678901234567890123456789012123456
1
12234567890123456789012345678901212345678901234567890123456789012123456 7
1 34567890123456789012345678901212345678901234567890123456789012123456 7
12234567890123456789012345678901212345678901234567890123456789012123456 7
1 34567890123456789012345678901212345678901234567890123456789012123456 7
1234567890123456789012345678901212345678901234567890123456789012123456 7
1234567890123456789012345678901212345678901234567890123456789012123456 7
1234567890123456789012345678901212345678901234567890123456789012123456 7
12345678901234567890123456789012123456789012345678901234567890121234567
Copyright © 2005 Thomson Peterson’s, a part of The Thomson Corporaton 35
SAT is a registered trademark of the College Entrance Examination Board, which
was not involved in the production of and does not endorse this product.
12345678901234567890123456789012123456789012345678901234567890121234567
1234567890123456789012345678901212345678901234567890123456789012123456 7
12 34567890123456789012345678901212345678901234567890123456789012123456 7
1234567890123456789012345678901212345678901234567890123456789012123456
Questions 15–16 refer to the following chart. 17. If l1i l2, then which of the following pairs 7
12 34567890123456789012345678901212345678901234567890123456789012123456 7
12 34567890123456789012345678901212345678901234567890123456789012123456
of angles must be congruent? 7
1234567890123456789012345678901212345678901234567890123456789012123456 7
12 34567890123456789012345678901212345678901234567890123456789012123456 7
1234567890123456789012345678901212345678901234567890123456789012123456 7
12 34567890123456789012345678901212345678901234567890123456789012123456 7
12 34567890123456789012345678901212345678901234567890123456789012123456 7
12 34567890123456789012345678901212345678901234567890123456789012123456 7
12 34567890123456789012345678901212345678901234567890123456789012123456 7
1 34567890123456789012345678901212345678901234567890123456789012123456 7
2
12 34567890123456789012345678901212345678901234567890123456789012123456 7
12 34567890123456789012345678901212345678901234567890123456789012123456 7
12 34567890123456789012345678901212345678901234567890123456789012123456 7
12 34567890123456789012345678901212345678901234567890123456789012123456 7
1 34567890123456789012345678901212345678901234567890123456789012123456 7
2
12 34567890123456789012345678901212345678901234567890123456789012123456 7
12 34567890123456789012345678901212345678901234567890123456789012123456 7
12 34567890123456789012345678901212345678901234567890123456789012123456 7
12 34567890123456789012345678901212345678901234567890123456789012123456 7
1 34567890123456789012345678901212345678901234567890123456789012123456 7
2
12 34567890123456789012345678901212345678901234567890123456789012123456 7
12 34567890123456789012345678901212345678901234567890123456789012123456 7
12 34567890123456789012345678901212345678901234567890123456789012123456 7
12 34567890123456789012345678901212345678901234567890123456789012123456
I. 6 and 12 7
2 34567890123456789012345678901212345678901234567890123456789012123456
1234567890123456789012345678901212345678901234567890123456789012123456 7
1234567890123456789012345678901212345678901234567890123456789012123456
II. 2 and 9 7
1234567890123456789012345678901212345678901234567890123456789012123456 7
III. 4 and
1234567890123456789012345678901212345678901234567890123456789012123456 10 7
1234567890123456789012345678901212345678901234567890123456789012123456 77
15. Which year had the least percentage
1234567890123456789012345678901212345678901234567890123456789012123456
(A) I only
7
1 difference in reported incidence of flu and
12 34567890123456789012345678901212345678901234567890123456789012123456
(B) II only 7
12 34567890123456789012345678901212345678901234567890123456789012123456
cold? 7
12 34567890123456789012345678901212345678901234567890123456789012123456
(C) III only 7
2 34567890123456789012345678901212345678901234567890123456789012123456
1234567890123456789012345678901212345678901234567890123456789012123456 7
1234567890123456789012345678901212345678901234567890123456789012123456
(A) 1960 (D) II and III only 7
1234567890123456789012345678901212345678901234567890123456789012123456
(B) 1970 7
1234567890123456789012345678901212345678901234567890123456789012123456
(E) I and II only 7
1234567890123456789012345678901212345678901234567890123456789012123456
(C) 1980 7
1234567890123456789012345678901212345678901234567890123456789012123456 7
1234567890123456789012345678901212345678901234567890123456789012123456
(D) 1990 2b 2c
7
1234567890123456789012345678901212345678901234567890123456789012123456 77
18. If 2a is a positive integer, and a, b,
1234567890123456789012345678901212345678901234567890123456789012123456
(E) 2000
7
1 and c are integers, then
1234567890123456789012345678901212345678901234567890123456789012123456 77
2 34567890123456789012345678901212345678901234567890123456789012123456
1234567890123456789012345678901212345678901234567890123456789012123456
(A) a must be negative.
7
1 16. In percentage terms, in what decade was
1234567890123456789012345678901212345678901234567890123456789012123456
(B) b must be negative. 7
12
2
34567890123456789012345678901212345678901234567890123456789012123456 7
the number of reported cold cases about
34567890123456789012345678901212345678901234567890123456789012123456 7
1234567890123456789012345678901212345678901234567890123456789012123456
(C) c must be negative.
1234567890123456789012345678901212345678901234567890123456789012123456
25% greater than the number of reported 7
1 (D) b must be an even positive integer. 7
12 34567890123456789012345678901212345678901234567890123456789012123456
flu cases? 7
12 34567890123456789012345678901212345678901234567890123456789012123456
(E) None of the above. 7
2 34567890123456789012345678901212345678901234567890123456789012123456
1234567890123456789012345678901212345678901234567890123456789012123456 7
(A) 1960
1234567890123456789012345678901212345678901234567890123456789012123456 7
1 (B) 1970 7
1234567890123456789012345678901212345678901234567890123456789012123456 7
1234567890123456789012345678901212345678901234567890123456789012123456
(C) 1980 7
12 34567890123456789012345678901212345678901234567890123456789012123456 7
12 34567890123456789012345678901212345678901234567890123456789012123456
(D) 1990 7
12 34567890123456789012345678901212345678901234567890123456789012123456 7
12 34567890123456789012345678901212345678901234567890123456789012123456
(E) 2000 7
1234567890123456789012345678901212345678901234567890123456789012123456 7
1234567890123456789012345678901212345678901234567890123456789012123456 77
2 34567890123456789012345678901212345678901234567890123456789012123456
1234567890123456789012345678901212345678901234567890123456789012123456 7
1234567890123456789012345678901212345678901234567890123456789012123456 7
1234567890123456789012345678901212345678901234567890123456789012123456 7
1
12 34567890123456789012345678901212345678901234567890123456789012123456 7
12 34567890123456789012345678901212345678901234567890123456789012123456 7
12 34567890123456789012345678901212345678901234567890123456789012123456 7
12 34567890123456789012345678901212345678901234567890123456789012123456 7
1234567890123456789012345678901212345678901234567890123456789012123456 7
1234567890123456789012345678901212345678901234567890123456789012123456
2 7
1 34567890123456789012345678901212345678901234567890123456789012123456 7
12 34567890123456789012345678901212345678901234567890123456789012123456 7
1234567890123456789012345678901212345678901234567890123456789012123456 7
12 34567890123456789012345678901212345678901234567890123456789012123456 7
1234567890123456789012345678901212345678901234567890123456789012123456 7
1234567890123456789012345678901212345678901234567890123456789012123456 7
1234567890123456789012345678901212345678901234567890123456789012123456 7
1234567890123456789012345678901212345678901234567890123456789012123456 7
12345678901234567890123456789012123456789012345678901234567890121234567
36 Copyright © 2005 Thomson Peterson’s, a part of The Thomson Corporaton
SAT is a registered trademark of the College Entrance Examination Board, which
was not involved in the production of and does not endorse this product.
12345678901234567890123456789012123456789012345678901234567890121234567
1234567890123456789012345678901212345678901234567890123456789012123456 7
12234567890123456789012345678901212345678901234567890123456789012123456 7
1 34567890123456789012345678901212345678901234567890123456789012123456
19. 21. There are 150 students in math courses at 7
12234567890123456789012345678901212345678901234567890123456789012123456 7
1234567890123456789012345678901212345678901234567890123456789012123456
Dagmar High School. 73 are in geometry, 7
1 34567890123456789012345678901212345678901234567890123456789012123456 7
12234567890123456789012345678901212345678901234567890123456789012123456
62 are in algebra, and 52 are in neither.
34567890123456789012345678901212345678901234567890123456789012123456
7
7
1
12234567890123456789012345678901212345678901234567890123456789012123456
How many students are in both geometry 7
1234567890123456789012345678901212345678901234567890123456789012123456 7
1234567890123456789012345678901212345678901234567890123456789012123456
and algebra? 7
1234567890123456789012345678901212345678901234567890123456789012123456
34567890123456789012345678901212345678901234567890123456789012123456
7
7
1
12234567890123456789012345678901212345678901234567890123456789012123456
(A) 32 7
1234567890123456789012345678901212345678901234567890123456789012123456 7
1234567890123456789012345678901212345678901234567890123456789012123456
(B) 33 7
1234567890123456789012345678901212345678901234567890123456789012123456
34567890123456789012345678901212345678901234567890123456789012123456
7
7
1 (C) 35
12234567890123456789012345678901212345678901234567890123456789012123456 7
1234567890123456789012345678901212345678901234567890123456789012123456
(D) 36 7
1234567890123456789012345678901212345678901234567890123456789012123456
If all the line segments in the above figure 7
1234567890123456789012345678901212345678901234567890123456789012123456
are congruent, then
(E) 37
34567890123456789012345678901212345678901234567890123456789012123456
7
7
1
12234567890123456789012345678901212345678901234567890123456789012123456 7
1234567890123456789012345678901212345678901234567890123456789012123456
(A) a . b 7
1234567890123456789012345678901212345678901234567890123456789012123456 7
1234567890123456789012345678901212345678901234567890123456789012123456
(B) a , b 7
34567890123456789012345678901212345678901234567890123456789012123456
1234567890123456789012345678901212345678901234567890123456789012123456 7
1234567890123456789012345678901212345678901234567890123456789012123456
(C) 2a 5 b 7
1234567890123456789012345678901212345678901234567890123456789012123456 7
1234567890123456789012345678901212345678901234567890123456789012123456
(D) a 5 b 7
1234567890123456789012345678901212345678901234567890123456789012123456 7
1234567890123456789012345678901212345678901234567890123456789012123456 77
(E) It cannot be determined.
1
12234567890123456789012345678901212345678901234567890123456789012123456 7
1234567890123456789012345678901212345678901234567890123456789012123456 7
1234567890123456789012345678901212345678901234567890123456789012123456
20. Two boys and two girls are assigned to sit 7
34567890123456789012345678901212345678901234567890123456789012123456
1234567890123456789012345678901212345678901234567890123456789012123456 7
1234567890123456789012345678901212345678901234567890123456789012123456
at a five-seat circular table, where the 7
1234567890123456789012345678901212345678901234567890123456789012123456
seats are numbered one through five. If 7
1234567890123456789012345678901212345678901234567890123456789012123456 77
1234567890123456789012345678901212345678901234567890123456789012123456
neither boy can sit by the open seat, seat
1234567890123456789012345678901212345678901234567890123456789012123456 7
1234567890123456789012345678901212345678901234567890123456789012123456
3, how many different seating arrange- 7
1234567890123456789012345678901212345678901234567890123456789012123456 77
1234567890123456789012345678901212345678901234567890123456789012123456
ments are possible?
7
1
2 34567890123456789012345678901212345678901234567890123456789012123456
1234567890123456789012345678901212345678901234567890123456789012123456 7
1234567890123456789012345678901212345678901234567890123456789012123456
(A) 2 7
1 7
1234567890123456789012345678901212345678901234567890123456789012123456
(B) 3 7
12234567890123456789012345678901212345678901234567890123456789012123456 7
1234567890123456789012345678901212345678901234567890123456789012123456 77
34567890123456789012345678901212345678901234567890123456789012123456
(C) 4
1234567890123456789012345678901212345678901234567890123456789012123456 7
1 (D) 5
12234567890123456789012345678901212345678901234567890123456789012123456 7
1234567890123456789012345678901212345678901234567890123456789012123456
(E) 6 7
1234567890123456789012345678901212345678901234567890123456789012123456 77
34567890123456789012345678901212345678901234567890123456789012123456
1234567890123456789012345678901212345678901234567890123456789012123456 7
1
1234567890123456789012345678901212345678901234567890123456789012123456 7
1234567890123456789012345678901212345678901234567890123456789012123456 7
12234567890123456789012345678901212345678901234567890123456789012123456 7
1234567890123456789012345678901212345678901234567890123456789012123456 7
1234567890123456789012345678901212345678901234567890123456789012123456 7
1234567890123456789012345678901212345678901234567890123456789012123456 7
1 34567890123456789012345678901212345678901234567890123456789012123456 7
1234567890123456789012345678901212345678901234567890123456789012123456 77
2 34567890123456789012345678901212345678901234567890123456789012123456
1234567890123456789012345678901212345678901234567890123456789012123456 7
1234567890123456789012345678901212345678901234567890123456789012123456 7
1234567890123456789012345678901212345678901234567890123456789012123456 7
1
12234567890123456789012345678901212345678901234567890123456789012123456 7
1234567890123456789012345678901212345678901234567890123456789012123456 7
1234567890123456789012345678901212345678901234567890123456789012123456 7
1234567890123456789012345678901212345678901234567890123456789012123456 7
1 34567890123456789012345678901212345678901234567890123456789012123456 7
1234567890123456789012345678901212345678901234567890123456789012123456 77
2 34567890123456789012345678901212345678901234567890123456789012123456
1
12234567890123456789012345678901212345678901234567890123456789012123456 7
1 34567890123456789012345678901212345678901234567890123456789012123456 7
12234567890123456789012345678901212345678901234567890123456789012123456 7
STOP
1 34567890123456789012345678901212345678901234567890123456789012123456
Do not proceed to the next section until time is up.
1234567890123456789012345678901212345678901234567890123456789012123456 7
7
1234567890123456789012345678901212345678901234567890123456789012123456 7
1234567890123456789012345678901212345678901234567890123456789012123456 7
12345678901234567890123456789012123456789012345678901234567890121234567
Copyright © 2005 Thomson Peterson’s, a part of The Thomson Corporaton 37
SAT is a registered trademark of the College Entrance Examination Board, which
was not involved in the production of and does not endorse this product.
Section 6
16 Questions j Time—20 Minutes

12345678901234567890123456789012123456789012345678901234567890121234567
12 34567890123456789012345678901212345678901234567890123456789012123456 7
12 34567890123456789012345678901212345678901234567890123456789012123456 7
1 34567890123456789012345678901212345678901234567890123456789012123456 7
2
12 34567890123456789012345678901212345678901234567890123456789012123456 7
12 34567890123456789012345678901212345678901234567890123456789012123456
Directions: Each passage below is followed by a set of questions. Read each passage, then 7
12 34567890123456789012345678901212345678901234567890123456789012123456
answer the accompanying questions, basing your answers on what is stated or implied in the 7
12 34567890123456789012345678901212345678901234567890123456789012123456 7
2 34567890123456789012345678901212345678901234567890123456789012123456
1234567890123456789012345678901212345678901234567890123456789012123456
passage and any introductory material provided. Mark the letter of your choice on the answer 7
1234567890123456789012345678901212345678901234567890123456789012123456 7
1234567890123456789012345678901212345678901234567890123456789012123456
sheet that best corresponds to the correct answer. 7
1234567890123456789012345678901212345678901234567890123456789012123456 77
1234567890123456789012345678901212345678901234567890123456789012123456 7
1234567890123456789012345678901212345678901234567890123456789012123456 7
1
12 34567890123456789012345678901212345678901234567890123456789012123456 7
12 34567890123456789012345678901212345678901234567890123456789012123456
Line In 1953, Watson and Crick unlocked the 2. The passage uses the phrase “wet and 7
12 34567890123456789012345678901212345678901234567890123456789012123456 7
2 34567890123456789012345678901212345678901234567890123456789012123456
1234567890123456789012345678901212345678901234567890123456789012123456
structure of the DNA molecule and set dirty” (line 5) to mean 7
1234567890123456789012345678901212345678901234567890123456789012123456 7
1234567890123456789012345678901212345678901234567890123456789012123456
into motion the modern study of genetics.
(A) haphazard guessing about the genetic 7
7
1234567890123456789012345678901212345678901234567890123456789012123456
1234567890123456789012345678901212345678901234567890123456789012123456
This advance allowed our study of life to 7
1234567890123456789012345678901212345678901234567890123456789012123456 code. 7
1234567890123456789012345678901212345678901234567890123456789012123456
(5) go beyond the so-called wet and dirty 7
1234567890123456789012345678901212345678901234567890123456789012123456
realm of biology, the complicated labora- (B) the work of Watson and Crick in 7
1234567890123456789012345678901212345678901234567890123456789012123456 7
1 tory study of proteins, cells, organelles, discovering DNA. 7
2 34567890123456789012345678901212345678901234567890123456789012123456
1234567890123456789012345678901212345678901234567890123456789012123456 7
1234567890123456789012345678901212345678901234567890123456789012123456
ions, and lipids. The study of life could (C) information-based biological 7
1 7
1234567890123456789012345678901212345678901234567890123456789012123456 7
now be performed with more abstract research.
12 34567890123456789012345678901212345678901234567890123456789012123456 7
12 34567890123456789012345678901212345678901234567890123456789012123456
(10) methods of analysis. By discovering the (D) the study of the genetic code. 7
12 34567890123456789012345678901212345678901234567890123456789012123456
basic structure of DNA, we had received 7
1234567890123456789012345678901212345678901234567890123456789012123456(E) involved laboratory practices in 7
12 34567890123456789012345678901212345678901234567890123456789012123456
our first glance into the information-based 7
12 34567890123456789012345678901212345678901234567890123456789012123456 studying basic biological entities. 7
1234567890123456789012345678901212345678901234567890123456789012123456 77
2 34567890123456789012345678901212345678901234567890123456789012123456
realm locked inside the genetic code.
1234567890123456789012345678901212345678901234567890123456789012123456 7
1 Although little-known today in the United
1234567890123456789012345678901212345678901234567890123456789012123456
1. Which of the following does the passage 7
1234567890123456789012345678901212345678901234567890123456789012123456
States, Clark Saunders (1859–1941) cast a large 7
2 34567890123456789012345678901212345678901234567890123456789012123456
1234567890123456789012345678901212345678901234567890123456789012123456
discuss as a change that the discovery of 7
1234567890123456789012345678901212345678901234567890123456789012123456
shadow in the first several decades of the 7
1234567890123456789012345678901212345678901234567890123456789012123456
DNA brought to the study of life? 7
1234567890123456789012345678901212345678901234567890123456789012123456 77
twentieth century, writing many widely read
1
12 34567890123456789012345678901212345678901234567890123456789012123456
(A) The study of lipids and proteins books on Native American, Spanish, and Anglo 7
12 34567890123456789012345678901212345678901234567890123456789012123456
became irrelevant. 7
12 34567890123456789012345678901212345678901234567890123456789012123456
folklore. He also wrote extensively on the 7
12 34567890123456789012345678901212345678901234567890123456789012123456
(B) New and more abstract methods of 7
1234567890123456789012345678901212345678901234567890123456789012123456
different cultures of California, the Sierras, and 7
2 34567890123456789012345678901212345678901234567890123456789012123456
1234567890123456789012345678901212345678901234567890123456789012123456
study were possible. 7
1234567890123456789012345678901212345678901234567890123456789012123456
the Southwest. He was a major and influential 7
1234567890123456789012345678901212345678901234567890123456789012123456
(C) Biology could then focus on mol- 7
1234567890123456789012345678901212345678901234567890123456789012123456
contributor to Sunset Magazine in its early 7
1 ecules rather than cells. 7
2 34567890123456789012345678901212345678901234567890123456789012123456
1234567890123456789012345678901212345678901234567890123456789012123456
(D) Modern genetics matured past its years. In his day, Saunders was important for 7
1 7
12 34567890123456789012345678901212345678901234567890123456789012123456
Mendelian roots. introducing much of the American public to a 7
1234567890123456789012345678901212345678901234567890123456789012123456 7
12
2
34567890123456789012345678901212345678901234567890123456789012123456 7
(E) Information-based study of genes person-sized
34567890123456789012345678901212345678901234567890123456789012123456
understanding of the “Old West.”
7
1
1234567890123456789012345678901212345678901234567890123456789012123456
became obsolete. 7
1234567890123456789012345678901212345678901234567890123456789012123456 77
1234567890123456789012345678901212345678901234567890123456789012123456
12345678901234567890123456789012123456789012345678901234567890121234567
38 Copyright © 2005 Thomson Peterson’s, a part of The Thomson Corporaton
SAT is a registered trademark of the College Entrance Examination Board, which
was not involved in the production of and does not endorse this product.
12345678901234567890123456789012123456789012345678901234567890121234567
1234567890123456789012345678901212345678901234567890123456789012123456 7
12234567890123456789012345678901212345678901234567890123456789012123456 7
1 34567890123456789012345678901212345678901234567890123456789012123456
3. The passage presents Saunders as a(n) The following two passages deal with the 7
12234567890123456789012345678901212345678901234567890123456789012123456 7
1234567890123456789012345678901212345678901234567890123456789012123456
political movements working for the woman’s 7
1 34567890123456789012345678901212345678901234567890123456789012123456
(A) influential contemporary western 7
12234567890123456789012345678901212345678901234567890123456789012123456
vote in America.
34567890123456789012345678901212345678901234567890123456789012123456
7
7
1 writer.
12234567890123456789012345678901212345678901234567890123456789012123456 7
1234567890123456789012345678901212345678901234567890123456789012123456
(B) important historian of the West. 7
1234567890123456789012345678901212345678901234567890123456789012123456
Passage 1 7
1234567890123456789012345678901212345678901234567890123456789012123456
(C) a specialist of Native American
34567890123456789012345678901212345678901234567890123456789012123456
7
7
1 Line The first organized assertion of woman’s
12234567890123456789012345678901212345678901234567890123456789012123456
studies. 7
1234567890123456789012345678901212345678901234567890123456789012123456
rights in the United States was made at the 7
1234567890123456789012345678901212345678901234567890123456789012123456
(D) widely read author in his own day. 7
1234567890123456789012345678901212345678901234567890123456789012123456
Seneca Falls convention in 1848. The
34567890123456789012345678901212345678901234567890123456789012123456
7
7
1 (E) the first editor of Sunset Magazine.
12234567890123456789012345678901212345678901234567890123456789012123456
convention, though, had little immediate 7
1234567890123456789012345678901212345678901234567890123456789012123456 7
1234567890123456789012345678901212345678901234567890123456789012123456
The history of rock and roll is inseparable from (5) impact because of the national issues that 7
1234567890123456789012345678901212345678901234567890123456789012123456 7
1 34567890123456789012345678901212345678901234567890123456789012123456
the development of blues and gospel music in would soon embroil the country. The 7
12234567890123456789012345678901212345678901234567890123456789012123456 7
1234567890123456789012345678901212345678901234567890123456789012123456
the southeastern United States. Though the contentious debates involving slavery and 7
1234567890123456789012345678901212345678901234567890123456789012123456 7
1234567890123456789012345678901212345678901234567890123456789012123456
genre gained mass appeal through legendary state’s rights that preceded the Civil War 7
34567890123456789012345678901212345678901234567890123456789012123456
1234567890123456789012345678901212345678901234567890123456789012123456
soon took center stage in national debates. 7
7
1234567890123456789012345678901212345678901234567890123456789012123456
figures such as Elvis Presley or the wildly
Thus woman’s rights issues would have to 7
1234567890123456789012345678901212345678901234567890123456789012123456
1234567890123456789012345678901212345678901234567890123456789012123456
popular Beatles, the musical roots of rock and (10) 7
1234567890123456789012345678901212345678901234567890123456789012123456 7
1234567890123456789012345678901212345678901234567890123456789012123456 77
roll extend far before such groups. In fact, wait until the war and its antecedent
1 many of the groups who popularized rock and
12234567890123456789012345678901212345678901234567890123456789012123456
problems had been addressed before they 7
1234567890123456789012345678901212345678901234567890123456789012123456 7
1234567890123456789012345678901212345678901234567890123456789012123456
roll were consciously attempting to emulate the would be addressed. 7
34567890123456789012345678901212345678901234567890123456789012123456
1234567890123456789012345678901212345678901234567890123456789012123456 7
1234567890123456789012345678901212345678901234567890123456789012123456
work of blues greats such as B. B. King or In 1869, two organizations were 7
1234567890123456789012345678901212345678901234567890123456789012123456 7
1234567890123456789012345678901212345678901234567890123456789012123456 77
Muddy Waters. The Rolling Stones are a good (15) formed that would play important roles in
1234567890123456789012345678901212345678901234567890123456789012123456
1234567890123456789012345678901212345678901234567890123456789012123456
example of this trend, which developed in the securing the woman’s right to vote. The 7
1234567890123456789012345678901212345678901234567890123456789012123456 7
1234567890123456789012345678901212345678901234567890123456789012123456 77
late fifties and early sixties. The Rolling Stones, first was the American Woman’s Suffrage
1234567890123456789012345678901212345678901234567890123456789012123456
1 both then and now, have always explicitly Association (AWSA). Leaving federal and
7
2 34567890123456789012345678901212345678901234567890123456789012123456
1234567890123456789012345678901212345678901234567890123456789012123456
stated their admiration and imitation of blues constitutional issues aside, the AWSA 7
1234567890123456789012345678901212345678901234567890123456789012123456 7
1 greats. (20) focused their attention on state-level 7
1234567890123456789012345678901212345678901234567890123456789012123456 7
12234567890123456789012345678901212345678901234567890123456789012123456
politics. They also restricted their ambi- 7
1234567890123456789012345678901212345678901234567890123456789012123456 7
1234567890123456789012345678901212345678901234567890123456789012123456
tions to securing the woman’s vote and 7
1 34567890123456789012345678901212345678901234567890123456789012123456
4. B. B. King is used in this passage as an 7
12234567890123456789012345678901212345678901234567890123456789012123456
downplayed discussion of women’s full 7
1234567890123456789012345678901212345678901234567890123456789012123456
example of a 7
1234567890123456789012345678901212345678901234567890123456789012123456 77
34567890123456789012345678901212345678901234567890123456789012123456
equality. Taking a different track, the
1234567890123456789012345678901212345678901234567890123456789012123456
(A) blues artist who was emulated by 7
1 (25) National Woman’s Suffrage Association
1234567890123456789012345678901212345678901234567890123456789012123456
early rock bands. 7
1234567890123456789012345678901212345678901234567890123456789012123456 7
(NWSA), led by Elizabeth Stanton and
2 34567890123456789012345678901212345678901234567890123456789012123456
1234567890123456789012345678901212345678901234567890123456789012123456
(B) musical artist influenced by Elvis 7
1234567890123456789012345678901212345678901234567890123456789012123456
Susan B. Anthony, believed that the only 7
1234567890123456789012345678901212345678901234567890123456789012123456
Presley. 7
1234567890123456789012345678901212345678901234567890123456789012123456 77
way to assure the long-term security of the
1 (C) musician who incorporated aspects
12234567890123456789012345678901212345678901234567890123456789012123456
woman’s vote was to ground it in the 7
1234567890123456789012345678901212345678901234567890123456789012123456
of rock and roll. 7
1234567890123456789012345678901212345678901234567890123456789012123456
(30) constitution. The NWSA challenged the 7
1234567890123456789012345678901212345678901234567890123456789012123456
(D) musician who often played with 7
1 34567890123456789012345678901212345678901234567890123456789012123456
exclusion of woman from the Fifteenth 7
2 34567890123456789012345678901212345678901234567890123456789012123456
1234567890123456789012345678901212345678901234567890123456789012123456
Muddy Waters. 7
1234567890123456789012345678901212345678901234567890123456789012123456
Amendment, the amendment that 7
1234567890123456789012345678901212345678901234567890123456789012123456
(E) gospel singer who influenced the 7
1234567890123456789012345678901212345678901234567890123456789012123456
extended the vote to African-American 7
1 Rolling Stones. men. Furthermore, the NWSA linked the 7
1234567890123456789012345678901212345678901234567890123456789012123456 77
2 34567890123456789012345678901212345678901234567890123456789012123456
1 fight for suffrage with other inequalities
12234567890123456789012345678901212345678901234567890123456789012123456
(35)
7
1 34567890123456789012345678901212345678901234567890123456789012123456
faced by woman, such as marriage laws, 7
12234567890123456789012345678901212345678901234567890123456789012123456
34567890123456789012345678901212345678901234567890123456789012123456
7
7
1 which greatly disadvantaged women.
1234567890123456789012345678901212345678901234567890123456789012123456 7
1234567890123456789012345678901212345678901234567890123456789012123456 77
1234567890123456789012345678901212345678901234567890123456789012123456
12345678901234567890123456789012123456789012345678901234567890121234567
Copyright © 2005 Thomson Peterson’s, a part of The Thomson Corporaton 39
SAT is a registered trademark of the College Entrance Examination Board, which
was not involved in the production of and does not endorse this product.
12345678901234567890123456789012123456789012345678901234567890121234567
1234567890123456789012345678901212345678901234567890123456789012123456 7
12 34567890123456789012345678901212345678901234567890123456789012123456 7
1234567890123456789012345678901212345678901234567890123456789012123456
By the late 1880s the differences that 5. The word “antecedent” in line 11 can best 7
12 34567890123456789012345678901212345678901234567890123456789012123456 7
12 34567890123456789012345678901212345678901234567890123456789012123456
separated the two organizations had be replaced by 7
1234567890123456789012345678901212345678901234567890123456789012123456 7
12
2
34567890123456789012345678901212345678901234567890123456789012123456
(40) receded in importance as the women’s
34567890123456789012345678901212345678901234567890123456789012123456
7
7
1 (A) antebellum.
12 34567890123456789012345678901212345678901234567890123456789012123456
movement had become a substantial and 7
12 34567890123456789012345678901212345678901234567890123456789012123456
(B) referent. 7
12 34567890123456789012345678901212345678901234567890123456789012123456
broad-based political force in the country. 7
12
2
34567890123456789012345678901212345678901234567890123456789012123456
(C)
34567890123456789012345678901212345678901234567890123456789012123456
causal. 7
7
1 In 1890, the two organizations joined
12 34567890123456789012345678901212345678901234567890123456789012123456
(D) subsequent. 7
12 34567890123456789012345678901212345678901234567890123456789012123456
forces under the title of the National 7
12 34567890123456789012345678901212345678901234567890123456789012123456
(E) abolitionist. 7
12
2
34567890123456789012345678901212345678901234567890123456789012123456 7
(45) American Woman’s Suffrage Association
34567890123456789012345678901212345678901234567890123456789012123456 7
1
12 34567890123456789012345678901212345678901234567890123456789012123456
(NAWSA). The NAWSA would go on to 6. Which of the following does the first 7
12 34567890123456789012345678901212345678901234567890123456789012123456 7
12 34567890123456789012345678901212345678901234567890123456789012123456
play a vital role in the further fight to passage say was the first organized push 7
12
2
34567890123456789012345678901212345678901234567890123456789012123456
34567890123456789012345678901212345678901234567890123456789012123456
7
7
1 achieve the woman’s vote. for woman’s suffrage?
12 34567890123456789012345678901212345678901234567890123456789012123456 7
12 34567890123456789012345678901212345678901234567890123456789012123456 7
12 34567890123456789012345678901212345678901234567890123456789012123456
Passage 2 (A) formation of the National Woman’s 7
12 34567890123456789012345678901212345678901234567890123456789012123456 7
2 34567890123456789012345678901212345678901234567890123456789012123456
1234567890123456789012345678901212345678901234567890123456789012123456 Suffrage Association 7
1234567890123456789012345678901212345678901234567890123456789012123456
In 1920, when Tennessee became the 7
1234567890123456789012345678901212345678901234567890123456789012123456
(B) formation of the American Woman’s 7
1234567890123456789012345678901212345678901234567890123456789012123456
(50) thirty-eighth state to approve the constitu- 7
1234567890123456789012345678901212345678901234567890123456789012123456 Suffrage Association 7
1234567890123456789012345678901212345678901234567890123456789012123456
tional amendment securing the woman’s 7
1 (C) convening of the Seneca Falls 7
12 34567890123456789012345678901212345678901234567890123456789012123456
right to vote, woman’s suffrage became 7
12 34567890123456789012345678901212345678901234567890123456789012123456 convention 7
12 34567890123456789012345678901212345678901234567890123456789012123456
enshrined in the constitution. But wom- 7
2 34567890123456789012345678901212345678901234567890123456789012123456
1234567890123456789012345678901212345678901234567890123456789012123456
(D) Tennessee passing the Twenty-Second 7
1234567890123456789012345678901212345678901234567890123456789012123456
an’s suffrage did not happen in one fell 7
1234567890123456789012345678901212345678901234567890123456789012123456 Amendment 7
1234567890123456789012345678901212345678901234567890123456789012123456
(55) swoop. The success of the woman’s
(E) “partial suffrages” of local woman’s 7
1234567890123456789012345678901212345678901234567890123456789012123456 7
1234567890123456789012345678901212345678901234567890123456789012123456
suffrage movement was the story of a
suffrage efforts 7
1234567890123456789012345678901212345678901234567890123456789012123456 7
1234567890123456789012345678901212345678901234567890123456789012123456 77
number of partial victories that led to the
1234567890123456789012345678901212345678901234567890123456789012123456
explicit endorsement of the woman’s right 7. What national event does the first passage 7
1
2 34567890123456789012345678901212345678901234567890123456789012123456
1234567890123456789012345678901212345678901234567890123456789012123456
to vote in the constitution. 7
1234567890123456789012345678901212345678901234567890123456789012123456
cite as pushing woman’s voting rights to 7
1 (60) As early as the 1870s and 1880s, 7
1234567890123456789012345678901212345678901234567890123456789012123456 7
the background of the national conscious-
12 34567890123456789012345678901212345678901234567890123456789012123456
women had begun to win the right to vote 7
12 34567890123456789012345678901212345678901234567890123456789012123456
ness? 7
12 34567890123456789012345678901212345678901234567890123456789012123456
in local affairs such as municipal elections, 7
1234567890123456789012345678901212345678901234567890123456789012123456 7
12 34567890123456789012345678901212345678901234567890123456789012123456
school board elections, or prohibition (A) Civil War 7
12 34567890123456789012345678901212345678901234567890123456789012123456 7
1234567890123456789012345678901212345678901234567890123456789012123456 77
2 34567890123456789012345678901212345678901234567890123456789012123456
measures. These “partial suffrages” (B) Suffrage movement
1234567890123456789012345678901212345678901234567890123456789012123456
(C) Prohibition 7
1 (65) demonstrated that women could in fact
1234567890123456789012345678901212345678901234567890123456789012123456
(D) Passage of the Fifteenth Amendment 7
1234567890123456789012345678901212345678901234567890123456789012123456 7
responsibly and reasonably participate in
12 34567890123456789012345678901212345678901234567890123456789012123456
(E) World War I 7
12 34567890123456789012345678901212345678901234567890123456789012123456
a representative democracy (at least as 7
12 34567890123456789012345678901212345678901234567890123456789012123456 7
12 34567890123456789012345678901212345678901234567890123456789012123456
voters). Once such successes were 7
1234567890123456789012345678901212345678901234567890123456789012123456
8. According to the first passage, the 7
2 34567890123456789012345678901212345678901234567890123456789012123456
1234567890123456789012345678901212345678901234567890123456789012123456
achieved and maintained over a period of 7
1234567890123456789012345678901212345678901234567890123456789012123456
National Woman’s Suffrage Association 7
1234567890123456789012345678901212345678901234567890123456789012123456
(70) time, restricting the full voting rights of 7
1234567890123456789012345678901212345678901234567890123456789012123456
focused their efforts on 7
1 woman became more and more suspect. If 7
2 34567890123456789012345678901212345678901234567890123456789012123456
1234567890123456789012345678901212345678901234567890123456789012123456 7
1234567890123456789012345678901212345678901234567890123456789012123456
women were helping decide who was on (A) local elections. 7
1234567890123456789012345678901212345678901234567890123456789012123456 7
1234567890123456789012345678901212345678901234567890123456789012123456 77
the local school board, why should they (B) constitutional issues.
1
12 not also have a voice in deciding who was
34567890123456789012345678901212345678901234567890123456789012123456
(C) prohibition efforts. 7
1234567890123456789012345678901212345678901234567890123456789012123456
president of the country? Such questions (D) school board elections. 7
12 34567890123456789012345678901212345678901234567890123456789012123456 7
(75)
1234567890123456789012345678901212345678901234567890123456789012123456
became more difficult for non-suffragists (E) state elections. 7
12
2
34567890123456789012345678901212345678901234567890123456789012123456
34567890123456789012345678901212345678901234567890123456789012123456
7
7
1 to answer, and thus the logic of restricting
1234567890123456789012345678901212345678901234567890123456789012123456 7
1234567890123456789012345678901212345678901234567890123456789012123456
the woman’s vote began to crumble. 7
1234567890123456789012345678901212345678901234567890123456789012123456 7
12345678901234567890123456789012123456789012345678901234567890121234567
40 Copyright © 2005 Thomson Peterson’s, a part of The Thomson Corporaton
SAT is a registered trademark of the College Entrance Examination Board, which
was not involved in the production of and does not endorse this product.
12345678901234567890123456789012123456789012345678901234567890121234567
1234567890123456789012345678901212345678901234567890123456789012123456 7
12234567890123456789012345678901212345678901234567890123456789012123456 7
1 34567890123456789012345678901212345678901234567890123456789012123456
9. The differences between the AWSA and 12. When is the earliest success of the 7
12234567890123456789012345678901212345678901234567890123456789012123456 7
1234567890123456789012345678901212345678901234567890123456789012123456
the NWSA were ultimately resolved when woman’s suffrage movement that the 7
1 34567890123456789012345678901212345678901234567890123456789012123456 7
12234567890123456789012345678901212345678901234567890123456789012123456
second passage
34567890123456789012345678901212345678901234567890123456789012123456
points to? 7
7
1 (A) the Twenty-Second Amendment
12234567890123456789012345678901212345678901234567890123456789012123456 7
1234567890123456789012345678901212345678901234567890123456789012123456
passed. (A) 1848 7
1234567890123456789012345678901212345678901234567890123456789012123456 7
1234567890123456789012345678901212345678901234567890123456789012123456
(B) the two organizations were com- (B) 1869
34567890123456789012345678901212345678901234567890123456789012123456
7
7
1
12234567890123456789012345678901212345678901234567890123456789012123456
bined to form the NAWSA. (C) 1870s 7
1234567890123456789012345678901212345678901234567890123456789012123456 7
1234567890123456789012345678901212345678901234567890123456789012123456
(C) the Civil War ended. (D) 1880s 7
1234567890123456789012345678901212345678901234567890123456789012123456
34567890123456789012345678901212345678901234567890123456789012123456
7
7
1 (D) prohibition passed. (E) 1920
12234567890123456789012345678901212345678901234567890123456789012123456 7
1234567890123456789012345678901212345678901234567890123456789012123456
(E) woman’s suffragists won significant 7
1234567890123456789012345678901212345678901234567890123456789012123456
13. Which of the following is NOT an 7
1234567890123456789012345678901212345678901234567890123456789012123456
victories in the 1890 general election.
34567890123456789012345678901212345678901234567890123456789012123456
7
7
1 example of a “partial suffrage” as
12234567890123456789012345678901212345678901234567890123456789012123456 7
1234567890123456789012345678901212345678901234567890123456789012123456
10. In Passage 1, the author’s attitude toward described in the second passage? 7
1234567890123456789012345678901212345678901234567890123456789012123456 7
1234567890123456789012345678901212345678901234567890123456789012123456
the subject matter is 7
1234567890123456789012345678901212345678901234567890123456789012123456 77
34567890123456789012345678901212345678901234567890123456789012123456
(A) A mayoral election
1234567890123456789012345678901212345678901234567890123456789012123456
1234567890123456789012345678901212345678901234567890123456789012123456
(A) intense scrutiny. (B) A school board measure 7
1234567890123456789012345678901212345678901234567890123456789012123456 7
1234567890123456789012345678901212345678901234567890123456789012123456 77
(B) distanced suspicion. (C) Passage of the Fifteenth Amendment
1234567890123456789012345678901212345678901234567890123456789012123456 7
1 (C) mild censure. (D) A state prohibition referendum
12234567890123456789012345678901212345678901234567890123456789012123456 7
1234567890123456789012345678901212345678901234567890123456789012123456
(D) appreciative description. (E) Impeaching a city council member 7
1234567890123456789012345678901212345678901234567890123456789012123456 7
1234567890123456789012345678901212345678901234567890123456789012123456 77
34567890123456789012345678901212345678901234567890123456789012123456
(E) enthusiastic support.
1234567890123456789012345678901212345678901234567890123456789012123456
14. The author of the second passage argues
1234567890123456789012345678901212345678901234567890123456789012123456 7
1234567890123456789012345678901212345678901234567890123456789012123456
11. Passage 2 locates the ultimate victory of that the “partial suffrages” were most 7
1234567890123456789012345678901212345678901234567890123456789012123456 7
1234567890123456789012345678901212345678901234567890123456789012123456 77
the woman’s suffrage movement with effective in bringing full voting rights for
1234567890123456789012345678901212345678901234567890123456789012123456
1234567890123456789012345678901212345678901234567890123456789012123456
which of the following events? woman because 7
1234567890123456789012345678901212345678901234567890123456789012123456 77
1
12234567890123456789012345678901212345678901234567890123456789012123456
(A) Tennessee approving the woman’s (A) through them woman were able to 7
1234567890123456789012345678901212345678901234567890123456789012123456 7
1 34567890123456789012345678901212345678901234567890123456789012123456 7
voting rights amendment elect prosuffrage representatives.
1234567890123456789012345678901212345678901234567890123456789012123456 7
12234567890123456789012345678901212345678901234567890123456789012123456
(B) Congress passing the Twenty-Second (B) they showed women voting ably.
34567890123456789012345678901212345678901234567890123456789012123456
7
(C) they demonstrated that woman could 7
1234567890123456789012345678901212345678901234567890123456789012123456
1234567890123456789012345678901212345678901234567890123456789012123456
Amendment 7
1 7
12234567890123456789012345678901212345678901234567890123456789012123456
(C) The combination of AWSA and participate in a full democracy. 7
1234567890123456789012345678901212345678901234567890123456789012123456 7
1234567890123456789012345678901212345678901234567890123456789012123456 77
34567890123456789012345678901212345678901234567890123456789012123456
NWSA into NAWSA (D) they demonstrated that woman could
1234567890123456789012345678901212345678901234567890123456789012123456
(D) Woman earning the full vote in handle the intricacies of foreign 7
1
1234567890123456789012345678901212345678901234567890123456789012123456
Wyoming policy. 7
1234567890123456789012345678901212345678901234567890123456789012123456 7
2 34567890123456789012345678901212345678901234567890123456789012123456
1234567890123456789012345678901212345678901234567890123456789012123456
(E) Women’s fruitful participation in (E) they established the power of the 7
1234567890123456789012345678901212345678901234567890123456789012123456 7
1234567890123456789012345678901212345678901234567890123456789012123456
local elections woman voter. 7
1234567890123456789012345678901212345678901234567890123456789012123456 77
1
12234567890123456789012345678901212345678901234567890123456789012123456 7
1234567890123456789012345678901212345678901234567890123456789012123456 7
1234567890123456789012345678901212345678901234567890123456789012123456 7
1234567890123456789012345678901212345678901234567890123456789012123456 7
1 34567890123456789012345678901212345678901234567890123456789012123456 7
1234567890123456789012345678901212345678901234567890123456789012123456 77
2 34567890123456789012345678901212345678901234567890123456789012123456
1234567890123456789012345678901212345678901234567890123456789012123456 7
1234567890123456789012345678901212345678901234567890123456789012123456 7
1234567890123456789012345678901212345678901234567890123456789012123456 7
1
12234567890123456789012345678901212345678901234567890123456789012123456 7
1 34567890123456789012345678901212345678901234567890123456789012123456 7
12234567890123456789012345678901212345678901234567890123456789012123456 7
1 34567890123456789012345678901212345678901234567890123456789012123456 7
12234567890123456789012345678901212345678901234567890123456789012123456 7
1 34567890123456789012345678901212345678901234567890123456789012123456 7
1234567890123456789012345678901212345678901234567890123456789012123456 7
1234567890123456789012345678901212345678901234567890123456789012123456 7
1234567890123456789012345678901212345678901234567890123456789012123456 7
12345678901234567890123456789012123456789012345678901234567890121234567
Copyright © 2005 Thomson Peterson’s, a part of The Thomson Corporaton 41
SAT is a registered trademark of the College Entrance Examination Board, which
was not involved in the production of and does not endorse this product.
12345678901234567890123456789012123456789012345678901234567890121234567
1234567890123456789012345678901212345678901234567890123456789012123456 7
12 34567890123456789012345678901212345678901234567890123456789012123456 7
1234567890123456789012345678901212345678901234567890123456789012123456
15. Which of the following questions is NOT 16. The author of the second passage would 7
12 34567890123456789012345678901212345678901234567890123456789012123456 7
12 34567890123456789012345678901212345678901234567890123456789012123456
addressed in either passage? most likely see the work of the 7
1234567890123456789012345678901212345678901234567890123456789012123456 7
12 34567890123456789012345678901212345678901234567890123456789012123456 7
1234567890123456789012345678901212345678901234567890123456789012123456
(A) When did the woman’s right to vote (A) AWSA as crucial for the ultimate 7
12 34567890123456789012345678901212345678901234567890123456789012123456 7
12 34567890123456789012345678901212345678901234567890123456789012123456 7
become a constitutional amendment? success of the suffrage movement.
12 34567890123456789012345678901212345678901234567890123456789012123456 7
12
2
34567890123456789012345678901212345678901234567890123456789012123456
(B) What effect did the Civil War have (B) NWSA as indispensable for “partial
34567890123456789012345678901212345678901234567890123456789012123456
7
7
1
12 34567890123456789012345678901212345678901234567890123456789012123456
on the woman’s suffrage movement? suffrages.” 7
12 34567890123456789012345678901212345678901234567890123456789012123456 7
12 34567890123456789012345678901212345678901234567890123456789012123456 7
(C) What are the names of two leaders of (C) NWSA as unimportant for the
12 34567890123456789012345678901212345678901234567890123456789012123456
passage of the woman’s voting rights 7
1234567890123456789012345678901212345678901234567890123456789012123456
the National Woman’s Suffrage 7
12 34567890123456789012345678901212345678901234567890123456789012123456 7
12 34567890123456789012345678901212345678901234567890123456789012123456 7
Association? amendment.
12 34567890123456789012345678901212345678901234567890123456789012123456 7
12
2
34567890123456789012345678901212345678901234567890123456789012123456
(D) What are “partial suffrages?” (D) Seneca Falls convention as the most
34567890123456789012345678901212345678901234567890123456789012123456
7
7
1 (E) Which constitutional amendment important single event in the
12 34567890123456789012345678901212345678901234567890123456789012123456 7
12 34567890123456789012345678901212345678901234567890123456789012123456
gave women the vote? women’s suffrage movement. 7
12 34567890123456789012345678901212345678901234567890123456789012123456 7
12 34567890123456789012345678901212345678901234567890123456789012123456
(E) the NAWSA as important for the 7
12 34567890123456789012345678901212345678901234567890123456789012123456 7
12 34567890123456789012345678901212345678901234567890123456789012123456
unity of the woman’s suffrage 7
12 34567890123456789012345678901212345678901234567890123456789012123456 7
12 34567890123456789012345678901212345678901234567890123456789012123456
movement. 7
12 34567890123456789012345678901212345678901234567890123456789012123456 7
1234567890123456789012345678901212345678901234567890123456789012123456 77
2 34567890123456789012345678901212345678901234567890123456789012123456
1
12 34567890123456789012345678901212345678901234567890123456789012123456 7
12 34567890123456789012345678901212345678901234567890123456789012123456 7
12 34567890123456789012345678901212345678901234567890123456789012123456 7
1234567890123456789012345678901212345678901234567890123456789012123456
2 7
1234567890123456789012345678901212345678901234567890123456789012123456 7
1234567890123456789012345678901212345678901234567890123456789012123456 7
1234567890123456789012345678901212345678901234567890123456789012123456 7
1234567890123456789012345678901212345678901234567890123456789012123456 7
1234567890123456789012345678901212345678901234567890123456789012123456 77
34567890123456789012345678901212345678901234567890123456789012123456
1234567890123456789012345678901212345678901234567890123456789012123456 7
1234567890123456789012345678901212345678901234567890123456789012123456 7
1234567890123456789012345678901212345678901234567890123456789012123456 7
1
12 34567890123456789012345678901212345678901234567890123456789012123456 7
12 34567890123456789012345678901212345678901234567890123456789012123456 7
1234567890123456789012345678901212345678901234567890123456789012123456 7
1234567890123456789012345678901212345678901234567890123456789012123456 7
12 34567890123456789012345678901212345678901234567890123456789012123456 7
12 34567890123456789012345678901212345678901234567890123456789012123456 7
12 34567890123456789012345678901212345678901234567890123456789012123456 7
1234567890123456789012345678901212345678901234567890123456789012123456 7
12 34567890123456789012345678901212345678901234567890123456789012123456 7
12 34567890123456789012345678901212345678901234567890123456789012123456 7
1234567890123456789012345678901212345678901234567890123456789012123456
2 7
1234567890123456789012345678901212345678901234567890123456789012123456 7
1 34567890123456789012345678901212345678901234567890123456789012123456 7
1234567890123456789012345678901212345678901234567890123456789012123456 7
1234567890123456789012345678901212345678901234567890123456789012123456 7
12 34567890123456789012345678901212345678901234567890123456789012123456 7
12 34567890123456789012345678901212345678901234567890123456789012123456 7
12 34567890123456789012345678901212345678901234567890123456789012123456 7
12 34567890123456789012345678901212345678901234567890123456789012123456 7
1234567890123456789012345678901212345678901234567890123456789012123456 7
1234567890123456789012345678901212345678901234567890123456789012123456
2 7
1234567890123456789012345678901212345678901234567890123456789012123456 7
1234567890123456789012345678901212345678901234567890123456789012123456 7
1234567890123456789012345678901212345678901234567890123456789012123456 7
1 34567890123456789012345678901212345678901234567890123456789012123456 7
1234567890123456789012345678901212345678901234567890123456789012123456 77
2 34567890123456789012345678901212345678901234567890123456789012123456
1234567890123456789012345678901212345678901234567890123456789012123456 7
1234567890123456789012345678901212345678901234567890123456789012123456 7
1234567890123456789012345678901212345678901234567890123456789012123456 7
1
12 34567890123456789012345678901212345678901234567890123456789012123456 7
1234567890123456789012345678901212345678901234567890123456789012123456 7
12 34567890123456789012345678901212345678901234567890123456789012123456 7
1234567890123456789012345678901212345678901234567890123456789012123456 7
12 34567890123456789012345678901212345678901234567890123456789012123456 7
STOP
1234567890123456789012345678901212345678901234567890123456789012123456
Do not proceed to the next section until time is up.
1234567890123456789012345678901212345678901234567890123456789012123456 7
7
1234567890123456789012345678901212345678901234567890123456789012123456 7
1234567890123456789012345678901212345678901234567890123456789012123456 7
12345678901234567890123456789012123456789012345678901234567890121234567
42 Copyright © 2005 Thomson Peterson’s, a part of The Thomson Corporaton
SAT is a registered trademark of the College Entrance Examination Board, which
was not involved in the production of and does not endorse this product.
Section 7
13 Questions j Time—20 Minutes

12345678901234567890123456789012123456789012345678901234567890121234567
12234567890123456789012345678901212345678901234567890123456789012123456 7
1234567890123456789012345678901212345678901234567890123456789012123456
34567890123456789012345678901212345678901234567890123456789012123456
7
7
1
12234567890123456789012345678901212345678901234567890123456789012123456 7
1234567890123456789012345678901212345678901234567890123456789012123456
Notes: 7
1234567890123456789012345678901212345678901234567890123456789012123456 7
1234567890123456789012345678901212345678901234567890123456789012123456
1. All numbers used are real numbers. 7
1234567890123456789012345678901212345678901234567890123456789012123456 77
34567890123456789012345678901212345678901234567890123456789012123456
1234567890123456789012345678901212345678901234567890123456789012123456
1234567890123456789012345678901212345678901234567890123456789012123456
2. All angle measurements can be assumed to be positive unless otherwise noted. 7
1234567890123456789012345678901212345678901234567890123456789012123456 77
1234567890123456789012345678901212345678901234567890123456789012123456
1234567890123456789012345678901212345678901234567890123456789012123456
3. All figures lie in the same plane unless otherwise noted. 7
1 7
12234567890123456789012345678901212345678901234567890123456789012123456 7
1234567890123456789012345678901212345678901234567890123456789012123456
4. Drawings that accompany questions are intended to provide information useful in 7
1234567890123456789012345678901212345678901234567890123456789012123456 7
1234567890123456789012345678901212345678901234567890123456789012123456 77
34567890123456789012345678901212345678901234567890123456789012123456
answering the question. The figures are drawn closely to scale unless otherwise noted.
1234567890123456789012345678901212345678901234567890123456789012123456 7
1234567890123456789012345678901212345678901234567890123456789012123456 7
1234567890123456789012345678901212345678901234567890123456789012123456 7
1234567890123456789012345678901212345678901234567890123456789012123456
1234567890123456789012345678901212345678901234567890123456789012123456
Directions for Student Produced Responses 7
1234567890123456789012345678901212345678901234567890123456789012123456 77
1234567890123456789012345678901212345678901234567890123456789012123456
Enter your responses to questions 1–13 in the
1234567890123456789012345678901212345678901234567890123456789012123456 Answer: 1.4 7
1 special grids provided on your answer sheet. 7
1234567890123456789012345678901212345678901234567890123456789012123456 77
2 34567890123456789012345678901212345678901234567890123456789012123456 Either position is correct.
1234567890123456789012345678901212345678901234567890123456789012123456
1 Input your answers as indicated in the directions 7
1234567890123456789012345678901212345678901234567890123456789012123456
below. 1 . 4 1 . 4 7
12234567890123456789012345678901212345678901234567890123456789012123456
34567890123456789012345678901212345678901234567890123456789012123456
7
7
1234567890123456789012345678901212345678901234567890123456789012123456 /
O O/ /O O/

1234567890123456789012345678901212345678901234567890123456789012123456
4 Decimal → O Þ O O
. . . O O Þ O
. . .
7
1 Answer: or 4/9 7
12234567890123456789012345678901212345678901234567890123456789012123456
9 point 0
O O0 O0 0O 0
O O0
7
1234567890123456789012345678901212345678901234567890123456789012123456 Þ Þ 7
34567890123456789012345678901212345678901234567890123456789012123456
1234567890123456789012345678901212345678901234567890123456789012123456 7
1
O O1 O1 1
O 1
O O1

1234567890123456789012345678901212345678901234567890123456789012123456 7
2 2
O O O O 2 2 2 2
O O O O 2 2
Write answer → 4 / 9
1 in boxes. 3 3
O O O O 3 3 3 3
O O O O 3 3
7
1234567890123456789012345678901212345678901234567890123456789012123456
Þ O ← Fraction line 4
O 4
O Þ O4 4
O 4O 4
O Þ 7
1234567890123456789012345678901212345678901234567890123456789012123456 7
/

2 34567890123456789012345678901212345678901234567890123456789012123456
1234567890123456789012345678901212345678901234567890123456789012123456 7
. .
O O O O . .

Note: You may begin your answer in any col- 7


1234567890123456789012345678901212345678901234567890123456789012123456
⎧O
1234567890123456789012345678901212345678901234567890123456789012123456 7
0
O O O0 0

umn, space permitting. Leave blank any columns 7


1234567890123456789012345678901212345678901234567890123456789012123456
1 1
O O O1 1

1 ⎪Þ
2 2
O O O O 2 2
7
12234567890123456789012345678901212345678901234567890123456789012123456
3 3
O O O O 3 3 not needed. 7
1234567890123456789012345678901212345678901234567890123456789012123456 7
Grid in →

1234567890123456789012345678901212345678901234567890123456789012123456 7
4
O O O4 4

1234567890123456789012345678901212345678901234567890123456789012123456
• 7
5 5 5 5
result. O O O O
Writing your answer in the boxes at the top
1 34567890123456789012345678901212345678901234567890123456789012123456

6 6
O O O O 6 6
7
2 34567890123456789012345678901212345678901234567890123456789012123456
1234567890123456789012345678901212345678901234567890123456789012123456
7
O O7 O7 O7 of the columns will help you accurately grid 7
1234567890123456789012345678901212345678901234567890123456789012123456 7

1234567890123456789012345678901212345678901234567890123456789012123456 7
8 8 8 8
O O O O
your answer, but it is not required. You will
1234567890123456789012345678901212345678901234567890123456789012123456
O O Þ O
7
9 9 9

1 only receive credit for an answer if the ovals 7


2 34567890123456789012345678901212345678901234567890123456789012123456
1234567890123456789012345678901212345678901234567890123456789012123456 7
1 are filled in properly. 7
12234567890123456789012345678901212345678901234567890123456789012123456 7
1 34567890123456789012345678901212345678901234567890123456789012123456
• Only fill in one oval in each column. 7
12234567890123456789012345678901212345678901234567890123456789012123456
34567890123456789012345678901212345678901234567890123456789012123456
7
7
1
1234567890123456789012345678901212345678901234567890123456789012123456 7
1234567890123456789012345678901212345678901234567890123456789012123456 7
1234567890123456789012345678901212345678901234567890123456789012123456 7
12345678901234567890123456789012123456789012345678901234567890121234567
12345678901234567890123456789012123456789012345678901234567890121234567
1234567890123456789012345678901212345678901234567890123456789012123456 7
12 34567890123456789012345678901212345678901234567890123456789012123456 7
1234567890123456789012345678901212345678901234567890123456789012123456
• If a problem has several correct answers, Decimal Accuracy 7
12 34567890123456789012345678901212345678901234567890123456789012123456 7
12 34567890123456789012345678901212345678901234567890123456789012123456
just grid in one of them. 7
1234567890123456789012345678901212345678901234567890123456789012123456 7
Decimal answers must be gridded as accu-
12 34567890123456789012345678901212345678901234567890123456789012123456
rately as possible. The answer 0.3333 . . . 7
1234567890123456789012345678901212345678901234567890123456789012123456
• There are no negative answers. 7
12 34567890123456789012345678901212345678901234567890123456789012123456 7
12 34567890123456789012345678901212345678901234567890123456789012123456

must be gridded as .333. 7
12 34567890123456789012345678901212345678901234567890123456789012123456 7
Never grid in mixed numbers. The answer
12 34567890123456789012345678901212345678901234567890123456789012123456
Less accurate values, such as .33 are not 7
1234567890123456789012345678901212345678901234567890123456789012123456
1 7
12 34567890123456789012345678901212345678901234567890123456789012123456
3 must be gridded as 16/5 or 3.2. If 7
12 34567890123456789012345678901212345678901234567890123456789012123456
5 acceptable. 7
12 34567890123456789012345678901212345678901234567890123456789012123456
is gridded, it will be read 7
12
2
34567890123456789012345678901212345678901234567890123456789012123456
3 1 / 5
34567890123456789012345678901212345678901234567890123456789012123456 1 7
7
1 31 1 Acceptable ways to grid 5 .3333 . . .
12 34567890123456789012345678901212345678901234567890123456789012123456
O Þ
/
as , not 3 . 3 7
12 34567890123456789012345678901212345678901234567890123456789012123456
. . . .
5 5 7
12 34567890123456789012345678901212345678901234567890123456789012123456 7
O O O O

12
2
34567890123456789012345678901212345678901234567890123456789012123456
0
O 0O 0O
34567890123456789012345678901212345678901234567890123456789012123456
7
7
1 Þ 1 / 3 . 3 3 3
12 34567890123456789012345678901212345678901234567890123456789012123456 7
1
O 1O 1O

12 34567890123456789012345678901212345678901234567890123456789012123456 Þ O
7
2 2
O O O O 2 2 / O O / /

12 34567890123456789012345678901212345678901234567890123456789012123456
Þ 3 3
O O O 3 .
O O O O . . . Þ O O O . . .
7
12 34567890123456789012345678901212345678901234567890123456789012123456
4
O 4
O 4O 4O 7
2 34567890123456789012345678901212345678901234567890123456789012123456
1234567890123456789012345678901212345678901234567890123456789012123456 7
O O O 0 0 0 O O O 0 0 0
Þ
1234567890123456789012345678901212345678901234567890123456789012123456 7
5
O 5
O 5O
Þ O O O 1 1 1 1
O O O O 1 1 1

1234567890123456789012345678901212345678901234567890123456789012123456 2 2 2 2 O O 2O 2
O 2 2
7
7
O O O O
1234567890123456789012345678901212345678901234567890123456789012123456 O Þ Þ Þ
1234567890123456789012345678901212345678901234567890123456789012123456 7
O O Þ O
3 3 3 3

1234567890123456789012345678901212345678901234567890123456789012123456 7
4
O O O O 4 4 4 4
O O O O 4 4 4

1 5
O O O O 5 5 5 5
O O O O 5 5 5
7
12 34567890123456789012345678901212345678901234567890123456789012123456 7 6 6 6 6 O O 6O 6
O 6 6

12 34567890123456789012345678901212345678901234567890123456789012123456 7
O O O O

12 34567890123456789012345678901212345678901234567890123456789012123456 7
1234567890123456789012345678901212345678901234567890123456789012123456
2 7
1234567890123456789012345678901212345678901234567890123456789012123456 7
1234567890123456789012345678901212345678901234567890123456789012123456 7
1234567890123456789012345678901212345678901234567890123456789012123456 7
1234567890123456789012345678901212345678901234567890123456789012123456
1. [2(3 ) 1 (4 2 3(4)) ] 5 7. Given P(3, 2) and Q(3, 6), what is the 7
1234567890123456789012345678901212345678901234567890123456789012123456 77
34567890123456789012345678901212345678901234567890123456789012123456
2 2 2

1234567890123456789012345678901212345678901234567890123456789012123456
y-coordinate of the midpoint of the line
1234567890123456789012345678901212345678901234567890123456789012123456 77
1234567890123456789012345678901212345678901234567890123456789012123456
1 2. Given that 22 2 3x . 13, what is a segment defined by these two points?
7
1234567890123456789012345678901212345678901234567890123456789012123456 77
2 34567890123456789012345678901212345678901234567890123456789012123456
positive integer that is less than x?
1234567890123456789012345678901212345678901234567890123456789012123456 7
1 8.
1234567890123456789012345678901212345678901234567890123456789012123456 7
12
2
34567890123456789012345678901212345678901234567890123456789012123456 7
3. A phone company charges 40 cents for a
34567890123456789012345678901212345678901234567890123456789012123456 7
1234567890123456789012345678901212345678901234567890123456789012123456
1234567890123456789012345678901212345678901234567890123456789012123456
completed long-distance phone call and 6 7
1 7
12 34567890123456789012345678901212345678901234567890123456789012123456 7
cents per minute on top of the initial fee.
12 34567890123456789012345678901212345678901234567890123456789012123456 7
12 34567890123456789012345678901212345678901234567890123456789012123456
How much, in dollars, would a 30-minute 7
12 34567890123456789012345678901212345678901234567890123456789012123456 7
1234567890123456789012345678901212345678901234567890123456789012123456 7
long-distance phone call cost?
1234567890123456789012345678901212345678901234567890123456789012123456 7
1234567890123456789012345678901212345678901234567890123456789012123456 7
12 34567890123456789012345678901212345678901234567890123456789012123456
4. What is the least common multiple of 18 7
12 34567890123456789012345678901212345678901234567890123456789012123456 7
12 34567890123456789012345678901212345678901234567890123456789012123456
and 24? 7
12 34567890123456789012345678901212345678901234567890123456789012123456 7
1234567890123456789012345678901212345678901234567890123456789012123456 7
12 34567890123456789012345678901212345678901234567890123456789012123456
5. If |x 2 3| . 2, then what is one integer 7
12 34567890123456789012345678901212345678901234567890123456789012123456 7
12 34567890123456789012345678901212345678901234567890123456789012123456 7
12 34567890123456789012345678901212345678901234567890123456789012123456 7
value that x cannot equal?
1234567890123456789012345678901212345678901234567890123456789012123456
What is the length of the hypotenuse? 7
12 34567890123456789012345678901212345678901234567890123456789012123456 7
12 34567890123456789012345678901212345678901234567890123456789012123456
6. If the kth term in a series is generated by 7
12 34567890123456789012345678901212345678901234567890123456789012123456 7
12 34567890123456789012345678901212345678901234567890123456789012123456
10~k 1 1! 9. What is the difference between the mean 7
1234567890123456789012345678901212345678901234567890123456789012123456
the equation n 5 , what is the 7
2 34567890123456789012345678901212345678901234567890123456789012123456
1234567890123456789012345678901212345678901234567890123456789012123456
k21 and the median of the number set {7, 4, 3, 7
1 7
12 34567890123456789012345678901212345678901234567890123456789012123456 7
eleventh term in the series? 6, 2, 2}?
1234567890123456789012345678901212345678901234567890123456789012123456 7
12 34567890123456789012345678901212345678901234567890123456789012123456 7
1234567890123456789012345678901212345678901234567890123456789012123456 7
1234567890123456789012345678901212345678901234567890123456789012123456 7
1234567890123456789012345678901212345678901234567890123456789012123456 7
1234567890123456789012345678901212345678901234567890123456789012123456 7
12345678901234567890123456789012123456789012345678901234567890121234567
44 Copyright © 2005 Thomson Peterson’s, a part of The Thomson Corporaton
SAT is a registered trademark of the College Entrance Examination Board, which
was not involved in the production of and does not endorse this product.
12345678901234567890123456789012123456789012345678901234567890121234567
1234567890123456789012345678901212345678901234567890123456789012123456 7
12234567890123456789012345678901212345678901234567890123456789012123456 7
1 34567890123456789012345678901212345678901234567890123456789012123456
10. If line a and line b are perpendicular, line 13. What is the one integer that is between 7
12234567890123456789012345678901212345678901234567890123456789012123456 7
1234567890123456789012345678901212345678901234567890123456789012123456
b and line c are perpendicular, and the 999 and 1199 and satisfies these two 7
1 34567890123456789012345678901212345678901234567890123456789012123456 7
12234567890123456789012345678901212345678901234567890123456789012123456
2 conditions:
34567890123456789012345678901212345678901234567890123456789012123456
7
7
1 slope of line a is , what is the slope of
12234567890123456789012345678901212345678901234567890123456789012123456
5 Its digits sum to seventeen. 7
1234567890123456789012345678901212345678901234567890123456789012123456 7
1234567890123456789012345678901212345678901234567890123456789012123456
line c? 7
1234567890123456789012345678901212345678901234567890123456789012123456
Its
34567890123456789012345678901212345678901234567890123456789012123456
tens and units digits are the same. 7
7
1
12234567890123456789012345678901212345678901234567890123456789012123456
11. If f(x) 5 x2 1 3 and x 5 y 1 2, when 7
1234567890123456789012345678901212345678901234567890123456789012123456 7
1234567890123456789012345678901212345678901234567890123456789012123456
y 5 3, f(x) 5 7
1234567890123456789012345678901212345678901234567890123456789012123456
34567890123456789012345678901212345678901234567890123456789012123456
7
7
1
12234567890123456789012345678901212345678901234567890123456789012123456 7
1234567890123456789012345678901212345678901234567890123456789012123456
12. If a square is inscribed in a circle and the 7
1234567890123456789012345678901212345678901234567890123456789012123456 7
1234567890123456789012345678901212345678901234567890123456789012123456
area of the square is eight, what is the
34567890123456789012345678901212345678901234567890123456789012123456
7
7
1
12234567890123456789012345678901212345678901234567890123456789012123456
longest distance that a straight line could 7
1234567890123456789012345678901212345678901234567890123456789012123456 7
1234567890123456789012345678901212345678901234567890123456789012123456
be that originated and terminated on the 7
1234567890123456789012345678901212345678901234567890123456789012123456 7
1234567890123456789012345678901212345678901234567890123456789012123456 77
34567890123456789012345678901212345678901234567890123456789012123456
circumference of the circle?
1234567890123456789012345678901212345678901234567890123456789012123456 7
1234567890123456789012345678901212345678901234567890123456789012123456 7
1234567890123456789012345678901212345678901234567890123456789012123456 7
1234567890123456789012345678901212345678901234567890123456789012123456 7
1234567890123456789012345678901212345678901234567890123456789012123456 7
1
12234567890123456789012345678901212345678901234567890123456789012123456 7
1234567890123456789012345678901212345678901234567890123456789012123456 7
1234567890123456789012345678901212345678901234567890123456789012123456 7
1234567890123456789012345678901212345678901234567890123456789012123456 77
34567890123456789012345678901212345678901234567890123456789012123456
1234567890123456789012345678901212345678901234567890123456789012123456 7
1234567890123456789012345678901212345678901234567890123456789012123456 7
1234567890123456789012345678901212345678901234567890123456789012123456 7
1234567890123456789012345678901212345678901234567890123456789012123456 7
1234567890123456789012345678901212345678901234567890123456789012123456 7
1234567890123456789012345678901212345678901234567890123456789012123456 7
1234567890123456789012345678901212345678901234567890123456789012123456 7
1234567890123456789012345678901212345678901234567890123456789012123456 7
1
12234567890123456789012345678901212345678901234567890123456789012123456 7
1234567890123456789012345678901212345678901234567890123456789012123456 7
1 34567890123456789012345678901212345678901234567890123456789012123456 7
1234567890123456789012345678901212345678901234567890123456789012123456 7
12234567890123456789012345678901212345678901234567890123456789012123456 7
1234567890123456789012345678901212345678901234567890123456789012123456 7
1234567890123456789012345678901212345678901234567890123456789012123456 7
1 34567890123456789012345678901212345678901234567890123456789012123456 7
12234567890123456789012345678901212345678901234567890123456789012123456 7
1234567890123456789012345678901212345678901234567890123456789012123456 7
1234567890123456789012345678901212345678901234567890123456789012123456 77
34567890123456789012345678901212345678901234567890123456789012123456
1234567890123456789012345678901212345678901234567890123456789012123456 7
1
1234567890123456789012345678901212345678901234567890123456789012123456 7
1234567890123456789012345678901212345678901234567890123456789012123456 7
12234567890123456789012345678901212345678901234567890123456789012123456 7
1234567890123456789012345678901212345678901234567890123456789012123456 7
1234567890123456789012345678901212345678901234567890123456789012123456 7
1234567890123456789012345678901212345678901234567890123456789012123456 7
1 34567890123456789012345678901212345678901234567890123456789012123456 7
1234567890123456789012345678901212345678901234567890123456789012123456 77
2 34567890123456789012345678901212345678901234567890123456789012123456
1234567890123456789012345678901212345678901234567890123456789012123456 7
1234567890123456789012345678901212345678901234567890123456789012123456 7
1234567890123456789012345678901212345678901234567890123456789012123456 7
1
12234567890123456789012345678901212345678901234567890123456789012123456 7
1234567890123456789012345678901212345678901234567890123456789012123456 7
1234567890123456789012345678901212345678901234567890123456789012123456 7
1234567890123456789012345678901212345678901234567890123456789012123456 7
1 34567890123456789012345678901212345678901234567890123456789012123456 7
1234567890123456789012345678901212345678901234567890123456789012123456 77
2 34567890123456789012345678901212345678901234567890123456789012123456
1
12234567890123456789012345678901212345678901234567890123456789012123456 7
1 34567890123456789012345678901212345678901234567890123456789012123456 7
12234567890123456789012345678901212345678901234567890123456789012123456 7
STOP
1 34567890123456789012345678901212345678901234567890123456789012123456
Do not proceed to the next section until time is up.
1234567890123456789012345678901212345678901234567890123456789012123456 7
7
1234567890123456789012345678901212345678901234567890123456789012123456 7
1234567890123456789012345678901212345678901234567890123456789012123456 7
12345678901234567890123456789012123456789012345678901234567890121234567
Copyright © 2005 Thomson Peterson’s, a part of The Thomson Corporaton 45
SAT is a registered trademark of the College Entrance Examination Board, which
was not involved in the production of and does not endorse this product.
Section 8
Time—25 Minutes

12345678901234567890123456789012123456789012345678901234567890121234567
12 34567890123456789012345678901212345678901234567890123456789012123456 7
12 34567890123456789012345678901212345678901234567890123456789012123456 7
1 34567890123456789012345678901212345678901234567890123456789012123456 7
2
12 34567890123456789012345678901212345678901234567890123456789012123456 7
12 34567890123456789012345678901212345678901234567890123456789012123456
Directions: Think carefully about the statement below and the assignment that follows it. 7
12 34567890123456789012345678901212345678901234567890123456789012123456 7
12 34567890123456789012345678901212345678901234567890123456789012123456 7
12 34567890123456789012345678901212345678901234567890123456789012123456 7
12 34567890123456789012345678901212345678901234567890123456789012123456
Consider the following statement. 7
12 34567890123456789012345678901212345678901234567890123456789012123456 7
12 34567890123456789012345678901212345678901234567890123456789012123456 7
12 34567890123456789012345678901212345678901234567890123456789012123456
Some people think that war is justified under certain circumstances. 7
1234567890123456789012345678901212345678901234567890123456789012123456 77
2 34567890123456789012345678901212345678901234567890123456789012123456
1 Assignment: Write an essay in which you agree or disagree with the preceding statement. Develop your
12 34567890123456789012345678901212345678901234567890123456789012123456 7
12 34567890123456789012345678901212345678901234567890123456789012123456
point of view on this statement and be sure to support your stance with sufficient details. 7
12 34567890123456789012345678901212345678901234567890123456789012123456 7
1234567890123456789012345678901212345678901234567890123456789012123456 77
2 34567890123456789012345678901212345678901234567890123456789012123456
1234567890123456789012345678901212345678901234567890123456789012123456 7
1234567890123456789012345678901212345678901234567890123456789012123456 7
1234567890123456789012345678901212345678901234567890123456789012123456 7
1234567890123456789012345678901212345678901234567890123456789012123456 7
1234567890123456789012345678901212345678901234567890123456789012123456 7
1234567890123456789012345678901212345678901234567890123456789012123456 7
1234567890123456789012345678901212345678901234567890123456789012123456 7
1234567890123456789012345678901212345678901234567890123456789012123456 7
1
12 34567890123456789012345678901212345678901234567890123456789012123456 7
12 34567890123456789012345678901212345678901234567890123456789012123456 7
1234567890123456789012345678901212345678901234567890123456789012123456 7
1234567890123456789012345678901212345678901234567890123456789012123456 7
12 34567890123456789012345678901212345678901234567890123456789012123456 7
12 34567890123456789012345678901212345678901234567890123456789012123456 7
12 34567890123456789012345678901212345678901234567890123456789012123456 7
1234567890123456789012345678901212345678901234567890123456789012123456 7
12 34567890123456789012345678901212345678901234567890123456789012123456 7
12 34567890123456789012345678901212345678901234567890123456789012123456 7
1234567890123456789012345678901212345678901234567890123456789012123456
2 7
1234567890123456789012345678901212345678901234567890123456789012123456 7
1 34567890123456789012345678901212345678901234567890123456789012123456 7
1234567890123456789012345678901212345678901234567890123456789012123456 7
1234567890123456789012345678901212345678901234567890123456789012123456 7
12 34567890123456789012345678901212345678901234567890123456789012123456 7
12 34567890123456789012345678901212345678901234567890123456789012123456 7
12 34567890123456789012345678901212345678901234567890123456789012123456 7
12 34567890123456789012345678901212345678901234567890123456789012123456 7
1234567890123456789012345678901212345678901234567890123456789012123456 7
1234567890123456789012345678901212345678901234567890123456789012123456
2 7
1234567890123456789012345678901212345678901234567890123456789012123456 7
1234567890123456789012345678901212345678901234567890123456789012123456 7
1234567890123456789012345678901212345678901234567890123456789012123456 7
1 34567890123456789012345678901212345678901234567890123456789012123456 7
1234567890123456789012345678901212345678901234567890123456789012123456 77
2 34567890123456789012345678901212345678901234567890123456789012123456
1234567890123456789012345678901212345678901234567890123456789012123456 7
1234567890123456789012345678901212345678901234567890123456789012123456 7
1234567890123456789012345678901212345678901234567890123456789012123456 7
1
12 34567890123456789012345678901212345678901234567890123456789012123456 7
1234567890123456789012345678901212345678901234567890123456789012123456 7
12 34567890123456789012345678901212345678901234567890123456789012123456 7
1234567890123456789012345678901212345678901234567890123456789012123456 7
12 34567890123456789012345678901212345678901234567890123456789012123456 7
1234567890123456789012345678901212345678901234567890123456789012123456 7
1234567890123456789012345678901212345678901234567890123456789012123456 7
1234567890123456789012345678901212345678901234567890123456789012123456 7
1234567890123456789012345678901212345678901234567890123456789012123456 7
12345678901234567890123456789012123456789012345678901234567890121234567
46 Copyright © 2005 Thomson Peterson’s, a part of The Thomson Corporaton
SAT is a registered trademark of the College Entrance Examination Board, which
was not involved in the production of and does not endorse this product.
12345678901234567890123456789012123456789012345678901234567890121234567
1234567890123456789012345678901212345678901234567890123456789012123456 7
12234567890123456789012345678901212345678901234567890123456789012123456 7
1 34567890123456789012345678901212345678901234567890123456789012123456 7
12234567890123456789012345678901212345678901234567890123456789012123456 7
1234567890123456789012345678901212345678901234567890123456789012123456 7
1 34567890123456789012345678901212345678901234567890123456789012123456 7
12234567890123456789012345678901212345678901234567890123456789012123456 7
1 34567890123456789012345678901212345678901234567890123456789012123456 7
12234567890123456789012345678901212345678901234567890123456789012123456 7
1234567890123456789012345678901212345678901234567890123456789012123456 7
1234567890123456789012345678901212345678901234567890123456789012123456 7
1234567890123456789012345678901212345678901234567890123456789012123456
34567890123456789012345678901212345678901234567890123456789012123456
7
7
1
12234567890123456789012345678901212345678901234567890123456789012123456 7
1234567890123456789012345678901212345678901234567890123456789012123456 7
1234567890123456789012345678901212345678901234567890123456789012123456 7
1234567890123456789012345678901212345678901234567890123456789012123456
34567890123456789012345678901212345678901234567890123456789012123456
7
7
1
12234567890123456789012345678901212345678901234567890123456789012123456 7
1234567890123456789012345678901212345678901234567890123456789012123456 7
1234567890123456789012345678901212345678901234567890123456789012123456 7
1234567890123456789012345678901212345678901234567890123456789012123456
34567890123456789012345678901212345678901234567890123456789012123456
7
7
1
12234567890123456789012345678901212345678901234567890123456789012123456 7
1234567890123456789012345678901212345678901234567890123456789012123456 7
1234567890123456789012345678901212345678901234567890123456789012123456 7
1234567890123456789012345678901212345678901234567890123456789012123456 7
1234567890123456789012345678901212345678901234567890123456789012123456 77
34567890123456789012345678901212345678901234567890123456789012123456
1234567890123456789012345678901212345678901234567890123456789012123456 7
1234567890123456789012345678901212345678901234567890123456789012123456 7
1234567890123456789012345678901212345678901234567890123456789012123456 7
1234567890123456789012345678901212345678901234567890123456789012123456 7
1234567890123456789012345678901212345678901234567890123456789012123456 7
1
12234567890123456789012345678901212345678901234567890123456789012123456 7
1234567890123456789012345678901212345678901234567890123456789012123456 7
1234567890123456789012345678901212345678901234567890123456789012123456 7
1234567890123456789012345678901212345678901234567890123456789012123456 77
34567890123456789012345678901212345678901234567890123456789012123456
1234567890123456789012345678901212345678901234567890123456789012123456 7
1234567890123456789012345678901212345678901234567890123456789012123456 7
1234567890123456789012345678901212345678901234567890123456789012123456 7
1234567890123456789012345678901212345678901234567890123456789012123456 7
1234567890123456789012345678901212345678901234567890123456789012123456 7
1234567890123456789012345678901212345678901234567890123456789012123456 7
1234567890123456789012345678901212345678901234567890123456789012123456 7
1234567890123456789012345678901212345678901234567890123456789012123456 7
1
12234567890123456789012345678901212345678901234567890123456789012123456 7
1234567890123456789012345678901212345678901234567890123456789012123456 7
1 34567890123456789012345678901212345678901234567890123456789012123456 7
1234567890123456789012345678901212345678901234567890123456789012123456 7
12234567890123456789012345678901212345678901234567890123456789012123456 7
1234567890123456789012345678901212345678901234567890123456789012123456 7
1234567890123456789012345678901212345678901234567890123456789012123456 7
1 34567890123456789012345678901212345678901234567890123456789012123456 7
12234567890123456789012345678901212345678901234567890123456789012123456 7
1234567890123456789012345678901212345678901234567890123456789012123456 7
1234567890123456789012345678901212345678901234567890123456789012123456 77
34567890123456789012345678901212345678901234567890123456789012123456
1234567890123456789012345678901212345678901234567890123456789012123456 7
1
1234567890123456789012345678901212345678901234567890123456789012123456 7
1234567890123456789012345678901212345678901234567890123456789012123456 7
12234567890123456789012345678901212345678901234567890123456789012123456 7
1234567890123456789012345678901212345678901234567890123456789012123456 7
1234567890123456789012345678901212345678901234567890123456789012123456 7
1234567890123456789012345678901212345678901234567890123456789012123456 7
1 34567890123456789012345678901212345678901234567890123456789012123456 7
1234567890123456789012345678901212345678901234567890123456789012123456 77
2 34567890123456789012345678901212345678901234567890123456789012123456
1234567890123456789012345678901212345678901234567890123456789012123456 7
1234567890123456789012345678901212345678901234567890123456789012123456 7
1234567890123456789012345678901212345678901234567890123456789012123456 7
1
12234567890123456789012345678901212345678901234567890123456789012123456 7
1234567890123456789012345678901212345678901234567890123456789012123456 7
1234567890123456789012345678901212345678901234567890123456789012123456 7
1234567890123456789012345678901212345678901234567890123456789012123456 7
1 34567890123456789012345678901212345678901234567890123456789012123456 7
1234567890123456789012345678901212345678901234567890123456789012123456 77
2 34567890123456789012345678901212345678901234567890123456789012123456
1
12234567890123456789012345678901212345678901234567890123456789012123456 7
1 34567890123456789012345678901212345678901234567890123456789012123456 7
12234567890123456789012345678901212345678901234567890123456789012123456 7
1 34567890123456789012345678901212345678901234567890123456789012123456 7
1234567890123456789012345678901212345678901234567890123456789012123456 7
1234567890123456789012345678901212345678901234567890123456789012123456 7
1234567890123456789012345678901212345678901234567890123456789012123456 7
12345678901234567890123456789012123456789012345678901234567890121234567
Copyright © 2005 Thomson Peterson’s, a part of The Thomson Corporaton 47
SAT is a registered trademark of the College Entrance Examination Board, which
was not involved in the production of and does not endorse this product.
12345678901234567890123456789012123456789012345678901234567890121234567
1234567890123456789012345678901212345678901234567890123456789012123456 7
12 34567890123456789012345678901212345678901234567890123456789012123456 7
1234567890123456789012345678901212345678901234567890123456789012123456 7
12 34567890123456789012345678901212345678901234567890123456789012123456 7
12 34567890123456789012345678901212345678901234567890123456789012123456 7
1234567890123456789012345678901212345678901234567890123456789012123456 7
12 34567890123456789012345678901212345678901234567890123456789012123456 7
1234567890123456789012345678901212345678901234567890123456789012123456 7
12 34567890123456789012345678901212345678901234567890123456789012123456 7
12 34567890123456789012345678901212345678901234567890123456789012123456 7
12 34567890123456789012345678901212345678901234567890123456789012123456 7
12 34567890123456789012345678901212345678901234567890123456789012123456 7
1 34567890123456789012345678901212345678901234567890123456789012123456 7
2
12 34567890123456789012345678901212345678901234567890123456789012123456 7
12 34567890123456789012345678901212345678901234567890123456789012123456 7
12 34567890123456789012345678901212345678901234567890123456789012123456 7
12 34567890123456789012345678901212345678901234567890123456789012123456 7
1 34567890123456789012345678901212345678901234567890123456789012123456 7
2
12 34567890123456789012345678901212345678901234567890123456789012123456 7
12 34567890123456789012345678901212345678901234567890123456789012123456 7
12 34567890123456789012345678901212345678901234567890123456789012123456 7
12 34567890123456789012345678901212345678901234567890123456789012123456 7
1 34567890123456789012345678901212345678901234567890123456789012123456 7
2
12 34567890123456789012345678901212345678901234567890123456789012123456 7
12 34567890123456789012345678901212345678901234567890123456789012123456 7
12 34567890123456789012345678901212345678901234567890123456789012123456 7
12 34567890123456789012345678901212345678901234567890123456789012123456 7
1234567890123456789012345678901212345678901234567890123456789012123456
2 7
1234567890123456789012345678901212345678901234567890123456789012123456 7
1234567890123456789012345678901212345678901234567890123456789012123456 7
1234567890123456789012345678901212345678901234567890123456789012123456 7
1234567890123456789012345678901212345678901234567890123456789012123456 7
1234567890123456789012345678901212345678901234567890123456789012123456 77
34567890123456789012345678901212345678901234567890123456789012123456
1
12 34567890123456789012345678901212345678901234567890123456789012123456 7
12 34567890123456789012345678901212345678901234567890123456789012123456 7
12 34567890123456789012345678901212345678901234567890123456789012123456 7
1234567890123456789012345678901212345678901234567890123456789012123456
2 7
1234567890123456789012345678901212345678901234567890123456789012123456 7
1234567890123456789012345678901212345678901234567890123456789012123456 7
1234567890123456789012345678901212345678901234567890123456789012123456 7
1234567890123456789012345678901212345678901234567890123456789012123456 7
1234567890123456789012345678901212345678901234567890123456789012123456 77
34567890123456789012345678901212345678901234567890123456789012123456
1234567890123456789012345678901212345678901234567890123456789012123456 7
1234567890123456789012345678901212345678901234567890123456789012123456 7
1234567890123456789012345678901212345678901234567890123456789012123456 7
1
12 34567890123456789012345678901212345678901234567890123456789012123456 7
12 34567890123456789012345678901212345678901234567890123456789012123456 7
1234567890123456789012345678901212345678901234567890123456789012123456 7
1234567890123456789012345678901212345678901234567890123456789012123456 7
12 34567890123456789012345678901212345678901234567890123456789012123456 7
12 34567890123456789012345678901212345678901234567890123456789012123456 7
12 34567890123456789012345678901212345678901234567890123456789012123456 7
1234567890123456789012345678901212345678901234567890123456789012123456 7
12 34567890123456789012345678901212345678901234567890123456789012123456 7
12 34567890123456789012345678901212345678901234567890123456789012123456 7
1234567890123456789012345678901212345678901234567890123456789012123456
2 7
1234567890123456789012345678901212345678901234567890123456789012123456 7
1 34567890123456789012345678901212345678901234567890123456789012123456 7
1234567890123456789012345678901212345678901234567890123456789012123456 7
1234567890123456789012345678901212345678901234567890123456789012123456 7
12 34567890123456789012345678901212345678901234567890123456789012123456 7
12 34567890123456789012345678901212345678901234567890123456789012123456 7
12 34567890123456789012345678901212345678901234567890123456789012123456 7
12 34567890123456789012345678901212345678901234567890123456789012123456 7
1234567890123456789012345678901212345678901234567890123456789012123456 7
1234567890123456789012345678901212345678901234567890123456789012123456
2 7
1234567890123456789012345678901212345678901234567890123456789012123456 7
1234567890123456789012345678901212345678901234567890123456789012123456 7
1234567890123456789012345678901212345678901234567890123456789012123456 7
1 34567890123456789012345678901212345678901234567890123456789012123456 7
1234567890123456789012345678901212345678901234567890123456789012123456 77
2 34567890123456789012345678901212345678901234567890123456789012123456
1234567890123456789012345678901212345678901234567890123456789012123456 7
1234567890123456789012345678901212345678901234567890123456789012123456 7
1234567890123456789012345678901212345678901234567890123456789012123456 7
1
12 34567890123456789012345678901212345678901234567890123456789012123456 7
1234567890123456789012345678901212345678901234567890123456789012123456 7
12 34567890123456789012345678901212345678901234567890123456789012123456 7
1234567890123456789012345678901212345678901234567890123456789012123456 7
12 34567890123456789012345678901212345678901234567890123456789012123456
When you are finished with your essay put your pencil down until the 7
1234567890123456789012345678901212345678901234567890123456789012123456
STOP 7
1234567890123456789012345678901212345678901234567890123456789012123456
time allotted is over. 7
1234567890123456789012345678901212345678901234567890123456789012123456 77
1234567890123456789012345678901212345678901234567890123456789012123456
12345678901234567890123456789012123456789012345678901234567890121234567
48 Copyright © 2005 Thomson Peterson’s, a part of The Thomson Corporaton
SAT is a registered trademark of the College Entrance Examination Board, which
was not involved in the production of and does not endorse this product.
Answers
and Explanations
1234567890123456789012345678901212345678901234567890123456789
123456789012345678901234567890121234567890123456789012345678 9
123456789012345678901234567890121234567890123456789012345678 9
123456789012345678901234567890121234567890123456789012345678
Section 1 9
123456789012345678901234567890121234567890123456789012345678 9
123456789012345678901234567890121234567890123456789012345678 9
23456789012345678901234567890121234567890123456789012345678
123456789012345678901234567890121234567890123456789012345678 9
1 1. C The passage begins, “Musical notes, like all sounds, are a result of the 9
123456789012345678901234567890121234567890123456789012345678
sound waves created by movement.” The author then goes on to talk 9
123456789012345678901234567890121234567890123456789012345678 9
123456789012345678901234567890121234567890123456789012345678
about musical notes and how they illustrate properties of sound waves. 9
123456789012345678901234567890121234567890123456789012345678 99
23456789012345678901234567890121234567890123456789012345678
123456789012345678901234567890121234567890123456789012345678
Choice (C) captures this idea.
123456789012345678901234567890121234567890123456789012345678 9
123456789012345678901234567890121234567890123456789012345678 9
123456789012345678901234567890121234567890123456789012345678
2. A Pitch is determined by the frequency of the sound wave. This eliminates 9
123456789012345678901234567890121234567890123456789012345678 9
123456789012345678901234567890121234567890123456789012345678
(B) and (E). Choice (C) seems to refer more to the intensity, so eliminate 9
123456789012345678901234567890121234567890123456789012345678 9
123456789012345678901234567890121234567890123456789012345678
it too. The final sentence says that pitch can be described either in 9
1 9
123456789012345678901234567890121234567890123456789012345678 99
23456789012345678901234567890121234567890123456789012345678
numbers or in letters, so eliminate (D). That leaves (A), the cor-
123456789012345678901234567890121234567890123456789012345678 9
1 rect answer.
123456789012345678901234567890121234567890123456789012345678 9
123456789012345678901234567890121234567890123456789012345678 9
123456789012345678901234567890121234567890123456789012345678
3. B The passage states that Langston Hughes “persuaded her to continue 9
123456789012345678901234567890121234567890123456789012345678 9
123456789012345678901234567890121234567890123456789012345678 9
her education in the North.” And the passage uses this fact to explain
123456789012345678901234567890121234567890123456789012345678 9
123456789012345678901234567890121234567890123456789012345678
her transfer to Northwestern. This is what (B), the correct an- 9
23456789012345678901234567890121234567890123456789012345678
123456789012345678901234567890121234567890123456789012345678 9
swer, suggests.
123456789012345678901234567890121234567890123456789012345678 9
1 9
123456789012345678901234567890121234567890123456789012345678
4. C The passage doesn’t specifically say that Walker was writing poetry 9
123456789012345678901234567890121234567890123456789012345678 9
123456789012345678901234567890121234567890123456789012345678 99
23456789012345678901234567890121234567890123456789012345678
before she entered New Orleans University. Eliminate (A). Hughes
123456789012345678901234567890121234567890123456789012345678 9
123456789012345678901234567890121234567890123456789012345678
recognized her talent, but he didn’t create it, so eliminate (B). Hughes
9
123456789012345678901234567890121234567890123456789012345678 9
1 recognized her talent before she transferred to Northwestern, so
23456789012345678901234567890121234567890123456789012345678
123456789012345678901234567890121234567890123456789012345678 9
123456789012345678901234567890121234567890123456789012345678
eliminate (D). The passage, if anything, implies that Walker wrote 9
123456789012345678901234567890121234567890123456789012345678 9
123456789012345678901234567890121234567890123456789012345678 99
poetry for some time before publishing anything, so eliminate (E). The
1
123456789012345678901234567890121234567890123456789012345678
passage makes reference to her parents’ occupations and encourage- 9
123456789012345678901234567890121234567890123456789012345678 9
123456789012345678901234567890121234567890123456789012345678
ment, implying that they had an influence on her decision to become 9
123456789012345678901234567890121234567890123456789012345678 9
123456789012345678901234567890121234567890123456789012345678 9
a poet.
123456789012345678901234567890121234567890123456789012345678 99
23456789012345678901234567890121234567890123456789012345678
1
123456789012345678901234567890121234567890123456789012345678
5. C The author is poking a bit of fun at the Ungers, so eliminate (A), (B), 9
123456789012345678901234567890121234567890123456789012345678 9
123456789012345678901234567890121234567890123456789012345678
and (E). His tone is more playful than downtrodden,
23456789012345678901234567890121234567890123456789012345678
so the answer 9
9
1 is (C).
123456789012345678901234567890121234567890123456789012345678 9
123456789012345678901234567890121234567890123456789012345678 99
123456789012345678901234567890121234567890123456789012345678
1234567890123456789012345678901212345678901234567890123456789
Copyright © 2005 Thomson Peterson’s, a part of The Thomson Corporaton 49
SAT is a registered trademark of the College Entrance Examination Board, which
was not involved in the production of and does not endorse this product.
1234567890123456789012345678901212345678901234567890123456789
123456789012345678901234567890121234567890123456789012345678 9
123456789012345678901234567890121234567890123456789012345678 9
123456789012345678901234567890121234567890123456789012345678 9
123456789012345678901234567890121234567890123456789012345678 9
123456789012345678901234567890121234567890123456789012345678
6. D The “Chicago beef-princess” suggests the wider high-class social world 9
123456789012345678901234567890121234567890123456789012345678 9
123456789012345678901234567890121234567890123456789012345678
in Chicago. When one thing stands in
23456789012345678901234567890121234567890123456789012345678
for another, it is a metaphor. The 9
9
1
123456789012345678901234567890121234567890123456789012345678
answer is (D). 9
123456789012345678901234567890121234567890123456789012345678 9
123456789012345678901234567890121234567890123456789012345678 9
123456789012345678901234567890121234567890123456789012345678
7. A Even if you do not know the definition
23456789012345678901234567890121234567890123456789012345678
of fatuity you can still get this 9
9
1 question. John is going from Hades, which we can assume is hot, to
123456789012345678901234567890121234567890123456789012345678 9
123456789012345678901234567890121234567890123456789012345678
Boston. He will probably not need the light suits and fans. The answer 9
123456789012345678901234567890121234567890123456789012345678 9
123456789012345678901234567890121234567890123456789012345678
is (A).
23456789012345678901234567890121234567890123456789012345678
9
9
1
123456789012345678901234567890121234567890123456789012345678 9
123456789012345678901234567890121234567890123456789012345678
8. B We know that John does not feel rejected, because he says he knows he 9
123456789012345678901234567890121234567890123456789012345678 9
123456789012345678901234567890121234567890123456789012345678
will always be welcome at home. Eliminate
23456789012345678901234567890121234567890123456789012345678
(A). On the other hand, he 9
9
1
123456789012345678901234567890121234567890123456789012345678
does feel something negative, or he wouldn’t cry. Eliminate (C) and (E). 9
123456789012345678901234567890121234567890123456789012345678 9
123456789012345678901234567890121234567890123456789012345678
The handshake and the fact that John’s tears are not mentioned until he 9
123456789012345678901234567890121234567890123456789012345678 9
123456789012345678901234567890121234567890123456789012345678 9
23456789012345678901234567890121234567890123456789012345678
has turned away from his father suggest that he is composed. The best
123456789012345678901234567890121234567890123456789012345678 9
123456789012345678901234567890121234567890123456789012345678
answer is (B). 9
123456789012345678901234567890121234567890123456789012345678 9
123456789012345678901234567890121234567890123456789012345678 9
123456789012345678901234567890121234567890123456789012345678
9. C If you were leaving home (and you were crying), why would you stop 9
1 9
123456789012345678901234567890121234567890123456789012345678
and look back? Most likely you would do so because you were sad to 9
123456789012345678901234567890121234567890123456789012345678
leave and wanted to get one last look before you went. Which of the 9
123456789012345678901234567890121234567890123456789012345678 9
23456789012345678901234567890121234567890123456789012345678
123456789012345678901234567890121234567890123456789012345678
answer choices matches this sentiment? Choice (C) does. The medita- 9
123456789012345678901234567890121234567890123456789012345678 9
123456789012345678901234567890121234567890123456789012345678
tion on what the sign says serves to emphasize the quaintness of the 9
123456789012345678901234567890121234567890123456789012345678 9
123456789012345678901234567890121234567890123456789012345678
town, of which John will no longer be a part. The other answers rely on 9
123456789012345678901234567890121234567890123456789012345678 9
123456789012345678901234567890121234567890123456789012345678
your being distracted from the main emotions of the story. 9
123456789012345678901234567890121234567890123456789012345678 9
123456789012345678901234567890121234567890123456789012345678 9
1 10. E Hades is hell in Greco-Roman mythology. Midas represents wealth. 9
23456789012345678901234567890121234567890123456789012345678
123456789012345678901234567890121234567890123456789012345678 9
123456789012345678901234567890121234567890123456789012345678
Unger resonates with the hunger the family feels for the wealth and 9
1 9
123456789012345678901234567890121234567890123456789012345678 9
prestige of the North. In other words, the names suggest that the story
123456789012345678901234567890121234567890123456789012345678 9
123456789012345678901234567890121234567890123456789012345678
uses the experiences of this one family to represent a larger situation. It 9
123456789012345678901234567890121234567890123456789012345678 9
123456789012345678901234567890121234567890123456789012345678 9
is an allegory, choice (E).
123456789012345678901234567890121234567890123456789012345678 9
123456789012345678901234567890121234567890123456789012345678 9
123456789012345678901234567890121234567890123456789012345678 9
23456789012345678901234567890121234567890123456789012345678
11. C Infused is used to mean that his work was filled with the experiences he
123456789012345678901234567890121234567890123456789012345678
had in Manchuria. Eliminate all but (B) and (C). Saturated has 9
1 9
123456789012345678901234567890121234567890123456789012345678
something of a negative tone, and the author praises Abe’s work, so 9
123456789012345678901234567890121234567890123456789012345678 9
23456789012345678901234567890121234567890123456789012345678
123456789012345678901234567890121234567890123456789012345678
eliminate (B). The answer is (C). 9
123456789012345678901234567890121234567890123456789012345678 9
123456789012345678901234567890121234567890123456789012345678 9
123456789012345678901234567890121234567890123456789012345678
12. E The metaphorical use of orbit and gravitational pull is used in 9
1 9
123456789012345678901234567890121234567890123456789012345678 9
23456789012345678901234567890121234567890123456789012345678
conjunction with the negative words “controlled” and “oppressive.”
9
123456789012345678901234567890121234567890123456789012345678 9
123456789012345678901234567890121234567890123456789012345678
Abe’s work is not controlled by oppressive forces. Eliminate (B), (C),
9
123456789012345678901234567890121234567890123456789012345678 9
1 and (D). Choices (A) and (E) are similar answers, but (E) better captures
23456789012345678901234567890121234567890123456789012345678
123456789012345678901234567890121234567890123456789012345678 9
123456789012345678901234567890121234567890123456789012345678
the author’s intent. 9
123456789012345678901234567890121234567890123456789012345678 9 9
123456789012345678901234567890121234567890123456789012345678 9
1 13. D Abe forged a medical certificate, so we know he was not actually sick.
23456789012345678901234567890121234567890123456789012345678
123456789012345678901234567890121234567890123456789012345678 9
1 You can eliminate (A) and (B). The passage makes no reference to Abe 9
123456789012345678901234567890121234567890123456789012345678 9
123456789012345678901234567890121234567890123456789012345678 9
helping the sick and injured, so eliminate (C). The sentence in the
123456789012345678901234567890121234567890123456789012345678
passage says that the forged medical certificate allowed him to avoid 9
123456789012345678901234567890121234567890123456789012345678 9
123456789012345678901234567890121234567890123456789012345678 9
123456789012345678901234567890121234567890123456789012345678 9
123456789012345678901234567890121234567890123456789012345678 9
1234567890123456789012345678901212345678901234567890123456789
50 Copyright © 2005 Thomson Peterson’s, a part of The Thomson Corporaton
SAT is a registered trademark of the College Entrance Examination Board, which
was not involved in the production of and does not endorse this product.
1234567890123456789012345678901212345678901234567890123456789
123456789012345678901234567890121234567890123456789012345678 9
123456789012345678901234567890121234567890123456789012345678 9
123456789012345678901234567890121234567890123456789012345678 9
123456789012345678901234567890121234567890123456789012345678 9
123456789012345678901234567890121234567890123456789012345678
fighting. Choice (D) corresponds with that meaning. Choice (E) can be 9
123456789012345678901234567890121234567890123456789012345678 9
123456789012345678901234567890121234567890123456789012345678
eliminated because you don’t know
23456789012345678901234567890121234567890123456789012345678
what his intentions were for after 9
9
1
123456789012345678901234567890121234567890123456789012345678
the war. 9
123456789012345678901234567890121234567890123456789012345678 9
123456789012345678901234567890121234567890123456789012345678 9
123456789012345678901234567890121234567890123456789012345678
14. E Even if you don’t know the defintion
23456789012345678901234567890121234567890123456789012345678
of apocalyptic, you probably 9
9
1 know that it is a negative word. Eliminate (D) (exhilarating is a positive
123456789012345678901234567890121234567890123456789012345678 9
123456789012345678901234567890121234567890123456789012345678
word). There is no reference to nuclear weapons in the passage, so 9
123456789012345678901234567890121234567890123456789012345678 9
123456789012345678901234567890121234567890123456789012345678
eliminate (A). There was famine, and Abe seems to have been strongly
23456789012345678901234567890121234567890123456789012345678
9
9
1
123456789012345678901234567890121234567890123456789012345678
affected by the loss of his father, but neither of these answers is specific 9
123456789012345678901234567890121234567890123456789012345678 9
123456789012345678901234567890121234567890123456789012345678
enough. Only (E) expressly answers the question. 9
123456789012345678901234567890121234567890123456789012345678 9
1 15. A This question is a little bit more difficult than some vocabulary 9
23456789012345678901234567890121234567890123456789012345678
123456789012345678901234567890121234567890123456789012345678 9
123456789012345678901234567890121234567890123456789012345678 9
123456789012345678901234567890121234567890123456789012345678
questions because you have to look in a few different places. The third 9
123456789012345678901234567890121234567890123456789012345678 9
123456789012345678901234567890121234567890123456789012345678 99
23456789012345678901234567890121234567890123456789012345678
paragraph, where the word appears, tells you that the avant-garde
123456789012345678901234567890121234567890123456789012345678
123456789012345678901234567890121234567890123456789012345678
group was political and that Abe worked in various genres. The fourth 9
123456789012345678901234567890121234567890123456789012345678 9
123456789012345678901234567890121234567890123456789012345678 99
paragraph refers to his earlier work, which was the work in the third
123456789012345678901234567890121234567890123456789012345678 9
1 paragraph, as “experimental and heavily political.” Since one of these
123456789012345678901234567890121234567890123456789012345678 9
123456789012345678901234567890121234567890123456789012345678
words is an answer choice (A), it is the best answer. 9
123456789012345678901234567890121234567890123456789012345678 9
23456789012345678901234567890121234567890123456789012345678
123456789012345678901234567890121234567890123456789012345678
16. C This question basically asks you to distinguish between the author’s 9
123456789012345678901234567890121234567890123456789012345678 9
123456789012345678901234567890121234567890123456789012345678
opinion and the basic facts of Abe’s career. Choices (A), (B), and (E) all 9
123456789012345678901234567890121234567890123456789012345678 99
123456789012345678901234567890121234567890123456789012345678
contain evaluative opinions, so eliminate them. The author expresses
123456789012345678901234567890121234567890123456789012345678 9
123456789012345678901234567890121234567890123456789012345678
strong opinions about the themes furusato and the emperor, but never 9
123456789012345678901234567890121234567890123456789012345678 9
123456789012345678901234567890121234567890123456789012345678
presents any facts about their influence on Japanese literature in the 9
1 9
123456789012345678901234567890121234567890123456789012345678 99
23456789012345678901234567890121234567890123456789012345678
world. The best answer is (C). The author presents it as a known fact
123456789012345678901234567890121234567890123456789012345678 9
1 that young Japanese artists after World War II were interested in
123456789012345678901234567890121234567890123456789012345678 9
123456789012345678901234567890121234567890123456789012345678
Marxism. 9
123456789012345678901234567890121234567890123456789012345678 99
23456789012345678901234567890121234567890123456789012345678
123456789012345678901234567890121234567890123456789012345678
1 17. B As always, go back to the passage to look for the context of the phrase. 9
123456789012345678901234567890121234567890123456789012345678 9
123456789012345678901234567890121234567890123456789012345678
Shortly after the phrase appears, the author says that readers have 9
23456789012345678901234567890121234567890123456789012345678
123456789012345678901234567890121234567890123456789012345678 9
123456789012345678901234567890121234567890123456789012345678
wrongly decided that Woman in the Dunes was Abe’s masterpiece. The 9
1 9
123456789012345678901234567890121234567890123456789012345678 9
author also refers to the lack of translations of Abe’s earlier works. The
123456789012345678901234567890121234567890123456789012345678 9
123456789012345678901234567890121234567890123456789012345678
answer that best summarizes these two things is (B). 9
123456789012345678901234567890121234567890123456789012345678 9
123456789012345678901234567890121234567890123456789012345678
18. D The author’s purpose in paragraph 4 is to suggest that too much 9
123456789012345678901234567890121234567890123456789012345678 9
123456789012345678901234567890121234567890123456789012345678
attention has been given to Abe’s later work, as you just determined in 9
23456789012345678901234567890121234567890123456789012345678
123456789012345678901234567890121234567890123456789012345678 9
123456789012345678901234567890121234567890123456789012345678
question 17. So the answer cannot be (A). There is only a brief 9
123456789012345678901234567890121234567890123456789012345678 9
123456789012345678901234567890121234567890123456789012345678
comparison to Abe’s contemporaries, so (B) is too specific. (E) is not 9
1 9
123456789012345678901234567890121234567890123456789012345678 99
23456789012345678901234567890121234567890123456789012345678
factually correct, since most of the work the passage discusses was
123456789012345678901234567890121234567890123456789012345678 9
123456789012345678901234567890121234567890123456789012345678
produced in Japan. You are left with (C) and (D). (C) is too neutral; this
9
123456789012345678901234567890121234567890123456789012345678 9
1 author is opinionated. She/he does not suggest that Abe’s later work is
23456789012345678901234567890121234567890123456789012345678
123456789012345678901234567890121234567890123456789012345678 9
1 bad, but rather that his early work also deserves attention. Choice (D) is 9
123456789012345678901234567890121234567890123456789012345678 9
123456789012345678901234567890121234567890123456789012345678 9
the best answer.
123456789012345678901234567890121234567890123456789012345678 9
123456789012345678901234567890121234567890123456789012345678 9
123456789012345678901234567890121234567890123456789012345678 9
123456789012345678901234567890121234567890123456789012345678 9
123456789012345678901234567890121234567890123456789012345678 9
1234567890123456789012345678901212345678901234567890123456789
Copyright © 2005 Thomson Peterson’s, a part of The Thomson Corporaton 51
SAT is a registered trademark of the College Entrance Examination Board, which
was not involved in the production of and does not endorse this product.
1234567890123456789012345678901212345678901234567890123456789
123456789012345678901234567890121234567890123456789012345678 9
123456789012345678901234567890121234567890123456789012345678 9
123456789012345678901234567890121234567890123456789012345678 9
123456789012345678901234567890121234567890123456789012345678 9
123456789012345678901234567890121234567890123456789012345678
19. B The author is most interested in literary works. There is no reason to 9
123456789012345678901234567890121234567890123456789012345678 9
123456789012345678901234567890121234567890123456789012345678
suspect that the author is an artist or
23456789012345678901234567890121234567890123456789012345678
writer. The tone is critical and 9
9
1
123456789012345678901234567890121234567890123456789012345678
scholarly. (B) is the best answer. 9
123456789012345678901234567890121234567890123456789012345678 9
123456789012345678901234567890121234567890123456789012345678 9
123456789012345678901234567890121234567890123456789012345678
20. D The author of this passage does express
23456789012345678901234567890121234567890123456789012345678
many strong opinions, but not 9
9
1 in regard to Marxism. You can therefore eliminate both (A) and (B). If
123456789012345678901234567890121234567890123456789012345678 9
123456789012345678901234567890121234567890123456789012345678
anything, she/he is more positive than negative about the influence of 9
123456789012345678901234567890121234567890123456789012345678 9
123456789012345678901234567890121234567890123456789012345678
Marxism on Abe’s work. Eliminate (C) and (E), which imply a negative
23456789012345678901234567890121234567890123456789012345678
9
9
1
123456789012345678901234567890121234567890123456789012345678
bias. The answer is (D). 9
123456789012345678901234567890121234567890123456789012345678 9
123456789012345678901234567890121234567890123456789012345678 9
123456789012345678901234567890121234567890123456789012345678
23456789012345678901234567890121234567890123456789012345678
9
9
1 Section 2
123456789012345678901234567890121234567890123456789012345678 9
123456789012345678901234567890121234567890123456789012345678 9
123456789012345678901234567890121234567890123456789012345678 9
123456789012345678901234567890121234567890123456789012345678
1. C With a calculator this problem is straightforward enough, but you do not 9
23456789012345678901234567890121234567890123456789012345678
123456789012345678901234567890121234567890123456789012345678 9
123456789012345678901234567890121234567890123456789012345678
need a calculator to solve this problem. The wording is a bit tricky, but to 9
123456789012345678901234567890121234567890123456789012345678 9
123456789012345678901234567890121234567890123456789012345678 9
find the percentage of 75 that 12 represents, you would place 12 over 75.
123456789012345678901234567890121234567890123456789012345678 9
123456789012345678901234567890121234567890123456789012345678
12 12/3 4 9
1 This can be simplified 5 5 . Percent means “of 100,” so if you 9
123456789012345678901234567890121234567890123456789012345678
75 75/3 25 9
123456789012345678901234567890121234567890123456789012345678
change the 25 in the denominator to 100, you’ll have your percentage in 9
123456789012345678901234567890121234567890123456789012345678 9
123456789012345678901234567890121234567890123456789012345678 9
23456789012345678901234567890121234567890123456789012345678
4 4 3 4 16
123456789012345678901234567890121234567890123456789012345678
the numerator. 5 5 . This is answer (C). 9
123456789012345678901234567890121234567890123456789012345678
25 25 3 4 100 9
123456789012345678901234567890121234567890123456789012345678 9
123456789012345678901234567890121234567890123456789012345678 9
123456789012345678901234567890121234567890123456789012345678
You could also have set up an algebraic equation, and then cross- 9
123456789012345678901234567890121234567890123456789012345678 9
123456789012345678901234567890121234567890123456789012345678 9
multiplied to find the answer.
123456789012345678901234567890121234567890123456789012345678 9 9
1
123456789012345678901234567890121234567890123456789012345678
12 n 9
123456789012345678901234567890121234567890123456789012345678
5 9
123456789012345678901234567890121234567890123456789012345678 9
75 100
123456789012345678901234567890121234567890123456789012345678 9
123456789012345678901234567890121234567890123456789012345678
~12!~100! 5 ~75!~n! 9
123456789012345678901234567890121234567890123456789012345678 9
23456789012345678901234567890121234567890123456789012345678
123456789012345678901234567890121234567890123456789012345678
1200 5 75n 9
1 9
123456789012345678901234567890121234567890123456789012345678 9
123456789012345678901234567890121234567890123456789012345678
1200 75n 9
23456789012345678901234567890121234567890123456789012345678
123456789012345678901234567890121234567890123456789012345678
5 9
123456789012345678901234567890121234567890123456789012345678
75 75 9
1 9
123456789012345678901234567890121234567890123456789012345678
16 5 n 9
123456789012345678901234567890121234567890123456789012345678 9
23456789012345678901234567890121234567890123456789012345678
123456789012345678901234567890121234567890123456789012345678 9
123456789012345678901234567890121234567890123456789012345678
2. D If a circle is inscribed in a square, then the circle is inside the square. 9
123456789012345678901234567890121234567890123456789012345678 9
123456789012345678901234567890121234567890123456789012345678 9
You can find the length of the square’s sides using the area formula for
9
1
123456789012345678901234567890121234567890123456789012345678
a square: 9
123456789012345678901234567890121234567890123456789012345678 9
123456789012345678901234567890121234567890123456789012345678 9
123456789012345678901234567890121234567890123456789012345678 9
2
A 5 s
123456789012345678901234567890121234567890123456789012345678 9
123456789012345678901234567890121234567890123456789012345678 9
23456789012345678901234567890121234567890123456789012345678
36 5 s2
123456789012345678901234567890121234567890123456789012345678 9 9
123456789012345678901234567890121234567890123456789012345678
65s
9
123456789012345678901234567890121234567890123456789012345678 9
1
123456789012345678901234567890121234567890123456789012345678 9
23456789012345678901234567890121234567890123456789012345678
It might help if you draw a circle inside a square to visualize the next
9
1
123456789012345678901234567890121234567890123456789012345678
part. The side length of the square is the same as the diameter of the 9
123456789012345678901234567890121234567890123456789012345678 9
123456789012345678901234567890121234567890123456789012345678
circle. Draw a diameter and you’ll see
23456789012345678901234567890121234567890123456789012345678
that it’s the same length as a side. 9
9
1
123456789012345678901234567890121234567890123456789012345678 9
123456789012345678901234567890121234567890123456789012345678 9
123456789012345678901234567890121234567890123456789012345678 9
1234567890123456789012345678901212345678901234567890123456789
52 Copyright © 2005 Thomson Peterson’s, a part of The Thomson Corporaton
SAT is a registered trademark of the College Entrance Examination Board, which
was not involved in the production of and does not endorse this product.
1234567890123456789012345678901212345678901234567890123456789
123456789012345678901234567890121234567890123456789012345678 9
123456789012345678901234567890121234567890123456789012345678 9
123456789012345678901234567890121234567890123456789012345678 9
123456789012345678901234567890121234567890123456789012345678 9
123456789012345678901234567890121234567890123456789012345678
This is a key relationship that the two figures share. Once you know the 9
123456789012345678901234567890121234567890123456789012345678 9
123456789012345678901234567890121234567890123456789012345678
circle’s diameter is 6, its radius must
23456789012345678901234567890121234567890123456789012345678
be half that, 3. This radius can be 9
9
1
123456789012345678901234567890121234567890123456789012345678
placed into the area formula for a circle: 9
123456789012345678901234567890121234567890123456789012345678 9
123456789012345678901234567890121234567890123456789012345678 9
123456789012345678901234567890121234567890123456789012345678 9
2
A5 pr
123456789012345678901234567890121234567890123456789012345678 9
123456789012345678901234567890121234567890123456789012345678
A 5 p~3!2 9
123456789012345678901234567890121234567890123456789012345678 9
123456789012345678901234567890121234567890123456789012345678
A 9p 9
123456789012345678901234567890121234567890123456789012345678
5
23456789012345678901234567890121234567890123456789012345678
9
9
1
123456789012345678901234567890121234567890123456789012345678
Choice (D) is correct. 9
123456789012345678901234567890121234567890123456789012345678 9
123456789012345678901234567890121234567890123456789012345678 9
123456789012345678901234567890121234567890123456789012345678
3. B First, you are looking for a line with
23456789012345678901234567890121234567890123456789012345678
a positive slope, which means that it 9
9
1
123456789012345678901234567890121234567890123456789012345678
rises as you go from left to right. This eliminates choices (A), (D), and (E). 9
123456789012345678901234567890121234567890123456789012345678 9
123456789012345678901234567890121234567890123456789012345678
Second, a slope of 1 means that it rises as much as it runs (it goes up at the 9
123456789012345678901234567890121234567890123456789012345678 9
123456789012345678901234567890121234567890123456789012345678 99
23456789012345678901234567890121234567890123456789012345678
same rate that it goes over). A line with a slope of 1 will be halfway
123456789012345678901234567890121234567890123456789012345678
123456789012345678901234567890121234567890123456789012345678
between a line that is completely horizontal and a line that is completely 9
123456789012345678901234567890121234567890123456789012345678 9
123456789012345678901234567890121234567890123456789012345678 99
vertical. Choice (B) is that line, since the rise in choice (C) is too gradual.
123456789012345678901234567890121234567890123456789012345678 9
1 4. C To answer this question, you have to determine how the series is
123456789012345678901234567890121234567890123456789012345678 9
123456789012345678901234567890121234567890123456789012345678
generated. The numbers are increasing, so it is very unlikely that either 9
123456789012345678901234567890121234567890123456789012345678 9
23456789012345678901234567890121234567890123456789012345678
123456789012345678901234567890121234567890123456789012345678
subtraction or division is involved. The numbers increase, and note 9
123456789012345678901234567890121234567890123456789012345678 9
123456789012345678901234567890121234567890123456789012345678
how fast the increase is. Addition can be ruled out since the increase 9
123456789012345678901234567890121234567890123456789012345678 99
123456789012345678901234567890121234567890123456789012345678
from one term to the next is too great; simple multiplication is also
123456789012345678901234567890121234567890123456789012345678 9
123456789012345678901234567890121234567890123456789012345678
unlikely. Look at the four terms closely, and you’ll notice that each 9
123456789012345678901234567890121234567890123456789012345678 9
123456789012345678901234567890121234567890123456789012345678
number is the square of an ascending integer. The series is one squared, 9
1 9
123456789012345678901234567890121234567890123456789012345678 99
23456789012345678901234567890121234567890123456789012345678
then two squared, then three squared, and so forth. This series then is
123456789012345678901234567890121234567890123456789012345678 9
1 generated by squaring the integers. Therefore, the eighth term is eight
123456789012345678901234567890121234567890123456789012345678 9
123456789012345678901234567890121234567890123456789012345678
squared, 64. Choice (C) is correct. 9
123456789012345678901234567890121234567890123456789012345678 99
23456789012345678901234567890121234567890123456789012345678
123456789012345678901234567890121234567890123456789012345678 9
1 5. B The area of a parallelogram is the height times the base. You do not
123456789012345678901234567890121234567890123456789012345678 9
123456789012345678901234567890121234567890123456789012345678
know the height, but you can determine it by using the geometry of a 9
23456789012345678901234567890121234567890123456789012345678
123456789012345678901234567890121234567890123456789012345678 9
123456789012345678901234567890121234567890123456789012345678
triangle. If you drop a perpendicular from the top left corner to the 9
1 9
123456789012345678901234567890121234567890123456789012345678 9
opposite side (which you will call the base), then you have a triangle
123456789012345678901234567890121234567890123456789012345678
whose height is the height of the parallelogram. Measures of adjacent 9
123456789012345678901234567890121234567890123456789012345678 9
123456789012345678901234567890121234567890123456789012345678
angles of parallelograms sum to 180, and so the bottom left-hand angle 9
123456789012345678901234567890121234567890123456789012345678 9
123456789012345678901234567890121234567890123456789012345678
measures 60° (this is because the bottom right interior angle is 120°, 9
123456789012345678901234567890121234567890123456789012345678 9
23456789012345678901234567890121234567890123456789012345678
123456789012345678901234567890121234567890123456789012345678
and 180 2 120 5 60). Surprise! This gives you a 30-60-90 triangle, and 9
123456789012345678901234567890121234567890123456789012345678 9
123456789012345678901234567890121234567890123456789012345678 = 9
123456789012345678901234567890121234567890123456789012345678 99
you can determine the height. Since the hypotenuse is 2 3, the height
1
123456789012345678901234567890121234567890123456789012345678
is 3. The base is 6, and so the area is: 9
123456789012345678901234567890121234567890123456789012345678 9
123456789012345678901234567890121234567890123456789012345678 9
123456789012345678901234567890121234567890123456789012345678
A 5 bh 9
123456789012345678901234567890121234567890123456789012345678 9
123456789012345678901234567890121234567890123456789012345678 99
23456789012345678901234567890121234567890123456789012345678
A 5 ~ 6 !~3 !
1
123456789012345678901234567890121234567890123456789012345678
A 5 18 9
123456789012345678901234567890121234567890123456789012345678 9
123456789012345678901234567890121234567890123456789012345678
23456789012345678901234567890121234567890123456789012345678
9
9
1 That’s choice (B).
123456789012345678901234567890121234567890123456789012345678 9
123456789012345678901234567890121234567890123456789012345678 9
123456789012345678901234567890121234567890123456789012345678 9
1234567890123456789012345678901212345678901234567890123456789
Copyright © 2005 Thomson Peterson’s, a part of The Thomson Corporaton 53
SAT is a registered trademark of the College Entrance Examination Board, which
was not involved in the production of and does not endorse this product.
1234567890123456789012345678901212345678901234567890123456789
123456789012345678901234567890121234567890123456789012345678 9
123456789012345678901234567890121234567890123456789012345678 9
123456789012345678901234567890121234567890123456789012345678 9
123456789012345678901234567890121234567890123456789012345678 9
123456789012345678901234567890121234567890123456789012345678
6. A To find the reciprocal, switch the numerator and the denominator. 9
123456789012345678901234567890121234567890123456789012345678 9
123456789012345678901234567890121234567890123456789012345678
Once you’ve flipped the fractions over,
23456789012345678901234567890121234567890123456789012345678
look for a fraction where the 9
9
1
123456789012345678901234567890121234567890123456789012345678
numerator is greater than the denominator and where the difference 9
123456789012345678901234567890121234567890123456789012345678 9
123456789012345678901234567890121234567890123456789012345678
between the two is greatest. Choices (A) and (E) look like the best 9
123456789012345678901234567890121234567890123456789012345678 9
123456789012345678901234567890121234567890123456789012345678
1
123456789012345678901234567890121234567890123456789012345678
candidates. The reciprocal of A is 4
123456789012345678901234567890121234567890123456789012345678
2
S 9
2
1
D
5 4 , and the reciprocal of
2
9
9
9
1 (E) is 4
1 13
3 3
S 1
D
123456789012345678901234567890121234567890123456789012345678
123456789012345678901234567890121234567890123456789012345678
5 4 . Choice (A) is the greatest.
23456789012345678901234567890121234567890123456789012345678
3
123456789012345678901234567890121234567890123456789012345678
9
9
9
9
123456789012345678901234567890121234567890123456789012345678 9
123456789012345678901234567890121234567890123456789012345678
7. E Another sketch with more of the angles numbered is helpful here. 9
123456789012345678901234567890121234567890123456789012345678
23456789012345678901234567890121234567890123456789012345678
9
9
1
123456789012345678901234567890121234567890123456789012345678 9
123456789012345678901234567890121234567890123456789012345678 9
123456789012345678901234567890121234567890123456789012345678 9
123456789012345678901234567890121234567890123456789012345678 9
123456789012345678901234567890121234567890123456789012345678 9
23456789012345678901234567890121234567890123456789012345678
123456789012345678901234567890121234567890123456789012345678 9
123456789012345678901234567890121234567890123456789012345678 9
123456789012345678901234567890121234567890123456789012345678 9
123456789012345678901234567890121234567890123456789012345678 9
123456789012345678901234567890121234567890123456789012345678 9 9
1
123456789012345678901234567890121234567890123456789012345678 9
123456789012345678901234567890121234567890123456789012345678 9
123456789012345678901234567890121234567890123456789012345678 9
123456789012345678901234567890121234567890123456789012345678 9
23456789012345678901234567890121234567890123456789012345678
123456789012345678901234567890121234567890123456789012345678 9
123456789012345678901234567890121234567890123456789012345678 9
123456789012345678901234567890121234567890123456789012345678 9
123456789012345678901234567890121234567890123456789012345678
Angle 1 is 90° because its measure and the measure of the angle 9
123456789012345678901234567890121234567890123456789012345678 9
123456789012345678901234567890121234567890123456789012345678
adjacent to it must sum to 180. Couple this fact with the given 9
123456789012345678901234567890121234567890123456789012345678 9
123456789012345678901234567890121234567890123456789012345678
information a 5 60, and it means that the measure of angle 2 is 30° 9
1 9
123456789012345678901234567890121234567890123456789012345678 9
23456789012345678901234567890121234567890123456789012345678
since the measures of the angles in that triangle must sum to 180.
123456789012345678901234567890121234567890123456789012345678 9 9
1
123456789012345678901234567890121234567890123456789012345678
Angle 3 is also 30° since it is vertical with angle 2. If you can find angle 9
123456789012345678901234567890121234567890123456789012345678 9
123456789012345678901234567890121234567890123456789012345678 9
23456789012345678901234567890121234567890123456789012345678
4, you could figure out angle x since you will have two of the three
123456789012345678901234567890121234567890123456789012345678 9
1 angles on that small triangle. To find the measure of angle 4, look at the 9
123456789012345678901234567890121234567890123456789012345678 9
123456789012345678901234567890121234567890123456789012345678
big triangle with angles 4, c, and the right angle. Since a right angle is 9
23456789012345678901234567890121234567890123456789012345678
123456789012345678901234567890121234567890123456789012345678 9
123456789012345678901234567890121234567890123456789012345678
90° and c 5 50, the measure of angle 4 is 40° since the measures of the 9
1 9
123456789012345678901234567890121234567890123456789012345678 9
angles in the larger triangle must sum to 180. If the measure of angle
123456789012345678901234567890121234567890123456789012345678 9
123456789012345678901234567890121234567890123456789012345678
4 5 40, and the measure of the angle 3 5 30, then 9
123456789012345678901234567890121234567890123456789012345678 9
123456789012345678901234567890121234567890123456789012345678
180 5 m∠3 1 m∠4 1 m∠x 9
123456789012345678901234567890121234567890123456789012345678 9
123456789012345678901234567890121234567890123456789012345678 9
123456789012345678901234567890121234567890123456789012345678 9
23456789012345678901234567890121234567890123456789012345678
180 5 30 1 40 1 x
9
123456789012345678901234567890121234567890123456789012345678 9
123456789012345678901234567890121234567890123456789012345678
180 5 70 1 x
123456789012345678901234567890121234567890123456789012345678 9 9
1 110 5 x
123456789012345678901234567890121234567890123456789012345678 9
123456789012345678901234567890121234567890123456789012345678 9
123456789012345678901234567890121234567890123456789012345678
Choice (E) is correct. 9
123456789012345678901234567890121234567890123456789012345678 9
123456789012345678901234567890121234567890123456789012345678
If you had no idea how to answer this question, you might have noticed 9
23456789012345678901234567890121234567890123456789012345678
123456789012345678901234567890121234567890123456789012345678 9
1 that the figures were drawn to scale. Looking at x, it certainly looks 9
123456789012345678901234567890121234567890123456789012345678 9
123456789012345678901234567890121234567890123456789012345678
greater than 90°. Choice (A) is highly unlikely as an answer, and (E) 9
123456789012345678901234567890121234567890123456789012345678
23456789012345678901234567890121234567890123456789012345678
9
9
1 would be your best guess since it’s the only choice greater than 90°.
123456789012345678901234567890121234567890123456789012345678 9
123456789012345678901234567890121234567890123456789012345678 9
123456789012345678901234567890121234567890123456789012345678 9
1234567890123456789012345678901212345678901234567890123456789
54 Copyright © 2005 Thomson Peterson’s, a part of The Thomson Corporaton
SAT is a registered trademark of the College Entrance Examination Board, which
was not involved in the production of and does not endorse this product.
1234567890123456789012345678901212345678901234567890123456789
123456789012345678901234567890121234567890123456789012345678 9
123456789012345678901234567890121234567890123456789012345678 9
123456789012345678901234567890121234567890123456789012345678 9
123456789012345678901234567890121234567890123456789012345678 9
123456789012345678901234567890121234567890123456789012345678
8. E A problem like this takes a careful step-by-step execution, but 9
123456789012345678901234567890121234567890123456789012345678 9
123456789012345678901234567890121234567890123456789012345678
fortunately nothing else is needed.
23456789012345678901234567890121234567890123456789012345678
9
9
1
123456789012345678901234567890121234567890123456789012345678
123456789012345678901234567890121234567890123456789012345678
@ ~ 2 2 21 22
! # @ ~ !21 22
#
123456789012345678901234567890121234567890123456789012345678
2 1 2 5 4 1 4
123456789012345678901234567890121234567890123456789012345678
5
1
8
FG
22
5
1
1
5 64, choice (E).
9
9
9
9
123456789012345678901234567890121234567890123456789012345678 9
123456789012345678901234567890121234567890123456789012345678 64 9
123456789012345678901234567890121234567890123456789012345678 9
123456789012345678901234567890121234567890123456789012345678
9. C Here you need to take care to read the chart correctly. Each complete 9
123456789012345678901234567890121234567890123456789012345678
23456789012345678901234567890121234567890123456789012345678
9
9
1 box represents 150,000 widgets. Company B made one-and-a-half
123456789012345678901234567890121234567890123456789012345678 9
123456789012345678901234567890121234567890123456789012345678
more boxes. That last box is not a full box, as you can see by its size and 9
123456789012345678901234567890121234567890123456789012345678 9
123456789012345678901234567890121234567890123456789012345678
by the dashed lines on the right end.
23456789012345678901234567890121234567890123456789012345678
Choice (B) represents one box 9
9
1
123456789012345678901234567890121234567890123456789012345678
worth of difference (150,000 widgets), while choice (D)represents two 9
123456789012345678901234567890121234567890123456789012345678 9
123456789012345678901234567890121234567890123456789012345678
complete boxes of widgets as the difference. Answer (C) is correct, since 9
123456789012345678901234567890121234567890123456789012345678 9
123456789012345678901234567890121234567890123456789012345678 99
23456789012345678901234567890121234567890123456789012345678
it shows a difference in production of one-and-a-half boxes (150,000
123456789012345678901234567890121234567890123456789012345678
123456789012345678901234567890121234567890123456789012345678
widgets 1 75,000 widgets 5 225,000). 9
123456789012345678901234567890121234567890123456789012345678 99
123456789012345678901234567890121234567890123456789012345678
10. A Since you are told that x is odd, you should suspect that the answer
123456789012345678901234567890121234567890123456789012345678 9
1 would have something to do with being odd or even or neither. That 9
123456789012345678901234567890121234567890123456789012345678 9
123456789012345678901234567890121234567890123456789012345678
makes choices (A) or (B) the prime suspects. x cannot be even because 9
123456789012345678901234567890121234567890123456789012345678 9
23456789012345678901234567890121234567890123456789012345678
123456789012345678901234567890121234567890123456789012345678
an even number raised to an even power must be even. Try giving x an 9
123456789012345678901234567890121234567890123456789012345678 9
123456789012345678901234567890121234567890123456789012345678
x 3
odd value, like 3. x 5 3 5 27. This satisfies the facts given in the 9
123456789012345678901234567890121234567890123456789012345678 9
123456789012345678901234567890121234567890123456789012345678
problem, since 27 is an odd number, so choice (A) is the answer. 9
123456789012345678901234567890121234567890123456789012345678 99
123456789012345678901234567890121234567890123456789012345678
123456789012345678901234567890121234567890123456789012345678
11. E Do not be unnerved by the newness of this concept. All that you need to 9
123456789012345678901234567890121234567890123456789012345678 9
1 know about factorials is provided in the explanation. So be a good 9
23456789012345678901234567890121234567890123456789012345678
123456789012345678901234567890121234567890123456789012345678 9
123456789012345678901234567890121234567890123456789012345678
test-taking robot: Take the numbers they give you and feed them into 9
1 9
123456789012345678901234567890121234567890123456789012345678 9
the formula.
123456789012345678901234567890121234567890123456789012345678 9
123456789012345678901234567890121234567890123456789012345678
6! 6 3 5 3 4 3 3 3 2 3 1 9
123456789012345678901234567890121234567890123456789012345678 9
123456789012345678901234567890121234567890123456789012345678
5 5 6 3 5 3 4 5 120, choice (E). 9
123456789012345678901234567890121234567890123456789012345678
3! 3 3 2 3 1 9
123456789012345678901234567890121234567890123456789012345678 9
123456789012345678901234567890121234567890123456789012345678 99
23456789012345678901234567890121234567890123456789012345678
If you don’t get flustered (and no self-respecting test-taking robot ever
123456789012345678901234567890121234567890123456789012345678 9
1 does), the problem is quite straightforward. You just apply the concept
123456789012345678901234567890121234567890123456789012345678 9
123456789012345678901234567890121234567890123456789012345678 9
of factorial and then multiply to find the answer.
123456789012345678901234567890121234567890123456789012345678 99
23456789012345678901234567890121234567890123456789012345678
123456789012345678901234567890121234567890123456789012345678 9
123456789012345678901234567890121234567890123456789012345678
12. B Here replace x with three, and then solve:
9
123456789012345678901234567890121234567890123456789012345678 9
1 2 2 2
f(3) 5 (3!) 5 (3 3 2 3 1) 5 (6) 5 36, answer (B).
123456789012345678901234567890121234567890123456789012345678 99
23456789012345678901234567890121234567890123456789012345678
123456789012345678901234567890121234567890123456789012345678 9
123456789012345678901234567890121234567890123456789012345678 9
123456789012345678901234567890121234567890123456789012345678 9
1
123456789012345678901234567890121234567890123456789012345678 9
123456789012345678901234567890121234567890123456789012345678 9
123456789012345678901234567890121234567890123456789012345678 9
123456789012345678901234567890121234567890123456789012345678 9
123456789012345678901234567890121234567890123456789012345678 9
123456789012345678901234567890121234567890123456789012345678 99
23456789012345678901234567890121234567890123456789012345678
1
123456789012345678901234567890121234567890123456789012345678 9
123456789012345678901234567890121234567890123456789012345678 9
123456789012345678901234567890121234567890123456789012345678 9
123456789012345678901234567890121234567890123456789012345678 9
123456789012345678901234567890121234567890123456789012345678 9
123456789012345678901234567890121234567890123456789012345678 9
123456789012345678901234567890121234567890123456789012345678 9
1234567890123456789012345678901212345678901234567890123456789
Copyright © 2005 Thomson Peterson’s, a part of The Thomson Corporaton 55
SAT is a registered trademark of the College Entrance Examination Board, which
was not involved in the production of and does not endorse this product.
1234567890123456789012345678901212345678901234567890123456789
123456789012345678901234567890121234567890123456789012345678 9
123456789012345678901234567890121234567890123456789012345678 9
123456789012345678901234567890121234567890123456789012345678 9
123456789012345678901234567890121234567890123456789012345678 9
123456789012345678901234567890121234567890123456789012345678
13. A This one’s the toughest factorial problem, but a clue is provided by the 9
123456789012345678901234567890121234567890123456789012345678 9
123456789012345678901234567890121234567890123456789012345678
answer choices. Most of the answers are
23456789012345678901234567890121234567890123456789012345678
in terms of y. Therefore, you 9
9
1
123456789012345678901234567890121234567890123456789012345678 y! 9
123456789012345678901234567890121234567890123456789012345678
are probably going to have to manipulate so that only y’s remain in 9
123456789012345678901234567890121234567890123456789012345678 x! 9
123456789012345678901234567890121234567890123456789012345678
the expression.
23456789012345678901234567890121234567890123456789012345678
9
9
1
123456789012345678901234567890121234567890123456789012345678 9
123456789012345678901234567890121234567890123456789012345678
To do this, let’s replace the x’s with y’s. Since, and y and x are integers, 9
123456789012345678901234567890121234567890123456789012345678 9
123456789012345678901234567890121234567890123456789012345678
x! 5 (y 1 2)!. Substituting this into the
23456789012345678901234567890121234567890123456789012345678
problem: 9
9
1
123456789012345678901234567890121234567890123456789012345678 9
123456789012345678901234567890121234567890123456789012345678
y! y! y! 1 9
123456789012345678901234567890121234567890123456789012345678
5 5 5 9
123456789012345678901234567890121234567890123456789012345678
x! ~y 1 2!! ~y 1 2!~y 1 1!y! ~y 1 2!~y 1 1!
23456789012345678901234567890121234567890123456789012345678
9
9
1
123456789012345678901234567890121234567890123456789012345678 9
123456789012345678901234567890121234567890123456789012345678
This is choice (A). You could also solve the problem by picking values 9
123456789012345678901234567890121234567890123456789012345678 9
123456789012345678901234567890121234567890123456789012345678
for the two variables that are consistent with y 1 2 5 x, and then 9
23456789012345678901234567890121234567890123456789012345678
123456789012345678901234567890121234567890123456789012345678 9
123456789012345678901234567890121234567890123456789012345678
plugging the values into the factorial fraction and also into the answer 9
123456789012345678901234567890121234567890123456789012345678 9
123456789012345678901234567890121234567890123456789012345678 9
choices. If only one answer choice matches the factorial fraction, then
123456789012345678901234567890121234567890123456789012345678 9
123456789012345678901234567890121234567890123456789012345678
you have the right answer. If two answers match, then pick another set 9
1 9
123456789012345678901234567890121234567890123456789012345678
of values for the variables and repeat the process. 9
123456789012345678901234567890121234567890123456789012345678 9
123456789012345678901234567890121234567890123456789012345678 9
123456789012345678901234567890121234567890123456789012345678 9
23456789012345678901234567890121234567890123456789012345678
14. A You will probably recall that the determinant of a matrix is found by
123456789012345678901234567890121234567890123456789012345678
cross-multiplying. Since you know the end result is 26, all you need is 9
123456789012345678901234567890121234567890123456789012345678 9
123456789012345678901234567890121234567890123456789012345678
the right set-up and the proper computation: 9
123456789012345678901234567890121234567890123456789012345678 9
123456789012345678901234567890121234567890123456789012345678 9
123456789012345678901234567890121234567890123456789012345678 9
1 5
S D
123456789012345678901234567890121234567890123456789012345678
n 4
123456789012345678901234567890121234567890123456789012345678
27
5 26 9
9
9
23456789012345678901234567890121234567890123456789012345678
123456789012345678901234567890121234567890123456789012345678 9
123456789012345678901234567890121234567890123456789012345678
~27!~n! 2 ~4!~5! 5 26 9
1 9
123456789012345678901234567890121234567890123456789012345678
27n 2 20 5 26 9
123456789012345678901234567890121234567890123456789012345678
23456789012345678901234567890121234567890123456789012345678
9
9
123456789012345678901234567890121234567890123456789012345678
123456789012345678901234567890121234567890123456789012345678
27n 2 20 1 20 5 26 1 20 9
1 9
123456789012345678901234567890121234567890123456789012345678
27n 514 9
123456789012345678901234567890121234567890123456789012345678 9
23456789012345678901234567890121234567890123456789012345678
123456789012345678901234567890121234567890123456789012345678
27n 14 9
123456789012345678901234567890121234567890123456789012345678
5 9
1 27 27 9
123456789012345678901234567890121234567890123456789012345678 9
123456789012345678901234567890121234567890123456789012345678 9
123456789012345678901234567890121234567890123456789012345678
n 5 22 9
123456789012345678901234567890121234567890123456789012345678 9
123456789012345678901234567890121234567890123456789012345678
15. E First note that the question asks which statements must be true. Some 9
123456789012345678901234567890121234567890123456789012345678 9
123456789012345678901234567890121234567890123456789012345678
statements could be true under the right conditions, but if they are not 9
123456789012345678901234567890121234567890123456789012345678 9
23456789012345678901234567890121234567890123456789012345678
9
123456789012345678901234567890121234567890123456789012345678
always true, they are not going to be the right answer for this problem.
123456789012345678901234567890121234567890123456789012345678 9 9
123456789012345678901234567890121234567890123456789012345678 9
1 Now, from p 1 q 5 2q 1 6, you can determine: p 5 q 1 6 by
23456789012345678901234567890121234567890123456789012345678
123456789012345678901234567890121234567890123456789012345678 9
123456789012345678901234567890121234567890123456789012345678
subtracting a q from both sides. If p is odd then q is odd, and if p is even 9
123456789012345678901234567890121234567890123456789012345678 9
123456789012345678901234567890121234567890123456789012345678 9
then q is even (since an odd plus an even is odd and an even plus an even
9
1
123456789012345678901234567890121234567890123456789012345678
is even). But neither of them has to be even or odd. Thus I and II are not 9
123456789012345678901234567890121234567890123456789012345678 9
123456789012345678901234567890121234567890123456789012345678 9
123456789012345678901234567890121234567890123456789012345678 9
123456789012345678901234567890121234567890123456789012345678 9
123456789012345678901234567890121234567890123456789012345678 9
123456789012345678901234567890121234567890123456789012345678 9
123456789012345678901234567890121234567890123456789012345678 9
123456789012345678901234567890121234567890123456789012345678 9
1234567890123456789012345678901212345678901234567890123456789
56 Copyright © 2005 Thomson Peterson’s, a part of The Thomson Corporaton
SAT is a registered trademark of the College Entrance Examination Board, which
was not involved in the production of and does not endorse this product.
1234567890123456789012345678901212345678901234567890123456789
123456789012345678901234567890121234567890123456789012345678 9
123456789012345678901234567890121234567890123456789012345678 9
123456789012345678901234567890121234567890123456789012345678 9
123456789012345678901234567890121234567890123456789012345678 9
123456789012345678901234567890121234567890123456789012345678
necessarily true. The same is true for III, since p and q could both be 9
123456789012345678901234567890121234567890123456789012345678 9
123456789012345678901234567890121234567890123456789012345678
odd, which would make their product
23456789012345678901234567890121234567890123456789012345678
odd. Choice (E) is the answer, 9
9
1
123456789012345678901234567890121234567890123456789012345678
since it’s the only choice left standing. 9
123456789012345678901234567890121234567890123456789012345678 9
123456789012345678901234567890121234567890123456789012345678 9
123456789012345678901234567890121234567890123456789012345678
One last point: this question should have
23456789012345678901234567890121234567890123456789012345678
been answered on the second 9
9
1 pass. Once you saw the Roman numerals, you should have realized that
123456789012345678901234567890121234567890123456789012345678 9
123456789012345678901234567890121234567890123456789012345678
it would take some time to answer, and that waiting until the second 9
123456789012345678901234567890121234567890123456789012345678 9
123456789012345678901234567890121234567890123456789012345678
pass would allow you more time to get to questions that might take
23456789012345678901234567890121234567890123456789012345678
9
9
1
123456789012345678901234567890121234567890123456789012345678
less time. 9
123456789012345678901234567890121234567890123456789012345678 9
123456789012345678901234567890121234567890123456789012345678 9
123456789012345678901234567890121234567890123456789012345678
16. B Look at the graph and you’ll see that
23456789012345678901234567890121234567890123456789012345678
there are three distinct regions in 9
9
1
123456789012345678901234567890121234567890123456789012345678
which x is greater than zero: 9
123456789012345678901234567890121234567890123456789012345678 9
123456789012345678901234567890121234567890123456789012345678 9
123456789012345678901234567890121234567890123456789012345678
(1) x to the left of negative three 9
23456789012345678901234567890121234567890123456789012345678
123456789012345678901234567890121234567890123456789012345678 9
123456789012345678901234567890121234567890123456789012345678
(2) x between negative one and one 9
123456789012345678901234567890121234567890123456789012345678 9
123456789012345678901234567890121234567890123456789012345678 99
(3) x to the right of three
123456789012345678901234567890121234567890123456789012345678 9
123456789012345678901234567890121234567890123456789012345678
Once your eyes give you that information, it’s up to your brain to
9
1
123456789012345678901234567890121234567890123456789012345678
decipher the wilderness of greater than/less than signs and find the 9
123456789012345678901234567890121234567890123456789012345678 9
123456789012345678901234567890121234567890123456789012345678
answer that describes these three regions correctly. Choice (B) does this. 9
123456789012345678901234567890121234567890123456789012345678 99
23456789012345678901234567890121234567890123456789012345678
123456789012345678901234567890121234567890123456789012345678
123456789012345678901234567890121234567890123456789012345678
17. E Since the question asks which number must be in Box E, the conditions 9
123456789012345678901234567890121234567890123456789012345678 9
123456789012345678901234567890121234567890123456789012345678 99
must only allow for one number to be in Box E. There is only one prime
123456789012345678901234567890121234567890123456789012345678
123456789012345678901234567890121234567890123456789012345678
number, 11, and so it goes in Box B. 9, 12, and 15 are evenly divisible by 9
123456789012345678901234567890121234567890123456789012345678 9
123456789012345678901234567890121234567890123456789012345678 99
three, and so they go in A, C, and D. You don’t know what order, they
1
123456789012345678901234567890121234567890123456789012345678
are in, but it doesn’t matter. That leaves 16 for Box E, which means 9
123456789012345678901234567890121234567890123456789012345678 9
123456789012345678901234567890121234567890123456789012345678 9
choice (E) is the answer.
123456789012345678901234567890121234567890123456789012345678 9
123456789012345678901234567890121234567890123456789012345678 9
123456789012345678901234567890121234567890123456789012345678
18. D Since B, D, E, and H are all midpoints on their respective lines, the 9
123456789012345678901234567890121234567890123456789012345678 9
123456789012345678901234567890121234567890123456789012345678 9
rectangle is divided into four equal rectangles. The diagonal CG divides
123456789012345678901234567890121234567890123456789012345678 9
123456789012345678901234567890121234567890123456789012345678
the two smaller rectangles it traverses into halves. One-half of each of 9
23456789012345678901234567890121234567890123456789012345678
123456789012345678901234567890121234567890123456789012345678 9
123456789012345678901234567890121234567890123456789012345678
these two rectangles is shaded, so a total of one of the smaller rectangles 9
1 is shaded. Since the smaller rectangles are equal in size and there are 9
123456789012345678901234567890121234567890123456789012345678 9
123456789012345678901234567890121234567890123456789012345678
four of them, one-fourth of the larger rectangle is shaded or 25%, 9
123456789012345678901234567890121234567890123456789012345678 9
123456789012345678901234567890121234567890123456789012345678
choice (D). 9
123456789012345678901234567890121234567890123456789012345678 9
123456789012345678901234567890121234567890123456789012345678 9
123456789012345678901234567890121234567890123456789012345678
19. D Since n is the same thing as n1, you are looking to raise n n so that the 2p
9
123456789012345678901234567890121234567890123456789012345678 99
23456789012345678901234567890121234567890123456789012345678
123456789012345678901234567890121234567890123456789012345678 9
123456789012345678901234567890121234567890123456789012345678
resulting exponent is 1. Remember that when you raise a number to an
9
123456789012345678901234567890121234567890123456789012345678 9
1 2p
23456789012345678901234567890121234567890123456789012345678
123456789012345678901234567890121234567890123456789012345678
exponent, you multiply the exponents. What multiplied with yields 9
123456789012345678901234567890121234567890123456789012345678 n 9
123456789012345678901234567890121234567890123456789012345678
2n 9
123456789012345678901234567890121234567890123456789012345678
one? The answer is its reciprocal, . 9
1 p 9
23456789012345678901234567890121234567890123456789012345678
123456789012345678901234567890121234567890123456789012345678 9
1 2n 9
123456789012345678901234567890121234567890123456789012345678
~ 2p
S 2p
DS 2n
D
n n ! 5 n n p 5 n . So (D) is the answer.
9
123456789012345678901234567890121234567890123456789012345678
p 1
9
123456789012345678901234567890121234567890123456789012345678
23456789012345678901234567890121234567890123456789012345678
9
9
1
123456789012345678901234567890121234567890123456789012345678 9
123456789012345678901234567890121234567890123456789012345678 9
123456789012345678901234567890121234567890123456789012345678 9
1234567890123456789012345678901212345678901234567890123456789
Copyright © 2005 Thomson Peterson’s, a part of The Thomson Corporaton 57
SAT is a registered trademark of the College Entrance Examination Board, which
was not involved in the production of and does not endorse this product.
1234567890123456789012345678901212345678901234567890123456789
123456789012345678901234567890121234567890123456789012345678 9
123456789012345678901234567890121234567890123456789012345678 9
123456789012345678901234567890121234567890123456789012345678 9
123456789012345678901234567890121234567890123456789012345678 9
123456789012345678901234567890121234567890123456789012345678
20. A A straightforward way to start this problem is simply to start with the 9
123456789012345678901234567890121234567890123456789012345678 9
123456789012345678901234567890121234567890123456789012345678
first prime number. 2 and 3 cannot be
23456789012345678901234567890121234567890123456789012345678
n, but 5 could since 3, 5, and 7 9
9
1
123456789012345678901234567890121234567890123456789012345678
could form a prime triplet. Continuing on with the prime numbers to 9
123456789012345678901234567890121234567890123456789012345678 9
123456789012345678901234567890121234567890123456789012345678
fifty, there is not another prime number whose closest primes are two 9
123456789012345678901234567890121234567890123456789012345678
23456789012345678901234567890121234567890123456789012345678
9
9
1 more and two less than itself. Thus the answer is (A).
123456789012345678901234567890121234567890123456789012345678 9
123456789012345678901234567890121234567890123456789012345678
21. B If a geometry figure is described but not drawn, it’s always a good idea 9
123456789012345678901234567890121234567890123456789012345678 9
123456789012345678901234567890121234567890123456789012345678
to sketch the figure yourself. Draw a circle and then one tangent line.
23456789012345678901234567890121234567890123456789012345678
9
9
1
123456789012345678901234567890121234567890123456789012345678
Next draw a second tangent line that is perpendicular to the first. Now 9
123456789012345678901234567890121234567890123456789012345678 9
123456789012345678901234567890121234567890123456789012345678
draw a third tangent that also is perpendicular to the first. Your sketch 9
123456789012345678901234567890121234567890123456789012345678
23456789012345678901234567890121234567890123456789012345678
9
9
1 can only look like this:
123456789012345678901234567890121234567890123456789012345678 9
123456789012345678901234567890121234567890123456789012345678 9
123456789012345678901234567890121234567890123456789012345678 9
123456789012345678901234567890121234567890123456789012345678 9
123456789012345678901234567890121234567890123456789012345678 9
23456789012345678901234567890121234567890123456789012345678
123456789012345678901234567890121234567890123456789012345678 9
123456789012345678901234567890121234567890123456789012345678 9
123456789012345678901234567890121234567890123456789012345678 9
123456789012345678901234567890121234567890123456789012345678 9
123456789012345678901234567890121234567890123456789012345678 9 9
1
123456789012345678901234567890121234567890123456789012345678 9
123456789012345678901234567890121234567890123456789012345678 9
123456789012345678901234567890121234567890123456789012345678 9
123456789012345678901234567890121234567890123456789012345678 9
23456789012345678901234567890121234567890123456789012345678
123456789012345678901234567890121234567890123456789012345678 9
123456789012345678901234567890121234567890123456789012345678 9
123456789012345678901234567890121234567890123456789012345678 9
123456789012345678901234567890121234567890123456789012345678 9
123456789012345678901234567890121234567890123456789012345678 9
123456789012345678901234567890121234567890123456789012345678 9
123456789012345678901234567890121234567890123456789012345678 9
123456789012345678901234567890121234567890123456789012345678 9 9
1
123456789012345678901234567890121234567890123456789012345678 9
123456789012345678901234567890121234567890123456789012345678 9
123456789012345678901234567890121234567890123456789012345678 9
123456789012345678901234567890121234567890123456789012345678
Your lines could be rotated around, but the interrelationships of the 9
123456789012345678901234567890121234567890123456789012345678 9
123456789012345678901234567890121234567890123456789012345678 9
23456789012345678901234567890121234567890123456789012345678
tangents must be the same. The only place to draw a fourth tangent line
123456789012345678901234567890121234567890123456789012345678 9
1 that will be perpendicular to two other tangent lines is at the bottom of 9
123456789012345678901234567890121234567890123456789012345678 9
123456789012345678901234567890121234567890123456789012345678
the circle. 9
123456789012345678901234567890121234567890123456789012345678 9
23456789012345678901234567890121234567890123456789012345678
123456789012345678901234567890121234567890123456789012345678 9
1 Does this figure look familiar? It should, since it has the same 9
123456789012345678901234567890121234567890123456789012345678 9
123456789012345678901234567890121234567890123456789012345678 9
appearance as the first geometry problem in this section, question 2.
23456789012345678901234567890121234567890123456789012345678
123456789012345678901234567890121234567890123456789012345678 9
123456789012345678901234567890121234567890123456789012345678
You have drawn a circle inscribed in a square. The fact that it is a 9
123456789012345678901234567890121234567890123456789012345678 9
123456789012345678901234567890121234567890123456789012345678 9
square could clue you in to the fact that there are two sets of parallel
9
1
123456789012345678901234567890121234567890123456789012345678
lines. You might also recall from your geometry class that two lines that 9
123456789012345678901234567890121234567890123456789012345678
are perpendicular to the same line are parallel to each other. This means 9
123456789012345678901234567890121234567890123456789012345678 9
123456789012345678901234567890121234567890123456789012345678
that both sets of tangent lines across the circle are parallel. Either way, 9
123456789012345678901234567890121234567890123456789012345678 9
23456789012345678901234567890121234567890123456789012345678
123456789012345678901234567890121234567890123456789012345678
the answer is (B). 9
123456789012345678901234567890121234567890123456789012345678 9
123456789012345678901234567890121234567890123456789012345678 9
123456789012345678901234567890121234567890123456789012345678
Choice (D) is close, but the circle is inscribed in the square, not the other 9
1 9
123456789012345678901234567890121234567890123456789012345678 9
23456789012345678901234567890121234567890123456789012345678
way around.
9
1
123456789012345678901234567890121234567890123456789012345678 9
123456789012345678901234567890121234567890123456789012345678 9
123456789012345678901234567890121234567890123456789012345678 9
123456789012345678901234567890121234567890123456789012345678 9
123456789012345678901234567890121234567890123456789012345678 9
123456789012345678901234567890121234567890123456789012345678 9
123456789012345678901234567890121234567890123456789012345678 9
1234567890123456789012345678901212345678901234567890123456789
58 Copyright © 2005 Thomson Peterson’s, a part of The Thomson Corporaton
SAT is a registered trademark of the College Entrance Examination Board, which
was not involved in the production of and does not endorse this product.
1234567890123456789012345678901212345678901234567890123456789
123456789012345678901234567890121234567890123456789012345678 9
123456789012345678901234567890121234567890123456789012345678 9
123456789012345678901234567890121234567890123456789012345678 9
123456789012345678901234567890121234567890123456789012345678 9
123456789012345678901234567890121234567890123456789012345678
The fact that questions 2 and 21 employ the same figure shows you 9
123456789012345678901234567890121234567890123456789012345678 9
123456789012345678901234567890121234567890123456789012345678
how two questions can use the same figure
23456789012345678901234567890121234567890123456789012345678
but vary greatly in terms of 9
9
1
123456789012345678901234567890121234567890123456789012345678
difficulty. A point to ponder is, “How did the test-takers make the last 9
123456789012345678901234567890121234567890123456789012345678 9
123456789012345678901234567890121234567890123456789012345678
question harder?” First and foremost, no figure was included. This 9
123456789012345678901234567890121234567890123456789012345678
23456789012345678901234567890121234567890123456789012345678
9
9
1 should spur you to draw any figure that is not given to you, and make a
123456789012345678901234567890121234567890123456789012345678 9
123456789012345678901234567890121234567890123456789012345678
hard problem a lot easier to visualize and answer. 9
123456789012345678901234567890121234567890123456789012345678 9
123456789012345678901234567890121234567890123456789012345678
23456789012345678901234567890121234567890123456789012345678
9
9
1
123456789012345678901234567890121234567890123456789012345678
Section 3 9
123456789012345678901234567890121234567890123456789012345678 9
123456789012345678901234567890121234567890123456789012345678
1. E You should always take a close look at underlined pronouns in 9
123456789012345678901234567890121234567890123456789012345678
23456789012345678901234567890121234567890123456789012345678
9
9
1 identifying sentence error questions. “Registered” in this sentence is an
123456789012345678901234567890121234567890123456789012345678 9
123456789012345678901234567890121234567890123456789012345678
adjective, and “who” is a pronoun that represents “voters.” Because it 9
123456789012345678901234567890121234567890123456789012345678 9
123456789012345678901234567890121234567890123456789012345678
is the subject of the verb, “who,” rather than “whom,” is correct. In 9
23456789012345678901234567890121234567890123456789012345678
123456789012345678901234567890121234567890123456789012345678 9
123456789012345678901234567890121234567890123456789012345678
addition, there are no other errors in this sentence. So, the answer is (E). 9
123456789012345678901234567890121234567890123456789012345678 99
123456789012345678901234567890121234567890123456789012345678
123456789012345678901234567890121234567890123456789012345678
2. C This sentence lacks parallel structure. “Debating” in the first part 9
123456789012345678901234567890121234567890123456789012345678 9
1 should be matched by “setting” in the second part. The answer is 9
123456789012345678901234567890121234567890123456789012345678 9
123456789012345678901234567890121234567890123456789012345678
therefore (C). 9
123456789012345678901234567890121234567890123456789012345678 9
23456789012345678901234567890121234567890123456789012345678
123456789012345678901234567890121234567890123456789012345678 9
123456789012345678901234567890121234567890123456789012345678
3. B There’s an error in diction here. “Affects” is a verb; “effects” is the 9
123456789012345678901234567890121234567890123456789012345678
noun form. “Side effects” is the desired phrase. Choice (B) is the 9
123456789012345678901234567890121234567890123456789012345678 99
123456789012345678901234567890121234567890123456789012345678
answer.
123456789012345678901234567890121234567890123456789012345678 9
123456789012345678901234567890121234567890123456789012345678 9
123456789012345678901234567890121234567890123456789012345678
4. B The error here is difficult to detect. The phrase over the last decade 9
123456789012345678901234567890121234567890123456789012345678 9
1 indicates that the growth is still continuing. This is also indicated in the 9
23456789012345678901234567890121234567890123456789012345678
123456789012345678901234567890121234567890123456789012345678 9
123456789012345678901234567890121234567890123456789012345678
present tense form of employs (D). Thus the first verb should be has 9
1 9
123456789012345678901234567890121234567890123456789012345678
grown instead of had grown. Choice (B) is the answer. 9
123456789012345678901234567890121234567890123456789012345678
23456789012345678901234567890121234567890123456789012345678
9
9
123456789012345678901234567890121234567890123456789012345678
123456789012345678901234567890121234567890123456789012345678
5. C Here is another case of missing parallel structure. This is a common 9
1 9
123456789012345678901234567890121234567890123456789012345678
mistake in this section. Reading widely and making herself need to be 9
123456789012345678901234567890121234567890123456789012345678 9
123456789012345678901234567890121234567890123456789012345678 99
23456789012345678901234567890121234567890123456789012345678
matched by another gerund in part C, such as conducting sophisticated
123456789012345678901234567890121234567890123456789012345678 9
1 research. The answer is (C).
123456789012345678901234567890121234567890123456789012345678 9
123456789012345678901234567890121234567890123456789012345678
6. E Using “one” might sound awkward to one, but it is not incorrect. Nor 9
123456789012345678901234567890121234567890123456789012345678 9
123456789012345678901234567890121234567890123456789012345678
are there any other errors. The answer is (E). 9
123456789012345678901234567890121234567890123456789012345678 9
123456789012345678901234567890121234567890123456789012345678 9
123456789012345678901234567890121234567890123456789012345678 9
7. D The tenses of the verbs in this sentence are not consistent; this is another
23456789012345678901234567890121234567890123456789012345678
123456789012345678901234567890121234567890123456789012345678 9
123456789012345678901234567890121234567890123456789012345678
favorite error in this type of question. Has undergone should be 9
123456789012345678901234567890121234567890123456789012345678 9
123456789012345678901234567890121234567890123456789012345678 99
matched with has exploded. The answer is (D).
1
123456789012345678901234567890121234567890123456789012345678 9
123456789012345678901234567890121234567890123456789012345678
8. D In comparing two things (films in this case), one should use the 9
123456789012345678901234567890121234567890123456789012345678 9
123456789012345678901234567890121234567890123456789012345678
comparative case of an adjective and not the superlative. Best, choice 9
123456789012345678901234567890121234567890123456789012345678 9
123456789012345678901234567890121234567890123456789012345678 99
23456789012345678901234567890121234567890123456789012345678
(D), is superlative and not comparative (better would be correct), and
1
123456789012345678901234567890121234567890123456789012345678
so it is incorrect. 9
123456789012345678901234567890121234567890123456789012345678 9
123456789012345678901234567890121234567890123456789012345678 9
123456789012345678901234567890121234567890123456789012345678 9
123456789012345678901234567890121234567890123456789012345678 9
123456789012345678901234567890121234567890123456789012345678 9
123456789012345678901234567890121234567890123456789012345678 9
1234567890123456789012345678901212345678901234567890123456789
Copyright © 2005 Thomson Peterson’s, a part of The Thomson Corporaton 59
SAT is a registered trademark of the College Entrance Examination Board, which
was not involved in the production of and does not endorse this product.
1234567890123456789012345678901212345678901234567890123456789
123456789012345678901234567890121234567890123456789012345678 9
123456789012345678901234567890121234567890123456789012345678 9
123456789012345678901234567890121234567890123456789012345678 9
123456789012345678901234567890121234567890123456789012345678 9
123456789012345678901234567890121234567890123456789012345678
9. C You agree with a noun, and that a phrase. (A) is idiomatically correct. 9
123456789012345678901234567890121234567890123456789012345678 9
123456789012345678901234567890121234567890123456789012345678
There is no error in (B). You may
23456789012345678901234567890121234567890123456789012345678
use reasonable for with the 9
9
1
123456789012345678901234567890121234567890123456789012345678
construction here (subject 1 infinitive). No error there. To not 9
123456789012345678901234567890121234567890123456789012345678 9
123456789012345678901234567890121234567890123456789012345678
acquiesce is acceptable for spoken speech, but in written language it 9
123456789012345678901234567890121234567890123456789012345678
23456789012345678901234567890121234567890123456789012345678
9
9
1 should be not to acquiesce. There’s the rub. The answer is (C).
123456789012345678901234567890121234567890123456789012345678 9
123456789012345678901234567890121234567890123456789012345678
10. C A country (Canada) requires the pronoun it, not they. The problem is 9
123456789012345678901234567890121234567890123456789012345678 9
123456789012345678901234567890121234567890123456789012345678
therefore in choice (C).
23456789012345678901234567890121234567890123456789012345678
9
9
1
123456789012345678901234567890121234567890123456789012345678 9
123456789012345678901234567890121234567890123456789012345678
11. E This sentence is unnecessarily wordy. Having published her first novel, 9
123456789012345678901234567890121234567890123456789012345678 9
123456789012345678901234567890121234567890123456789012345678
the author is a lot snappier and more
23456789012345678901234567890121234567890123456789012345678
direct. The answer is (E). 9
9
1
123456789012345678901234567890121234567890123456789012345678 9
123456789012345678901234567890121234567890123456789012345678
12. D Here, despite is not used in a grammatically correct way. Despite the 9
123456789012345678901234567890121234567890123456789012345678 9
123456789012345678901234567890121234567890123456789012345678
fact that is grammatically correct, but it is unnecessarily wordy and it is 9
23456789012345678901234567890121234567890123456789012345678
123456789012345678901234567890121234567890123456789012345678 9
123456789012345678901234567890121234567890123456789012345678
not appropriately joined with a semicolon. How about just but? That’s 9
123456789012345678901234567890121234567890123456789012345678 9
123456789012345678901234567890121234567890123456789012345678 9
the answer (D).
123456789012345678901234567890121234567890123456789012345678 9
123456789012345678901234567890121234567890123456789012345678 9
1 13. C This sentence might have sounded okay to you, but it has a misplaced 9
123456789012345678901234567890121234567890123456789012345678
modifier. These are very popular mistakes in the sentence correction 9
123456789012345678901234567890121234567890123456789012345678 9
123456789012345678901234567890121234567890123456789012345678
questions. The phrase after he had worked on his serve for a few days 9
123456789012345678901234567890121234567890123456789012345678 9
23456789012345678901234567890121234567890123456789012345678
123456789012345678901234567890121234567890123456789012345678
modifies the noun player, so player must be right next to it. Only (B) 9
123456789012345678901234567890121234567890123456789012345678 9
123456789012345678901234567890121234567890123456789012345678
and (C) accomplish this. Choice (B), however, changes the sense of the 9
123456789012345678901234567890121234567890123456789012345678 9
123456789012345678901234567890121234567890123456789012345678 9
original sentence. The answer is (C).
123456789012345678901234567890121234567890123456789012345678 9
123456789012345678901234567890121234567890123456789012345678 9
123456789012345678901234567890121234567890123456789012345678
14. C In sentence construction parallel parts of a sentence should have 9
1 9
123456789012345678901234567890121234567890123456789012345678 9
23456789012345678901234567890121234567890123456789012345678
parallel forms. In this sentence the verbs portray and reveal are parallel,
123456789012345678901234567890121234567890123456789012345678 9
1 but in the original sentence construction they are not in parallel form. 9
123456789012345678901234567890121234567890123456789012345678 9
123456789012345678901234567890121234567890123456789012345678
Choice (C) corrects this mistake. 9
123456789012345678901234567890121234567890123456789012345678 9
23456789012345678901234567890121234567890123456789012345678
123456789012345678901234567890121234567890123456789012345678 9
1 15. B This is another favorite of the test writers: Any time you have “not 9
123456789012345678901234567890121234567890123456789012345678
only” you need to signal the contrasting phrase with “but also.” The 9
123456789012345678901234567890121234567890123456789012345678 9
23456789012345678901234567890121234567890123456789012345678
123456789012345678901234567890121234567890123456789012345678
answer here is therefore (B). 9
123456789012345678901234567890121234567890123456789012345678 9
1 16. A There is no error here. The answer is (A). 9
123456789012345678901234567890121234567890123456789012345678 9
123456789012345678901234567890121234567890123456789012345678 9
123456789012345678901234567890121234567890123456789012345678 9
123456789012345678901234567890121234567890123456789012345678
17. A There is not a grammatical error in the underlined portion, and none of 9
123456789012345678901234567890121234567890123456789012345678 9
123456789012345678901234567890121234567890123456789012345678
the alternatives improve upon the original. Because you do not repeat 9
123456789012345678901234567890121234567890123456789012345678 9
123456789012345678901234567890121234567890123456789012345678 9
23456789012345678901234567890121234567890123456789012345678
the subject, there is no comma needed before the underlined portion.
9
123456789012345678901234567890121234567890123456789012345678 9
123456789012345678901234567890121234567890123456789012345678
Were you to choose answer (C), you would need to add a comma.
9
123456789012345678901234567890121234567890123456789012345678 9
1 Choice (A) is the answer.
123456789012345678901234567890121234567890123456789012345678 9
23456789012345678901234567890121234567890123456789012345678
9
123456789012345678901234567890121234567890123456789012345678
18. D Here the error is logical rather than grammatical per se. The first part of
9
123456789012345678901234567890121234567890123456789012345678 9
123456789012345678901234567890121234567890123456789012345678
the sentence emphasizes sympathy, but the second part focuses on
9
1
23456789012345678901234567890121234567890123456789012345678
123456789012345678901234567890121234567890123456789012345678
negative human qualities. You need a contrasting coordinator. That 9
1 9
123456789012345678901234567890121234567890123456789012345678
leaves (D) and (E). (D) is the better choice. 9
123456789012345678901234567890121234567890123456789012345678 9
123456789012345678901234567890121234567890123456789012345678 9
123456789012345678901234567890121234567890123456789012345678 9
123456789012345678901234567890121234567890123456789012345678 9
123456789012345678901234567890121234567890123456789012345678 9
123456789012345678901234567890121234567890123456789012345678 9
1234567890123456789012345678901212345678901234567890123456789
60 Copyright © 2005 Thomson Peterson’s, a part of The Thomson Corporaton
SAT is a registered trademark of the College Entrance Examination Board, which
was not involved in the production of and does not endorse this product.
1234567890123456789012345678901212345678901234567890123456789
123456789012345678901234567890121234567890123456789012345678 9
123456789012345678901234567890121234567890123456789012345678 9
123456789012345678901234567890121234567890123456789012345678 9
123456789012345678901234567890121234567890123456789012345678 9
123456789012345678901234567890121234567890123456789012345678
19. E It was not because Pancho Villa’s raid was part of the tumult of the 9
123456789012345678901234567890121234567890123456789012345678 9
123456789012345678901234567890121234567890123456789012345678
Mexican revolution that it prompted
23456789012345678901234567890121234567890123456789012345678
a retaliatory expedition. There- 9
9
1
123456789012345678901234567890121234567890123456789012345678
fore is misused. Simply delete it and you have a correct sentence. The 9
123456789012345678901234567890121234567890123456789012345678 9
123456789012345678901234567890121234567890123456789012345678
answer is (E). 9
123456789012345678901234567890121234567890123456789012345678 9
1 20. A There is nothing grammatically wrong with the underlined portion, and 9
23456789012345678901234567890121234567890123456789012345678
123456789012345678901234567890121234567890123456789012345678 9
123456789012345678901234567890121234567890123456789012345678
none of the other choices improve on the wording of the sentence. (A) is 9
123456789012345678901234567890121234567890123456789012345678 9
123456789012345678901234567890121234567890123456789012345678
the best choice.
23456789012345678901234567890121234567890123456789012345678
9
9
1
123456789012345678901234567890121234567890123456789012345678 9
123456789012345678901234567890121234567890123456789012345678
21. E Right away, you should notice two sentences in passive voice and think 9
123456789012345678901234567890121234567890123456789012345678 9
123456789012345678901234567890121234567890123456789012345678
about making them active. Only (D)
23456789012345678901234567890121234567890123456789012345678
and (E) do that. (D) includes an 9
9
1
123456789012345678901234567890121234567890123456789012345678
imprecise 2ing verb. The test writers love to throw these around. 9
123456789012345678901234567890121234567890123456789012345678 9
123456789012345678901234567890121234567890123456789012345678
Sometimes they are the right answer, but you should always scrutinize 9
123456789012345678901234567890121234567890123456789012345678 9
123456789012345678901234567890121234567890123456789012345678 99
23456789012345678901234567890121234567890123456789012345678
them. Here (E) is a much sharper sentence.
123456789012345678901234567890121234567890123456789012345678 9
123456789012345678901234567890121234567890123456789012345678
123456789012345678901234567890121234567890123456789012345678
22. A Doesn’t sentence 2 seem too specific? It is really an explanation for why 9
123456789012345678901234567890121234567890123456789012345678 9
123456789012345678901234567890121234567890123456789012345678 99
the men couldn’t use their limbs. It should therefore follow sentence 4.
1
123456789012345678901234567890121234567890123456789012345678
The answer is (A). 9
123456789012345678901234567890121234567890123456789012345678 9
123456789012345678901234567890121234567890123456789012345678
23. D What is the logical connection between the two sentences? The first 9
123456789012345678901234567890121234567890123456789012345678 99
23456789012345678901234567890121234567890123456789012345678
123456789012345678901234567890121234567890123456789012345678
deals with the highest trip. The second deals with the largest balloon.
123456789012345678901234567890121234567890123456789012345678 9
123456789012345678901234567890121234567890123456789012345678
Now has nothing to do with that. Neither does in the nineteenth 9
123456789012345678901234567890121234567890123456789012345678 9
123456789012345678901234567890121234567890123456789012345678 99
century. You don’t want to begin with either of these. Moreover
123456789012345678901234567890121234567890123456789012345678
123456789012345678901234567890121234567890123456789012345678
represents paragraph 2 as an extension of the ideas in paragraph 1, 9
123456789012345678901234567890121234567890123456789012345678 9
1 which is also inaccurate. The easiest thing to do is simply get rid of 9
23456789012345678901234567890121234567890123456789012345678
123456789012345678901234567890121234567890123456789012345678 9
123456789012345678901234567890121234567890123456789012345678
now, (D). 9
1 9
123456789012345678901234567890121234567890123456789012345678 9
123456789012345678901234567890121234567890123456789012345678
24. C Sentence combination is huge in this
23456789012345678901234567890121234567890123456789012345678
section. This example is trickier 9
9
123456789012345678901234567890121234567890123456789012345678
123456789012345678901234567890121234567890123456789012345678
than most. It already has an and in the first sentence, so if you use and 9
1 9
123456789012345678901234567890121234567890123456789012345678
again your sentence will start to sound like a run-on. Here, too, the 9
123456789012345678901234567890121234567890123456789012345678 9
123456789012345678901234567890121234567890123456789012345678 99
23456789012345678901234567890121234567890123456789012345678
2ing verb is imprecise. Which should really go very close to the noun it
123456789012345678901234567890121234567890123456789012345678
modifies, so eliminate (A). But implies a contrast, when all of these 9
1
123456789012345678901234567890121234567890123456789012345678
ideas are similar, so you can eliminate (E). Go with the semicolon (C). 9
123456789012345678901234567890121234567890123456789012345678 9
23456789012345678901234567890121234567890123456789012345678
123456789012345678901234567890121234567890123456789012345678 9
123456789012345678901234567890121234567890123456789012345678
25. D All of whom returned safely is not a complete sentence. It modifies 9
123456789012345678901234567890121234567890123456789012345678 9
123456789012345678901234567890121234567890123456789012345678
“passengers” in the preceding sentence. Only (D) addresses that major 9
1 9
123456789012345678901234567890121234567890123456789012345678 99
23456789012345678901234567890121234567890123456789012345678
problem!
123456789012345678901234567890121234567890123456789012345678 9
123456789012345678901234567890121234567890123456789012345678 9
123456789012345678901234567890121234567890123456789012345678
26. E What’s missing in this sentence is where. As it stands now, it implies
9
1
123456789012345678901234567890121234567890123456789012345678 99
23456789012345678901234567890121234567890123456789012345678
that literacy program is the direct object of working. Choices (D) and
123456789012345678901234567890121234567890123456789012345678 9
123456789012345678901234567890121234567890123456789012345678
(E) correct the error, but (D) makes undesirable changes to the verb
9
123456789012345678901234567890121234567890123456789012345678 9
1 tenses. (E) is the best answer.
123456789012345678901234567890121234567890123456789012345678 99
23456789012345678901234567890121234567890123456789012345678
1
123456789012345678901234567890121234567890123456789012345678 9
123456789012345678901234567890121234567890123456789012345678 9
123456789012345678901234567890121234567890123456789012345678 9
123456789012345678901234567890121234567890123456789012345678 9
123456789012345678901234567890121234567890123456789012345678 9
123456789012345678901234567890121234567890123456789012345678 9
123456789012345678901234567890121234567890123456789012345678 9
1234567890123456789012345678901212345678901234567890123456789
Copyright © 2005 Thomson Peterson’s, a part of The Thomson Corporaton 61
SAT is a registered trademark of the College Entrance Examination Board, which
was not involved in the production of and does not endorse this product.
1234567890123456789012345678901212345678901234567890123456789
123456789012345678901234567890121234567890123456789012345678 9
123456789012345678901234567890121234567890123456789012345678 9
123456789012345678901234567890121234567890123456789012345678 9
123456789012345678901234567890121234567890123456789012345678 9
123456789012345678901234567890121234567890123456789012345678
27. B That had automobiles should not be separated by commas because it is 9
123456789012345678901234567890121234567890123456789012345678 9
123456789012345678901234567890121234567890123456789012345678
an integral part of the category being
23456789012345678901234567890121234567890123456789012345678
described, not an added 9
9
1
123456789012345678901234567890121234567890123456789012345678
description. But it isn’t correct in written English to write people that. It 9
123456789012345678901234567890121234567890123456789012345678 9
123456789012345678901234567890121234567890123456789012345678
has to be people who (or people whom if what follows positions the 9
123456789012345678901234567890121234567890123456789012345678
23456789012345678901234567890121234567890123456789012345678
9
9
1 people as the object of a verb). The answer is (B).
123456789012345678901234567890121234567890123456789012345678 9
123456789012345678901234567890121234567890123456789012345678
28. D The sentence as it stands is a bit of a disaster. It sounds like a run-on: it 9
123456789012345678901234567890121234567890123456789012345678 9
123456789012345678901234567890121234567890123456789012345678
just goes on and on like the Energizer Bunny. So what you will want to
23456789012345678901234567890121234567890123456789012345678
9
9
1
123456789012345678901234567890121234567890123456789012345678
do is make it more direct, showcasing the important parts and 9
123456789012345678901234567890121234567890123456789012345678 9
123456789012345678901234567890121234567890123456789012345678
subordinating the descriptions that are really secondary. You also need 9
123456789012345678901234567890121234567890123456789012345678
23456789012345678901234567890121234567890123456789012345678
9
9
1 a comma after It is hard to believe. Start with the easiest thing, and
123456789012345678901234567890121234567890123456789012345678 9
123456789012345678901234567890121234567890123456789012345678
eliminate (A) and (C) off the bat. Which of (B), (D), and (E) makes the 9
123456789012345678901234567890121234567890123456789012345678 9
123456789012345678901234567890121234567890123456789012345678
sentence more direct? Definitely not (E). Choice (B) gets rid of the 9
23456789012345678901234567890121234567890123456789012345678
123456789012345678901234567890121234567890123456789012345678 9
123456789012345678901234567890121234567890123456789012345678
second comma/and combination, which could be good. But is a colon 9
123456789012345678901234567890121234567890123456789012345678 9
123456789012345678901234567890121234567890123456789012345678 9
really in order here? No. The best answer is (D).
123456789012345678901234567890121234567890123456789012345678 9
123456789012345678901234567890121234567890123456789012345678 9
1 29. A This is a little tricky because the repetition here does serve a purpose; it 9
123456789012345678901234567890121234567890123456789012345678 9
123456789012345678901234567890121234567890123456789012345678
isn’t just extra wordage that got in the author’s way. Basically, the 9
123456789012345678901234567890121234567890123456789012345678 9
123456789012345678901234567890121234567890123456789012345678 9
23456789012345678901234567890121234567890123456789012345678
sentences are a list. When you have clauses that form a list (or other
123456789012345678901234567890121234567890123456789012345678 9
123456789012345678901234567890121234567890123456789012345678
things requiring lots of words and/or punctuation), you separate them 9
123456789012345678901234567890121234567890123456789012345678
with semicolons rather than commas. (A) looks good. All of the other 9
123456789012345678901234567890121234567890123456789012345678 9
123456789012345678901234567890121234567890123456789012345678
answers, except (D), change the sense of the original ever so slightly. (D) 9
123456789012345678901234567890121234567890123456789012345678 9
123456789012345678901234567890121234567890123456789012345678
could be possible if it had and before the last clause, but (A) is still 9
123456789012345678901234567890121234567890123456789012345678 9 9
1 better.
123456789012345678901234567890121234567890123456789012345678 9
123456789012345678901234567890121234567890123456789012345678 9
123456789012345678901234567890121234567890123456789012345678
30. B While it would be possible to add a comma after convenience, it doesn’t 9
123456789012345678901234567890121234567890123456789012345678 9
123456789012345678901234567890121234567890123456789012345678
make much sense to add one after necessity.
23456789012345678901234567890121234567890123456789012345678
Changing you live to one 9
9
123456789012345678901234567890121234567890123456789012345678
123456789012345678901234567890121234567890123456789012345678
lives is possible, but not required. So is adding Desert. The comma after 9
1 9
123456789012345678901234567890121234567890123456789012345678
apparently isn’t strictly required, but it is desirable. The only absolutely 9
123456789012345678901234567890121234567890123456789012345678 9
123456789012345678901234567890121234567890123456789012345678 9
23456789012345678901234567890121234567890123456789012345678
necessary change is to replace “there” with “they are” (choice B).
123456789012345678901234567890121234567890123456789012345678 9
1 “There are” might have been more difficult to rule against (though still 9
123456789012345678901234567890121234567890123456789012345678 9
123456789012345678901234567890121234567890123456789012345678 9
incorrect), but the sentence doesn’t even say there are; it just says there.
123456789012345678901234567890121234567890123456789012345678 9
23456789012345678901234567890121234567890123456789012345678
123456789012345678901234567890121234567890123456789012345678 9
123456789012345678901234567890121234567890123456789012345678 9
123456789012345678901234567890121234567890123456789012345678 9 9
1
123456789012345678901234567890121234567890123456789012345678 9
123456789012345678901234567890121234567890123456789012345678 9
123456789012345678901234567890121234567890123456789012345678 9
123456789012345678901234567890121234567890123456789012345678 9
123456789012345678901234567890121234567890123456789012345678 9
123456789012345678901234567890121234567890123456789012345678 9
23456789012345678901234567890121234567890123456789012345678
123456789012345678901234567890121234567890123456789012345678 9
123456789012345678901234567890121234567890123456789012345678 9
123456789012345678901234567890121234567890123456789012345678 9 9
1
123456789012345678901234567890121234567890123456789012345678 9
123456789012345678901234567890121234567890123456789012345678 9
123456789012345678901234567890121234567890123456789012345678 9
123456789012345678901234567890121234567890123456789012345678 9
123456789012345678901234567890121234567890123456789012345678 9
123456789012345678901234567890121234567890123456789012345678 9
123456789012345678901234567890121234567890123456789012345678 9
123456789012345678901234567890121234567890123456789012345678 9
123456789012345678901234567890121234567890123456789012345678 9
1234567890123456789012345678901212345678901234567890123456789
62 Copyright © 2005 Thomson Peterson’s, a part of The Thomson Corporaton
SAT is a registered trademark of the College Entrance Examination Board, which
was not involved in the production of and does not endorse this product.
1234567890123456789012345678901212345678901234567890123456789
123456789012345678901234567890121234567890123456789012345678 9
123456789012345678901234567890121234567890123456789012345678 9
123456789012345678901234567890121234567890123456789012345678 9
123456789012345678901234567890121234567890123456789012345678 9
123456789012345678901234567890121234567890123456789012345678
Section 4 9
123456789012345678901234567890121234567890123456789012345678 9
123456789012345678901234567890121234567890123456789012345678
1. A What is the name of the kind of talk that is delivered at a funeral? 9
123456789012345678901234567890121234567890123456789012345678 9
123456789012345678901234567890121234567890123456789012345678
Eulogy. If you know this, the answer pops out at you. If you did not 9
123456789012345678901234567890121234567890123456789012345678 9
123456789012345678901234567890121234567890123456789012345678
know it, consider each of the choices in their turn. Epigraph is a quote 9
123456789012345678901234567890121234567890123456789012345678
23456789012345678901234567890121234567890123456789012345678
9
9
1 at the beginning of a piece of writing. Eponymy is something with the
123456789012345678901234567890121234567890123456789012345678 9
123456789012345678901234567890121234567890123456789012345678
same name as something else. Epitaph is what is written on a 9
123456789012345678901234567890121234567890123456789012345678 9
123456789012345678901234567890121234567890123456789012345678
gravestone. That leaves (A) and (B). Elegy
23456789012345678901234567890121234567890123456789012345678
is a poem written in memory. 9
9
1
123456789012345678901234567890121234567890123456789012345678
You don’t “give” a poem. That leaves (A), the correct answer. 9
123456789012345678901234567890121234567890123456789012345678 9
123456789012345678901234567890121234567890123456789012345678 9
123456789012345678901234567890121234567890123456789012345678
2. C On this dual-blank sentence, let’s do the
23456789012345678901234567890121234567890123456789012345678
first blank first since we know 9
9
1
123456789012345678901234567890121234567890123456789012345678
that the blank was an encouragement to the rest of the team. Good 9
123456789012345678901234567890121234567890123456789012345678 9
123456789012345678901234567890121234567890123456789012345678
spirits would be an encouragement to the rest of the team. You can 9
123456789012345678901234567890121234567890123456789012345678
eliminate (D) and (E). As for the second blank, what does a string of 9
123456789012345678901234567890121234567890123456789012345678 99
23456789012345678901234567890121234567890123456789012345678
123456789012345678901234567890121234567890123456789012345678
defeats do to a team? It discourages them. (A), elated, does not match
123456789012345678901234567890121234567890123456789012345678 9
123456789012345678901234567890121234567890123456789012345678
this. Nor does (B), inundated. But (C), dispirited, fits well and you’ve 9
123456789012345678901234567890121234567890123456789012345678 99
123456789012345678901234567890121234567890123456789012345678
already eliminated (D) and (E). Choice (C) is the best answer.
9
1
123456789012345678901234567890121234567890123456789012345678 9
123456789012345678901234567890121234567890123456789012345678
3. B You might not know what resorted means, but if you know it’s a 9
123456789012345678901234567890121234567890123456789012345678 9
123456789012345678901234567890121234567890123456789012345678 99
23456789012345678901234567890121234567890123456789012345678
negative word, you can make an educated guess. Which of the answer
123456789012345678901234567890121234567890123456789012345678
123456789012345678901234567890121234567890123456789012345678
choices is also a negative verb? (A), (C), and (D) are not. (E) is not a 9
123456789012345678901234567890121234567890123456789012345678 9
123456789012345678901234567890121234567890123456789012345678 99
good answer because swindling has nothing to do with campaigning.
123456789012345678901234567890121234567890123456789012345678
123456789012345678901234567890121234567890123456789012345678
Choice (B) is the best choice. 9
123456789012345678901234567890121234567890123456789012345678 99
123456789012345678901234567890121234567890123456789012345678 9
1 4. E The second half of the sentence gives more clues, so you ought to start
123456789012345678901234567890121234567890123456789012345678 9
123456789012345678901234567890121234567890123456789012345678
there. What are cats most likely to do to dogs? Avoid them, 9
123456789012345678901234567890121234567890123456789012345678
probably—which will lead you to (E), the correct answer. But for good 9
123456789012345678901234567890121234567890123456789012345678 9
123456789012345678901234567890121234567890123456789012345678
measure, let’s eliminate the other possibilities. For a cat to undermine a 9
123456789012345678901234567890121234567890123456789012345678 9
123456789012345678901234567890121234567890123456789012345678
dog isn’t logical. Being undermined is something that happens to 9
123456789012345678901234567890121234567890123456789012345678 9
123456789012345678901234567890121234567890123456789012345678
humans or projects, so you can definitely eliminate (D). One could say 9
123456789012345678901234567890121234567890123456789012345678 9
23456789012345678901234567890121234567890123456789012345678
123456789012345678901234567890121234567890123456789012345678
that a cat enticed a dog to do something, but it isn’t good usage simply 9
123456789012345678901234567890121234567890123456789012345678 9
1 to say that the cat enticed the dog. Eliminate (A). Is it likely for a cat to 9
123456789012345678901234567890121234567890123456789012345678 9
123456789012345678901234567890121234567890123456789012345678
gracefully apprehend a dog? No. Eliminate (C). The only possibilities 9
23456789012345678901234567890121234567890123456789012345678
123456789012345678901234567890121234567890123456789012345678 9
123456789012345678901234567890121234567890123456789012345678
left are (B) and (E). A cat might possibly defeat a dog in battle, but use 9
123456789012345678901234567890121234567890123456789012345678 9
123456789012345678901234567890121234567890123456789012345678 99
the other clues. Defeat and “gracefully” don’t go well together, and it
1
123456789012345678901234567890121234567890123456789012345678
doesn’t make sense for a battle to happen while the cat is creeping 9
123456789012345678901234567890121234567890123456789012345678 9
123456789012345678901234567890121234567890123456789012345678
across the lawn. Eliminate (B). The answer is (E). 9
123456789012345678901234567890121234567890123456789012345678 9
123456789012345678901234567890121234567890123456789012345678 9
123456789012345678901234567890121234567890123456789012345678 99
23456789012345678901234567890121234567890123456789012345678
5. A Attack the second blank first. The most likely adjective to describe
123456789012345678901234567890121234567890123456789012345678 9
123456789012345678901234567890121234567890123456789012345678
attention will be something like undivided or rapt. (A), which includes
9
123456789012345678901234567890121234567890123456789012345678 9
1 rapt, is the answer. (B), which includes spellbound, is also possible. But
23456789012345678901234567890121234567890123456789012345678
123456789012345678901234567890121234567890123456789012345678
pointed anecdotes doesn’t make sense, so the answer is (A). 9
1 9
123456789012345678901234567890121234567890123456789012345678 9
123456789012345678901234567890121234567890123456789012345678 9
123456789012345678901234567890121234567890123456789012345678 9
123456789012345678901234567890121234567890123456789012345678 9
123456789012345678901234567890121234567890123456789012345678 9
123456789012345678901234567890121234567890123456789012345678 9
123456789012345678901234567890121234567890123456789012345678 9
1234567890123456789012345678901212345678901234567890123456789
Copyright © 2005 Thomson Peterson’s, a part of The Thomson Corporaton 63
SAT is a registered trademark of the College Entrance Examination Board, which
was not involved in the production of and does not endorse this product.
1234567890123456789012345678901212345678901234567890123456789
123456789012345678901234567890121234567890123456789012345678 9
123456789012345678901234567890121234567890123456789012345678 9
123456789012345678901234567890121234567890123456789012345678 9
123456789012345678901234567890121234567890123456789012345678 9
123456789012345678901234567890121234567890123456789012345678
6. E You know the word is going to be negative: both bogged and 9
123456789012345678901234567890121234567890123456789012345678 9
123456789012345678901234567890121234567890123456789012345678
contentious tell you so. Eliminate
23456789012345678901234567890121234567890123456789012345678
(B). Now think that the word is 9
9
1
123456789012345678901234567890121234567890123456789012345678
basically going to mean mess. You can eliminate (A) and (C). 9
123456789012345678901234567890121234567890123456789012345678 9
123456789012345678901234567890121234567890123456789012345678
Conundrum is a confusing problem, not really a messy situation. (E) is 9
123456789012345678901234567890121234567890123456789012345678
23456789012345678901234567890121234567890123456789012345678
9
9
1 the best answer.
123456789012345678901234567890121234567890123456789012345678 9
123456789012345678901234567890121234567890123456789012345678
7. B Here the first blank seems more approachable. The reference to a clear 9
123456789012345678901234567890121234567890123456789012345678 9
123456789012345678901234567890121234567890123456789012345678
leader indicates that the outcome was known. Eliminate (A) and (E).
23456789012345678901234567890121234567890123456789012345678
9
9
1
123456789012345678901234567890121234567890123456789012345678
For the second blank, the clue is that the leader misstepped and so rest 9
123456789012345678901234567890121234567890123456789012345678 9
123456789012345678901234567890121234567890123456789012345678
of the competitors must have gotten a chance at the title, but they 9
123456789012345678901234567890121234567890123456789012345678
23456789012345678901234567890121234567890123456789012345678
9
9
1 weren’t assured a victory. Eliminate (D) and (C). The answer is (B).
123456789012345678901234567890121234567890123456789012345678 9
123456789012345678901234567890121234567890123456789012345678 9
123456789012345678901234567890121234567890123456789012345678
8. D Consider the first blank. The word though indicates that the drug was 9
123456789012345678901234567890121234567890123456789012345678 9
123456789012345678901234567890121234567890123456789012345678 9
23456789012345678901234567890121234567890123456789012345678
intended to be beneficial but ultimately was not. Do any of the answer
123456789012345678901234567890121234567890123456789012345678 9
123456789012345678901234567890121234567890123456789012345678
choices mean not beneficial? (A), (D), and (E) do. How would the 9
123456789012345678901234567890121234567890123456789012345678 9
123456789012345678901234567890121234567890123456789012345678 9
medical community respond to a bad result? Ostensibly they would
123456789012345678901234567890121234567890123456789012345678 9
1 think that a bad result was bad. That eliminates (A) and (E). This 9
123456789012345678901234567890121234567890123456789012345678 9
123456789012345678901234567890121234567890123456789012345678
leaves (D). 9
123456789012345678901234567890121234567890123456789012345678 9
23456789012345678901234567890121234567890123456789012345678
123456789012345678901234567890121234567890123456789012345678
9. B A life that only lasts 24 hours is what in comparison to a normal human 9
123456789012345678901234567890121234567890123456789012345678 9
123456789012345678901234567890121234567890123456789012345678
life? It is short. Which of the answer choices contains the notion of 9
123456789012345678901234567890121234567890123456789012345678 9
123456789012345678901234567890121234567890123456789012345678
shortness in its meaning? (B), ephemeral, does. 9
123456789012345678901234567890121234567890123456789012345678 9
123456789012345678901234567890121234567890123456789012345678 9
123456789012345678901234567890121234567890123456789012345678
10. E What is a species likely to do in an environment? It either grows in 9
123456789012345678901234567890121234567890123456789012345678 9
1 number or diminishes in number. Each of the first words, except in (B), 9
23456789012345678901234567890121234567890123456789012345678
123456789012345678901234567890121234567890123456789012345678 9
123456789012345678901234567890121234567890123456789012345678
could mean one of those things. Eliminate (B). When you discover that 9
1 9
123456789012345678901234567890121234567890123456789012345678 9
there is an abundance of food, you know that the first word will suggest
123456789012345678901234567890121234567890123456789012345678 9
123456789012345678901234567890121234567890123456789012345678
that the kangaroos increased in numbers. Eliminate (A) and (D). Now 9
123456789012345678901234567890121234567890123456789012345678 9
123456789012345678901234567890121234567890123456789012345678 9
you need the second part of the sentence. To grow in numbers, the
123456789012345678901234567890121234567890123456789012345678 9
123456789012345678901234567890121234567890123456789012345678
kangaroos will need an absence or near absence of predators. Eliminate 9
23456789012345678901234567890121234567890123456789012345678
123456789012345678901234567890121234567890123456789012345678 9
123456789012345678901234567890121234567890123456789012345678
(C). You are left with (E). 9
1 9
123456789012345678901234567890121234567890123456789012345678
11. C The best clue in this sentence is “fears.” Citizens with fears can only be 9
123456789012345678901234567890121234567890123456789012345678 9
23456789012345678901234567890121234567890123456789012345678
123456789012345678901234567890121234567890123456789012345678
concerned or alarmed. That leaves (B) and (C). It’s not particularly 9
123456789012345678901234567890121234567890123456789012345678 9
123456789012345678901234567890121234567890123456789012345678
logical to say that a speech is designed to ignore something. On the 9
123456789012345678901234567890121234567890123456789012345678 9 9
1 other hand, it is common to use assuage with fears. The best answer
123456789012345678901234567890121234567890123456789012345678 9
123456789012345678901234567890121234567890123456789012345678
is C. 9
123456789012345678901234567890121234567890123456789012345678 9
123456789012345678901234567890121234567890123456789012345678 9
123456789012345678901234567890121234567890123456789012345678 9
12. A Let’s attack the first blank. If the female is a fencing champion then she
23456789012345678901234567890121234567890123456789012345678
123456789012345678901234567890121234567890123456789012345678 9
123456789012345678901234567890121234567890123456789012345678
must be skillful with her rapier (her sword). Which of the first answer 9
123456789012345678901234567890121234567890123456789012345678 9
123456789012345678901234567890121234567890123456789012345678 9
choices matches skillful? Choices (A), (B), and (E) do. (C) is possible but
9
1
123456789012345678901234567890121234567890123456789012345678
not likely. As for the second blank, the conjunction but indicates that 9
123456789012345678901234567890121234567890123456789012345678 9
123456789012345678901234567890121234567890123456789012345678
her skillfulness in fencing is in contrast to her lack of skill in other 9
123456789012345678901234567890121234567890123456789012345678 9
123456789012345678901234567890121234567890123456789012345678
sports. Which of the remaining second
23456789012345678901234567890121234567890123456789012345678
answer choices matches with 9
9
1
123456789012345678901234567890121234567890123456789012345678
this pre-guess? Only awkward, choice (A), does. 9
123456789012345678901234567890121234567890123456789012345678 9
123456789012345678901234567890121234567890123456789012345678 9
1234567890123456789012345678901212345678901234567890123456789
64 Copyright © 2005 Thomson Peterson’s, a part of The Thomson Corporaton
SAT is a registered trademark of the College Entrance Examination Board, which
was not involved in the production of and does not endorse this product.
1234567890123456789012345678901212345678901234567890123456789
123456789012345678901234567890121234567890123456789012345678 9
123456789012345678901234567890121234567890123456789012345678 9
123456789012345678901234567890121234567890123456789012345678 9
123456789012345678901234567890121234567890123456789012345678 9
123456789012345678901234567890121234567890123456789012345678
13. B There is a contrast drawn in the sentence between receiving accolades— 9
123456789012345678901234567890121234567890123456789012345678 9
123456789012345678901234567890121234567890123456789012345678
praise, awards—and Jane Goodall’s
23456789012345678901234567890121234567890123456789012345678
initial standing in her field. She 9
9
1
123456789012345678901234567890121234567890123456789012345678
must have met with a lack of support or outright disapproval. Eliminate 9
123456789012345678901234567890121234567890123456789012345678 9
123456789012345678901234567890121234567890123456789012345678
(E) because it is illogical. Eliminate (C) because it goes with, rather than 9
123456789012345678901234567890121234567890123456789012345678
23456789012345678901234567890121234567890123456789012345678
9
9
1 against, accolades. An acolyte is someone who assists a clergyman, so
123456789012345678901234567890121234567890123456789012345678 9
123456789012345678901234567890121234567890123456789012345678
you can eliminate (A). You are left with (B) and (D). A charlatan is a 9
123456789012345678901234567890121234567890123456789012345678 9
123456789012345678901234567890121234567890123456789012345678
fake, an incompetent. If the sentence
23456789012345678901234567890121234567890123456789012345678
said, “Some people thought she 9
9
1 was a ——,” charlatan might work, but it says she actually was “a —-.”
123456789012345678901234567890121234567890123456789012345678 9
123456789012345678901234567890121234567890123456789012345678
She couldn’t have been a fake and later gotten awards. Eliminate (D). 9
123456789012345678901234567890121234567890123456789012345678 9
123456789012345678901234567890121234567890123456789012345678
You are left with (B), a maverick, an independent thinker, a dissenter,
23456789012345678901234567890121234567890123456789012345678
9
9
1
123456789012345678901234567890121234567890123456789012345678
a pioneer. 9
123456789012345678901234567890121234567890123456789012345678 9
123456789012345678901234567890121234567890123456789012345678 9
123456789012345678901234567890121234567890123456789012345678
14. D This sentence is contrasting the views of Alston and Mario (the conjunc- 9
23456789012345678901234567890121234567890123456789012345678
123456789012345678901234567890121234567890123456789012345678 9
123456789012345678901234567890121234567890123456789012345678
tion but clues you into this fact). Alston thinks that the lecture was 9
123456789012345678901234567890121234567890123456789012345678 9
123456789012345678901234567890121234567890123456789012345678 99
impressive, which probably means smart, accurate, logical. Mario’s view
123456789012345678901234567890121234567890123456789012345678
123456789012345678901234567890121234567890123456789012345678
is in contrast to this. You can eliminate (A) and (C). Recondite is not 9
1 9
123456789012345678901234567890121234567890123456789012345678
likely to be a word to describe a lecture, so eliminate (B). You are left 9
123456789012345678901234567890121234567890123456789012345678 9
123456789012345678901234567890121234567890123456789012345678
with specious or fictitious. Specious means logically false; fictitious comes 9
23456789012345678901234567890121234567890123456789012345678
123456789012345678901234567890121234567890123456789012345678 9
123456789012345678901234567890121234567890123456789012345678
from fiction, and presumably the philosopher didn’t tell a story but 9
123456789012345678901234567890121234567890123456789012345678 9
123456789012345678901234567890121234567890123456789012345678 99
rather made an argument. Choice (D) is the best answer.
123456789012345678901234567890121234567890123456789012345678 9
123456789012345678901234567890121234567890123456789012345678
15. A If you know that insinuation is a negative word, you can guess that the
123456789012345678901234567890121234567890123456789012345678 9
123456789012345678901234567890121234567890123456789012345678
first blank will describe a logical response to a negative thing. Balk is a 9
123456789012345678901234567890121234567890123456789012345678 99
1 common word in this situation, but if you don’t know that use the
123456789012345678901234567890121234567890123456789012345678 9
123456789012345678901234567890121234567890123456789012345678
process of elimination. You can eliminate (B) and probably (D) because 9
123456789012345678901234567890121234567890123456789012345678 9
123456789012345678901234567890121234567890123456789012345678
they are not negative words. Move to the next blank. If the official’s 9
123456789012345678901234567890121234567890123456789012345678 9
123456789012345678901234567890121234567890123456789012345678 99
23456789012345678901234567890121234567890123456789012345678
response is negative, it’s most logical that he is accused of having
123456789012345678901234567890121234567890123456789012345678 9
1 something to do with the economic woes. Eliminate (C). That leaves (A)
123456789012345678901234567890121234567890123456789012345678 9
123456789012345678901234567890121234567890123456789012345678
and (E) as the most likely answers. But you don’t “rile” at something; 9
23456789012345678901234567890121234567890123456789012345678
123456789012345678901234567890121234567890123456789012345678 9
123456789012345678901234567890121234567890123456789012345678
it’s not good usage. Eliminate (E) and you are left with (A). 9
1 9
123456789012345678901234567890121234567890123456789012345678 9
123456789012345678901234567890121234567890123456789012345678 9
16. D The author is speaking generally in this first paragraph. Global
23456789012345678901234567890121234567890123456789012345678
123456789012345678901234567890121234567890123456789012345678 9
123456789012345678901234567890121234567890123456789012345678
warming and species extinction are two big, general problems; he refers 9
123456789012345678901234567890121234567890123456789012345678 9
123456789012345678901234567890121234567890123456789012345678 99
to them in a positive light at “maintaining global climate and genetic
1
123456789012345678901234567890121234567890123456789012345678
resources.” “Genetic resources” refers diverse species of plants and 9
123456789012345678901234567890121234567890123456789012345678
animals, choice (D). 9
123456789012345678901234567890121234567890123456789012345678 9
123456789012345678901234567890121234567890123456789012345678 9
123456789012345678901234567890121234567890123456789012345678
17. E The author mentions that Los Amigos is relatively pristine, and that the 9
23456789012345678901234567890121234567890123456789012345678
123456789012345678901234567890121234567890123456789012345678 9
123456789012345678901234567890121234567890123456789012345678
rainforest is facing threats. Eliminate (A) and (B). He isn’t talking in the 9
123456789012345678901234567890121234567890123456789012345678 9
123456789012345678901234567890121234567890123456789012345678
passage about restoring the rainforest, but preventing future damage. 9
1 9
123456789012345678901234567890121234567890123456789012345678 99
23456789012345678901234567890121234567890123456789012345678
Eliminate (C). He does not say that every other part of the rainforest is
1
123456789012345678901234567890121234567890123456789012345678
already destroyed beyond repair. Your logic should tell you that. 9
123456789012345678901234567890121234567890123456789012345678 9
123456789012345678901234567890121234567890123456789012345678
Eliminate (D) and you are left with
23456789012345678901234567890121234567890123456789012345678
(E), the correct answer. 9
9
1
123456789012345678901234567890121234567890123456789012345678 9
123456789012345678901234567890121234567890123456789012345678 9
123456789012345678901234567890121234567890123456789012345678 9
1234567890123456789012345678901212345678901234567890123456789
Copyright © 2005 Thomson Peterson’s, a part of The Thomson Corporaton 65
SAT is a registered trademark of the College Entrance Examination Board, which
was not involved in the production of and does not endorse this product.
1234567890123456789012345678901212345678901234567890123456789
123456789012345678901234567890121234567890123456789012345678 9
123456789012345678901234567890121234567890123456789012345678 9
123456789012345678901234567890121234567890123456789012345678 9
123456789012345678901234567890121234567890123456789012345678 9
123456789012345678901234567890121234567890123456789012345678
18. A First go back and get the context of the use of this phrase. It refers to 9
123456789012345678901234567890121234567890123456789012345678 9
123456789012345678901234567890121234567890123456789012345678
land being set aside for conservation
23456789012345678901234567890121234567890123456789012345678
use. The only possibility is (A). 9
9
1
123456789012345678901234567890121234567890123456789012345678 9
123456789012345678901234567890121234567890123456789012345678
19. D This is a difficult question because it requires you to infer the answer. 9
123456789012345678901234567890121234567890123456789012345678 9
123456789012345678901234567890121234567890123456789012345678
The best way to do that is to eliminate
23456789012345678901234567890121234567890123456789012345678
the least likely answers and then 9
9
1 see what’s left. The passage tells you that the agreement was “the first
123456789012345678901234567890121234567890123456789012345678 9
123456789012345678901234567890121234567890123456789012345678
long-term permanently renewable conservation concession.” There are 9
123456789012345678901234567890121234567890123456789012345678 9
123456789012345678901234567890121234567890123456789012345678
two references to time in this sentence, so the answer must have to do
23456789012345678901234567890121234567890123456789012345678
9
9
1
123456789012345678901234567890121234567890123456789012345678
with time—that leaves (B), (D), and (E). The author isn’t really 9
123456789012345678901234567890121234567890123456789012345678 9
123456789012345678901234567890121234567890123456789012345678
interested in the legal aspects, though, so eliminate (E). Because he 9
123456789012345678901234567890121234567890123456789012345678
23456789012345678901234567890121234567890123456789012345678
9
9
1 includes both “long-term” and “renewable,” the agreement probably
123456789012345678901234567890121234567890123456789012345678 9
123456789012345678901234567890121234567890123456789012345678
wasn’t the first contract that was simply one or the other. Eliminate (B). 9
123456789012345678901234567890121234567890123456789012345678 9
123456789012345678901234567890121234567890123456789012345678
That leaves you with (D), the correct answer. 9
123456789012345678901234567890121234567890123456789012345678 9
23456789012345678901234567890121234567890123456789012345678
123456789012345678901234567890121234567890123456789012345678 9
123456789012345678901234567890121234567890123456789012345678
20. C This is a question that you should be asking yourself as you read through 9
123456789012345678901234567890121234567890123456789012345678 9
123456789012345678901234567890121234567890123456789012345678 9
the passage. The passage begins by discussing the importance of conser-
123456789012345678901234567890121234567890123456789012345678 9
1 vation efforts in Amazonia and then links the work at the Los Amigos 9
123456789012345678901234567890121234567890123456789012345678 9
123456789012345678901234567890121234567890123456789012345678
watershed with this goal. The correct answer will contain both of these 9
123456789012345678901234567890121234567890123456789012345678 9
123456789012345678901234567890121234567890123456789012345678 9
23456789012345678901234567890121234567890123456789012345678
things. (A) is too general. (B) isn’t accurate—he doesn’t focus on elimi-
123456789012345678901234567890121234567890123456789012345678 9
123456789012345678901234567890121234567890123456789012345678
nating bad things but on continuing good things. (C) sounds good. (D) 9
123456789012345678901234567890121234567890123456789012345678
is incorrect because the passage is not primarily about the Peruvian gov- 9
123456789012345678901234567890121234567890123456789012345678 9
123456789012345678901234567890121234567890123456789012345678
ernment. (E) points to one issue that the passage discusses but lacks 9
123456789012345678901234567890121234567890123456789012345678 9
123456789012345678901234567890121234567890123456789012345678
many of the other issues the passage discusses. (C) is the best answer. 9
123456789012345678901234567890121234567890123456789012345678 9 9
1 21. D This question calls for a little nuance. He does advocate for his project,
123456789012345678901234567890121234567890123456789012345678 9
123456789012345678901234567890121234567890123456789012345678 9
123456789012345678901234567890121234567890123456789012345678
but does not position it against other projects. Eliminate (A). (B) is too 9
123456789012345678901234567890121234567890123456789012345678 9
123456789012345678901234567890121234567890123456789012345678
general. (C) is not accurate—he does
23456789012345678901234567890121234567890123456789012345678
not condemn the government. (D) 9
9
123456789012345678901234567890121234567890123456789012345678
123456789012345678901234567890121234567890123456789012345678
sounds good. (E) uses language that is too strong—he is not a zealot, 9
1 9
123456789012345678901234567890121234567890123456789012345678
but a scientist making his case in calm, rational language. (D) is correct. 9
123456789012345678901234567890121234567890123456789012345678 9
23456789012345678901234567890121234567890123456789012345678
123456789012345678901234567890121234567890123456789012345678 9
123456789012345678901234567890121234567890123456789012345678
22. E The author positions his project as complementary to other projects. These 9
1 9
123456789012345678901234567890121234567890123456789012345678 9
scientists are examples of the other amenable projects. The answer is (E).
123456789012345678901234567890121234567890123456789012345678 9
123456789012345678901234567890121234567890123456789012345678
23. E This is a tricky answer because the right choice is the one you’d least 9
123456789012345678901234567890121234567890123456789012345678 9
123456789012345678901234567890121234567890123456789012345678
expect. The author focuses on working with plants in the watershed, but 9
123456789012345678901234567890121234567890123456789012345678 9
123456789012345678901234567890121234567890123456789012345678
in the last word of the passage mentions an “herbarium,” which through 9
23456789012345678901234567890121234567890123456789012345678
123456789012345678901234567890121234567890123456789012345678 9
123456789012345678901234567890121234567890123456789012345678
context clues and word study, you can guess means a laboratory where 9
123456789012345678901234567890121234567890123456789012345678 9
123456789012345678901234567890121234567890123456789012345678
plants are grown. Eliminate (A). The author mentions studying “human- 9
1 9
123456789012345678901234567890121234567890123456789012345678 9
23456789012345678901234567890121234567890123456789012345678
plant” interactions in paragraph 6. Eliminate (B). Somewhat surprisingly,
9
123456789012345678901234567890121234567890123456789012345678 9
123456789012345678901234567890121234567890123456789012345678
the author is in favor of pharmaceutical use of Amazon plants, as he
9
123456789012345678901234567890121234567890123456789012345678 9
1 indicates in paragraphs 1 and 5 and implies in paragraph 6. Eliminate (C).
23456789012345678901234567890121234567890123456789012345678
123456789012345678901234567890121234567890123456789012345678 9
1 (D) is obviously not the answer. You might think that because he focuses 9
123456789012345678901234567890121234567890123456789012345678 9
123456789012345678901234567890121234567890123456789012345678 9
on naming, he means labeling, but in fact it is a scientist on another project,
123456789012345678901234567890121234567890123456789012345678 9
123456789012345678901234567890121234567890123456789012345678
Robin Foster, who actually labeled plants. (E) is the answer. 9
123456789012345678901234567890121234567890123456789012345678 9
123456789012345678901234567890121234567890123456789012345678 9
123456789012345678901234567890121234567890123456789012345678 9
1234567890123456789012345678901212345678901234567890123456789
66 Copyright © 2005 Thomson Peterson’s, a part of The Thomson Corporaton
SAT is a registered trademark of the College Entrance Examination Board, which
was not involved in the production of and does not endorse this product.
1234567890123456789012345678901212345678901234567890123456789
123456789012345678901234567890121234567890123456789012345678 9
123456789012345678901234567890121234567890123456789012345678 9
123456789012345678901234567890121234567890123456789012345678 9
123456789012345678901234567890121234567890123456789012345678 9
123456789012345678901234567890121234567890123456789012345678
24. B As always, first go back and read the section cited in the question. The 9
123456789012345678901234567890121234567890123456789012345678 9
123456789012345678901234567890121234567890123456789012345678
sentence in which “providing names” occurs,
23456789012345678901234567890121234567890123456789012345678
mentions communication 9
9
1
123456789012345678901234567890121234567890123456789012345678
about plants and the animals that use them. You will recall that earlier 9
123456789012345678901234567890121234567890123456789012345678 9
123456789012345678901234567890121234567890123456789012345678
in the passage, it was stated that one of the major projects in studying 9
123456789012345678901234567890121234567890123456789012345678
23456789012345678901234567890121234567890123456789012345678
9
9
1 Amazonia was discovering new species. One hurdle for communication
123456789012345678901234567890121234567890123456789012345678 9
123456789012345678901234567890121234567890123456789012345678
among scientists once a species is discovered is standardizing the name 9
123456789012345678901234567890121234567890123456789012345678 9
123456789012345678901234567890121234567890123456789012345678
of the species. This is how “providing
23456789012345678901234567890121234567890123456789012345678
names” will facilitate communi- 9
9
1 cation. Choice (B) correctly points this out. (If you had difficulty with
123456789012345678901234567890121234567890123456789012345678 9
123456789012345678901234567890121234567890123456789012345678
this question, notice that all the other choices mention issues not 9
123456789012345678901234567890121234567890123456789012345678 9
123456789012345678901234567890121234567890123456789012345678
directly addressed in the passage. That is a strong indicator that an
23456789012345678901234567890121234567890123456789012345678
9
9
1
123456789012345678901234567890121234567890123456789012345678
answer is incorrect.) 9
123456789012345678901234567890121234567890123456789012345678 9
123456789012345678901234567890121234567890123456789012345678 9
123456789012345678901234567890121234567890123456789012345678
25. A The author’s full argument goes, “To be informed, we must develop 9
23456789012345678901234567890121234567890123456789012345678
123456789012345678901234567890121234567890123456789012345678 9
123456789012345678901234567890121234567890123456789012345678
knowledge. To develop knowledge, we must collect, organize, and 9
123456789012345678901234567890121234567890123456789012345678 9
123456789012345678901234567890121234567890123456789012345678 99
disseminate information. In this sense, botanical information has
123456789012345678901234567890121234567890123456789012345678
123456789012345678901234567890121234567890123456789012345678
conservation value.” The author is arguing that being informed is 9
1 9
123456789012345678901234567890121234567890123456789012345678
essential for conservationism, and so in this sense botanical informa- 9
123456789012345678901234567890121234567890123456789012345678 9
123456789012345678901234567890121234567890123456789012345678
tion has conservation value. So even though (B), (C), (D), and (E) are all 9
23456789012345678901234567890121234567890123456789012345678
123456789012345678901234567890121234567890123456789012345678 9
123456789012345678901234567890121234567890123456789012345678
things the author might agree with, only (A) captures the meaning of 9
123456789012345678901234567890121234567890123456789012345678 9
123456789012345678901234567890121234567890123456789012345678 99
the argument made here.
123456789012345678901234567890121234567890123456789012345678 9
123456789012345678901234567890121234567890123456789012345678
26. B You might confuse repeated use of the word watershed with an actual
123456789012345678901234567890121234567890123456789012345678 9
123456789012345678901234567890121234567890123456789012345678
discussion of water pollution, but the author doesn’t mention water 9
123456789012345678901234567890121234567890123456789012345678 99
1 pollution explicitly. The answer is (B). If you don’t get this right away,
123456789012345678901234567890121234567890123456789012345678 9
123456789012345678901234567890121234567890123456789012345678
you can arrive at it by eliminating the others. He does clearly mention 9
123456789012345678901234567890121234567890123456789012345678 9
123456789012345678901234567890121234567890123456789012345678
all of the other choices. 9
123456789012345678901234567890121234567890123456789012345678
23456789012345678901234567890121234567890123456789012345678
9
9
123456789012345678901234567890121234567890123456789012345678
123456789012345678901234567890121234567890123456789012345678
27. C The author is talking about how his work at Los Amigos relates to 9
1 9
123456789012345678901234567890121234567890123456789012345678
other conservation projects, and how the Los Amigos area is related to 9
123456789012345678901234567890121234567890123456789012345678 9
123456789012345678901234567890121234567890123456789012345678 99
23456789012345678901234567890121234567890123456789012345678
other environmentally protected areas. Only (C) captures that meaning.
123456789012345678901234567890121234567890123456789012345678 9
1
123456789012345678901234567890121234567890123456789012345678 9
123456789012345678901234567890121234567890123456789012345678
Section 5 9
123456789012345678901234567890121234567890123456789012345678 9
123456789012345678901234567890121234567890123456789012345678 9
123456789012345678901234567890121234567890123456789012345678
1. C For every positive integer, there is a negative integer the same distance 9
123456789012345678901234567890121234567890123456789012345678 9
123456789012345678901234567890121234567890123456789012345678 9
from zero. This means that there are an equal number of positive and
23456789012345678901234567890121234567890123456789012345678
123456789012345678901234567890121234567890123456789012345678
negative integers. Therefore each set contains the same number of 9
123456789012345678901234567890121234567890123456789012345678 9
123456789012345678901234567890121234567890123456789012345678
members, which is choice (C). 9
123456789012345678901234567890121234567890123456789012345678 99
1
123456789012345678901234567890121234567890123456789012345678
2. B Four sweaters cost p dollars, and so 12 sweaters at the regular price 9
123456789012345678901234567890121234567890123456789012345678 9
123456789012345678901234567890121234567890123456789012345678
would cost 3p (4 3 3 5 12). Since the sweaters are half-off, the 12 9
123456789012345678901234567890121234567890123456789012345678 9
123456789012345678901234567890121234567890123456789012345678
3p 9
23456789012345678901234567890121234567890123456789012345678
123456789012345678901234567890121234567890123456789012345678
sweaters only cost half as much, , choice (B). 9
1 2 9
123456789012345678901234567890121234567890123456789012345678 9
123456789012345678901234567890121234567890123456789012345678 9
123456789012345678901234567890121234567890123456789012345678 9
123456789012345678901234567890121234567890123456789012345678 9
123456789012345678901234567890121234567890123456789012345678 9
123456789012345678901234567890121234567890123456789012345678 9
123456789012345678901234567890121234567890123456789012345678 9
1234567890123456789012345678901212345678901234567890123456789
Copyright © 2005 Thomson Peterson’s, a part of The Thomson Corporaton 67
SAT is a registered trademark of the College Entrance Examination Board, which
was not involved in the production of and does not endorse this product.
1234567890123456789012345678901212345678901234567890123456789
123456789012345678901234567890121234567890123456789012345678 9
123456789012345678901234567890121234567890123456789012345678 9
123456789012345678901234567890121234567890123456789012345678 9
123456789012345678901234567890121234567890123456789012345678 9
123456789012345678901234567890121234567890123456789012345678
3. C This fraction looks complicated, but realize that the numerator and de- 9
123456789012345678901234567890121234567890123456789012345678 9
123456789012345678901234567890121234567890123456789012345678
nominator are both fractions. You might
23456789012345678901234567890121234567890123456789012345678
then see that the numerator 9
9
1 2
SD denominator
4
SD
123456789012345678901234567890121234567890123456789012345678
123456789012345678901234567890121234567890123456789012345678
is less than the , and
123456789012345678901234567890121234567890123456789012345678
3 5
so this fraction is less than 9
9
9
123456789012345678901234567890121234567890123456789012345678
one. That eliminates (D) and (E) as possible answer choices. You have to
23456789012345678901234567890121234567890123456789012345678
9
9
1
123456789012345678901234567890121234567890123456789012345678
simplify the fraction to find the answer, and since you’re dividing frac- 9
123456789012345678901234567890121234567890123456789012345678 9
123456789012345678901234567890121234567890123456789012345678
tions, remember to multiply by the reciprocal of the divisor: 9
123456789012345678901234567890121234567890123456789012345678
23456789012345678901234567890121234567890123456789012345678
9
9
1
123456789012345678901234567890121234567890123456789012345678
2 9
123456789012345678901234567890121234567890123456789012345678 9
123456789012345678901234567890121234567890123456789012345678
3 2 4 2 5 10 5 9
123456789012345678901234567890121234567890123456789012345678
5 4 5 3 5 5 . (C) is the answer.
23456789012345678901234567890121234567890123456789012345678
9
9
1 4 3 5 3 4 12 6
123456789012345678901234567890121234567890123456789012345678 9
123456789012345678901234567890121234567890123456789012345678
5 9
123456789012345678901234567890121234567890123456789012345678 9
123456789012345678901234567890121234567890123456789012345678 9
123456789012345678901234567890121234567890123456789012345678 9
23456789012345678901234567890121234567890123456789012345678
4. E Here you just need to plug in and be careful. If x 5 3 then y 5 1 because
123456789012345678901234567890121234567890123456789012345678 9
123456789012345678901234567890121234567890123456789012345678
3y 5 x 9
123456789012345678901234567890121234567890123456789012345678 9
123456789012345678901234567890121234567890123456789012345678
3y 5 3 9
123456789012345678901234567890121234567890123456789012345678 9
1 9
123456789012345678901234567890121234567890123456789012345678
3y 3 9
123456789012345678901234567890121234567890123456789012345678
5 9
123456789012345678901234567890121234567890123456789012345678
3 3 9
23456789012345678901234567890121234567890123456789012345678
123456789012345678901234567890121234567890123456789012345678 9
123456789012345678901234567890121234567890123456789012345678
y51 9
123456789012345678901234567890121234567890123456789012345678 9
123456789012345678901234567890121234567890123456789012345678 9
123456789012345678901234567890121234567890123456789012345678
If y 5 1, then 9
123456789012345678901234567890121234567890123456789012345678 9
123456789012345678901234567890121234567890123456789012345678
10 9
123456789012345678901234567890121234567890123456789012345678
y5 9
123456789012345678901234567890121234567890123456789012345678
z 9
1 9
23456789012345678901234567890121234567890123456789012345678
123456789012345678901234567890121234567890123456789012345678
10 9
123456789012345678901234567890121234567890123456789012345678 9
1 1 5 9
123456789012345678901234567890121234567890123456789012345678 9
z
123456789012345678901234567890121234567890123456789012345678 9
10
S D
123456789012345678901234567890121234567890123456789012345678
123456789012345678901234567890121234567890123456789012345678
~z!1 5 z ~z!
123456789012345678901234567890121234567890123456789012345678 9
9
9
123456789012345678901234567890121234567890123456789012345678 9
123456789012345678901234567890121234567890123456789012345678
z 5 10 9
123456789012345678901234567890121234567890123456789012345678 9
23456789012345678901234567890121234567890123456789012345678
123456789012345678901234567890121234567890123456789012345678 9
1 This is choice (E). 9
123456789012345678901234567890121234567890123456789012345678 9
123456789012345678901234567890121234567890123456789012345678 9
123456789012345678901234567890121234567890123456789012345678
5. C y 5 mx 1 b is the equation for a line, so the wiggly lines of choices (D) 9
123456789012345678901234567890121234567890123456789012345678 9
123456789012345678901234567890121234567890123456789012345678
and (E) cannot be correct. A negative m means the slope is negative, so 9
123456789012345678901234567890121234567890123456789012345678 9
123456789012345678901234567890121234567890123456789012345678 9
the line must slant down when viewed from left to right. That
23456789012345678901234567890121234567890123456789012345678
123456789012345678901234567890121234567890123456789012345678 9
123456789012345678901234567890121234567890123456789012345678
eliminates (A). The y-intercept b is positive, so the line must cross the 9
123456789012345678901234567890121234567890123456789012345678 9
123456789012345678901234567890121234567890123456789012345678 9
y-axis above the x-axis. This is true of (C), so it’s the answer.
9
1
123456789012345678901234567890121234567890123456789012345678
6. A This problem requires a little careful decoding, but all the steps are 9
123456789012345678901234567890121234567890123456789012345678 9
123456789012345678901234567890121234567890123456789012345678
straightforward. The first part of the order is 2 H-M, which are two 9
123456789012345678901234567890121234567890123456789012345678 9
123456789012345678901234567890121234567890123456789012345678
medium hats. Usually the hats are $12 each, so two would cost $24, but 9
23456789012345678901234567890121234567890123456789012345678
123456789012345678901234567890121234567890123456789012345678 9
1 since all medium-sized items are 25% off, the price is reduced. Twenty- 9
123456789012345678901234567890121234567890123456789012345678 9
123456789012345678901234567890121234567890123456789012345678
five percent of 24 is 6, so together the two medium hats cost $18 ($24 9
123456789012345678901234567890121234567890123456789012345678
23456789012345678901234567890121234567890123456789012345678
9
9
1 minus $6). The next part is 2 H-L, which is two large hats, and together
123456789012345678901234567890121234567890123456789012345678 9
123456789012345678901234567890121234567890123456789012345678 9
123456789012345678901234567890121234567890123456789012345678 9
1234567890123456789012345678901212345678901234567890123456789
68 Copyright © 2005 Thomson Peterson’s, a part of The Thomson Corporaton
SAT is a registered trademark of the College Entrance Examination Board, which
was not involved in the production of and does not endorse this product.
1234567890123456789012345678901212345678901234567890123456789
123456789012345678901234567890121234567890123456789012345678 9
123456789012345678901234567890121234567890123456789012345678 9
123456789012345678901234567890121234567890123456789012345678 9
123456789012345678901234567890121234567890123456789012345678 9
123456789012345678901234567890121234567890123456789012345678
they cost $24. The last part of the order is 1 SH-M, one medium shirt, 9
123456789012345678901234567890121234567890123456789012345678 9
123456789012345678901234567890121234567890123456789012345678
which costs $9 since it is 25% off ($3
23456789012345678901234567890121234567890123456789012345678
is 25% of $12, the normal cost of 9
9
1
123456789012345678901234567890121234567890123456789012345678
the medium shirt). Adding up 18 1 24 1 9 5 51, answer (A). 9
123456789012345678901234567890121234567890123456789012345678 9
123456789012345678901234567890121234567890123456789012345678 9
123456789012345678901234567890121234567890123456789012345678
7. A The task here is to solve for x. The
23456789012345678901234567890121234567890123456789012345678
test makers are betting that the 9
9
1 square root symbol will throw you off a bit.
123456789012345678901234567890121234567890123456789012345678 9
123456789012345678901234567890121234567890123456789012345678 9
123456789012345678901234567890121234567890123456789012345678 9
123456789012345678901234567890121234567890123456789012345678
5= x 1 15 5 30
23456789012345678901234567890121234567890123456789012345678
9
9
1
123456789012345678901234567890121234567890123456789012345678
= 9
123456789012345678901234567890121234567890123456789012345678
5 x 1 15 2 15 5 30 2 15 9
123456789012345678901234567890121234567890123456789012345678 9
123456789012345678901234567890121234567890123456789012345678
=
23456789012345678901234567890121234567890123456789012345678
9
9
1 5 x 5 15
123456789012345678901234567890121234567890123456789012345678 9
123456789012345678901234567890121234567890123456789012345678
5=x 15 9
123456789012345678901234567890121234567890123456789012345678
5 9
123456789012345678901234567890121234567890123456789012345678
5 5 9
123456789012345678901234567890121234567890123456789012345678 99
23456789012345678901234567890121234567890123456789012345678
123456789012345678901234567890121234567890123456789012345678
123456789012345678901234567890121234567890123456789012345678
=x 5 3 9
123456789012345678901234567890121234567890123456789012345678 99
123456789012345678901234567890121234567890123456789012345678
x59
123456789012345678901234567890121234567890123456789012345678 9
1 Answer (A). 9
123456789012345678901234567890121234567890123456789012345678 9
123456789012345678901234567890121234567890123456789012345678 9
123456789012345678901234567890121234567890123456789012345678
8. D Here there is no shortcut. We have to factor and then simplify. 9
23456789012345678901234567890121234567890123456789012345678
123456789012345678901234567890121234567890123456789012345678 9
123456789012345678901234567890121234567890123456789012345678 9
123456789012345678901234567890121234567890123456789012345678 9
2
x 2 x 2 12 ~ x 2 4!~ x 1 3 ! 5 ~ !~ x -4 x 1 3! 5
123456789012345678901234567890121234567890123456789012345678
5 9
123456789012345678901234567890121234567890123456789012345678 99
2x 2
1 2x 2 12 2 ~ x2
1 x 2 6! 2 ~ x 1 3!~ x 2 2!
123456789012345678901234567890121234567890123456789012345678
x24
123456789012345678901234567890121234567890123456789012345678 9
123456789012345678901234567890121234567890123456789012345678
~ !
. (D) is the answer. 9
123456789012345678901234567890121234567890123456789012345678 99
2 x 2 2
1
123456789012345678901234567890121234567890123456789012345678
9. C The first thing to do is to have it clearly in mind what the graph of 9
123456789012345678901234567890121234567890123456789012345678 9
123456789012345678901234567890121234567890123456789012345678 9
2
f(x) 5 x looks like.
123456789012345678901234567890121234567890123456789012345678 9
123456789012345678901234567890121234567890123456789012345678 9
123456789012345678901234567890121234567890123456789012345678 9
123456789012345678901234567890121234567890123456789012345678 9
123456789012345678901234567890121234567890123456789012345678 9
123456789012345678901234567890121234567890123456789012345678 9
123456789012345678901234567890121234567890123456789012345678 9
123456789012345678901234567890121234567890123456789012345678 99
23456789012345678901234567890121234567890123456789012345678
123456789012345678901234567890121234567890123456789012345678 9
1
123456789012345678901234567890121234567890123456789012345678 9
123456789012345678901234567890121234567890123456789012345678 9
123456789012345678901234567890121234567890123456789012345678 9
123456789012345678901234567890121234567890123456789012345678 9
123456789012345678901234567890121234567890123456789012345678 9
123456789012345678901234567890121234567890123456789012345678 9
123456789012345678901234567890121234567890123456789012345678 9
123456789012345678901234567890121234567890123456789012345678 99
23456789012345678901234567890121234567890123456789012345678
123456789012345678901234567890121234567890123456789012345678 9
123456789012345678901234567890121234567890123456789012345678 9
123456789012345678901234567890121234567890123456789012345678 9
1
123456789012345678901234567890121234567890123456789012345678 9
123456789012345678901234567890121234567890123456789012345678 9
123456789012345678901234567890121234567890123456789012345678 9
123456789012345678901234567890121234567890123456789012345678 9
123456789012345678901234567890121234567890123456789012345678 9
123456789012345678901234567890121234567890123456789012345678 99
23456789012345678901234567890121234567890123456789012345678
1
123456789012345678901234567890121234567890123456789012345678 9
123456789012345678901234567890121234567890123456789012345678 9
123456789012345678901234567890121234567890123456789012345678 9
123456789012345678901234567890121234567890123456789012345678 9
123456789012345678901234567890121234567890123456789012345678 9
123456789012345678901234567890121234567890123456789012345678 9
123456789012345678901234567890121234567890123456789012345678 9
1234567890123456789012345678901212345678901234567890123456789
Copyright © 2005 Thomson Peterson’s, a part of The Thomson Corporaton 69
SAT is a registered trademark of the College Entrance Examination Board, which
was not involved in the production of and does not endorse this product.
1234567890123456789012345678901212345678901234567890123456789
123456789012345678901234567890121234567890123456789012345678 9
123456789012345678901234567890121234567890123456789012345678 9
123456789012345678901234567890121234567890123456789012345678 9
123456789012345678901234567890121234567890123456789012345678 9
123456789012345678901234567890121234567890123456789012345678 9
2 2
This shape comes about as a result of the x term. When f(x) 5 x , the
123456789012345678901234567890121234567890123456789012345678 9
123456789012345678901234567890121234567890123456789012345678
fact that all x values are squared makes
23456789012345678901234567890121234567890123456789012345678
every y value positive (keep in 9
9
1
123456789012345678901234567890121234567890123456789012345678
mind that f(x) acts as the y value). This graph has no negative y values, 9
123456789012345678901234567890121234567890123456789012345678 9
123456789012345678901234567890121234567890123456789012345678
but to find the right answer, it must now be rotated clockwise. This 9
123456789012345678901234567890121234567890123456789012345678
23456789012345678901234567890121234567890123456789012345678
9
9
1 means rotating to the right.
123456789012345678901234567890121234567890123456789012345678 9
123456789012345678901234567890121234567890123456789012345678
Here’s a good way to visualize this. Hold up your left hand, and make a 9
123456789012345678901234567890121234567890123456789012345678 9
123456789012345678901234567890121234567890123456789012345678
bowl-shape with your fingers on one side and your thumb on the other.
23456789012345678901234567890121234567890123456789012345678
9
9
1
123456789012345678901234567890121234567890123456789012345678
The tips of your fingers and thumbs should all be point towards the ceil- 9
123456789012345678901234567890121234567890123456789012345678 9
123456789012345678901234567890121234567890123456789012345678
ing. This is your graph before rotation. Now take your left hand and shift 9
123456789012345678901234567890121234567890123456789012345678
23456789012345678901234567890121234567890123456789012345678
9
9
1 it 90° to the right; in essence, snap your wrist down. The tips of your
123456789012345678901234567890121234567890123456789012345678 9
123456789012345678901234567890121234567890123456789012345678
fingers and thumb should now be pointing to the right. Find the answer 9
123456789012345678901234567890121234567890123456789012345678 9
123456789012345678901234567890121234567890123456789012345678
choice that looks like the position of your left hand. (C) is the answer. 9
123456789012345678901234567890121234567890123456789012345678 9
23456789012345678901234567890121234567890123456789012345678
123456789012345678901234567890121234567890123456789012345678 9
123456789012345678901234567890121234567890123456789012345678
10. E The correct ordered pair has to satisfy two conditions, x 2 y . 2 and 9
123456789012345678901234567890121234567890123456789012345678 9
123456789012345678901234567890121234567890123456789012345678 9
x 1 y . 4. Let’s start with the first condition and take it from there. In
123456789012345678901234567890121234567890123456789012345678 9
1 order for x 2 y . 2, x must be greater than y by more than two. This 9
123456789012345678901234567890121234567890123456789012345678 9
123456789012345678901234567890121234567890123456789012345678
eliminates choices (A), (B), and (D). Now you have only two choices left 9
123456789012345678901234567890121234567890123456789012345678 9
123456789012345678901234567890121234567890123456789012345678 9
23456789012345678901234567890121234567890123456789012345678
to check the second condition, x 1 y . 4. If you add the x and y values
123456789012345678901234567890121234567890123456789012345678 9
123456789012345678901234567890121234567890123456789012345678
of choice (C) together you get 4, but the value must be greater than 4. 9
123456789012345678901234567890121234567890123456789012345678
Choice (E) has to be the correct answer. 9
123456789012345678901234567890121234567890123456789012345678 9
123456789012345678901234567890121234567890123456789012345678 9
123456789012345678901234567890121234567890123456789012345678
11. A The best way to approach a problem like this is to use one of the 9
123456789012345678901234567890121234567890123456789012345678 9
123456789012345678901234567890121234567890123456789012345678
equations and solve for one of the variables in terms of the other. Then 9
1 9
123456789012345678901234567890121234567890123456789012345678 9
23456789012345678901234567890121234567890123456789012345678
you can place that answer into the second equation.
123456789012345678901234567890121234567890123456789012345678 9 9
1
123456789012345678901234567890121234567890123456789012345678
This problem succumbs quite nicely to this strategy. Take the first 9
123456789012345678901234567890121234567890123456789012345678 9
123456789012345678901234567890121234567890123456789012345678 9
23456789012345678901234567890121234567890123456789012345678
equation and solve for y in terms of x. Cross-multiplication makes
123456789012345678901234567890121234567890123456789012345678 9
1 this simple. 9
123456789012345678901234567890121234567890123456789012345678 9
123456789012345678901234567890121234567890123456789012345678 9
23456789012345678901234567890121234567890123456789012345678
123456789012345678901234567890121234567890123456789012345678
x 1 9
123456789012345678901234567890121234567890123456789012345678
5 9
1 y 2 9
123456789012345678901234567890121234567890123456789012345678 9
123456789012345678901234567890121234567890123456789012345678
2x 5 y 9
123456789012345678901234567890121234567890123456789012345678 9
123456789012345678901234567890121234567890123456789012345678 9
123456789012345678901234567890121234567890123456789012345678
You can now substitute this information into x 1 y. Subbing in for y: 9
123456789012345678901234567890121234567890123456789012345678 9
123456789012345678901234567890121234567890123456789012345678 9
x 1 y 5 x 1 2x 5 3x, which is (A).
123456789012345678901234567890121234567890123456789012345678 9
23456789012345678901234567890121234567890123456789012345678
9
123456789012345678901234567890121234567890123456789012345678 9
123456789012345678901234567890121234567890123456789012345678
12. B There are two main ways to approach this problem. You can come up
9
123456789012345678901234567890121234567890123456789012345678 9
1 with a snazzy algebraic formula, or you can go to the answer choices
23456789012345678901234567890121234567890123456789012345678
123456789012345678901234567890121234567890123456789012345678 9
123456789012345678901234567890121234567890123456789012345678
and start cranking in numbers. Let’s use the crank method. 9
123456789012345678901234567890121234567890123456789012345678 9 9
123456789012345678901234567890121234567890123456789012345678
Whichever method you use, you must first determine how much
9
1
23456789012345678901234567890121234567890123456789012345678
123456789012345678901234567890121234567890123456789012345678
Quentin paid for the three hot dogs. This is done by subtracting $7.34 9
1 9
123456789012345678901234567890121234567890123456789012345678
from $10, giving you $2.66. This is the price, including tax, of three 9
123456789012345678901234567890121234567890123456789012345678 9
123456789012345678901234567890121234567890123456789012345678
hot dogs. 9
123456789012345678901234567890121234567890123456789012345678 9
123456789012345678901234567890121234567890123456789012345678 9
123456789012345678901234567890121234567890123456789012345678 9
123456789012345678901234567890121234567890123456789012345678 9
1234567890123456789012345678901212345678901234567890123456789
70 Copyright © 2005 Thomson Peterson’s, a part of The Thomson Corporaton
SAT is a registered trademark of the College Entrance Examination Board, which
was not involved in the production of and does not endorse this product.
1234567890123456789012345678901212345678901234567890123456789
123456789012345678901234567890121234567890123456789012345678 9
123456789012345678901234567890121234567890123456789012345678 9
123456789012345678901234567890121234567890123456789012345678 9
123456789012345678901234567890121234567890123456789012345678 9
123456789012345678901234567890121234567890123456789012345678
With the crank method, starting with the middle answer choice (C) 9
123456789012345678901234567890121234567890123456789012345678 9
123456789012345678901234567890121234567890123456789012345678
often yields a clue, even if it’s incorrect.
23456789012345678901234567890121234567890123456789012345678
Choice (C) has a hot dog at 86 9
9
1
123456789012345678901234567890121234567890123456789012345678
cents. Add 6 or 7 cents sales tax to this, and then multiply by three. 9
123456789012345678901234567890121234567890123456789012345678 9
123456789012345678901234567890121234567890123456789012345678
You get: 9
123456789012345678901234567890121234567890123456789012345678
23456789012345678901234567890121234567890123456789012345678
9
9
1 3(86 1 7) 5 3(93)5279
123456789012345678901234567890121234567890123456789012345678 9
123456789012345678901234567890121234567890123456789012345678 9
123456789012345678901234567890121234567890123456789012345678 9
123456789012345678901234567890121234567890123456789012345678
23456789012345678901234567890121234567890123456789012345678
9
9
1 You can add the entire amount of sales tax per dollar, 7 cents, or you
123456789012345678901234567890121234567890123456789012345678
can take into account the fact that 86 cents is not quite an entire 9
123456789012345678901234567890121234567890123456789012345678 9
123456789012345678901234567890121234567890123456789012345678 9
Tip

dollar, so Quentin might not owe the entire 7 cents sales tax. This
123456789012345678901234567890121234567890123456789012345678
23456789012345678901234567890121234567890123456789012345678
9
9
1 might seem vague, but the question does allow for some wiggle room
123456789012345678901234567890121234567890123456789012345678 9
123456789012345678901234567890121234567890123456789012345678
since it asks “which is closest to the price . . .” Whether you use 6 or 9
123456789012345678901234567890121234567890123456789012345678 9
123456789012345678901234567890121234567890123456789012345678
7 cents sales tax, the same answer works best both ways. 9
123456789012345678901234567890121234567890123456789012345678 99
23456789012345678901234567890121234567890123456789012345678
123456789012345678901234567890121234567890123456789012345678 9
123456789012345678901234567890121234567890123456789012345678
123456789012345678901234567890121234567890123456789012345678
You know that Quentin spent $2.66, or 266 cents. Answer choice (C) 9
123456789012345678901234567890121234567890123456789012345678 9
123456789012345678901234567890121234567890123456789012345678 99
comes out at 279 cents, which is 13 cents too many. This means that (C)
1
123456789012345678901234567890121234567890123456789012345678
is incorrect because it’s too great. If (C) is too great, then (D) and (E) are 9
123456789012345678901234567890121234567890123456789012345678 9
123456789012345678901234567890121234567890123456789012345678
even greater, so they must also be incorrect. This leaves (A) and (B). Try 9
23456789012345678901234567890121234567890123456789012345678
123456789012345678901234567890121234567890123456789012345678 9
123456789012345678901234567890121234567890123456789012345678
(B). If it also too great, then the answer must be (A). Here’s (B). 9
123456789012345678901234567890121234567890123456789012345678 99
123456789012345678901234567890121234567890123456789012345678
3(82 1 7) 5 3(89) 5 267
123456789012345678901234567890121234567890123456789012345678 9
123456789012345678901234567890121234567890123456789012345678 9
123456789012345678901234567890121234567890123456789012345678
This is only one penny off. If you use a 6-cent sales tax, it is only 2 cents 9
123456789012345678901234567890121234567890123456789012345678 9
123456789012345678901234567890121234567890123456789012345678
off. (B) is closest to the price of the hot dog, so it’s the correct answer. 9
1 9
123456789012345678901234567890121234567890123456789012345678 99
23456789012345678901234567890121234567890123456789012345678
123456789012345678901234567890121234567890123456789012345678 9
1
123456789012345678901234567890121234567890123456789012345678 9
123456789012345678901234567890121234567890123456789012345678 9
123456789012345678901234567890121234567890123456789012345678 9
123456789012345678901234567890121234567890123456789012345678 9
123456789012345678901234567890121234567890123456789012345678 9
123456789012345678901234567890121234567890123456789012345678 9
123456789012345678901234567890121234567890123456789012345678 9
123456789012345678901234567890121234567890123456789012345678 99
23456789012345678901234567890121234567890123456789012345678
123456789012345678901234567890121234567890123456789012345678 9
1
123456789012345678901234567890121234567890123456789012345678 9
123456789012345678901234567890121234567890123456789012345678 9
123456789012345678901234567890121234567890123456789012345678 9
123456789012345678901234567890121234567890123456789012345678 9
123456789012345678901234567890121234567890123456789012345678 9
123456789012345678901234567890121234567890123456789012345678 9
123456789012345678901234567890121234567890123456789012345678 9
123456789012345678901234567890121234567890123456789012345678 99
23456789012345678901234567890121234567890123456789012345678
123456789012345678901234567890121234567890123456789012345678 9
123456789012345678901234567890121234567890123456789012345678 9
123456789012345678901234567890121234567890123456789012345678 9
1
123456789012345678901234567890121234567890123456789012345678 9
123456789012345678901234567890121234567890123456789012345678 9
123456789012345678901234567890121234567890123456789012345678 9
123456789012345678901234567890121234567890123456789012345678 9
123456789012345678901234567890121234567890123456789012345678 9
123456789012345678901234567890121234567890123456789012345678 99
23456789012345678901234567890121234567890123456789012345678
1
123456789012345678901234567890121234567890123456789012345678 9
123456789012345678901234567890121234567890123456789012345678 9
123456789012345678901234567890121234567890123456789012345678 9
123456789012345678901234567890121234567890123456789012345678 9
123456789012345678901234567890121234567890123456789012345678 9
123456789012345678901234567890121234567890123456789012345678 9
123456789012345678901234567890121234567890123456789012345678 9
1234567890123456789012345678901212345678901234567890123456789
Copyright © 2005 Thomson Peterson’s, a part of The Thomson Corporaton 71
SAT is a registered trademark of the College Entrance Examination Board, which
was not involved in the production of and does not endorse this product.
1234567890123456789012345678901212345678901234567890123456789
123456789012345678901234567890121234567890123456789012345678 9
123456789012345678901234567890121234567890123456789012345678 9
123456789012345678901234567890121234567890123456789012345678 9
123456789012345678901234567890121234567890123456789012345678 9
123456789012345678901234567890121234567890123456789012345678
13. D Since the triangles are congruent, you know that the bottom right angle 9
123456789012345678901234567890121234567890123456789012345678 9
123456789012345678901234567890121234567890123456789012345678
on the left triangle is a right angle. You
23456789012345678901234567890121234567890123456789012345678
also know that the side opposite 9
9
1
123456789012345678901234567890121234567890123456789012345678
x=3 9
123456789012345678901234567890121234567890123456789012345678
of b is , and from this you can deduce that the side adjacent to b 9
123456789012345678901234567890121234567890123456789012345678
2 9
123456789012345678901234567890121234567890123456789012345678
23456789012345678901234567890121234567890123456789012345678
9
9
1 x
123456789012345678901234567890121234567890123456789012345678
is by using the Pythagorean theorem. Here’s what the computation 9
123456789012345678901234567890121234567890123456789012345678
2 9
123456789012345678901234567890121234567890123456789012345678
would look like: 9
123456789012345678901234567890121234567890123456789012345678
23456789012345678901234567890121234567890123456789012345678
9
9
1
123456789012345678901234567890121234567890123456789012345678
a 2
b 2
c 2 9
123456789012345678901234567890121234567890123456789012345678
1 5 9
123456789012345678901234567890121234567890123456789012345678 9
1 = S D
3x
2
123456789012345678901234567890121234567890123456789012345678
23456789012345678901234567890121234567890123456789012345678
1b 5x2 2
123456789012345678901234567890121234567890123456789012345678
2
123456789012345678901234567890121234567890123456789012345678
9
9
9
9
123456789012345678901234567890121234567890123456789012345678 9
123456789012345678901234567890121234567890123456789012345678
3 2 2 2 9
23456789012345678901234567890121234567890123456789012345678
123456789012345678901234567890121234567890123456789012345678
x 1 b 5 x 9
123456789012345678901234567890121234567890123456789012345678
4 9
123456789012345678901234567890121234567890123456789012345678 9
123456789012345678901234567890121234567890123456789012345678
3 2 3 2 3 2 9
123456789012345678901234567890121234567890123456789012345678
x 2 x 1b 5x 2 x 2 2
9
123456789012345678901234567890121234567890123456789012345678 9
4 4 4
9
1
123456789012345678901234567890121234567890123456789012345678
x2 9
123456789012345678901234567890121234567890123456789012345678
2 9
123456789012345678901234567890121234567890123456789012345678
b 5 9
123456789012345678901234567890121234567890123456789012345678 9
23456789012345678901234567890121234567890123456789012345678
4
123456789012345678901234567890121234567890123456789012345678 9
123456789012345678901234567890121234567890123456789012345678
x 9
123456789012345678901234567890121234567890123456789012345678
b 5 9
123456789012345678901234567890121234567890123456789012345678 9
2
123456789012345678901234567890121234567890123456789012345678 9
123456789012345678901234567890121234567890123456789012345678 9
123456789012345678901234567890121234567890123456789012345678
The sides of this right triangle, then, are in the proportions of a 9
123456789012345678901234567890121234567890123456789012345678 9
1 30-60-90 triangle. And since angle b is opposite the second largest side, 9
23456789012345678901234567890121234567890123456789012345678
123456789012345678901234567890121234567890123456789012345678 9
123456789012345678901234567890121234567890123456789012345678
its measure is 60°, choice (D). 9
1 9
123456789012345678901234567890121234567890123456789012345678
14. C You can infer that the figure is a square since all the sides must be 9
123456789012345678901234567890121234567890123456789012345678
23456789012345678901234567890121234567890123456789012345678
9
9
123456789012345678901234567890121234567890123456789012345678
congruent because they are all formed by points equidistant from zero.
123456789012345678901234567890121234567890123456789012345678 9
1 To find the area of the square, you only need the length of one side. You 9
123456789012345678901234567890121234567890123456789012345678 9
123456789012345678901234567890121234567890123456789012345678
can get this two ways: you can use the distance formula for two points 9
23456789012345678901234567890121234567890123456789012345678
123456789012345678901234567890121234567890123456789012345678 9
123456789012345678901234567890121234567890123456789012345678
or you can take a shortcut and realize that the square is cut into four 9
1 9
123456789012345678901234567890121234567890123456789012345678
45-45-90 triangles by the y- and x-axes. The length of each non- 9
123456789012345678901234567890121234567890123456789012345678 9
123456789012345678901234567890121234567890123456789012345678 9
23456789012345678901234567890121234567890123456789012345678
hypotenuse side of these triangles is 2, so the hypotenuse must be
123456789012345678901234567890121234567890123456789012345678 9 9
123456789012345678901234567890121234567890123456789012345678
2=2. This hypotenuse is also the length of the square’s side, so place
9
123456789012345678901234567890121234567890123456789012345678 9
1 this value into the area of a square formula:
123456789012345678901234567890121234567890123456789012345678 9
23456789012345678901234567890121234567890123456789012345678
9
123456789012345678901234567890121234567890123456789012345678 9
123456789012345678901234567890121234567890123456789012345678
A 5 s2
9
123456789012345678901234567890121234567890123456789012345678 9
1 2
123456789012345678901234567890121234567890123456789012345678 9
23456789012345678901234567890121234567890123456789012345678
A 5 ~ 2= 2!
123456789012345678901234567890121234567890123456789012345678 9 9
123456789012345678901234567890121234567890123456789012345678
A543258 9
123456789012345678901234567890121234567890123456789012345678 9
1
123456789012345678901234567890121234567890123456789012345678 9
23456789012345678901234567890121234567890123456789012345678
Choice (C) is the answer.
9
1
123456789012345678901234567890121234567890123456789012345678 9
123456789012345678901234567890121234567890123456789012345678 9
123456789012345678901234567890121234567890123456789012345678 9
123456789012345678901234567890121234567890123456789012345678 9
123456789012345678901234567890121234567890123456789012345678 9
123456789012345678901234567890121234567890123456789012345678 9
123456789012345678901234567890121234567890123456789012345678 9
1234567890123456789012345678901212345678901234567890123456789
72 Copyright © 2005 Thomson Peterson’s, a part of The Thomson Corporaton
SAT is a registered trademark of the College Entrance Examination Board, which
was not involved in the production of and does not endorse this product.
1234567890123456789012345678901212345678901234567890123456789
123456789012345678901234567890121234567890123456789012345678 9
123456789012345678901234567890121234567890123456789012345678 9
123456789012345678901234567890121234567890123456789012345678 9
123456789012345678901234567890121234567890123456789012345678 9
123456789012345678901234567890121234567890123456789012345678
15. E The smallest percentage difference is going to be the year where the flu 9
123456789012345678901234567890121234567890123456789012345678 9
123456789012345678901234567890121234567890123456789012345678
column and the cold column are closest.
23456789012345678901234567890121234567890123456789012345678
(The converse is not true for 9
9
1
123456789012345678901234567890121234567890123456789012345678
the greatest percentage difference.) Eyeing the chart is the fastest way to 9
123456789012345678901234567890121234567890123456789012345678 9
123456789012345678901234567890121234567890123456789012345678
determine the answer. The values in the columns for 2000 look and are 9
123456789012345678901234567890121234567890123456789012345678
23456789012345678901234567890121234567890123456789012345678
9
9
1 the closest, and so the percentage difference is the least. (E) is
123456789012345678901234567890121234567890123456789012345678 9
123456789012345678901234567890121234567890123456789012345678
the answer. 9
123456789012345678901234567890121234567890123456789012345678 9
123456789012345678901234567890121234567890123456789012345678
16. D Cold cases (white bar) have to be greater than flu cases (shaded bar), so
23456789012345678901234567890121234567890123456789012345678
9
9
1
123456789012345678901234567890121234567890123456789012345678
there are only two decades that could be the correct answer, 1970 and 9
123456789012345678901234567890121234567890123456789012345678 9
123456789012345678901234567890121234567890123456789012345678
1990, choices (B) and (D). In 1970, the number of cold cases is about 9
123456789012345678901234567890121234567890123456789012345678
23456789012345678901234567890121234567890123456789012345678
9
9
1 twice as many as the flu (2,000 to 1,000), so the percentage difference
123456789012345678901234567890121234567890123456789012345678 9
123456789012345678901234567890121234567890123456789012345678
would be 100%. This makes 1990, choice (D), the answer by default, 9
123456789012345678901234567890121234567890123456789012345678 9
123456789012345678901234567890121234567890123456789012345678
but you can see that the percentage difference works out to about 25% 9
23456789012345678901234567890121234567890123456789012345678
123456789012345678901234567890121234567890123456789012345678 9
123456789012345678901234567890121234567890123456789012345678
since there were roughly 4,000 cold cases and 3,000 flu cases. 9
123456789012345678901234567890121234567890123456789012345678 99
123456789012345678901234567890121234567890123456789012345678
123456789012345678901234567890121234567890123456789012345678
17. A You may want to save Roman numeral questions for the second pass! A 9
123456789012345678901234567890121234567890123456789012345678 9
1 judicious use of POE will help you out on this one, though. Looking at 9
123456789012345678901234567890121234567890123456789012345678 9
123456789012345678901234567890121234567890123456789012345678
I, angles 6 and 12 are both formed by the same intersecting line, and 9
123456789012345678901234567890121234567890123456789012345678 9
123456789012345678901234567890121234567890123456789012345678 99
23456789012345678901234567890121234567890123456789012345678
they are congruent. Since I must be true, you can take this information
123456789012345678901234567890121234567890123456789012345678
123456789012345678901234567890121234567890123456789012345678
and cross out any answer choices that do not have I in them. 9
123456789012345678901234567890121234567890123456789012345678
Surprisingly, this gets rid (B), (C), and (D). The answer is either (A) (I 9
123456789012345678901234567890121234567890123456789012345678 99
123456789012345678901234567890121234567890123456789012345678
only) or (E) (I and II only). Since these are the only choices, you do not
123456789012345678901234567890121234567890123456789012345678 9
123456789012345678901234567890121234567890123456789012345678
even have to worry about III. Check II, and if it must be true, then the 9
123456789012345678901234567890121234567890123456789012345678 99
1 answer is (E). If it isn’t true, the answer must be (A). As it turns out, 2
123456789012345678901234567890121234567890123456789012345678 9
123456789012345678901234567890121234567890123456789012345678
and 9 are not congruent, so the answer is choice (A). 9
123456789012345678901234567890121234567890123456789012345678 9
123456789012345678901234567890121234567890123456789012345678 9
123456789012345678901234567890121234567890123456789012345678
18. A This problem looks rather complicated, but the negative in front of the
23456789012345678901234567890121234567890123456789012345678
9
9
123456789012345678901234567890121234567890123456789012345678
123456789012345678901234567890121234567890123456789012345678 9
2c
a simplifies it. If 2a2b is positive, then a must be negative and raised
1 9
123456789012345678901234567890121234567890123456789012345678
to an odd power. To get your head around this, consider that if a were 9
123456789012345678901234567890121234567890123456789012345678 9
123456789012345678901234567890121234567890123456789012345678 99
23456789012345678901234567890121234567890123456789012345678
positive, the entire expression could not be positive. So (A) must
123456789012345678901234567890121234567890123456789012345678 9
1 be true.
123456789012345678901234567890121234567890123456789012345678 9
123456789012345678901234567890121234567890123456789012345678 9
123456789012345678901234567890121234567890123456789012345678
19. D The first thing to recognize here is that the figure is composed of three 9
123456789012345678901234567890121234567890123456789012345678
triangles, and since all the line segments are congruent, all the triangles 9
123456789012345678901234567890121234567890123456789012345678 9
123456789012345678901234567890121234567890123456789012345678
are equilateral and congruent to each other. Equilateral triangles are all 9
123456789012345678901234567890121234567890123456789012345678 9
23456789012345678901234567890121234567890123456789012345678
123456789012345678901234567890121234567890123456789012345678
60-60-60 triangles, and so both a and b are 60. Choice (D) is true. 9
123456789012345678901234567890121234567890123456789012345678 9
123456789012345678901234567890121234567890123456789012345678 9
123456789012345678901234567890121234567890123456789012345678
20. C Since the boys cannot sit by seat 3, both girls always have to be sitting in 9
1 9
123456789012345678901234567890121234567890123456789012345678 99
23456789012345678901234567890121234567890123456789012345678
seat 2 and 4. The boys have to be in seats 1 and 5.
123456789012345678901234567890121234567890123456789012345678 9
123456789012345678901234567890121234567890123456789012345678 9
123456789012345678901234567890121234567890123456789012345678
If the first girl is in seat 2, then the second is in seat 4. If the first boy is
9
1
123456789012345678901234567890121234567890123456789012345678 99
23456789012345678901234567890121234567890123456789012345678
in seat 1 the second boy is in seat 5. That is one arrangement. The boys
1
123456789012345678901234567890121234567890123456789012345678
could switch seats, and that is a second arrangement. The girls could 9
123456789012345678901234567890121234567890123456789012345678 9
123456789012345678901234567890121234567890123456789012345678
also switch seats, and this would yield
23456789012345678901234567890121234567890123456789012345678
another two possible arrange- 9
9
1
123456789012345678901234567890121234567890123456789012345678
ments. That gives four total, (C). 9
123456789012345678901234567890121234567890123456789012345678 99
123456789012345678901234567890121234567890123456789012345678
1234567890123456789012345678901212345678901234567890123456789
Copyright © 2005 Thomson Peterson’s, a part of The Thomson Corporaton 73
SAT is a registered trademark of the College Entrance Examination Board, which
was not involved in the production of and does not endorse this product.
1234567890123456789012345678901212345678901234567890123456789
123456789012345678901234567890121234567890123456789012345678 9
123456789012345678901234567890121234567890123456789012345678 9
123456789012345678901234567890121234567890123456789012345678 9
123456789012345678901234567890121234567890123456789012345678 9
123456789012345678901234567890121234567890123456789012345678
21. E There is a little formula that can be used, and you can draw up a Venn 9
123456789012345678901234567890121234567890123456789012345678 9
123456789012345678901234567890121234567890123456789012345678
diagram if you like, but if you keep
23456789012345678901234567890121234567890123456789012345678
your head you can reason your way 9
9
1
123456789012345678901234567890121234567890123456789012345678
through this problem. If 52 students are in neither class, then 98 9
123456789012345678901234567890121234567890123456789012345678 9
123456789012345678901234567890121234567890123456789012345678
students are in algebra or geometry. The question states that “73 are in 9
123456789012345678901234567890121234567890123456789012345678
23456789012345678901234567890121234567890123456789012345678
9
9
1 geometry and 62 are in algebra,” so that would be a total of 73 1 62 5
123456789012345678901234567890121234567890123456789012345678 9
123456789012345678901234567890121234567890123456789012345678
135 students. Since this number is greater than the number (98) of 9
123456789012345678901234567890121234567890123456789012345678 9
123456789012345678901234567890121234567890123456789012345678
students you found earlier, the difference
23456789012345678901234567890121234567890123456789012345678
between 135 and 98 must be 9
9
1 the number of students enrolled in both algebra and geometry. This
123456789012345678901234567890121234567890123456789012345678 9
123456789012345678901234567890121234567890123456789012345678
difference is 37 (135 2 98 5 37), choice (E). 9
123456789012345678901234567890121234567890123456789012345678 9
123456789012345678901234567890121234567890123456789012345678
23456789012345678901234567890121234567890123456789012345678
9
9
1
123456789012345678901234567890121234567890123456789012345678 9
123456789012345678901234567890121234567890123456789012345678
Section 6 9
123456789012345678901234567890121234567890123456789012345678 9
123456789012345678901234567890121234567890123456789012345678
1. B In reference to the discovery of DNA, the passage states, the study of 9
23456789012345678901234567890121234567890123456789012345678
123456789012345678901234567890121234567890123456789012345678 9
123456789012345678901234567890121234567890123456789012345678
life could now be performed with more abstract methods of analysis. 9
123456789012345678901234567890121234567890123456789012345678 9
123456789012345678901234567890121234567890123456789012345678
(B) makes the same point, using the same key word, while all the other 9
123456789012345678901234567890121234567890123456789012345678 9
123456789012345678901234567890121234567890123456789012345678 9
choices either go beyond what the passage actually states (e.g. (A), the
9
1
123456789012345678901234567890121234567890123456789012345678
passage does not say that the study of lipids and proteins became 9
123456789012345678901234567890121234567890123456789012345678 9
123456789012345678901234567890121234567890123456789012345678
irrelevant; (C) basically says the same thing) or bring in topics not 9
23456789012345678901234567890121234567890123456789012345678
123456789012345678901234567890121234567890123456789012345678 9
123456789012345678901234567890121234567890123456789012345678
mentioned in the passage (e.g. (D), Mendelian genetics is not mentioned 9
123456789012345678901234567890121234567890123456789012345678 9
123456789012345678901234567890121234567890123456789012345678 9
in the passage).
123456789012345678901234567890121234567890123456789012345678 9
123456789012345678901234567890121234567890123456789012345678 9
123456789012345678901234567890121234567890123456789012345678
2. E The passage contrasts “wet and dirty“ study of lipids and proteins with 9
123456789012345678901234567890121234567890123456789012345678
the information-based study of DNA. In this sense “wet and dirty“ 9
123456789012345678901234567890121234567890123456789012345678 9 9
1 involves intensive laboratory work with things like lipids and proteins.
123456789012345678901234567890121234567890123456789012345678 9
123456789012345678901234567890121234567890123456789012345678
It is not haphazard guessing (A). Choice (E) states this correctly, also 9
123456789012345678901234567890121234567890123456789012345678 9
123456789012345678901234567890121234567890123456789012345678
drawing on language in the passage (information-based). 9
123456789012345678901234567890121234567890123456789012345678
23456789012345678901234567890121234567890123456789012345678
9
9
123456789012345678901234567890121234567890123456789012345678
123456789012345678901234567890121234567890123456789012345678
3. D The passage says that Saunders was a writer on Western topics who was 9
1 9
123456789012345678901234567890121234567890123456789012345678
widely read in the past. (D) fits with this. (A) does not. The passage does 9
123456789012345678901234567890121234567890123456789012345678 9
123456789012345678901234567890121234567890123456789012345678 9
23456789012345678901234567890121234567890123456789012345678
not say anything like (B), (C), or (E). Thus (D) is the best choice.
123456789012345678901234567890121234567890123456789012345678 9 9
1
123456789012345678901234567890121234567890123456789012345678
4. A The passage reads, many of the groups who popularized rock and roll 9
123456789012345678901234567890121234567890123456789012345678 9
123456789012345678901234567890121234567890123456789012345678 9
23456789012345678901234567890121234567890123456789012345678
consciously were attempting to emulate the work of blues greats such as
9
123456789012345678901234567890121234567890123456789012345678 9
123456789012345678901234567890121234567890123456789012345678
B. B. King. Choice (A) is an accurate paraphrase of the information
9
123456789012345678901234567890121234567890123456789012345678 9
1 given in the passage.
123456789012345678901234567890121234567890123456789012345678 9
23456789012345678901234567890121234567890123456789012345678
9
123456789012345678901234567890121234567890123456789012345678
5. C Antecedent means coming before. Even if you don’t know the word,
123456789012345678901234567890121234567890123456789012345678 9 9
123456789012345678901234567890121234567890123456789012345678
you can use word analysis to figure out that it has something to do with
9
1
123456789012345678901234567890121234567890123456789012345678 9
23456789012345678901234567890121234567890123456789012345678
before. Don’t confuse ante- with anti- (against). You can eliminate (D)
9
123456789012345678901234567890121234567890123456789012345678 9
123456789012345678901234567890121234567890123456789012345678
and (E). Referent is a synonym for antecedent only in grammatical
9
123456789012345678901234567890121234567890123456789012345678 9
1 usage. Eliminate (B). You are left with antebellum, meaning before the
23456789012345678901234567890121234567890123456789012345678
123456789012345678901234567890121234567890123456789012345678 9
1 war, or causal, meaning causing. Causal is more specific and more 9
123456789012345678901234567890121234567890123456789012345678 9
123456789012345678901234567890121234567890123456789012345678 9
logical. (C) is the answer.
123456789012345678901234567890121234567890123456789012345678 9
123456789012345678901234567890121234567890123456789012345678 9
123456789012345678901234567890121234567890123456789012345678 9
123456789012345678901234567890121234567890123456789012345678 9
123456789012345678901234567890121234567890123456789012345678 9
1234567890123456789012345678901212345678901234567890123456789
74 Copyright © 2005 Thomson Peterson’s, a part of The Thomson Corporaton
SAT is a registered trademark of the College Entrance Examination Board, which
was not involved in the production of and does not endorse this product.
1234567890123456789012345678901212345678901234567890123456789
123456789012345678901234567890121234567890123456789012345678 9
123456789012345678901234567890121234567890123456789012345678 9
123456789012345678901234567890121234567890123456789012345678 9
123456789012345678901234567890121234567890123456789012345678 9
123456789012345678901234567890121234567890123456789012345678
6. C The passage begins by describing the Seneca Falls convention as the first 9
123456789012345678901234567890121234567890123456789012345678 9
123456789012345678901234567890121234567890123456789012345678
organized attempt for woman’s voting
23456789012345678901234567890121234567890123456789012345678
rights (read the first sentences of 9
9
1
123456789012345678901234567890121234567890123456789012345678
the passage to see this). AWSA and NWSA came after Seneca Falls. (D) 9
123456789012345678901234567890121234567890123456789012345678 9
123456789012345678901234567890121234567890123456789012345678
and (E) refer to information provided in Passage 2. Choice (C) is 9
123456789012345678901234567890121234567890123456789012345678
23456789012345678901234567890121234567890123456789012345678
9
9
1 the answer.
123456789012345678901234567890121234567890123456789012345678 9
123456789012345678901234567890121234567890123456789012345678
7. A The passage states that the Seneca Falls conference did not have an 9
123456789012345678901234567890121234567890123456789012345678 9
123456789012345678901234567890121234567890123456789012345678
immediate effect because the nation became embroiled in issues related
23456789012345678901234567890121234567890123456789012345678
9
9
1
123456789012345678901234567890121234567890123456789012345678
to the coming Civil War. Knowledge of the dates of the Civil War will 9
123456789012345678901234567890121234567890123456789012345678 9
123456789012345678901234567890121234567890123456789012345678
help you avoid confusing it with World War I. It was the Civil War, (A), 9
123456789012345678901234567890121234567890123456789012345678
23456789012345678901234567890121234567890123456789012345678
9
9
1 that pushed the woman’s voting rights movement to the background of
123456789012345678901234567890121234567890123456789012345678 9
123456789012345678901234567890121234567890123456789012345678
the national consciousness. 9
123456789012345678901234567890121234567890123456789012345678 9
123456789012345678901234567890121234567890123456789012345678 9
123456789012345678901234567890121234567890123456789012345678 99
23456789012345678901234567890121234567890123456789012345678
8. B Here it is best to go back to the passage and clarify in your own mind
123456789012345678901234567890121234567890123456789012345678
123456789012345678901234567890121234567890123456789012345678
the distinctions drawn between the NWSA and the AWSA. The NWSA, 9
123456789012345678901234567890121234567890123456789012345678 9
123456789012345678901234567890121234567890123456789012345678 99
which the question asks about, focused their efforts on federal and
123456789012345678901234567890121234567890123456789012345678 9
1 constitutional issues, whereas the AWSA focused on state-level issues.
123456789012345678901234567890121234567890123456789012345678 9
123456789012345678901234567890121234567890123456789012345678
So (B) is the correct answer. 9
123456789012345678901234567890121234567890123456789012345678 9
23456789012345678901234567890121234567890123456789012345678
123456789012345678901234567890121234567890123456789012345678
9. B Don’t be distracted by the wrong answers. Without even reading the 9
123456789012345678901234567890121234567890123456789012345678 9
123456789012345678901234567890121234567890123456789012345678
passage, you could guess that the conflict between two organizations 9
123456789012345678901234567890121234567890123456789012345678 99
123456789012345678901234567890121234567890123456789012345678
was resolved when they combined. The answer is (B).
123456789012345678901234567890121234567890123456789012345678 9
123456789012345678901234567890121234567890123456789012345678 9
123456789012345678901234567890121234567890123456789012345678
10. D The author of Passage 1 doesn’t have a very strong opinion, so you can 9
123456789012345678901234567890121234567890123456789012345678 9
1 eliminate (A) and (E). Now you need to decide if the author’s opinion, 9
23456789012345678901234567890121234567890123456789012345678
123456789012345678901234567890121234567890123456789012345678 9
123456789012345678901234567890121234567890123456789012345678
however subtle, is positive or negative. It seems positive. For example, 9
1 9
123456789012345678901234567890121234567890123456789012345678 9
the author describes the work of the NAWSA as important. Thus the
123456789012345678901234567890121234567890123456789012345678 9
123456789012345678901234567890121234567890123456789012345678
correct answer will be positive. That eliminates (B) and (C). The answer 9
123456789012345678901234567890121234567890123456789012345678 9
123456789012345678901234567890121234567890123456789012345678 9
is (D).
123456789012345678901234567890121234567890123456789012345678 9
123456789012345678901234567890121234567890123456789012345678 9
123456789012345678901234567890121234567890123456789012345678 99
23456789012345678901234567890121234567890123456789012345678
11. A The first question is to determine what the “ultimate victory of the
123456789012345678901234567890121234567890123456789012345678
woman’s suffrage movement“ is. The first paragraph of the second 9
1
123456789012345678901234567890121234567890123456789012345678
passage makes it clear the author views the passing of the amendment 9
123456789012345678901234567890121234567890123456789012345678 9
23456789012345678901234567890121234567890123456789012345678
123456789012345678901234567890121234567890123456789012345678
to the constitution as the “ultimate victory,” and this occurred with 9
123456789012345678901234567890121234567890123456789012345678 9
123456789012345678901234567890121234567890123456789012345678
Tennessee approving the amendment. So (A) is the answer. 9
123456789012345678901234567890121234567890123456789012345678 99
1 12. C The earliest time that the second passage points to is the 1870s (the first
123456789012345678901234567890121234567890123456789012345678 9
123456789012345678901234567890121234567890123456789012345678 9
123456789012345678901234567890121234567890123456789012345678
passage refers to the Seneca Falls convention in 1848), and so (C) is 9
123456789012345678901234567890121234567890123456789012345678 9
123456789012345678901234567890121234567890123456789012345678 9
the answer.
123456789012345678901234567890121234567890123456789012345678 99
23456789012345678901234567890121234567890123456789012345678
123456789012345678901234567890121234567890123456789012345678 9
123456789012345678901234567890121234567890123456789012345678
13. C The second passage describes “partial suffrage“ as the right to, “vote in
9
123456789012345678901234567890121234567890123456789012345678 9
1 local affairs such as municipal elections, school board elections, or
23456789012345678901234567890121234567890123456789012345678
123456789012345678901234567890121234567890123456789012345678 9
1 prohibition measures.” All of the examples but (C) refer to local affairs. 9
123456789012345678901234567890121234567890123456789012345678 9
123456789012345678901234567890121234567890123456789012345678 9
Choice (C) refers to something mentioned only in Passage 1 (which was,
123456789012345678901234567890121234567890123456789012345678
moreover, before the 1848 Seneca Falls convention). 9
123456789012345678901234567890121234567890123456789012345678 9
123456789012345678901234567890121234567890123456789012345678 9
123456789012345678901234567890121234567890123456789012345678 9
123456789012345678901234567890121234567890123456789012345678 9
1234567890123456789012345678901212345678901234567890123456789
Copyright © 2005 Thomson Peterson’s, a part of The Thomson Corporaton 75
SAT is a registered trademark of the College Entrance Examination Board, which
was not involved in the production of and does not endorse this product.
1234567890123456789012345678901212345678901234567890123456789
123456789012345678901234567890121234567890123456789012345678 9
123456789012345678901234567890121234567890123456789012345678 9
123456789012345678901234567890121234567890123456789012345678 9
123456789012345678901234567890121234567890123456789012345678 9
123456789012345678901234567890121234567890123456789012345678
14. B The second passage argues that the “partial suffrages“ showed that 9
123456789012345678901234567890121234567890123456789012345678 9
123456789012345678901234567890121234567890123456789012345678
woman could “responsibly and reasonably
23456789012345678901234567890121234567890123456789012345678
participate in a representa- 9
9
1
123456789012345678901234567890121234567890123456789012345678
tive democracy.” These examples made the reasoning of nonsuffragists, 9
123456789012345678901234567890121234567890123456789012345678 9
123456789012345678901234567890121234567890123456789012345678
that woman were not fit to vote, difficult to maintain. Choice (B) 9
123456789012345678901234567890121234567890123456789012345678
23456789012345678901234567890121234567890123456789012345678
9
9
1 correctly points to this same idea.
123456789012345678901234567890121234567890123456789012345678 9
123456789012345678901234567890121234567890123456789012345678
15. E To answer a question like this you have to clearly have in mind which 9
123456789012345678901234567890121234567890123456789012345678 9
123456789012345678901234567890121234567890123456789012345678
passage discusses what. Neither passage mentions the number of the
23456789012345678901234567890121234567890123456789012345678
9
9
1
123456789012345678901234567890121234567890123456789012345678
amendment that gave women the right to vote. The second passage 9
123456789012345678901234567890121234567890123456789012345678 9
123456789012345678901234567890121234567890123456789012345678
mentions when the amendment became law in the first paragraph, so 9
123456789012345678901234567890121234567890123456789012345678
23456789012345678901234567890121234567890123456789012345678
9
9
1 you can eliminate (A). The first passage mentions the Civil War in the
123456789012345678901234567890121234567890123456789012345678 9
123456789012345678901234567890121234567890123456789012345678
first paragraph. The first passage mentions the leaders of NWSA in the 9
123456789012345678901234567890121234567890123456789012345678 9
123456789012345678901234567890121234567890123456789012345678
second paragraph. The second passage is all about “partial suffrages,” 9
23456789012345678901234567890121234567890123456789012345678
123456789012345678901234567890121234567890123456789012345678 9
123456789012345678901234567890121234567890123456789012345678
and you’ve confirmed this in question 14. The only possible answer 9
123456789012345678901234567890121234567890123456789012345678 9
123456789012345678901234567890121234567890123456789012345678 9
is (E).
123456789012345678901234567890121234567890123456789012345678 9
123456789012345678901234567890121234567890123456789012345678 9
1 16. A Recall that the author of the second passage saw the “partial 9
123456789012345678901234567890121234567890123456789012345678 9
123456789012345678901234567890121234567890123456789012345678
suffrages,” the local successes of the woman’s movement in the 1870s 9
123456789012345678901234567890121234567890123456789012345678 9
123456789012345678901234567890121234567890123456789012345678 9
23456789012345678901234567890121234567890123456789012345678
and 1880s, as crucial to the ultimate success of the movement. The first
123456789012345678901234567890121234567890123456789012345678 9
123456789012345678901234567890121234567890123456789012345678
passage says that AWSA focused on state-level issues. Choice (A) states 9
123456789012345678901234567890121234567890123456789012345678
as much and is the best choice. NWSA was specifically interested in 9
123456789012345678901234567890121234567890123456789012345678 9
123456789012345678901234567890121234567890123456789012345678
federal and constitutional issues, so (B) is factually incorrect. The 9
123456789012345678901234567890121234567890123456789012345678 9
123456789012345678901234567890121234567890123456789012345678
author of passage two agrees that the constitutional amendment was 9
123456789012345678901234567890121234567890123456789012345678 9 9
1 the ultimate success of the suffrage movement, so he or she would not
123456789012345678901234567890121234567890123456789012345678 9
123456789012345678901234567890121234567890123456789012345678
agree with (C). (D) also contradicts that view. (E) is more difficult to 9
123456789012345678901234567890121234567890123456789012345678 9
123456789012345678901234567890121234567890123456789012345678
eliminate, but the author of Passage 2 doesn’t mention any differences 9
123456789012345678901234567890121234567890123456789012345678 9
123456789012345678901234567890121234567890123456789012345678 9
23456789012345678901234567890121234567890123456789012345678
within the suffrage movement. Choice (A) is the best answer.
123456789012345678901234567890121234567890123456789012345678 9 9
1
123456789012345678901234567890121234567890123456789012345678 9
123456789012345678901234567890121234567890123456789012345678 9
123456789012345678901234567890121234567890123456789012345678 9
23456789012345678901234567890121234567890123456789012345678
123456789012345678901234567890121234567890123456789012345678 9 9
1
123456789012345678901234567890121234567890123456789012345678 9
123456789012345678901234567890121234567890123456789012345678 9
123456789012345678901234567890121234567890123456789012345678 9
123456789012345678901234567890121234567890123456789012345678 9
123456789012345678901234567890121234567890123456789012345678 9
123456789012345678901234567890121234567890123456789012345678 9
123456789012345678901234567890121234567890123456789012345678 9
123456789012345678901234567890121234567890123456789012345678 9
23456789012345678901234567890121234567890123456789012345678
123456789012345678901234567890121234567890123456789012345678 9
123456789012345678901234567890121234567890123456789012345678 9
123456789012345678901234567890121234567890123456789012345678 9 9
1
123456789012345678901234567890121234567890123456789012345678 9
123456789012345678901234567890121234567890123456789012345678 9
123456789012345678901234567890121234567890123456789012345678 9
123456789012345678901234567890121234567890123456789012345678 9
123456789012345678901234567890121234567890123456789012345678 9
123456789012345678901234567890121234567890123456789012345678 9
23456789012345678901234567890121234567890123456789012345678
9
1
123456789012345678901234567890121234567890123456789012345678 9
123456789012345678901234567890121234567890123456789012345678 9
123456789012345678901234567890121234567890123456789012345678 9
123456789012345678901234567890121234567890123456789012345678 9
123456789012345678901234567890121234567890123456789012345678 9
123456789012345678901234567890121234567890123456789012345678 9
123456789012345678901234567890121234567890123456789012345678 9
1234567890123456789012345678901212345678901234567890123456789
76 Copyright © 2005 Thomson Peterson’s, a part of The Thomson Corporaton
SAT is a registered trademark of the College Entrance Examination Board, which
was not involved in the production of and does not endorse this product.
1234567890123456789012345678901212345678901234567890123456789
123456789012345678901234567890121234567890123456789012345678 9
123456789012345678901234567890121234567890123456789012345678 9
123456789012345678901234567890121234567890123456789012345678 9
123456789012345678901234567890121234567890123456789012345678 9
123456789012345678901234567890121234567890123456789012345678
Section 7 9
123456789012345678901234567890121234567890123456789012345678 9
123456789012345678901234567890121234567890123456789012345678
1. This one is all about PEMDAS. 9
123456789012345678901234567890121234567890123456789012345678 9
123456789012345678901234567890121234567890123456789012345678 9
123456789012345678901234567890121234567890123456789012345678
@ 2~32!2 1 ~4 2 3~4!!2# 5 2~9!2 1 ~4 2 12!2 9
123456789012345678901234567890121234567890123456789012345678 9
123456789012345678901234567890121234567890123456789012345678
@ ~ ! ~ !
23456789012345678901234567890121234567890123456789012345678
2
# 9
9
1 5 2 81 1 28 162 1 64
123456789012345678901234567890121234567890123456789012345678 9
123456789012345678901234567890121234567890123456789012345678
5 162 1 64 9
123456789012345678901234567890121234567890123456789012345678 9
123456789012345678901234567890121234567890123456789012345678
5 226
23456789012345678901234567890121234567890123456789012345678
9
9
1
123456789012345678901234567890121234567890123456789012345678 9
123456789012345678901234567890121234567890123456789012345678
2. Remember when you multiply or divide an inequality by a negative, you 9
123456789012345678901234567890121234567890123456789012345678 9
123456789012345678901234567890121234567890123456789012345678
switch the direction of the inequality sign:
23456789012345678901234567890121234567890123456789012345678
9
9
1
123456789012345678901234567890121234567890123456789012345678 9
123456789012345678901234567890121234567890123456789012345678
22 2 3x . 13 9
123456789012345678901234567890121234567890123456789012345678 9
123456789012345678901234567890121234567890123456789012345678
22 2 22 2 3x . 13 2 22 9
123456789012345678901234567890121234567890123456789012345678 99
23456789012345678901234567890121234567890123456789012345678
123456789012345678901234567890121234567890123456789012345678
123456789012345678901234567890121234567890123456789012345678
23x . 29 9
123456789012345678901234567890121234567890123456789012345678 9
123456789012345678901234567890121234567890123456789012345678
23x 29 9
123456789012345678901234567890121234567890123456789012345678
, 9
1 23 23 9
123456789012345678901234567890121234567890123456789012345678 9
123456789012345678901234567890121234567890123456789012345678
x,3 9
123456789012345678901234567890121234567890123456789012345678 9
23456789012345678901234567890121234567890123456789012345678
123456789012345678901234567890121234567890123456789012345678 9
123456789012345678901234567890121234567890123456789012345678
The only two positive integers less than 3 are 2 or 1. Either one is an 9
123456789012345678901234567890121234567890123456789012345678 9
123456789012345678901234567890121234567890123456789012345678
acceptable answer. 9
123456789012345678901234567890121234567890123456789012345678 99
123456789012345678901234567890121234567890123456789012345678
123456789012345678901234567890121234567890123456789012345678
3. The cost will include the initial charge and the minute rate times thirty, 0.40 9
123456789012345678901234567890121234567890123456789012345678 9
123456789012345678901234567890121234567890123456789012345678 99
1 0.06(30) 5 0.40 1 1.80 5 2.20.
1
123456789012345678901234567890121234567890123456789012345678 9
123456789012345678901234567890121234567890123456789012345678
4. A good, no-nonsense way to find the least common multiple (LCM) is to 9
123456789012345678901234567890121234567890123456789012345678
multiply both numbers by increasing positive integers until both multiply to 9
123456789012345678901234567890121234567890123456789012345678 9
123456789012345678901234567890121234567890123456789012345678
the same number. 9
123456789012345678901234567890121234567890123456789012345678 9
123456789012345678901234567890121234567890123456789012345678 9
123456789012345678901234567890121234567890123456789012345678 9
18 3 2 5 36 24 3 2 5 48
123456789012345678901234567890121234567890123456789012345678 9
123456789012345678901234567890121234567890123456789012345678
18 3 3 5 54 24 3 3 5 72 9
23456789012345678901234567890121234567890123456789012345678
123456789012345678901234567890121234567890123456789012345678 9
123456789012345678901234567890121234567890123456789012345678
18 3 4 5 72 24 3 4 5 96 9
1 9
123456789012345678901234567890121234567890123456789012345678 9
18 3 5 5 90 24 3 6 5 120
123456789012345678901234567890121234567890123456789012345678 9
123456789012345678901234567890121234567890123456789012345678
18 3 6 5 108 24 3 6 5 144 9
123456789012345678901234567890121234567890123456789012345678 9
123456789012345678901234567890121234567890123456789012345678
18 3 7 5 126 9
123456789012345678901234567890121234567890123456789012345678 9
18 3 8
123456789012345678901234567890121234567890123456789012345678 9
5 144
123456789012345678901234567890121234567890123456789012345678 99
23456789012345678901234567890121234567890123456789012345678
123456789012345678901234567890121234567890123456789012345678
Both numbers multiply out to 72, so this is the answer. Creating these two
9
123456789012345678901234567890121234567890123456789012345678 9
123456789012345678901234567890121234567890123456789012345678
tables would take a lot of time if you didn’t have a calculator, but since you
9
1
123456789012345678901234567890121234567890123456789012345678 99
23456789012345678901234567890121234567890123456789012345678
do, make the most of it.
123456789012345678901234567890121234567890123456789012345678 9
123456789012345678901234567890121234567890123456789012345678 9
123456789012345678901234567890121234567890123456789012345678 9
1
123456789012345678901234567890121234567890123456789012345678 9
123456789012345678901234567890121234567890123456789012345678 9
123456789012345678901234567890121234567890123456789012345678 9
123456789012345678901234567890121234567890123456789012345678 9
123456789012345678901234567890121234567890123456789012345678 9
123456789012345678901234567890121234567890123456789012345678 9
123456789012345678901234567890121234567890123456789012345678 9
123456789012345678901234567890121234567890123456789012345678 9
123456789012345678901234567890121234567890123456789012345678 9
1234567890123456789012345678901212345678901234567890123456789
Copyright © 2005 Thomson Peterson’s, a part of The Thomson Corporaton 77
SAT is a registered trademark of the College Entrance Examination Board, which
was not involved in the production of and does not endorse this product.
1234567890123456789012345678901212345678901234567890123456789
123456789012345678901234567890121234567890123456789012345678 9
123456789012345678901234567890121234567890123456789012345678 9
123456789012345678901234567890121234567890123456789012345678 9
123456789012345678901234567890121234567890123456789012345678 9
123456789012345678901234567890121234567890123456789012345678
5. If x is any negative number, then the inequality will hold true because the 9
123456789012345678901234567890121234567890123456789012345678 9
123456789012345678901234567890121234567890123456789012345678
absolute value bars will knock the value
23456789012345678901234567890121234567890123456789012345678
up well over 2. If you don’t see this, 9
9
1
123456789012345678901234567890121234567890123456789012345678
plug in some negative numbers. 9
123456789012345678901234567890121234567890123456789012345678 9
123456789012345678901234567890121234567890123456789012345678 9
123456789012345678901234567890121234567890123456789012345678
Thinking positively, if x . 5 then the
23456789012345678901234567890121234567890123456789012345678
inequality is also true. The only 9
9
1 remaining integers are 0–5. Plugging zero into the inequality, it still holds
123456789012345678901234567890121234567890123456789012345678 9
123456789012345678901234567890121234567890123456789012345678
true. But if you plug 1–5 in, the inequality is not true. Thus any integer 1, 2, 9
123456789012345678901234567890121234567890123456789012345678 9
123456789012345678901234567890121234567890123456789012345678
3, 4, 5 is correct.
23456789012345678901234567890121234567890123456789012345678
9
9
1
123456789012345678901234567890121234567890123456789012345678 9
123456789012345678901234567890121234567890123456789012345678
6. This problem works very much like a function machine. Take the number 9
123456789012345678901234567890121234567890123456789012345678 9
123456789012345678901234567890121234567890123456789012345678
they give you and place it into the equation,
23456789012345678901234567890121234567890123456789012345678
then solve. For the eleventh 9
9
1
123456789012345678901234567890121234567890123456789012345678
term, k will be equal to 11, so 9
123456789012345678901234567890121234567890123456789012345678 9
123456789012345678901234567890121234567890123456789012345678
10~k 1 1! 10~11 1 1! 10~12! 120 9
123456789012345678901234567890121234567890123456789012345678 9
23456789012345678901234567890121234567890123456789012345678
123456789012345678901234567890121234567890123456789012345678
5 5 5 5 12 9
123456789012345678901234567890121234567890123456789012345678
k21 11 2 1 10 10 9
123456789012345678901234567890121234567890123456789012345678 9
123456789012345678901234567890121234567890123456789012345678 9
7. If you do not see the answer by visualizing the points, you can always use the
123456789012345678901234567890121234567890123456789012345678 9
123456789012345678901234567890121234567890123456789012345678
formula for the midpoint. It is just the average of the coordinates (in this 9
1 9
123456789012345678901234567890121234567890123456789012345678
case, the y coordinates) 9
123456789012345678901234567890121234567890123456789012345678 9
123456789012345678901234567890121234567890123456789012345678
y1 1 y2 2 1 6 8 9
23456789012345678901234567890121234567890123456789012345678
123456789012345678901234567890121234567890123456789012345678
5 5 4. 9
123456789012345678901234567890121234567890123456789012345678
2
5
2 2 9
123456789012345678901234567890121234567890123456789012345678 9
123456789012345678901234567890121234567890123456789012345678 9
123456789012345678901234567890121234567890123456789012345678
8. You can use trigonometric functions to solve this problem, or you could use 9
123456789012345678901234567890121234567890123456789012345678 9
123456789012345678901234567890121234567890123456789012345678
the Pythagorean theorem since this is a right triangle (the measure of the 9
123456789012345678901234567890121234567890123456789012345678 9
123456789012345678901234567890121234567890123456789012345678 9
angles have to sum to 180 and you know the measures of two of the three
9
1
123456789012345678901234567890121234567890123456789012345678
angles, 40 and 50, sum to 90. This means the third angle must be 90.) 9
123456789012345678901234567890121234567890123456789012345678 9
123456789012345678901234567890121234567890123456789012345678 9
123456789012345678901234567890121234567890123456789012345678 9
2 2 2
a 1 b 5 c
123456789012345678901234567890121234567890123456789012345678 9
123456789012345678901234567890121234567890123456789012345678
42 1 ~2=5! 5 c2
2
9
123456789012345678901234567890121234567890123456789012345678 9
123456789012345678901234567890121234567890123456789012345678 9
123456789012345678901234567890121234567890123456789012345678
16 1 20 5 c 2
9
123456789012345678901234567890121234567890123456789012345678 9
23456789012345678901234567890121234567890123456789012345678
123456789012345678901234567890121234567890123456789012345678
36 5 c 2
9
123456789012345678901234567890121234567890123456789012345678 9 9
1
123456789012345678901234567890121234567890123456789012345678
65c 9
123456789012345678901234567890121234567890123456789012345678 9
123456789012345678901234567890121234567890123456789012345678 9
23456789012345678901234567890121234567890123456789012345678
The answer is 6.
123456789012345678901234567890121234567890123456789012345678 9 9
123456789012345678901234567890121234567890123456789012345678
9. The mean is the average, so we have to add up all the numbers and divide by
9
123456789012345678901234567890121234567890123456789012345678 9
1 the total:
123456789012345678901234567890121234567890123456789012345678 9
23456789012345678901234567890121234567890123456789012345678
9
123456789012345678901234567890121234567890123456789012345678 9
123456789012345678901234567890121234567890123456789012345678
7 1 4 1 3 1 6 1 2 1 2 24
9
123456789012345678901234567890121234567890123456789012345678
5 5 4.
9
1 6 6
123456789012345678901234567890121234567890123456789012345678 9
23456789012345678901234567890121234567890123456789012345678
9
123456789012345678901234567890121234567890123456789012345678 9
123456789012345678901234567890121234567890123456789012345678
The median is the number in the middle if you line up all the numbers from
9
123456789012345678901234567890121234567890123456789012345678 9
1 greatest to least. Since there is an even number of numbers, the median will
23456789012345678901234567890121234567890123456789012345678
123456789012345678901234567890121234567890123456789012345678 9
1 be the average of the two middle numbers, {2, 2, 3, 4, 6, 7}. The median then 9
123456789012345678901234567890121234567890123456789012345678 9
123456789012345678901234567890121234567890123456789012345678 1 9
123456789012345678901234567890121234567890123456789012345678
is the average of 3 and 4, or 3.5. The difference
23456789012345678901234567890121234567890123456789012345678
between the two is 0.5 or . 9
9
1 2
123456789012345678901234567890121234567890123456789012345678 9
123456789012345678901234567890121234567890123456789012345678 9
123456789012345678901234567890121234567890123456789012345678 9
1234567890123456789012345678901212345678901234567890123456789
78 Copyright © 2005 Thomson Peterson’s, a part of The Thomson Corporaton
SAT is a registered trademark of the College Entrance Examination Board, which
was not involved in the production of and does not endorse this product.
1234567890123456789012345678901212345678901234567890123456789
123456789012345678901234567890121234567890123456789012345678 9
123456789012345678901234567890121234567890123456789012345678 9
123456789012345678901234567890121234567890123456789012345678 9
123456789012345678901234567890121234567890123456789012345678 9
123456789012345678901234567890121234567890123456789012345678
10. Draw this picture! Two lines that are perpendicular to the same line are 9
123456789012345678901234567890121234567890123456789012345678 9
123456789012345678901234567890121234567890123456789012345678
parallel. To see this, draw one line perpendicular
23456789012345678901234567890121234567890123456789012345678
to another, and then draw 9
9
1
123456789012345678901234567890121234567890123456789012345678
a third line perpendicular to the second; it will be parallel to the first line). 9
123456789012345678901234567890121234567890123456789012345678 9
123456789012345678901234567890121234567890123456789012345678 2 9
123456789012345678901234567890121234567890123456789012345678
That means a and c are parallel, which
23456789012345678901234567890121234567890123456789012345678
implies they have the same slope, . 9
9
1 5
123456789012345678901234567890121234567890123456789012345678 9
123456789012345678901234567890121234567890123456789012345678
11. This might appear complicated, but a few careful substitutions and you will 9
123456789012345678901234567890121234567890123456789012345678 9
123456789012345678901234567890121234567890123456789012345678
have the answer. When, y 5 3, x 5 5 because
23456789012345678901234567890121234567890123456789012345678
x 5 y 1 2. 9
9
1
123456789012345678901234567890121234567890123456789012345678 9
123456789012345678901234567890121234567890123456789012345678
Now you need to plug this value of x into the function, 9
123456789012345678901234567890121234567890123456789012345678 9
123456789012345678901234567890121234567890123456789012345678
23456789012345678901234567890121234567890123456789012345678
9
9
1 f(x) 5 x 2
1 3
123456789012345678901234567890121234567890123456789012345678 9
123456789012345678901234567890121234567890123456789012345678
2
f~5! 5 5 1 3 5 25 1 3 5 28 9
123456789012345678901234567890121234567890123456789012345678 9
123456789012345678901234567890121234567890123456789012345678 9
123456789012345678901234567890121234567890123456789012345678 99
23456789012345678901234567890121234567890123456789012345678
12. The first step is to sketch the inscribed square.
123456789012345678901234567890121234567890123456789012345678 9
123456789012345678901234567890121234567890123456789012345678 9
123456789012345678901234567890121234567890123456789012345678 9
123456789012345678901234567890121234567890123456789012345678 9
123456789012345678901234567890121234567890123456789012345678 9
1
123456789012345678901234567890121234567890123456789012345678 9
123456789012345678901234567890121234567890123456789012345678 9
123456789012345678901234567890121234567890123456789012345678 9
123456789012345678901234567890121234567890123456789012345678 99
23456789012345678901234567890121234567890123456789012345678
123456789012345678901234567890121234567890123456789012345678 9
123456789012345678901234567890121234567890123456789012345678 9
123456789012345678901234567890121234567890123456789012345678 9
123456789012345678901234567890121234567890123456789012345678 9
123456789012345678901234567890121234567890123456789012345678 9
123456789012345678901234567890121234567890123456789012345678 9
123456789012345678901234567890121234567890123456789012345678 9
123456789012345678901234567890121234567890123456789012345678 9
1
123456789012345678901234567890121234567890123456789012345678 9
123456789012345678901234567890121234567890123456789012345678 9
123456789012345678901234567890121234567890123456789012345678
The sides of the square are =8 since the area is 8. You get this by working 9
123456789012345678901234567890121234567890123456789012345678 9
123456789012345678901234567890121234567890123456789012345678
backward from the area of a square formula: 9
123456789012345678901234567890121234567890123456789012345678 99
23456789012345678901234567890121234567890123456789012345678
123456789012345678901234567890121234567890123456789012345678 9
1 A 5 s2
123456789012345678901234567890121234567890123456789012345678 9
123456789012345678901234567890121234567890123456789012345678
8 s2 9
123456789012345678901234567890121234567890123456789012345678 99
23456789012345678901234567890121234567890123456789012345678
5
123456789012345678901234567890121234567890123456789012345678 9
1 =8 5 s
123456789012345678901234567890121234567890123456789012345678 9
123456789012345678901234567890121234567890123456789012345678
= 9
123456789012345678901234567890121234567890123456789012345678 99
23456789012345678901234567890121234567890123456789012345678
4 3 2 5 s
123456789012345678901234567890121234567890123456789012345678 9
123456789012345678901234567890121234567890123456789012345678
2=2 5 s 9
123456789012345678901234567890121234567890123456789012345678 9
1
23456789012345678901234567890121234567890123456789012345678
123456789012345678901234567890121234567890123456789012345678
The longest straight line that originates and terminates on the circumference 9
123456789012345678901234567890121234567890123456789012345678 9
123456789012345678901234567890121234567890123456789012345678
is the diagonal of the square. Now that you have the side of the square, you 9
123456789012345678901234567890121234567890123456789012345678 99
1 can find the length of its diagonal because two sides of the square and its
123456789012345678901234567890121234567890123456789012345678 9
123456789012345678901234567890121234567890123456789012345678
hypotenuse make a 45-45-90 right triangle. The diagonal is the hypotenuse, 9
123456789012345678901234567890121234567890123456789012345678 9
123456789012345678901234567890121234567890123456789012345678
= 9
123456789012345678901234567890121234567890123456789012345678 9
so it will be 2 times greater than a side.
123456789012345678901234567890121234567890123456789012345678 99
23456789012345678901234567890121234567890123456789012345678
1 This means the diagonal of the square is:
123456789012345678901234567890121234567890123456789012345678 9
123456789012345678901234567890121234567890123456789012345678 9
123456789012345678901234567890121234567890123456789012345678
~2=2!~=2! 5 2=4 5 2 3 2 5 4. 9
123456789012345678901234567890121234567890123456789012345678 9
123456789012345678901234567890121234567890123456789012345678 9
123456789012345678901234567890121234567890123456789012345678 9
123456789012345678901234567890121234567890123456789012345678 9
1234567890123456789012345678901212345678901234567890123456789
Copyright © 2005 Thomson Peterson’s, a part of The Thomson Corporaton 79
SAT is a registered trademark of the College Entrance Examination Board, which
was not involved in the production of and does not endorse this product.
1234567890123456789012345678901212345678901234567890123456789
123456789012345678901234567890121234567890123456789012345678 9
123456789012345678901234567890121234567890123456789012345678 9
123456789012345678901234567890121234567890123456789012345678 9
123456789012345678901234567890121234567890123456789012345678 9
123456789012345678901234567890121234567890123456789012345678
13. Our range of numbers to meet the conditions is 1000 to 1198. The only way 9
123456789012345678901234567890121234567890123456789012345678 9
123456789012345678901234567890121234567890123456789012345678
to meet the two conditions in the 1100’s is to
23456789012345678901234567890121234567890123456789012345678
have the ones and units sum to 9
9
1
123456789012345678901234567890121234567890123456789012345678
15 since with the two 1’s (in the hundreds and thousands place) will make 9
123456789012345678901234567890121234567890123456789012345678 9
123456789012345678901234567890121234567890123456789012345678
the sum 17. But this is impossible, since two numbers that are the same 9
123456789012345678901234567890121234567890123456789012345678
23456789012345678901234567890121234567890123456789012345678
9
9
1 cannot sum to an odd number. Therefore, the answer must be in the 1000’s.
123456789012345678901234567890121234567890123456789012345678 9
123456789012345678901234567890121234567890123456789012345678
1011 is far too less a sum of digits, and 1099 is too great, but not by much. 9
123456789012345678901234567890121234567890123456789012345678 9
123456789012345678901234567890121234567890123456789012345678
Try 1088. It sums to 17.
23456789012345678901234567890121234567890123456789012345678
9
9
1
123456789012345678901234567890121234567890123456789012345678 9
123456789012345678901234567890121234567890123456789012345678 9
123456789012345678901234567890121234567890123456789012345678
Section 8 9
123456789012345678901234567890121234567890123456789012345678
23456789012345678901234567890121234567890123456789012345678
9
9
1
123456789012345678901234567890121234567890123456789012345678
As you might expect, answers will vary. If possible, politely ask a teacher, 9
123456789012345678901234567890121234567890123456789012345678 9
123456789012345678901234567890121234567890123456789012345678
fellow student, or some other person knowledgeable about formal essay 9
123456789012345678901234567890121234567890123456789012345678
writing to review your essay and provide feedback on ways in which the 9
123456789012345678901234567890121234567890123456789012345678 9
23456789012345678901234567890121234567890123456789012345678
123456789012345678901234567890121234567890123456789012345678
essay is commendable and on areas where it could be improved. 9
123456789012345678901234567890121234567890123456789012345678 9
123456789012345678901234567890121234567890123456789012345678 9
123456789012345678901234567890121234567890123456789012345678 9
123456789012345678901234567890121234567890123456789012345678 9 9
1
123456789012345678901234567890121234567890123456789012345678 9
123456789012345678901234567890121234567890123456789012345678 9
123456789012345678901234567890121234567890123456789012345678 9
123456789012345678901234567890121234567890123456789012345678 9
23456789012345678901234567890121234567890123456789012345678
123456789012345678901234567890121234567890123456789012345678 9
123456789012345678901234567890121234567890123456789012345678 9
123456789012345678901234567890121234567890123456789012345678 9
123456789012345678901234567890121234567890123456789012345678 9
123456789012345678901234567890121234567890123456789012345678 9
123456789012345678901234567890121234567890123456789012345678 9
123456789012345678901234567890121234567890123456789012345678 9
123456789012345678901234567890121234567890123456789012345678 9 9
1
123456789012345678901234567890121234567890123456789012345678 9
123456789012345678901234567890121234567890123456789012345678 9
123456789012345678901234567890121234567890123456789012345678 9
123456789012345678901234567890121234567890123456789012345678 9
123456789012345678901234567890121234567890123456789012345678 9
123456789012345678901234567890121234567890123456789012345678 9
123456789012345678901234567890121234567890123456789012345678 9
123456789012345678901234567890121234567890123456789012345678 9
123456789012345678901234567890121234567890123456789012345678 9
123456789012345678901234567890121234567890123456789012345678 9
123456789012345678901234567890121234567890123456789012345678 9
23456789012345678901234567890121234567890123456789012345678
123456789012345678901234567890121234567890123456789012345678 9 9
1
123456789012345678901234567890121234567890123456789012345678 9
123456789012345678901234567890121234567890123456789012345678 9
123456789012345678901234567890121234567890123456789012345678 9
123456789012345678901234567890121234567890123456789012345678 9
123456789012345678901234567890121234567890123456789012345678 9
123456789012345678901234567890121234567890123456789012345678 9
123456789012345678901234567890121234567890123456789012345678 9
123456789012345678901234567890121234567890123456789012345678 9
23456789012345678901234567890121234567890123456789012345678
123456789012345678901234567890121234567890123456789012345678 9
123456789012345678901234567890121234567890123456789012345678 9
123456789012345678901234567890121234567890123456789012345678 9 9
1
123456789012345678901234567890121234567890123456789012345678 9
123456789012345678901234567890121234567890123456789012345678 9
123456789012345678901234567890121234567890123456789012345678 9
123456789012345678901234567890121234567890123456789012345678 9
123456789012345678901234567890121234567890123456789012345678 9
123456789012345678901234567890121234567890123456789012345678 9
23456789012345678901234567890121234567890123456789012345678
9
1
123456789012345678901234567890121234567890123456789012345678 9
123456789012345678901234567890121234567890123456789012345678 9
123456789012345678901234567890121234567890123456789012345678 9
123456789012345678901234567890121234567890123456789012345678 9
123456789012345678901234567890121234567890123456789012345678 9
123456789012345678901234567890121234567890123456789012345678 9
123456789012345678901234567890121234567890123456789012345678 9
1234567890123456789012345678901212345678901234567890123456789
80 Copyright © 2005 Thomson Peterson’s, a part of The Thomson Corporaton
SAT is a registered trademark of the College Entrance Examination Board, which
was not involved in the production of and does not endorse this product.
12345678901234567890123456789012123456789012345678901234567890121234567
1234567890123456789012345678901212345678901234567890123456789012123456 7
12234567890123456789012345678901212345678901234567890123456789012123456 7
1 34567890123456789012345678901212345678901234567890123456789012123456 7
12234567890123456789012345678901212345678901234567890123456789012123456 7
1234567890123456789012345678901212345678901234567890123456789012123456 7
Scoring Worksheet
1 34567890123456789012345678901212345678901234567890123456789012123456 7
12234567890123456789012345678901212345678901234567890123456789012123456
1 34567890123456789012345678901212345678901234567890123456789012123456 7
7
12234567890123456789012345678901212345678901234567890123456789012123456 7
1234567890123456789012345678901212345678901234567890123456789012123456 7
1234567890123456789012345678901212345678901234567890123456789012123456
MATH 7
1234567890123456789012345678901212345678901234567890123456789012123456
34567890123456789012345678901212345678901234567890123456789012123456
7
7
1
12234567890123456789012345678901212345678901234567890123456789012123456
Number Number Raw 7
1234567890123456789012345678901212345678901234567890123456789012123456 7
1234567890123456789012345678901212345678901234567890123456789012123456
Correct Incorrect 5 Score 7
1234567890123456789012345678901212345678901234567890123456789012123456
34567890123456789012345678901212345678901234567890123456789012123456
7
7
1
12234567890123456789012345678901212345678901234567890123456789012123456 7
1234567890123456789012345678901212345678901234567890123456789012123456 7
1234567890123456789012345678901212345678901234567890123456789012123456
Section 2 2 (.25 3 ) 5 7
1234567890123456789012345678901212345678901234567890123456789012123456
34567890123456789012345678901212345678901234567890123456789012123456
7
7
1
12234567890123456789012345678901212345678901234567890123456789012123456 7
1234567890123456789012345678901212345678901234567890123456789012123456 7
1234567890123456789012345678901212345678901234567890123456789012123456
Section 5 2 (.25 3 ) 5 7
1234567890123456789012345678901212345678901234567890123456789012123456 7
1234567890123456789012345678901212345678901234567890123456789012123456 77
34567890123456789012345678901212345678901234567890123456789012123456
1234567890123456789012345678901212345678901234567890123456789012123456 7
1234567890123456789012345678901212345678901234567890123456789012123456
Section 7 2 (.25 3 ) 5
1234567890123456789012345678901212345678901234567890123456789012123456 7
1234567890123456789012345678901212345678901234567890123456789012123456 77
1234567890123456789012345678901212345678901234567890123456789012123456 7
1
12234567890123456789012345678901212345678901234567890123456789012123456
CRITICAL READING 7
1234567890123456789012345678901212345678901234567890123456789012123456 7
1234567890123456789012345678901212345678901234567890123456789012123456 7
34567890123456789012345678901212345678901234567890123456789012123456
1234567890123456789012345678901212345678901234567890123456789012123456
Sections 7
1234567890123456789012345678901212345678901234567890123456789012123456 7
1234567890123456789012345678901212345678901234567890123456789012123456
1, 4, and 6 7
1234567890123456789012345678901212345678901234567890123456789012123456 77
2 (.25 3 ) 5
1234567890123456789012345678901212345678901234567890123456789012123456 7
1234567890123456789012345678901212345678901234567890123456789012123456 7
1234567890123456789012345678901212345678901234567890123456789012123456 7
1234567890123456789012345678901212345678901234567890123456789012123456
WRITING
1234567890123456789012345678901212345678901234567890123456789012123456 7
1 7
2 34567890123456789012345678901212345678901234567890123456789012123456
1234567890123456789012345678901212345678901234567890123456789012123456 7
1234567890123456789012345678901212345678901234567890123456789012123456
Section 3 2 (.25 3 ) 5 7
1 7
1234567890123456789012345678901212345678901234567890123456789012123456 7
12234567890123456789012345678901212345678901234567890123456789012123456 7
1234567890123456789012345678901212345678901234567890123456789012123456
Section 8 Go to www.petersons.com/satessayedge/ 7
1234567890123456789012345678901212345678901234567890123456789012123456 7
1 34567890123456789012345678901212345678901234567890123456789012123456 7
12234567890123456789012345678901212345678901234567890123456789012123456
for instant online scoring and feedback. 7
1234567890123456789012345678901212345678901234567890123456789012123456 7
1234567890123456789012345678901212345678901234567890123456789012123456 77
34567890123456789012345678901212345678901234567890123456789012123456
1234567890123456789012345678901212345678901234567890123456789012123456 7
1
1234567890123456789012345678901212345678901234567890123456789012123456 7
1234567890123456789012345678901212345678901234567890123456789012123456 7
12234567890123456789012345678901212345678901234567890123456789012123456 7
1234567890123456789012345678901212345678901234567890123456789012123456 7
1234567890123456789012345678901212345678901234567890123456789012123456 7
1234567890123456789012345678901212345678901234567890123456789012123456 7
1 34567890123456789012345678901212345678901234567890123456789012123456 7
1234567890123456789012345678901212345678901234567890123456789012123456 77
2 34567890123456789012345678901212345678901234567890123456789012123456
1234567890123456789012345678901212345678901234567890123456789012123456 7
1234567890123456789012345678901212345678901234567890123456789012123456 7
1234567890123456789012345678901212345678901234567890123456789012123456 7
1
12234567890123456789012345678901212345678901234567890123456789012123456 7
1234567890123456789012345678901212345678901234567890123456789012123456 7
1234567890123456789012345678901212345678901234567890123456789012123456 7
1234567890123456789012345678901212345678901234567890123456789012123456 7
1 34567890123456789012345678901212345678901234567890123456789012123456 7
1234567890123456789012345678901212345678901234567890123456789012123456 77
2 34567890123456789012345678901212345678901234567890123456789012123456
1
12234567890123456789012345678901212345678901234567890123456789012123456 7
1 34567890123456789012345678901212345678901234567890123456789012123456 7
12234567890123456789012345678901212345678901234567890123456789012123456 7
1 34567890123456789012345678901212345678901234567890123456789012123456 7
1234567890123456789012345678901212345678901234567890123456789012123456 7
1234567890123456789012345678901212345678901234567890123456789012123456 7
1234567890123456789012345678901212345678901234567890123456789012123456 7
12345678901234567890123456789012123456789012345678901234567890121234567
Copyright © 2005 Thomson Peterson’s, a part of The Thomson Corporaton 81
SAT is a registered trademark of the College Entrance Examination Board, which
was not involved in the production of and does not endorse this product.
12345678901234567890123456789012123456789012345678901234567890121234567
1234567890123456789012345678901212345678901234567890123456789012123456 7
12 34567890123456789012345678901212345678901234567890123456789012123456 7
12 Score Charts
1234567890123456789012345678901212345678901234567890123456789012123456 7
12 34567890123456789012345678901212345678901234567890123456789012123456 7
34567890123456789012345678901212345678901234567890123456789012123456 7
1234567890123456789012345678901212345678901234567890123456789012123456 7
12 34567890123456789012345678901212345678901234567890123456789012123456 7
1234567890123456789012345678901212345678901234567890123456789012123456 7
12 34567890123456789012345678901212345678901234567890123456789012123456
MATH CRITICAL READING 7
12 34567890123456789012345678901212345678901234567890123456789012123456 7
12 34567890123456789012345678901212345678901234567890123456789012123456 7
12
2
34567890123456789012345678901212345678901234567890123456789012123456
Raw Math Scaled Raw Math Scaled Raw Verbal Scaled Raw Verbal Scaled
34567890123456789012345678901212345678901234567890123456789012123456
7
7
1 Score Score Score Score Score Score Score Score
12 34567890123456789012345678901212345678901234567890123456789012123456 7
12 34567890123456789012345678901212345678901234567890123456789012123456 7
60 800 28 500 78 800 37 510
12 34567890123456789012345678901212345678901234567890123456789012123456
77 800 36 510 7
12
2
34567890123456789012345678901212345678901234567890123456789012123456
59 800 27 490
34567890123456789012345678901212345678901234567890123456789012123456
7
7
1 58 790 26 490 76 800 35 500
12 34567890123456789012345678901212345678901234567890123456789012123456 7
12 34567890123456789012345678901212345678901234567890123456789012123456
57 770 25 480 75 790 34 500 7
12 34567890123456789012345678901212345678901234567890123456789012123456 7
12
2
34567890123456789012345678901212345678901234567890123456789012123456
56 760 24 470 74 780 33 490
34567890123456789012345678901212345678901234567890123456789012123456
7
7
1 55 73 770 32 490
12 34567890123456789012345678901212345678901234567890123456789012123456
740 23 460 7
12 34567890123456789012345678901212345678901234567890123456789012123456
72 760 31 480 7
12 34567890123456789012345678901212345678901234567890123456789012123456
54 720 22 460
7
12 34567890123456789012345678901212345678901234567890123456789012123456
53 710 21 450 71 750 30 480 7
2 34567890123456789012345678901212345678901234567890123456789012123456
1234567890123456789012345678901212345678901234567890123456789012123456
70 740 29 470 7
1234567890123456789012345678901212345678901234567890123456789012123456
52 700 20 440 7
1234567890123456789012345678901212345678901234567890123456789012123456
69 730 28 460 7
1234567890123456789012345678901212345678901234567890123456789012123456
51 690 19 430 7
1234567890123456789012345678901212345678901234567890123456789012123456
68 720 27 460 7
1234567890123456789012345678901212345678901234567890123456789012123456
50 680 18 420
67 710 26 450 7
1 49 670 17 420 7
12 34567890123456789012345678901212345678901234567890123456789012123456
66 700 25 450 7
12 34567890123456789012345678901212345678901234567890123456789012123456
48 660 16 410 7
12 34567890123456789012345678901212345678901234567890123456789012123456
65 700 24 440 7
2 34567890123456789012345678901212345678901234567890123456789012123456
1234567890123456789012345678901212345678901234567890123456789012123456
47 650 15 410 7
1234567890123456789012345678901212345678901234567890123456789012123456
64 690 23 440 7
1234567890123456789012345678901212345678901234567890123456789012123456
46 640 14 400 63 680 22 430 7
1234567890123456789012345678901212345678901234567890123456789012123456 7
1234567890123456789012345678901212345678901234567890123456789012123456 77
45 630 13 390 62 670 21 420
1234567890123456789012345678901212345678901234567890123456789012123456
44 620 12 380
1234567890123456789012345678901212345678901234567890123456789012123456
61 670 20 410 7
1234567890123456789012345678901212345678901234567890123456789012123456
43 610 11 370 60 660 19 410 7
1234567890123456789012345678901212345678901234567890123456789012123456 7
1 42 600 10 360 59 650 18 400 7
2 34567890123456789012345678901212345678901234567890123456789012123456
1234567890123456789012345678901212345678901234567890123456789012123456 7
1234567890123456789012345678901212345678901234567890123456789012123456
41 600 9 350 58 640 17 390 7
1 40 7
1234567890123456789012345678901212345678901234567890123456789012123456
590 8 340 57 640 16 380
7
12
2
34567890123456789012345678901212345678901234567890123456789012123456
39 580 7 330 56 630 15 380
34567890123456789012345678901212345678901234567890123456789012123456
7
7
1234567890123456789012345678901212345678901234567890123456789012123456
1234567890123456789012345678901212345678901234567890123456789012123456
38 570 6 320 55 620 14 370 7
1 37 7
12 34567890123456789012345678901212345678901234567890123456789012123456 7
560 5 310 54 610 13 360
12 34567890123456789012345678901212345678901234567890123456789012123456
53 610 12 360 7
12 34567890123456789012345678901212345678901234567890123456789012123456
36 560 4 300 7
12 34567890123456789012345678901212345678901234567890123456789012123456
35 550 3 280 52 600 11 350 7
1234567890123456789012345678901212345678901234567890123456789012123456 7
1234567890123456789012345678901212345678901234567890123456789012123456
34 540 2 270 51 600 10 340 7
1234567890123456789012345678901212345678901234567890123456789012123456
50 590 9 330 7
2 34567890123456789012345678901212345678901234567890123456789012123456
1234567890123456789012345678901212345678901234567890123456789012123456
33 540 1 250 7
1234567890123456789012345678901212345678901234567890123456789012123456
49 590 8 320 7
1234567890123456789012345678901212345678901234567890123456789012123456
32 530 0 240 7
1234567890123456789012345678901212345678901234567890123456789012123456
48 580 7 310 7
1 31 520 21 220 7
2 34567890123456789012345678901212345678901234567890123456789012123456
1234567890123456789012345678901212345678901234567890123456789012123456
30 510 22 210
47 570 6 300 7
1234567890123456789012345678901212345678901234567890123456789012123456
46 570 5 290 7
1234567890123456789012345678901212345678901234567890123456789012123456
29 510 23 and 200 7
1234567890123456789012345678901212345678901234567890123456789012123456
45 560 4 270 7
1 below 7
1234567890123456789012345678901212345678901234567890123456789012123456 77
2 34567890123456789012345678901212345678901234567890123456789012123456
44 550 3 260
1234567890123456789012345678901212345678901234567890123456789012123456
43 550 2 250
7
1234567890123456789012345678901212345678901234567890123456789012123456 7
1234567890123456789012345678901212345678901234567890123456789012123456
42 540 1 240
7
1
1234567890123456789012345678901212345678901234567890123456789012123456 77
2 34567890123456789012345678901212345678901234567890123456789012123456
41 540 0 230
1 40 530 220
12 34567890123456789012345678901212345678901234567890123456789012123456
21
7
1234567890123456789012345678901212345678901234567890123456789012123456
39 520 22 210 7
12
2
34567890123456789012345678901212345678901234567890123456789012123456
34567890123456789012345678901212345678901234567890123456789012123456
7
7
1 38 520 23 and 200
1234567890123456789012345678901212345678901234567890123456789012123456
below 7
1234567890123456789012345678901212345678901234567890123456789012123456 77
1234567890123456789012345678901212345678901234567890123456789012123456
12345678901234567890123456789012123456789012345678901234567890121234567
82 Copyright © 2005 Thomson Peterson’s, a part of The Thomson Corporaton
SAT is a registered trademark of the College Entrance Examination Board, which
was not involved in the production of and does not endorse this product.

You might also like